You are on page 1of 337

SSC Selection

2023
Post XI
( Official Papers )
12 sets
English
Join @MathsByGaganPratap Telegram Channel for free PDFs

Phase-XI/2023/Selection Posts and Selection Posts/Ladakh/2023


Roll Number Gagan Pratap Maths
Candidate Name Gagan Pratap Maths
Venue Name iON Digital Zone iDZ 2 Moula Ali
Exam Date 27/06/2023
Exam Time 9:00 AM - 10:00 AM
Subject Selection Post Matriculation Level

Section : General Intelligence

hs
Q.1 In a code language, 'BANYAN' is written as ‘EDQBDQ’ and ‘LAUNCH’ is written as
‘ODXQFK’. How will 'STRIKER' be written in that language?
Ans 1. VWULNHU

2. SCFIRET

at
3. ELBURPT

4. UNDARSA

Question ID : 26433056264

M
Status : Answered
Chosen Option : 1

Q.2 Select the option that indicates the correct arrangement of the given words in a logical
and meaningful order.
1. Month
2. Day
ap
3. Year
4. Hour
5. Minute
at
Ans 1. 3, 4, 2, 1, 5

2. 3, 1, 2, 4, 5

3. 3, 1, 2, 5, 4
Pr

4. 3, 2, 1, 4, 5

Question ID : 264330125997
Status : Answered
Chosen Option : 2
an

Q.3 Which of the following numbers will replace the question mark (?) in the given series?
54, 46, 34, 26, 14 , ?
Ans 1. 5

2. 7
ag

3. 6

4. 8

Question ID : 264330142987
Status : Answered
G

Chosen Option : 3
Join @MathsByGaganPratap Telegram Channel for free PDFs
Q.4 Select the option that represents the letters that, when sequentially placed from left to
right in the blanks below, will complete the letter series.

M _ O_ _ N O _ _ N _ P M _ O _
Ans 1. N P M P M O N P

2. N P O P M O P O

3. N P M P M P M N

4. O P M P M O P M

Question ID : 264330142848
Status : Answered
Chosen Option : 1

hs
Q.5 Select the option that is related to the third word in the same way as the second word
is related to the first word. (The words must be considered as meaningful English
words and must NOT be related to each other based on the number of letters/number
of consonants/vowels in the word)
Omelette : Egg :: Road : ?

at
Ans 1. Ore

2. Latex

3. Asphalt

M
4. Flax

Question ID : 264330142866
Status : Answered
ap Chosen Option : 3

Q.6 ‘A # B’ means ‘A is the brother of B’.


‘A @ B’ means ‘A is the daughter of B’.
‘A & B’ means ‘A is the husband of B’.
‘A % B’ means ‘A is the wife of B’.
at
If L @ M % F # Q & P @ N, then how is F related to P?
Ans 1. Husband

2. Brother
Pr

3. Husband’s brother

4. Father

Question ID : 264330142900
Status : Answered
an

Chosen Option : 3

Q.7 Select the option that represents the correct order of the given words as they would
appear in an English dictionary.

1. Devoted
ag

2. Determine
3. Devastating
4. Diaphragm
5. Diagnose
6. Different
Ans 1. 2, 3, 1, 4, 6, 5
G

2. 2, 1, 5, 4, 6, 3

3. 2, 3, 1, 5, 4, 6

4. 2, 3, 1, 5, 6, 4

Question ID : 264330142924
Status : Answered
Chosen Option : 3
Join @MathsByGaganPratap Telegram Channel for free PDFs
Q.8 Three statements are given followed by four conclusions numbered I, II, III and IV.
Assuming the statements to be true, even if they seem to be at variance with
commonly known facts, decide which of the conclusions logically follow(s) from the
statements.

Statements:
Some parrots are pigs.
Some pigs are poles.
All poles are pets.

Conclusions:
I. Some poles are pigs.
II. Some pets are parrots.
III. All the parrots are pets.
IV. Some pets are pigs.
Ans 1. Only conclusion III follows

hs
2. Only conclusions I and IV follow

3. Only conclusion IV follows

4. Only conclusions I and II follow

at
Question ID : 264330142957
Status : Answered
Chosen Option : 2

M
Q.9 ‘A # B’ means ‘A is the wife of B’.
‘A @ B’ means ‘A is the father of B’.
‘A % B’ means ‘A is the sister of B’.
‘A & B’ means ‘A is the daughter of B’.
If A % G & H # B @ Z % M, then how is H related to M?
Ans 1. Mother-in-law

2. Daughter
ap
3. Son

4. Mother
at
Question ID : 26433068265
Status : Answered
Chosen Option : 4
Pr

Q.10 Select the option that represents the letters that, when sequentially placed from left to
right in the blanks, will complete the letter series.
RM_G_M__R_TG_MT_
Ans 1. T R T G M R G
an

2. T R G T G M R

3. T M T G M T G

4. T R G T M R G
ag

Question ID : 26433056778
Status : Answered
Chosen Option : 1

Q.11 Select the option that is related to the fourth term in the same way as the first term is
related to the second term and the fifth term is related to the sixth term
G

16 : 36 :: ? : 50 :: 9 : 22
Ans 1. 23 

2. 34

3. 22

4. 31

Question ID : 26433040808
Status : Answered
Chosen Option : 1
Join @MathsByGaganPratap Telegram Channel for free PDFs
Q.12

Ans
1.

2.

3.

hs
4.

Question ID : 264330143274
Status : Answered

at
Chosen Option : 4

Q.13 Select the option that is related to the fifth term in the same way as the second term is
related to the first term and the fourth term is related to the third term.

M
VISION : IVISNO :: COLOUR : OCOLRU :: YELLOW : ?
Ans 1. LEYWOL

2. EYLLWO

3. WOLLEY

4. EYLOLW
ap
Question ID : 264330142844
Status : Answered
at
Chosen Option : 2

Q.14 Select the correct combination of mathematical signs to sequentially replace the *
signs and to balance the given equation.
289 * 121 * 154 * 307 * (34 * 3) * 2
Pr

Ans 1. ÷, −, +, −, ×, =

2. ×, +, −, =, +, ÷

3. +, −, =, −, ×, ÷

4. ÷, +, −, ×, +, =
an

Question ID : 26433057643
Status : Not Answered
Chosen Option : --
ag

Q.15 Which letter-cluster will replace the question mark (?) and complete the given series?
CURB, ESPC, ?, IOLE, KMJF
Ans 1. GQND

2. HRND
G

3. GRND

4. HQND

Question ID : 264330142914
Status : Answered
Chosen Option : 1
Join @MathsByGaganPratap Telegram Channel for free PDFs
Q.16 Select the option that is related to the third word in the same way as the second word
is related to the first word. (The words must be considered as meaningful English
words and must NOT be related to each other based on the number of letters/number
of consonants/vowels in the word)

Choreographer : Ballet :: Dramatist : ?


Ans 1. Design

2. Play

3. Composition

4. Movie

Question ID : 264330142864
Status : Answered

hs
Chosen Option : 4

Q.17

at
M
Ans

1.
ap
at
2.
Pr

3.
an

4.
ag

Question ID : 264330143286
G

Status : Answered
Chosen Option : 1
Join @MathsByGaganPratap Telegram Channel for free PDFs
Q.18

Ans

1.

hs
2.

at
3.

M
4. ap
Question ID : 264330143285
Status : Answered
at
Chosen Option : 2

Q.19 Select the correct combination of mathematical signs to sequentially replace the *
signs and to balance the given equation.
119 * 7 * 63 * 24 * 3 * 8
Pr

Ans 1. ÷, +, –, ×, =

2. –, +, ÷, ×, =

3. +, ÷, ×, –, =

4. +, ÷, –, ×, =
an

Question ID : 264330142886
Status : Answered
Chosen Option : 1
ag

Q.20 Select the option that is related to the fifth term in the same way as the second term is
related to the first term and the fourth term is related to the third term.
INPUT : STOMH :: OUTPUT : STOSTN :: SOLVE : ?
Ans 1. DUKNR

2. TPMXF
G

3. DUMPT

4. PTUKD

Question ID : 264330142841
Status : Answered
Chosen Option : 1
Join @MathsByGaganPratap Telegram Channel for free PDFs
Q.21 Three statements are given followed by three conclusions numbered I, II and III.
Assuming the statements to be true, even if they seem to be at variance with
commonly known facts, decide which of the conclusions logically follow(s) from the
statements.

Statements:
Some cribs are pants.
No crib is a tap.
All pants are shells.

Conclusions:
I. All shells being cribs is a possibility.
II. At least some taps are not cribs.
III. Some pants are not taps.
Ans 1. Only conclusions I and III follow

2. Only conclusions II and III follow

hs
3. Only conclusion I follows

4. All conclusions I, II and III follow

at
Question ID : 264330142954
Status : Answered
Chosen Option : 2

M
Q.22 Select the option that indicates the correct arrangement of the given words in the
order in which they appear in an English dictionary.
1. Bunion
2. Bureau
3. Bundle
4. Bunting

Ans
5. Bungle
1. 4, 5, 1, 3, 2
ap
2. 4, 5, 3, 1, 2

3. 5, 3, 4, 2, 1
at
4. 3, 5, 1, 4, 2

Question ID : 264330126643
Status : Answered
Pr

Chosen Option : 4

Q.23 Three statements are given, followed by three conclusions numbered I, II and III.
Assuming the statements to be true, even if they seem to be at variance with
commonly known facts, decide which of the conclusions logically follow(s) from the
statements.
an

Statements:
All computers are footballs.
Some footballs are round.
Some round are stools.

Conclusions:
ag

I. Some footballs are computers.


II. Some round are footballs.
III. Some stools are round.
Ans 1. All the conclusions follow

2. Only conclusions II and III follow


G

3. Only conclusions I and II follow

4. Only conclusions I and III follow

Question ID : 26433092649
Status : Answered
Chosen Option : 1
Join @MathsByGaganPratap Telegram Channel for free PDFs
Q.24 Three statements are followed by three conclusions numbered I, II and III. You have to
consider these statements to be true, even if they seem to be at variance with
commonly known facts. Decide which of the given conclusions logically follow(s) from
the given statements.

Statements:
Most suits are trousers.
Some shirts are ties.
All suitcases are watches.

Conclusions:
(I) All trousers are suits.
(II) Some shirts are suitcases.
(III) Some shirts are not watches.
Ans 1. Either conclusion I or conclusion III follows

2. None of the conclusions follow

hs
3. Only conclusion II follows

4. Only conclusion I follows

at
Question ID : 264330142978
Status : Answered
Chosen Option : 1

M
Q.25 Select the combination of letters that when sequentially placed in the blanks of the
given series, will complete the series.
G_BAGS_BGSBC_SB_
Ans 1. DGSB

2. SCBD

3. BDSC
ap
4. SBGD

Question ID : 26433057816
at
Status : Answered
Chosen Option : 4
Pr

Section : General Awareness

Q.1 Which water-soluble vitamin found in various foods, including liver, milk, eggs and
fish, helps to keep your body's blood and nerve cells healthy?
Ans 1. Vitamin C

2. Vitamin B12
an

3. Vitamin E

4. Vitamin A

Question ID : 264330142714
ag

Status : Answered
Chosen Option : 4

Q.2 According to the Census of India 2011, which of the following groups of union
territories has the highest literacy rate?
Ans
G

1. Chandigarh and Andaman & Nicobar

2. Delhi and Chandigarh

3. Delhi and Lakshadweep

4. Lakshadweep and Daman & Diu

Question ID : 264330142800
Status : Answered
Chosen Option : 2
Join @MathsByGaganPratap Telegram Channel for free PDFs
Q.3 Which of the following classical dance forms is related to the worship of Lord
Jagannath?
Ans 1. Odissi

2. Kathak

3. Sattriya

4. Bharatanatyam

Question ID : 264330141227
Status : Answered
Chosen Option : 1

Q.4 Who among the following personalities has received both Grammy and Oscar awards?

hs
Ans 1. Jawed Akhtar

2. Jagjit Singh

3. Gulzar

at
4. Amitabh Bhattacharya

Question ID : 264330141387
Status : Answered

M
Chosen Option : 4

Q.5 Who among the following launched the Virtual Smart Grid Knowledge Centre and
Innovation Park in March 2022?
Ans 1. Giriraj Singh

2. Kiren Rijiju 
ap
3. Hardeep Singh Puri 

4. Raj Kumar Singh 


at
Question ID : 26433061837
Status : Answered
Chosen Option : 3
Pr

Q.6 Who was the first Sultan of the Delhi Sultanate to start military expeditions into
southern India?
Ans 1. Ghiyasuddin Balban

2. Alauddin Khalji
an

3. Muhammad Tughluq

4. Shamsuddin Iltutmish

Question ID : 26433089872
Status : Answered
ag

Chosen Option : 2

Q.7 Which of the following harvest festivals is celebrated in Karnataka in the months of
November-December?
Ans 1. Hampi
G

2. Dasara

3. Huthri

4. Kambala

Question ID : 264330141230
Status : Answered
Chosen Option : 2
Join @MathsByGaganPratap Telegram Channel for free PDFs
Q.8 Padma Awardee Sudha Singh is a sports person in which of the following fields?
Ans 1. Athletics

2. Shooting

3. Swimming

4. Table Tennis

Question ID : 264330142688
Status : Answered
Chosen Option : 1

Q.9 Who among the following national leaders was called ‘Grand Old Man of India’ in
British India?

hs
Ans 1. Gopal Krishna Gokhale

2. Dadabhai Naoroji

3. Prithvis Chandra Ray

at
4. Mahadev Govind Ranade

Question ID : 26433065564
Status : Answered

M
Chosen Option : 2

Q.10 Who became only the third Indian to win a singles tennis match at Olympics in July
2021?
Ans 1. Ramkumar Ramanathan

2. Sumit Nagal
ap
3. Mukund Sasikumar

4. Prajnesh Gunneswaran
at
Question ID : 264330142687
Status : Answered
Chosen Option : 4
Pr

Q.11 India was named ‘Bharat’ after the name of the Bharat clan. In which of the following
Vedas is this clan first mentioned?
Ans 1. Atharvaveda

2. Yajurveda
an

3. Samaveda

4. Rigveda

Question ID : 264330142801
Status : Answered
ag

Chosen Option : 4

Q.12 The Hoysaleshvara temple was built with which of the following stones by a Hoysala
king in 1150?
Ans 1. Dark Schist Stone
G

2. White Marble

3. Sandstone

4. Grey Basalt

Question ID : 26433090743
Status : Answered
Chosen Option : 1
Join @MathsByGaganPratap Telegram Channel for free PDFs
Q.13 Which potassium salt is a mineral supplement used to treat or prevent low levels of
potassium in the blood?
Ans 1. Potassium chloride

2. Potassium iodide

3. Potassium citrate

4. Potassium nitrate

Question ID : 26433055754
Status : Answered
Chosen Option : 2

Q.14 What is the rank of India in the production of rice globally? (As of year 2020)

hs
Ans 1. Third

2. Second

3. Fourth

at
4. First

Question ID : 26433086895
Status : Answered

M
Chosen Option : 4

Q.15 Which of the following autobiographies was written by the Indian film actor Balraj
Sahni?
Ans 1. The Act of Life

2. Main Ek Harfanmaula
ap
3. Meri Filmi Aatmakatha

4. I Am Not an Island
at
Question ID : 264330141402
Status : Not Answered
Chosen Option : --
Pr

Q.16 Which are the only major rivers of south India that flow into the Arabian sea?
Ans 1. The Chambal and the Kosi

2. The Mahanadi and the Brahmaputra

3. The Krishna and the Cauvery


an

4. The Narmada and the Tapti

Question ID : 26433081918
Status : Answered
ag

Chosen Option : 4

Q.17 What is the basic requirement for an ecosystem to function and sustain?
Ans 1. Input of solar energy

2. Soil structure
G

3. Organisms

4. Micro-organisms

Question ID : 26433064762
Status : Answered
Chosen Option : 1
Join @MathsByGaganPratap Telegram Channel for free PDFs
Q.18 In which of the following years was the first edition of the Thomas Cup, an
international men’s team championship in badminton organised?
Ans 1. 1949

2. 1952

3. 1955

4. 1961

Question ID : 26433059364
Status : Not Answered
Chosen Option : --

Q.19 Rani Gaidinliu is associated with which of the following movements?

hs
Ans 1. Heraka

2. Meira Paibi

3. Nupi Lan

at
4. Nisha Bandh

Question ID : 26433099173
Not Attempted and
Status :

M
Marked For Review
Chosen Option : --

Q.20 In January 2022, The Union Cabinet, chaired by Prime Minister Narendra Modi,
approved the extension of the tenure of the National Commission for Safai

Ans 1. five
ap
Karamcharis (NCSK) for __________ years beyond 31 March 2022.

2. four

3. three
at
4. two

Question ID : 26433081806
Not Attempted and
Pr

Status :
Marked For Review
Chosen Option : --

Q.21 To which state of India did Fathima Beevi, the first woman Supreme Court Judge of
India, belong?
Ans 1. Kerala
an

2. Andhra Pradesh

3. Tamil Nadu

4. Karnataka
ag

Question ID : 264330113427
Status : Answered
Chosen Option : 3
G

Q.22 Ratikant Mohapatra, an awardee of ‘Sangeet Natak Akademi Award’, is known for
which of the following classical dances of India?
Ans 1. Odissi

2. Manipuri

3. Kathak

4. Kathakali

Question ID : 26433097103
Status : Answered
Chosen Option : 3
Join @MathsByGaganPratap Telegram Channel for free PDFs
Q.23 As per the 2011 census of India, the male literacy rate in India is ________.
Ans
1.

2.

3.

4.

Question ID : 26433089251
Status : Answered
Chosen Option : 4

hs
Q.24 Which of the following Articles of the Constitution of India defines that the
Fundamental Duties are only meant for the Indian Citizens and not for foreigners?
Ans 1. Article 46 B

at
2. Article 51 A

3. Article 67 A

4. Article 81 C

M
Question ID : 26433086790
Status : Answered
Chosen Option : 2
ap
Q.25 Name the instrument that is used by meteorologist to measure the solar radiation from
the region of the hemisphere of incidence on a flat surface?
Ans 1. Hygrometer

2. Barometer
at
3. Pyranometer

4. Anemometer

Question ID : 264330142718
Pr

Status : Answered
Chosen Option : 3

Section : Quantitative Aptitude


an

Q.1 In a circular race along a track of length 3600 m, X and Y run at speeds of 27 km/h and
45 km/h, respectively. Suppose they start at the same time and in the same direction,
when will they meet again at the starting point?
Ans 1. 1440 sec

2. 720 sec
ag

3. 2200 sec

4. 1200 sec

Question ID : 26433083092
Status : Not Answered
G

Chosen Option : --
Join @MathsByGaganPratap Telegram Channel for free PDFs
Q.2 Aravind deposited ₹18,500 for 5 years at simple interest. After 5 years, he received
₹4,625 as interest. The annual rate of interest was:
Ans 1. 6%

2. 6.5%

3. 5%

4. 7%

Question ID : 264330142260
Status : Answered
Chosen Option : 3

Q.3 A grocer sells pulses at a profit of 13% and uses weights which are 24% less than the

hs
market weight. The percentage of profit (correct to 2 decimal places) earned by him
will be:
Ans 1. 45.56%

2. 37.75%

at
3. 48.68%

4. 42.35%

M
Question ID : 26433081473
Status : Answered
Chosen Option : 3

Q.4 Which of the following numbers is NOT divisible by 72?


Ans 1. 359784
ap
2. 426816

3. 486280

4. 754344
at

Question ID : 264330142676
Not Attempted and
Status :
Marked For Review
Pr

Chosen Option : --

Q.5 The third proportional of 16 and 24 will be ______.


Ans 1. 40

2. 30
an

3. 36

4. 32

Question ID : 264330142532
ag

Status : Answered
Chosen Option : 3

Q.6 Class A has 20 students scoring average marks of 75, and class B has 30 students
scoring average marks of 60. What is the average of both the classes together?
G

Ans 1. 64

2. 66

3. 68

4. 62

Question ID : 264330103604
Status : Answered
Chosen Option : 2
Join @MathsByGaganPratap Telegram Channel for free PDFs
Q.7 A number is first increased by 5% and then it is further increased by 20%. The original
number is increased by:
Ans 1. 16%

2. 30%

3. 18%

4. 26%

Question ID : 264330142551
Status : Answered
Chosen Option : 4

Q.8 Marked price of a TV is listed at ₹24,500. If it’s selling price is ₹20,090, then what is the

hs
discount percentage?
Ans 1. 22%

2. 18%

3. 20%

at
4. 15%

Question ID : 264330102755

M
Status : Answered
Chosen Option : 2

Q.9 A, B and C can separately complete a work in 20, 25 and 30 days, respectively. If A
works on the first day alone, B on second day alone and C on the third day alone, and

Ans
ap
this sequence goes on further, in how many days will the entire work be finished?

1.
at
2.

3.
Pr

4.

Question ID : 26433075571
an

Status : Answered
Chosen Option : 4

Q.10 The mean weight of 32 students in a school is 50 kg. If the weight of the teacher be
included, the mean rises by 500 grams. Find the weight of the teacher.
ag

Ans 1. 63.5 kg 

2. 66.5 kg  

3. 65.5 kg  

4. 64.5 kg 
G

Question ID : 26433058655
Status : Answered
Chosen Option : 1
Join @MathsByGaganPratap Telegram Channel for free PDFs
Q.11 The internal and external diameters of a hollow hemispherical bowl are 6 cm and 10
cm, respectively. If it is melted and recast into a solid cylinder of diameter 14 cm, what
is the height of the cylinder in cm?
Ans 1. 1.1

2. 1.22

3. 1.33

4. 1

Question ID : 26433041645
Status : Not Answered
Chosen Option : --

hs
Q.12 Food is available for 27 days for 35 students in a hostel. For how many days will this
food be sufficient for 45 students?
Ans 1. 25 

2.  22 

at
3.  23 

4. 21

M
Question ID : 26433061084
Status : Answered
Chosen Option : 4

Q.13 If the cost price is 66% of the selling price, then what is the profit percentage (correct

Ans
to 2 decimal places)?
1. 51.52%
ap
2. 50.52%

3. 60.25%
at
4. 55.62%

Question ID : 264330142239
Status : Answered
Pr

Chosen Option : 1

Q.14 Venkat spends 65% of his income and is able to save ₹1,100 per month. His monthly
expenses (correct up to two decimals) are:
Ans
1.
an

2.

3.
ag

4.

Question ID : 26433076372
Status : Answered
Chosen Option : 2
G
Join @MathsByGaganPratap Telegram Channel for free PDFs
Q.15 The simple interest earned on ₹7,000 in 2 years at the rate of R% per annum equals to
the simple interest earned on ₹5,000 at the rate of 5% per annum in 14 years. The value
of R (in percentage) is:
Ans 1.

2.

3.

4.

Question ID : 264330102358
Status : Answered
Chosen Option : 3

hs
Q.16 Somesh is a shopkeeper who gives two successive discounts on an umbrella marked
₹560. The first discount given is 17%. If the customer Mahesh pays ₹420 for the
umbrella, then find the second discount given (correct to two places of decimals).
Ans 1. 4.69%

at
2. 9.64%

3. 6.49%

4. 6.94%

M
Question ID : 264330130089
Status : Not Answered
Chosen Option : --
ap
Q.17 The sum of two numbers 10373 + 24871 is divisible by:
Ans 1. 7

2. 8
at
3. 6

4. 13

Question ID : 264330142548
Pr

Status : Answered
Chosen Option : 3

Q.18 The students in three batches of a dance class are in ratio 2 : 3 : 5. If 20 students
increase in each batch the ratio changes to 4 : 5 : 7. Find the total number of students
in the three batches before the increase.
an

Ans 1. 120

2. 100

3. 150
ag

4. 80

Question ID : 264330142664
Status : Answered
Chosen Option : 2
G
Join @MathsByGaganPratap Telegram Channel for free PDFs
Q.19 If the cost price is 94% of the selling price, then find the profit percentage.
Ans
1.

2.

3.

4.

Question ID : 26433076175
Status : Answered
Chosen Option : 3

hs
Q.20 Find the length of a diagonal of a cuboid whose dimensions are 12 m, 10 m and 8 m.
(up to one decimal point)
Ans 1. 18m

at
2. 16.5m

3. 17m

4. 17.54m

M
Question ID : 26433075511
Status : Answered
ap Chosen Option : 2

Q.21 If 20% of a number is subtracted from a second number and the second number
decreases to its 70%, then what is the ratio of the first number to the second number?
Ans 1. 3:2

2. 4:3
at
3. 2:3

4. 3:4
Pr

Question ID : 26433075650
Status : Not Answered
Chosen Option : --

Q.22
an

Ans 1. 4∶1

2. 1∶4

3. 3∶4
ag

4. 1∶3

Question ID : 264330142554
Status : Answered
Chosen Option : 2
G

Q.23 A person covers 48 km at the speed of 8 km/h, 36 km at the speed of 18 km/h and 16
km at the speed of 8 km/h. What is his average speed in covering the whole distance?
Ans 1. 9 km/h

2. 11 km/h

3. 10.5 km/h

4. 10 km/h

Question ID : 264330142262
Status : Not Answered
Chosen Option : --
Join @MathsByGaganPratap Telegram Channel for free PDFs
Q.24 An item is bought on a condition that three equal instalments of ₹3,993 are to be paid
at a rate of 10% compound interest, compounded annually. The cost of the item is:
Ans 1. ₹10,000

2. ₹9,050

3. ₹9,590

4. ₹9,930

Question ID : 264330103731
Status : Not Answered
Chosen Option : --

Q.25 In Class I, there are 12 students of average age 20 years and in Class II, there are 16

hs
students of average age 23 years. What will be the approximate average age for all the
students of these two classes?
Ans 1. 20.7 years

2. 1.53 years

at
3. 21.7 years

4. 43 years

M
Question ID : 26433073121
Status : Answered
Chosen Option : 1

Section : English Language


ap
Q.1 Select the most appropriate meaning of the given idiom.

He brought his servant to book for breaking the beautiful showpiece.


Ans 1. Scolded his servant
at
2. Relieved his servant

3. Rewarded his servant

4. Sent his servant away


Pr

Question ID : 264330122973
Status : Not Answered
Chosen Option : --

Q.2 Select the most appropriate meaning of the given idiom.


an

Easier said than done


Ans 1. Not as easy as it appears to be 

2. Not as moderate as it appears to be  


ag

3. Not as restricted as it appears to be  

4. Not as difficult as it appears to be  

Question ID : 26433061014
Status : Answered
G

Chosen Option : 1
Join @MathsByGaganPratap Telegram Channel for free PDFs
Q.3 Select the correct spelling of the underlined word from the options.

She is the most underated cricketer in the country.


Ans 1. underateed

2. underatted

3. undderated

4. underrated 

Question ID : 26433088782
Status : Answered
Chosen Option : 2

hs
Q.4 Select the option that can be used as a one-word substitute for the underlined group
of words in the given sentence.

Adversity always presents opportunities for the examination of one's own conscious
thoughts and feelings.
Ans 1. introspection

at
2. judgement 

3. evaluation 

M
4. cognition 

Question ID : 26433073850
Status : Answered
Chosen Option : 1
ap
Q.5 Select the most appropriate synonym of the underlined word in the following
sentence.

Biosphere reserves are ‘learning places for sustainable development’.


at
Ans 1. brief 

2. maintainable

3. temporary 
Pr

4. unsuitable

Question ID : 26433059798
Status : Answered
Chosen Option : 2
an

Q.6 Select the option that expresses the given sentence in indirect speech.
Aravind said, “My teacher is correcting the answer sheets.”
Ans 1. Aravind said his teacher was be corrected the answer sheets.  

2. Aravind said that his teacher is being correcting the answer sheets.  
ag

3. Aravind said that his teacher was correcting the answer sheets. 

4. Aravind said that teacher is corrected the answer sheets.  

Question ID : 264330135982
Status : Answered
G

Chosen Option : 3
Join @MathsByGaganPratap Telegram Channel for free PDFs
Q.7 Select the option that can be used as a one-word substitute for the given group of
words.

The wide variety of shared and different personal and group characteristics among
human beings
Ans 1. Diversity

2. Viscosity

3. Community

4. Intensity

Question ID : 26433060300
Status : Answered
Chosen Option : 3

hs
Q.8 Select the most appropriate meaning of the given idiom.

To show a clean pair of heels


Ans 1. To clean shoes fast

at
2. Show clean shoes

3. Show clean feet

M
4. To run fast

Question ID : 26433098328
Status : Answered
Chosen Option : 4
ap
Q.9 Fill in the blank with the most appropriate word that can be used here based on the
given context.

We as human beings usually tend to forget that life itself is an accumulated process of
_________.
at
Ans 1. learning 

2. recalling

3. remembering
Pr

4. testing

Question ID : 26433059829
Status : Answered
Chosen Option : 1
an

Q.10 Parts of the following sentence have been underlined and given as options. Select the
option that contains a spelling error.

In the unscientific age or community, there are official repositories of wisdom, such as
Egyptian preists and Tibetan Lamas.
ag

Ans 1. repositories

2. Egyptian

3. unscientific

4. preists
G

Question ID : 26433088434
Status : Answered
Chosen Option : 1
Join @MathsByGaganPratap Telegram Channel for free PDFs
Q.11 Select the idiom that gives the most appropriate meaning of the underlined phrase in
the following sentence.

‘Honesty is the best policy’ does not remain valid in the present times of corruption
and greed.
Ans 1. Go through thick and thin

2. Put up

3. Keep pace with

4. Hold good

Question ID : 26433086436
Status : Answered
Chosen Option : 1

hs
Q.12 Select the most appropriate option to fill in the blank.
You must _____________ what you cannot cure.
Ans 1. embracing

at
2. endure 

3. state

4. take

M
Question ID : 26433060056
Status : Answered
ap Chosen Option : 2

Q.13 Identify the most appropriate ANTONYM of the underlined word in the given sentence.

Precious stones are expensive because they are not common.


Ans 1. Abundant 
at
2. Rare  

3. Natural

4. Extinct 
Pr

Question ID : 26433060801
Status : Answered
Chosen Option : 2

Q.14 Choose the correct spelling of the underlined word in the given sentence.
an

We are going on a hiking expidition.


Ans 1. expedition

2. expedetion 

3. exhipidition 
ag

4. ixpedition

Question ID : 26433084611
Status : Answered
G

Chosen Option : 1
Join @MathsByGaganPratap Telegram Channel for free PDFs
Q.15 Select the option that expresses the given sentence in active voice.
The manager was being scolded by Rushikesh.
Ans 1. Rushikesh was scolding the manager.

2. Rushikesh scolded the manager.

3. Rushikesh has been scolding the manager.

4. Rushikesh will be scolding the manager.

Question ID : 264330134203
Status : Answered
Chosen Option : 1

Comprehension:

hs
In the following passage, some words have been deleted. Read the passage carefully and
select the most appropriate option to fill in each blank.

Power foods are foods that provide (1) _______ levels of nutrients like fibre, potassium and
minerals. With people becoming increasingly health conscious today, a lot of fitness trainers
encourage their clients to include these foods in their daily diet to (2) ________ muscle

at
development. There are various ways of incorporating power foods in your daily diet. Of
course, the key to enjoying power foods is proper preparation of these foods, the (3) _______
of season-fresh foods and identifying your choice of flavour among power foods.
Some of the recommended power food combinations are those that are prepared in our
kitchens on a regular basis. Take for instance, the combination of chickpeas and onions. This

M
combination is a powerful source of iron which is required by the body to transport oxygen to
its various parts. Iron (4) ______ can lead to anaemia, fatigue, brain fog and tiredness. A
study by the Journal of Agricultural and Food Chemistry says that sulphur compounds in
onion and garlic help in the absorption of iron and zinc from chickpeas. The combination is a
hit with teenagers who need to be (5) _______ about getting iron in their diet. A quick way to
prepare this power food is to make a chickpea salad with chopped onions, chaat masala and
cilantro.

SubQuestion No : 16
ap
Q.16 Select the most appropriate option to fill in blank number 1.
Ans 1. thriving
at
2. rich 

3. flourishing

4. prosperous
Pr

Question ID : 26433072017
Status : Answered
Chosen Option : 2
an
ag
G
Join @MathsByGaganPratap Telegram Channel for free PDFs
Comprehension:
In the following passage, some words have been deleted. Read the passage carefully and
select the most appropriate option to fill in each blank.

Power foods are foods that provide (1) _______ levels of nutrients like fibre, potassium and
minerals. With people becoming increasingly health conscious today, a lot of fitness trainers
encourage their clients to include these foods in their daily diet to (2) ________ muscle
development. There are various ways of incorporating power foods in your daily diet. Of
course, the key to enjoying power foods is proper preparation of these foods, the (3) _______
of season-fresh foods and identifying your choice of flavour among power foods.
Some of the recommended power food combinations are those that are prepared in our
kitchens on a regular basis. Take for instance, the combination of chickpeas and onions. This
combination is a powerful source of iron which is required by the body to transport oxygen to
its various parts. Iron (4) ______ can lead to anaemia, fatigue, brain fog and tiredness. A
study by the Journal of Agricultural and Food Chemistry says that sulphur compounds in
onion and garlic help in the absorption of iron and zinc from chickpeas. The combination is a

hs
hit with teenagers who need to be (5) _______ about getting iron in their diet. A quick way to
prepare this power food is to make a chickpea salad with chopped onions, chaat masala and
cilantro.

SubQuestion No : 17
Q.17 Select the most appropriate option to fill in blank number 2.

at
Ans 1. spike

2. magnify

3. intensify

M
4. increase

Question ID : 26433072018
ap Status : Answered
Chosen Option : 4

Comprehension:
In the following passage, some words have been deleted. Read the passage carefully and
select the most appropriate option to fill in each blank.
at
Power foods are foods that provide (1) _______ levels of nutrients like fibre, potassium and
minerals. With people becoming increasingly health conscious today, a lot of fitness trainers
encourage their clients to include these foods in their daily diet to (2) ________ muscle
development. There are various ways of incorporating power foods in your daily diet. Of
course, the key to enjoying power foods is proper preparation of these foods, the (3) _______
Pr

of season-fresh foods and identifying your choice of flavour among power foods.
Some of the recommended power food combinations are those that are prepared in our
kitchens on a regular basis. Take for instance, the combination of chickpeas and onions. This
combination is a powerful source of iron which is required by the body to transport oxygen to
its various parts. Iron (4) ______ can lead to anaemia, fatigue, brain fog and tiredness. A
study by the Journal of Agricultural and Food Chemistry says that sulphur compounds in
onion and garlic help in the absorption of iron and zinc from chickpeas. The combination is a
an

hit with teenagers who need to be (5) _______ about getting iron in their diet. A quick way to
prepare this power food is to make a chickpea salad with chopped onions, chaat masala and
cilantro.

SubQuestion No : 18
Q.18 Select the most appropriate option to fill in blank number 3.
ag

Ans 1. wield

2. apply

3. handle

4. use 
G

Question ID : 26433072019
Status : Answered
Chosen Option : 1
Join @MathsByGaganPratap Telegram Channel for free PDFs
Comprehension:
In the following passage, some words have been deleted. Read the passage carefully and
select the most appropriate option to fill in each blank.

Power foods are foods that provide (1) _______ levels of nutrients like fibre, potassium and
minerals. With people becoming increasingly health conscious today, a lot of fitness trainers
encourage their clients to include these foods in their daily diet to (2) ________ muscle
development. There are various ways of incorporating power foods in your daily diet. Of
course, the key to enjoying power foods is proper preparation of these foods, the (3) _______
of season-fresh foods and identifying your choice of flavour among power foods.
Some of the recommended power food combinations are those that are prepared in our
kitchens on a regular basis. Take for instance, the combination of chickpeas and onions. This
combination is a powerful source of iron which is required by the body to transport oxygen to
its various parts. Iron (4) ______ can lead to anaemia, fatigue, brain fog and tiredness. A
study by the Journal of Agricultural and Food Chemistry says that sulphur compounds in
onion and garlic help in the absorption of iron and zinc from chickpeas. The combination is a

hs
hit with teenagers who need to be (5) _______ about getting iron in their diet. A quick way to
prepare this power food is to make a chickpea salad with chopped onions, chaat masala and
cilantro.

SubQuestion No : 19
Q.19 Select the most appropriate option to fill in blank number 4.

at
Ans 1. deficiency

2. inadequacy

3. lack 

M
4. drought

Question ID : 26433072020
ap Status : Answered
Chosen Option : 1

Comprehension:
In the following passage, some words have been deleted. Read the passage carefully and
select the most appropriate option to fill in each blank.
at
Power foods are foods that provide (1) _______ levels of nutrients like fibre, potassium and
minerals. With people becoming increasingly health conscious today, a lot of fitness trainers
encourage their clients to include these foods in their daily diet to (2) ________ muscle
development. There are various ways of incorporating power foods in your daily diet. Of
course, the key to enjoying power foods is proper preparation of these foods, the (3) _______
Pr

of season-fresh foods and identifying your choice of flavour among power foods.
Some of the recommended power food combinations are those that are prepared in our
kitchens on a regular basis. Take for instance, the combination of chickpeas and onions. This
combination is a powerful source of iron which is required by the body to transport oxygen to
its various parts. Iron (4) ______ can lead to anaemia, fatigue, brain fog and tiredness. A
study by the Journal of Agricultural and Food Chemistry says that sulphur compounds in
onion and garlic help in the absorption of iron and zinc from chickpeas. The combination is a
an

hit with teenagers who need to be (5) _______ about getting iron in their diet. A quick way to
prepare this power food is to make a chickpea salad with chopped onions, chaat masala and
cilantro.

SubQuestion No : 20
Q.20 Select the most appropriate option to fill in blank number 5.
ag

Ans 1. vibrant

2. diligent 

3. swamped

4. immersed
G

Question ID : 26433072021
Status : Answered
Chosen Option : 1
Join @MathsByGaganPratap Telegram Channel for free PDFs
Comprehension:
Read the given passage and answer the questions that follow.

“For me, trees have always been the most penetrating preachers. I revere them when they
live in tribes and families, in forests and groves. And even more I revere them when they
stand alone. They are like lonely persons. Not like hermits who have stolen away out of some
weakness, but like great, solitary men, like Beethoven and Nietzsche. In their highest boughs
the world rustles, their roots rest in infinity; but they do not lose themselves there, they
struggle with all the force of their lives for one thing only: to fulfil themselves according to their
own laws, to build up their own form, to represent themselves. Nothing is holier, nothing is
more exemplary than a beautiful, strong tree. When a tree is cut down and reveals its naked
death-wound to the sun, one can read its whole history in the luminous, inscribed disk of its
trunk: in the rings of its years, its scars, all the struggle, all the suffering, all the sickness, all
the happiness and prosperity stand truly written, the narrow years and the luxurious years,
the attacks withstood, the storms endured. And every young farm-boy knows that the hardest
and noblest wood has the narrowest rings, that high on the mountains and in continuing

hs
danger the most indestructible, the strongest, the ideal trees grow.”

SubQuestion No : 21
Q.21 Where can one read the complete history of a tree?
Ans 1. On its leaves in sunny days

at
2. On its branches when it is full-grown

3. On its skin when it is counted

4. On its trunk when it is cut down

M
Question ID : 26433076129
Status : Answered
Chosen Option : 1

Comprehension:
ap
Read the given passage and answer the questions that follow.

“For me, trees have always been the most penetrating preachers. I revere them when they
live in tribes and families, in forests and groves. And even more I revere them when they
stand alone. They are like lonely persons. Not like hermits who have stolen away out of some
at
weakness, but like great, solitary men, like Beethoven and Nietzsche. In their highest boughs
the world rustles, their roots rest in infinity; but they do not lose themselves there, they
struggle with all the force of their lives for one thing only: to fulfil themselves according to their
own laws, to build up their own form, to represent themselves. Nothing is holier, nothing is
more exemplary than a beautiful, strong tree. When a tree is cut down and reveals its naked
Pr

death-wound to the sun, one can read its whole history in the luminous, inscribed disk of its
trunk: in the rings of its years, its scars, all the struggle, all the suffering, all the sickness, all
the happiness and prosperity stand truly written, the narrow years and the luxurious years,
the attacks withstood, the storms endured. And every young farm-boy knows that the hardest
and noblest wood has the narrowest rings, that high on the mountains and in continuing
danger the most indestructible, the strongest, the ideal trees grow.”

SubQuestion No : 22
an

Q.22 Where does the world rustle, according to the given passage?
Ans 1. Around the trunk of trees

2. Over the forest in the sunny days

3. In the highest boughs of trees


ag

4. In the deepest roots of trees

Question ID : 26433076128
Status : Not Answered
G

Chosen Option : --
Join @MathsByGaganPratap Telegram Channel for free PDFs
Comprehension:
Read the given passage and answer the questions that follow.

“For me, trees have always been the most penetrating preachers. I revere them when they
live in tribes and families, in forests and groves. And even more I revere them when they
stand alone. They are like lonely persons. Not like hermits who have stolen away out of some
weakness, but like great, solitary men, like Beethoven and Nietzsche. In their highest boughs
the world rustles, their roots rest in infinity; but they do not lose themselves there, they
struggle with all the force of their lives for one thing only: to fulfil themselves according to their
own laws, to build up their own form, to represent themselves. Nothing is holier, nothing is
more exemplary than a beautiful, strong tree. When a tree is cut down and reveals its naked
death-wound to the sun, one can read its whole history in the luminous, inscribed disk of its
trunk: in the rings of its years, its scars, all the struggle, all the suffering, all the sickness, all
the happiness and prosperity stand truly written, the narrow years and the luxurious years,
the attacks withstood, the storms endured. And every young farm-boy knows that the hardest
and noblest wood has the narrowest rings, that high on the mountains and in continuing

hs
danger the most indestructible, the strongest, the ideal trees grow.”

SubQuestion No : 23
Q.23 Select the most suitable word given in the passage that means ‘very good and
perfect’.
Ans 1. Holier

at
2. Penetrating

3. Luminous

4. Exemplary

M
Question ID : 26433076130
Status : Not Answered
ap Chosen Option : --

Comprehension:
Read the given passage and answer the questions that follow.

“For me, trees have always been the most penetrating preachers. I revere them when they
live in tribes and families, in forests and groves. And even more I revere them when they
at
stand alone. They are like lonely persons. Not like hermits who have stolen away out of some
weakness, but like great, solitary men, like Beethoven and Nietzsche. In their highest boughs
the world rustles, their roots rest in infinity; but they do not lose themselves there, they
struggle with all the force of their lives for one thing only: to fulfil themselves according to their
own laws, to build up their own form, to represent themselves. Nothing is holier, nothing is
Pr

more exemplary than a beautiful, strong tree. When a tree is cut down and reveals its naked
death-wound to the sun, one can read its whole history in the luminous, inscribed disk of its
trunk: in the rings of its years, its scars, all the struggle, all the suffering, all the sickness, all
the happiness and prosperity stand truly written, the narrow years and the luxurious years,
the attacks withstood, the storms endured. And every young farm-boy knows that the hardest
and noblest wood has the narrowest rings, that high on the mountains and in continuing
danger the most indestructible, the strongest, the ideal trees grow.”
an

SubQuestion No : 24
Q.24 Which of the following labels is most appropriate for the author of the given passage?
Ans 1. Arrogant theorist

2. Nostalgic dreamer
ag

3. Environment enthusiast 

4. Social activist

Question ID : 26433076132
Status : Not Answered
G

Chosen Option : --
Join @MathsByGaganPratap Telegram Channel for free PDFs
Comprehension:
Read the given passage and answer the questions that follow.

“For me, trees have always been the most penetrating preachers. I revere them when they
live in tribes and families, in forests and groves. And even more I revere them when they
stand alone. They are like lonely persons. Not like hermits who have stolen away out of some
weakness, but like great, solitary men, like Beethoven and Nietzsche. In their highest boughs
the world rustles, their roots rest in infinity; but they do not lose themselves there, they
struggle with all the force of their lives for one thing only: to fulfil themselves according to their
own laws, to build up their own form, to represent themselves. Nothing is holier, nothing is
more exemplary than a beautiful, strong tree. When a tree is cut down and reveals its naked
death-wound to the sun, one can read its whole history in the luminous, inscribed disk of its
trunk: in the rings of its years, its scars, all the struggle, all the suffering, all the sickness, all
the happiness and prosperity stand truly written, the narrow years and the luxurious years,
the attacks withstood, the storms endured. And every young farm-boy knows that the hardest
and noblest wood has the narrowest rings, that high on the mountains and in continuing

hs
danger the most indestructible, the strongest, the ideal trees grow.”

SubQuestion No : 25
Q.25 What is the tone of the speaker?
Ans 1. Emotional

at
2. Apathetic

3. Vituperative

4. Sarcastic

M
Question ID : 26433076131
Status : Not Answered
Chosen Option : --
ap
at
Pr
an
ag
G
Join @MathsByGaganPratap Telegram Channel for free PDFs

Phase-XI/2023/Selection Posts and Selection Posts/Ladakh/2023


Roll Number Gagan Pratap Maths
Candidate Name Gagan Pratap Maths
Venue Name iON Digital Zone iDZ 2 Mathura Road
Exam Date 27/06/2023
Exam Time 11:45 AM - 12:45 PM
Subject Selection Post Matriculation Level

Section : General Intelligence

hs
Q.1 Select the correct combination of mathematical signs to sequentially replace the *
signs and to balance the given equation.
62*14*7*18*26
Ans 1. +,×, =,−

2. −,+, =,×

at
3. =,÷,×,+

4. =,×,+,−

M
Question ID : 264330142883
Status : Answered
Chosen Option : 3

Q.2
ap
at

Ans
Pr

1.
an

2.
ag

3.
G

4.

Question ID : 264330143293
Status : Answered
Chosen Option : 1
Join @MathsByGaganPratap Telegram Channel for free PDFs
Q.3

Ans
1.

2.

3.

hs
4.

Question ID : 264330143275
Status : Answered

at
Chosen Option : 1

Q.4 Select the option that represents the letters that, when sequentially placed from left to
right in the blanks below, will complete the letter series.

M
_G _J Y _ U _ _G _ _Y G U _
Ans 1. Y U G J Y U J J

2. Y U G J Y U Y J ap
3. Y U G G Y U J J

4. Y U G J Y U J U

Question ID : 264330142853
Status : Answered
at
Chosen Option : 1

Q.5 Three statements are given followed by three conclusions numbered I, II and III.
Assuming the statements to be true, even if they seem to be at variance with
Pr

commonly known facts, decide which of the conclusions logically follow(s) from the
statements.

Statements:
All tanks are plates.
All cots are plates.
Some oats are cots.
an

Conclusions:
I. Some tanks are cots.
II. Some plates are tanks.
III. Some plates are oats.
Ans 1. Either conclusion I or II follows
ag

2. Only conclusion II follows

3. Only conclusion I follows

4. Both conclusions II and III follow


G

Question ID : 264330142958
Status : Answered
Chosen Option : 4
Join @MathsByGaganPratap Telegram Channel for free PDFs
Q.6 ‘A # B’ means ‘A is the brother of B’.
‘A @ B’ means ‘A is the daughter of B’.
‘A & B’ means ‘A is the husband of B’.
‘A % B’ means ‘A is the wife of B’.
If W % D # G @ B & M @ I, then how is D related to M?
Ans 1. Son

2. Father’s father

3. Father

4. Grandson

Question ID : 264330142903
Status : Answered
Chosen Option : 1

hs
Q.7 Select the correct combination of mathematical signs to sequentially replace the *
signs and to balance the given equation.
165 * 33 * 26 * 17 * 2 * 19 * 16
Ans 1. ÷, +, −, ×, +, =

at
2. ÷, −, +, −, ×, =

3. ×, +, −, =, +, ÷

M
4. ÷, +, =, +, −, ×

Question ID : 26433057642
Status : Answered
Chosen Option : 1
ap
Q.8 Select the option that indicates the correct arrangement of the given words in a logical
and meaningful order.
1. Seedling
2. Sapling
3. Tree
at
4. Seed
5. Forest
Ans 1. 4, 1, 2, 3, 5

2. 2, 4, 3, 5, 1
Pr

3. 1, 3, 4, 2, 5

4. 3, 1, 5, 4, 2

Question ID : 264330125995
an

Status : Answered
Chosen Option : 1
ag
G
Join @MathsByGaganPratap Telegram Channel for free PDFs
Q.9

Ans

1.

hs
2.

at
3.

M
4.
ap
Question ID : 264330143284
Status : Answered
Chosen Option : 1
at
Q.10 Three statements are given followed by three conclusions numbered I, II and III.
Assuming the statements to be true, even if they seem to be at variance with
commonly known facts, decide which of the conclusions logically follow(s) from the
statements.
Pr

Statements:
No mug is a bucket.
All mugs are cups.
Some cups are spoons.

Conclusions:
I. Some spoons are buckets.
an

II. Some mugs are spoons.


III. Some cups are not buckets.
Ans 1. Only conclusions II and III follow

2. Only conclusions I and II follow


ag

3. All conclusions follow

4. Only conclusion III follows

Question ID : 26433091717
Status : Answered
G

Chosen Option : 4
Join @MathsByGaganPratap Telegram Channel for free PDFs
Q.11 Select the option that indicates the correct arrangement of the given words in the
order in which they appear in an English dictionary.
1. Memoir
2. Menial
3. Melamine
4. Melancholy
5. Memory
Ans 1. 4, 5, 1, 2, 3

2. 4, 3, 2, 1, 5

3. 3, 4, 1, 5, 2

4. 5, 3, 1, 4, 2

Question ID : 264330126645

hs
Status : Answered
Chosen Option : 3

Q.12 ‘A + B’ means ‘A is the brother of B’.


‘A – B’ means ‘A is the mother of B’.

at
‘A × B’ means ‘A is the father of B’.
‘A ÷ B’ means ‘A is the sister of B’.
If C × M ÷ S × K – T + Q, then which of the following statements is NOT correct?
Ans 1. M is T's father's sister.

M
2. C is K’s father’s father.

3. K is the mother of Q.

4. S is Q’s mother’s father.


ap Question ID : 26433095002
Status : Answered
Chosen Option : 1

Q.13 Select the option that is related to the fifth letter-cluster in the same way as the second
at
letter-cluster is related to the first letter-cluster and the fourth letter-cluster is related
to the third letter-cluster.

RED : GHU :: BLUE : HXOE :: PINK : ?


Pr

Ans 1. NQLS

2. PLOK

3. MNHO

4. JHGF
an

Question ID : 264330142858
Status : Answered
Chosen Option : 1
ag
G
Join @MathsByGaganPratap Telegram Channel for free PDFs
Q.14 Three statements are given, followed by three conclusions numbered I, II and III.
Assuming the statements to be true, even if they seem to be at variance with
commonly known facts, decide which of the conclusions logically follow(s) from the
statements.

Statements:
All schools are colleges.
All colleges are houses.
All houses are roads.

Conclusions:
I. All schools are roads.
II. All schools are houses.
III. All colleges are roads.
Ans 1.  Only conclusions I and III follow.

2. Only conclusions II and III follow. 

hs
3.  Only conclusions I and II follow.

4. All conclusions follow. 

at
Question ID : 26433095997
Status : Answered
Chosen Option : 4

M
Q.15 Select the option that represents the letters that, when sequentially placed from left to
right in the blanks below, will complete the letter series.

Q _ C _ Q S _B _ S _ _Q S _ _
Ans 1. C B C Q C B B C ap
2. S B C Q C B C B

3. S B C B C B Q C

4. B B C B C B Q C
at
Question ID : 264330142851
Status : Answered
Chosen Option : 2
Pr

Q.16 Select the option that is related to the third word in the same way as the second word
is related to the first word. (The words must be considered as meaningful English
words and must NOT be related to each other based on the number of letters/number
of consonants/vowels in the word)
Oology : Eggs :: Palaeontology : ?
Ans 1. Soil
an

2. Fossils

3. Man

4. Writings
ag

Question ID : 264330142869
Not Attempted and
Status :
Marked For Review
Chosen Option : --
G
Join @MathsByGaganPratap Telegram Channel for free PDFs
Q.17 Select the option that represents the correct order of the given words as they would
appear in an English dictionary.

1. MUSLIN
2. MOUSSE
3. MOUTH
4. MOW
5. MOVE
6. MUCK
Ans 1. 2,4,3,5,6,1

2. 5,3,4,2,1,6

3. 2,3,5,4,6,1

4. 1,2,3,4,5,6

hs
Question ID : 264330142855
Status : Answered
Chosen Option : 3

at
Q.18 Three statements are followed by three conclusions numbered I, II and III. You have to
consider these statements to be true, even if they seem to be at variance with
commonly known facts. Decide which of the given conclusions logically follow(s) from
the given statements.

M
Statements:
No pigeon is a dove.
Some doves are sparrows.
All sparrows are birds.

Conclusions:
(I) Some sparrows are not pigeons.
(II) Some sparrows are doves.
(III) Some birds are doves.
ap
Ans 1. Only conclusion I follows

2. Only conclusion II follows


at
3. Either conclusion I or conclusion III follows

4. All the conclusions I, II and III follow


Pr

Question ID : 264330142980
Status : Answered
Chosen Option : 4

Q.19 Which letter cluster will replace the question mark (?) to complete the given series?
SPIN, RQJM, ?, PSLK, OTMJ
an

Ans 1. QRKL

2.  QRLK

3. RQLK

4.  QPKL
ag

Question ID : 26433056234
Status : Answered
Chosen Option : 1
G

Q.20 Which of the following numbers will replace the questionmark (?) in the given number
series?
25, 115, 214, 322, ?, 565, 700
Ans 1. 439

2. 427

3. 448

4. 432

Question ID : 264330142881
Status : Answered
Chosen Option : 1
Join @MathsByGaganPratap Telegram Channel for free PDFs
Q.21 Select the option that is related to the third term in the same way as the second term is
related to the first term and the sixth term is related to the fifth term.
MISSION : NOSSIIM :: PHOENIX : ? :: ORGANIC: CIGANRO
Ans 1. XHOENIP

2. XIOENHP

3. PHEONIX

4. PHOEXIN

Question ID : 264330142837
Status : Answered
Chosen Option : 2

hs
Q.22 Select the option that represents the letters that, when sequentially placed from left to
right in the blanks below, will complete the letter series.
L_H__LKHC__KH__L_HCW
Ans 1. K C W W L C W K

2. W C L K L W K L

at
3. K K C W L K C W

4. L C W K K W C L

M
Question ID : 26433068991
Status : Answered
Chosen Option : 1
ap
Q.23 Select the option that is related to the fourth term in the same way as the first term is
related to the second term and the fifth term is related to the sixth term
8 : 508 :: ? : 2193 :: 2 : 4
Ans 1. 15

2. 13 
at
3. 19

4. 17
Pr

Question ID : 26433040811
Status : Answered
Chosen Option : 2

Q.24 Select the option that is related to the fifth term in the same way as the second term is
related to the first term and the fourth term is related to the third term.
an

EARLY :YLRAE :: IMAGE : EGAMI :: LASER : ?


Ans 1. RASEL

2. RESAL

3. ERSLA
ag

4. EASRL

Question ID : 264330142833
Status : Answered
Chosen Option : 2
G
Join @MathsByGaganPratap Telegram Channel for free PDFs
Q.25 ‘A # B’ means ‘A is the daughter of B’.
‘A % B’ means ‘A is the husband of B’.
‘A & B’ means ‘A is the brother of B’.
‘A @ B’ means ‘A is the sister of B’.
If S # P % Q @ T & V % X , then how is S related to X?
Ans 1. Husband’s sister’s daughter

2. Brother

3. Sister

4. Husband’s brother’s daughter

Question ID : 26433091637
Status : Answered
Chosen Option : 1

hs
Section : General Awareness

Q.1 In which of the following sports is the technique clean and jerk used?

at
Ans 1. Wrestling

2. Hammer throw

3. Body building

M
4. Weightlifting

Question ID : 264330142823
ap Status : Answered
Chosen Option : 4

Q.2 Which compound is used to remove the skins from tomatoes, potatoes and other
fruits and vegetables for canning and as an ingredient in food preservatives that helps
prevent mold and bacteria from growing in food?
Ans 1. Sodium hydroxide
at
2. Magnesium bisulphite

3. Aluminium phosphide

4. Sodium bicarbonate
Pr

Question ID : 264330142708
Status : Not Answered
Chosen Option : --
an

Q.3 How is the molecular weight of fatty acid determined?


Ans 1. Iodine value

2. Acid value

3. Peroxide value
ag

4. Saponification value

Question ID : 26433064452
Status : Not Answered
Chosen Option : --
G
Join @MathsByGaganPratap Telegram Channel for free PDFs
Q.4 If four resistors of 3 Ω, 4 Ω, 5 Ω and 6 Ω are connected in series, what will be the
combined resistance of the four resistors?
Ans 1. 3 Ω

2. 9 Ω

3. 18 Ω

4. 10 Ω

Question ID : 26433053976
Status : Answered
Chosen Option : 3

Q.5 Which of the following instruments is performed by Nandini and Ragini Shankar?

hs
Ans 1. Veena

2. Sitar

3. Violin

at
4. Mridangam

Question ID : 264330141393
Status : Not Answered

M
Chosen Option : --

Q.6 What was the theme of Independence Day in the year 2015 in India?
Ans 1. Ye India ka Time Hai ap
2. Towards True Freedom

3. Yad Karo Kurbani

4. One Country, One People


at
Question ID : 264330141331
Status : Not Answered
Chosen Option : --
Pr

Q.7 What was the style of architecture used in Laxmi Vilas Palace of Vadodara, Gujarat?
Ans 1. Indo-Islamic

2. Hindu Temple

3. Indo-Saracenic
an

4. Mughal

Question ID : 26433090746
Status : Answered
Chosen Option : 3
ag

Q.8 Which of the following is the highest peak of South India?


Ans 1. Doda Betta

2. Mahendragiri
G

3. Anai Mudi

4. Makurti

Question ID : 26433097121
Status : Answered
Chosen Option : 1
Join @MathsByGaganPratap Telegram Channel for free PDFs
Q.9 In which of the following years, did the Nawab of Awadh accept the Subsidiary
Alliance, introduced by Lord Wellesley, Governor General of British India?
Ans 1. 1801

2. 1806

3. 1796

4. 1809

Question ID : 26433065544
Status : Answered
Chosen Option : 1

Q.10 Kamalini and Nalini Asthana, the dancer duo have been conferred with the Padma Shri

hs
in 2022. They are renowned for which of the following dance forms?
Ans 1. Kuchipudi

2. Manipuri

3. Bharatnatyam

at
4. Kathak

Question ID : 26433099196

M
Status : Not Answered
Chosen Option : --

Q.11 Which of the following is considered as small earthquakes in underground caves and
mines which are caused by the seismic waves produced by the eruption of rock on the

Ans
surface?
1. Explosion earthquake
ap
2. Collapse earthquake

3. Reservoir induced earthquakes


at
4. Volcanic earthquake

Question ID : 264330142721
Status : Answered
Pr

Chosen Option : 2

Q.12 Which of the following classical dance forms begins with a drum playing performance
called ‘Kelikottu’?
Ans 1. Bharatanatyam
an

2. Odissi

3. Kuchipudi

4. Kathakali
ag

Question ID : 264330141322
Status : Not Answered
Chosen Option : --

Q.13 Mariyappan Thangavelu and Sharad Kumar won Silver and Bronze in which sport at
G

the Tokyo Paralympics?


Ans 1. High Jump

2. Shot Put

3. Weightlifting

4. Shooting

Question ID : 264330142685
Status : Not Answered
Chosen Option : --
Join @MathsByGaganPratap Telegram Channel for free PDFs
Q.14 Which of the following Articles of the Constitution of India states that no religious
instructions shall be provided in any education institution wholly maintained out of
state funds?
Ans 1. Article 40

2. Article 68

3. Article 54

4. Article 28

Question ID : 26433086813
Status : Answered
Chosen Option : 4

hs
Q.15 Who was appointed as the Chief Economic Advisor of the Government of India in
January 2022?
Ans 1. M Jagdish Kumar

2. Sanjeev Sanyal

at
3. V Anantha Nageswaran

4. Madhavi Puri

M
Question ID : 26433072895
Status : Answered
Chosen Option : 1

Q.16 When were the general elections to the Legislative Assembly of the National Capital

Ans
Territory held for the first time?
1. 1990
ap
2. 1993

3. 1992
at
4. 1991

Question ID : 264330113421
Status : Not Answered
Pr

Chosen Option : --

Q.17 In 1921–22, farmers of the Malabar seacoast of Kerala undertook a great uprising,
which is known as the _________.
Ans 1. Munda uprising
an

2. Moplah uprising

3. Santhal uprising

4. Kol uprising
ag

Question ID : 264330106577
Status : Answered
Chosen Option : 2

Q.18 Which of the following indicators presents a picture of occupational structure and
G

unemployment in India?
Ans 1. Literacy rate

2. Per capita income

3. Sex ratio

4. Work participation rate

Question ID : 264330109516
Status : Answered
Chosen Option : 2
Join @MathsByGaganPratap Telegram Channel for free PDFs
Q.19 Karmayogi Bharat owns and operates the digital assets and platforms for the online
training of _______________.
Ans 1. civil servants

2. defence personnel

3. social activists

4. academicians

Question ID : 26433079548
Status : Not Answered
Chosen Option : --

Q.20 Who is the author of the novel ‘A Suitable Boy’ which is one of the longest novel?

hs
Ans 1. Salman Rushdie

2. R K Narayan

3. Vikram Seth

at
4. Rohinton Mistry

Question ID : 264330141390
Status : Not Answered

M
Chosen Option : --

Q.21 Who is given the credit for starting the Gupta era?
Ans 1. Samudragupta ap
2. Chandragupta II

3. Kumaragupta

4. Chandragupta I
at
Question ID : 264330108996
Status : Answered
Chosen Option : 3
Pr

Q.22 Rohan Bopanna and Ramkumar Ramanathan got a direct entry into the doubles main
draw of the fourth edition of which of the following tennis tournaments that began in
January 2022?
Ans 1. Qatar ExxonMobil Open

2. Dubai Duty Free Tennis Championships


an

3. BNP Paribas Open

4. Tata Open Maharashtra

Question ID : 264330142690
Status : Not Answered
ag

Chosen Option : --

Q.23 What does the “Green Revolution” refers to in Indian Agriculture?


Ans 1. Increase in Agro based industries
G

2. Increase in forest cover

3. Increase in land under grassland

4. Spectacular increase in production of food grains

Question ID : 26433089203
Status : Answered
Chosen Option : 4
Join @MathsByGaganPratap Telegram Channel for free PDFs
Q.24 Who is a marginal worker as per the standard census definition in India?
Ans 1. A person who works for less than 100 days in a year

2. A person who works for less than 200 days in a year

3. A person who works for less than 183 days in a year 

4. A person who works for less than 150 days in a year 

Question ID : 264330109872
Status : Not Answered
Chosen Option : --

Q.25 Buland Darwaza at Fatehpur Sikri is a gateway to which of the following mosques?

hs
Ans 1. Moti Masjid

2. Jama Masjid

3. Adhai Din Ka Jhonpra

4. Jamali Kamali

at
Question ID : 26433081946
Status : Answered
Chosen Option : 1

M
Section : Quantitative Aptitude

Q.1 P borrows ₹5,00,000 from the bank which charges 18% simple interest per annum at
ap
the start of the year. He pays 11 equal monthly instalments of ₹40,000 and his bike at
the end of the year to pay the amount. What is the value of the bike given?
Ans 1. 1,50,000

2. 1,75,000

3. 1,25,000
at
4. 1,10,000

Question ID : 264330112104
Pr

Not Attempted and


Status :
Marked For Review
Chosen Option : --

Q.2 Find the fourth proportion to the numbers 20, 32 and 36.
Ans 1. 52
an

2. 48

3. 49.4

4. 57.6
ag

Question ID : 264330142533
Status : Answered
Chosen Option : 4
G
Join @MathsByGaganPratap Telegram Channel for free PDFs
Q.3

Ans
1.

2.

3.

4.

Question ID : 26433091917
Status : Answered

hs
Chosen Option : 4

Q.4 If 45 men can build a wall of 360 m length in 18 days, how many men will take 15 days
to build a similar type of wall of length 180 m?
Ans 1. 29 men

at
2. 27 men

3. 21 men

4. 25 men

M
Question ID : 26433065005
Status : Answered
Chosen Option : 2

Q.5
ap
Ans 1. 10.5 CM
at
2. 9 cm

3. 10 CM
Pr

4. 9.5 cm

Question ID : 264330142761
Status : Answered
Chosen Option : 1
an

Q.6 What is the remainder when 4999 is divided by 7?


Ans 1. 2

2. 4

3. 1
ag

4. 3

Question ID : 264330126163
Status : Answered
G

Chosen Option : 3
Join @MathsByGaganPratap Telegram Channel for free PDFs
Q.7 A merchant bought two qualities of rice at the rate of Rs.120/kg and Rs.220/kg. In 72 kg
of the second quality rice, how much rice (in kg) of the first quality should be mixed so
that the resulting mixture may be sold at the rate of Rs.280/kg to earn a profit of 40%?
Ans 1. 20 

2. 22

3. 15

4. 18

Question ID : 26433063197
Status : Answered
Chosen Option : 4

hs
Q.8 Sanjib's salary was initially reduced by 50% and then boosted by 50%. What
percentage of his salary does he lose?
Ans 1. 0%

2. 15%

at
3. 25%

4. 10%

M
Question ID : 264330142582
Status : Answered
Chosen Option : 3

Q.9 If the salary of Reeta is 25% more than the salary of Geeta, then the salary of Geeta is

Ans 1. 20%
ap
how much percentage less than the salary of Reeta?

2. 15%

3. 25%
at
4. 30%

Question ID : 26433075648
Status : Answered
Pr

Chosen Option : 1

Q.10 A shopkeeper gives a 25% discount on the marked price of an item. If the selling price
of the item is ₹150, then its marked price is:
Ans 1. ₹300
an

2. ₹500

3. ₹200

4. ₹100
ag

Question ID : 26433058968
Status : Answered
Chosen Option : 3

Q.11 A bus travels at 70 km/h. How much distance will it travel in 36 minutes?
G

Ans 1. 45 km

2. 40 km

3. 39 km

4. 42 km

Question ID : 264330142245
Status : Answered
Chosen Option : 4
Join @MathsByGaganPratap Telegram Channel for free PDFs
Q.12 A is the average of 10 given numbers. B is the average after 2 of the numbers were
replaced by 3 other different numbers. The average of the removed numbers is 48 and
the average of the newly included numbers is 56. If A + B = 438, then the value of A − B
is:
Ans 1. 21

2. 14

3. 12

4. 26

Question ID : 264330103692
Status : Answered
Chosen Option : 2

hs
Q.13 A shopkeeper is purchasing goods from a wholesaler. The wholesaler is selling 1265
units of goods to the shopkeeper after gaining the sale price of 165 units of goods.
What is the gain percentage of the wholesaler?
Ans
1.

at
2.

3.

M
4.

Question ID : 26433082503
ap Status : Answered
Chosen Option : 1

Q.14 The simple interest on a sum of ₹8,000 for 2 years is ₹720. What is rate of interest per
annum?
at
Ans 1. 5.5%

2. 5%

3. 6%
Pr

4. 4.5%

Question ID : 264330142276
Status : Answered
Chosen Option : 4
an

Q.15 In a division problem, the divisor is 12 times the quotient and 9 times the remainder. If
the remainder is 96, then find the dividend.
Ans 1. 62304

2. 62404
ag

3. 62205

4. 62208

Question ID : 264330142235
G

Status : Answered
Chosen Option : 1
Join @MathsByGaganPratap Telegram Channel for free PDFs
Q.16 While playing cards, a man loses 75% of his money in the first round, 75% of the
remaining in the second round, and 75% of the remaining in the third round. If he is
left with Rs.100, how much money does he have initially?
Ans 1. Rs.6,400

2. Rs.2,400

3. Rs.1,600

4. Rs.3,200

Question ID : 26433073107
Status : Answered
Chosen Option : 1

hs
Q.17 The mean proportional of 11 and 44 is:
Ans 1. 22

2. 27.5

3. 25

at
4. 33

Question ID : 264330103330

M
Status : Answered
Chosen Option : 1

Q.18 A store declares a discount scheme as “Buy 1 shampoo bottle and get 60% off on
another shampoo of same quality, size and of the same price.” What is the profit
ap
percentage of the store, if each shampoo bottle has the cost price 60% below the
marked price?
Ans 1. 60%

2. 64%

3. 75%
at
4. 54%

Question ID : 26433063212
Pr

Status : Answered
Chosen Option : 3

Q.19 A shopkeeper sells pens for ₹12 each, notebooks for ₹25 each and notebook covers
for ₹8 each. What is the average price per item (in ₹) if he sells 5 pens, 8 notebooks
and 16 notebook covers (rounded off to two decimal places)?
an

Ans 1. 13.38

2. 18.13

3. 13.83

4. 18.31
ag

Question ID : 26433085425
Status : Answered
Chosen Option : 1
G

Q.20 A and B can do a piece of work in 16 days and 12 days, respectively. Both work for 4
days and then A goes away. Find how long will B take to complete the remaining work.
Ans 1. 8 days

2. 7 days

3. 6 days

4. 5 days

Question ID : 26433075507
Status : Answered
Chosen Option : 4
Join @MathsByGaganPratap Telegram Channel for free PDFs
Q.21 By selling 20 items, a shopkeeper gains the selling price of 5 items. His gain
percentage is:
Ans
1.

2.

3.

4.

Question ID : 26433087357

hs
Status : Answered
Chosen Option : 3

Q.22 A bus is running at a speed of 60 km/h to cover a distance in 50 min. To reduce the
time of the journey to 20 min, at what speed (in km/h) should the bus run?

at
Ans 1. 120

2. 90

3. 140

M
4. 150

Question ID : 264330142525
Status : Answered
ap Chosen Option : 4

Q.23 In a parallelogram, one of the parallel sides is 16 cm and the other side is 12 cm. If the
perpendicular distance between the two parallel sides of dimension 16 cm is 24 cm,
then the perpendicular distance between its other two parallel sides is :
at
Ans 1. 16 cm

2. 12 cm

3. 32 cm
Pr

4. 24 cm

Question ID : 26433081736
Not Attempted and
Status :
Marked For Review
Chosen Option : --
an

Q.24 A certain sum, at a certain rate of simple interest, amounts to ₹2,225 in 4 years and to
₹2,606 in 8 years. Find the sum (in ₹).
Ans 1. 1,344
ag

2. 1,240

3. 1,143

4. 1,844

Question ID : 264330102356
G

Status : Answered
Chosen Option : 4
Join @MathsByGaganPratap Telegram Channel for free PDFs
Q.25 A TV manufacturer sells an item to a wholesale dealer at a profit of 8%. The wholesaler
sells the same to a retailer at a profit of 10%. The retailer in turn sells it to a customer
for ₹11,050 thereby earning a profit of 15%. The cost price of the manufacturer is:
(Consider integral part only)
Ans 1. ₹8,000

2. ₹8,088

3. ₹7,088

4. ₹8,888

Question ID : 26433081022
Status : Answered
Chosen Option : 2

hs
Section : English Language

Q.1 Select the most appropriate option to fill in the blank.

at
The streets are empty today as ______ a holiday.
Ans 1. was

2. its

3. it’s

M
4. is

Question ID : 26433060079
Status : Answered
ap Chosen Option : 3

Q.2 Select the option that expresses the given sentence in active voice.
Paddy is grown by the farmers of this village.
Ans 1. The farmers of this village grew paddy.
at
2. The farmers of this village have grown paddy.

3. The farmers of this village grow paddy.

4. The farmers of this village had grown paddy


Pr

Question ID : 264330135919
Status : Answered
Chosen Option : 3
an

Q.3 Select the most appropriate ANTONYM to replace the italicised word.

There is a/an abundance of fertile soil and magnificent grazing land.


Ans 1. dearth 

2. plenitude
ag

3. adequacy

4. plethora 

Question ID : 26433059647
G

Status : Answered
Chosen Option : 1
Join @MathsByGaganPratap Telegram Channel for free PDFs
Q.4 Choose the correct spelling of the underlined word in the given sentence.

The world is governed by several elemenyts.


Ans 1. elyments

2. elements 

3. aliments

4. eliments 

Question ID : 26433084612
Status : Answered
Chosen Option : 2

hs
Q.5 Identify the sentence that contains no spelling errors.
Ans 1. An Ethical Hacker identifies software vulanerabilities so that business owners may
adress them before a baleful hacker uncovers them.
2. An Ethickal Hacker identifies software vulnerabilities so that bussiness owners may
address them before a baleful hacker uncovers them. 

at
3. An Ethical Hacker identifies software vulnerabilities so that business owners may
address them before a baleful hacker uncovers them.
4. An Ethickal Hacker identifies software vulanerabilities so that business owners may
adress them before a baleful hacker uncovers them.

M
Question ID : 26433088542
Status : Answered
ap Chosen Option : 3

Q.6 Choose the option which means the same as the underlined segment.

His deep study of the case uncovered more problems .


Ans 1. opened a Pandora’s box
at
2. opened the fire on

3. opened conversation

4. opened a season on
Pr

Question ID : 264330120537
Status : Answered
Chosen Option : 1

Q.7 Select the option that can be used as a one-word substitute for the underlined group
an

of words in the given sentence.

He was severe in his manner, due to his rigidly puritanical outlook.


Ans 1. austere

2. faithful 
ag

3. precise

4. authentic

Question ID : 26433073849
G

Status : Answered
Chosen Option : 1
Join @MathsByGaganPratap Telegram Channel for free PDFs
Q.8 Identify the INCORRECTLY spelt word from the options given.
Ans 1. Bangel

2. Angle

3. Angel

4. Bagel

Question ID : 26433088463
Status : Answered
Chosen Option : 1

Q.9 Select the most appropriate ANTONYM of the word ‘Unworthy’ from the given
sentence.

hs
Unethical behaviour like saying mean or nasty things is intolerable to a noble person
Ans 1. Intolerable  

2. Noble

3. Nasty

at
4. Unethical  

Question ID : 264330143251

M
Status : Answered
Chosen Option : 2

Q.10 Select the most appropriate option to fill in the blank.


ap
Although the jury system is fundamentally ________, no one has ever come up with a
better one.
Ans 1. sorted

2. appealing

3. flawless
at
4. flawed

Question ID : 26433060986
Pr

Status : Answered
Chosen Option : 4

Q.11 Identify the idiom that best expresses the meaning of the underlined group of words.

He is in a very bad situation after being caught with drugs at school.


an

Ans 1. Last resort

2. In dire straits 

3. Square an account

4. At cross purposes
ag

Question ID : 264330121051
Status : Answered
Chosen Option : 2
G
Join @MathsByGaganPratap Telegram Channel for free PDFs
Q.12 Select the option that can be used as a one-word substitute for the given group of
words.

A floating container anchored to the sea bottom, so used for directing ships and
warning them of possible danger
Ans 1. Buoy

2. Deck

3. Cabin

4. Channel

Question ID : 26433060301
Status : Answered
Chosen Option : 1

hs
Q.13 Select the most appropriate meaning of the underlined idiom/phrase in the given
sentence.

Henry said he would help with the rent, but he left me in the lurch.

at
Ans 1. To comfort someone

2. To desert someone

3. To impersonate someone

M
4. To blame someone

Question ID : 26433088224
Status : Answered
ap Chosen Option : 2

Q.14 Select the idiom that gives the most appropriate meaning of the underlined phrase in
the following sentence.

My neighbour has decided to join the air force.


at
Ans 1. Made up his mind

2. Made a clean sweep

3. Made a fuss
Pr

4. Made his own way

Question ID : 26433086438
Status : Answered
Chosen Option : 1
an

Q.15 Select the correct direct narration of the given sentence.


Prem said that he had been listening to music for an hour.
Ans 1. Prem said, “I have been listening to music for an hour.”

2. Prem said, “He has listening to music for an hour.”


ag

3. Prem said, “He has be listened to music for an hour.”

4. Prem said, “I had listened to music for an hour.”

Question ID : 264330132787
G

Status : Marked For Review


Chosen Option : 1
Join @MathsByGaganPratap Telegram Channel for free PDFs
Comprehension:
In the following passage, some words have been deleted. Read the passage carefully and
select the most appropriate option to fill in each blank.
All drivers are affected by road rage, whether they are directly or indirectly (1)________.
Individual coping qualities (2)________ the behaviour of stressful drivers. Instead of being
enraged over circumstances beyond their control, drivers (3)_________ learn to be patient.
On the road, we must be (4)_________ to control our own emotions. Road raged drivers are
also more likely to (5)_________ more traffic offences.

SubQuestion No : 16
Q.16 Select the most appropriate option to fill in blank number 1.
Ans 1. involves

2. involving

3. involve

hs
4. involved

Question ID : 26433096335
Status : Answered

at
Chosen Option : 4

Comprehension:
In the following passage, some words have been deleted. Read the passage carefully and

M
select the most appropriate option to fill in each blank.
All drivers are affected by road rage, whether they are directly or indirectly (1)________.
Individual coping qualities (2)________ the behaviour of stressful drivers. Instead of being
enraged over circumstances beyond their control, drivers (3)_________ learn to be patient.
On the road, we must be (4)_________ to control our own emotions. Road raged drivers are
also more likely to (5)_________ more traffic offences.

SubQuestion No : 17
ap
Q.17 Select the most appropriate option to fill in blank number 2.
Ans 1. influence

2. increase
at
3. apply

4. multiply
Pr

Question ID : 26433096336
Status : Answered
Chosen Option : 1

Comprehension:
In the following passage, some words have been deleted. Read the passage carefully and
an

select the most appropriate option to fill in each blank.


All drivers are affected by road rage, whether they are directly or indirectly (1)________.
Individual coping qualities (2)________ the behaviour of stressful drivers. Instead of being
enraged over circumstances beyond their control, drivers (3)_________ learn to be patient.
On the road, we must be (4)_________ to control our own emotions. Road raged drivers are
also more likely to (5)_________ more traffic offences.
ag

SubQuestion No : 18
Q.18 Select the most appropriate option to fill in blank number 3.
Ans 1. must

2. ought
G

3. can

4. to

Question ID : 26433096337
Status : Answered
Chosen Option : 1
Join @MathsByGaganPratap Telegram Channel for free PDFs
Comprehension:
In the following passage, some words have been deleted. Read the passage carefully and
select the most appropriate option to fill in each blank.
All drivers are affected by road rage, whether they are directly or indirectly (1)________.
Individual coping qualities (2)________ the behaviour of stressful drivers. Instead of being
enraged over circumstances beyond their control, drivers (3)_________ learn to be patient.
On the road, we must be (4)_________ to control our own emotions. Road raged drivers are
also more likely to (5)_________ more traffic offences.

SubQuestion No : 19
Q.19 Select the most appropriate option to fill in blank number 4.
Ans 1. able

2. consistent

3. oblige

hs
4. brave

Question ID : 26433096338
Status : Answered

at
Chosen Option : 1

Comprehension:
In the following passage, some words have been deleted. Read the passage carefully and

M
select the most appropriate option to fill in each blank.
All drivers are affected by road rage, whether they are directly or indirectly (1)________.
Individual coping qualities (2)________ the behaviour of stressful drivers. Instead of being
enraged over circumstances beyond their control, drivers (3)_________ learn to be patient.
On the road, we must be (4)_________ to control our own emotions. Road raged drivers are
also more likely to (5)_________ more traffic offences.

SubQuestion No : 20
ap
Q.20 Select the most appropriate option to fill in blank number 5.
Ans 1. incur

2. proceed
at
3. reflect

4. drag
Pr

Question ID : 26433096339
Status : Answered
Chosen Option : 1
an
ag
G
Join @MathsByGaganPratap Telegram Channel for free PDFs
Comprehension:
Read the given passage and answer the question that follows.

A sparrow is a small bird which is found throughout the world. There are many different
species of sparrows. Sparrows are only about four to six inches in length. Many people
appreciate their beautiful song. Sparrows prefer to build their nests in low places, usually on
the ground, clumps of grass, low trees and low bushes. In cities they build their nests in
building nooks or holes. They rarely build their nests in high places. They build their nests out
of twigs, grasses and plant fibres. Their nests are usually small and well-built structures.
Female sparrows lay four to six eggs at a time. The eggs are white with reddish brown spots.
They hatch between eleven to fourteen days. Both the male and female parents care for the
young. Insects are fed to the young after hatching. The large feet of the sparrows are used for
scratching seeds. Adult sparrows mainly eat seeds. Sparrows can be found almost
everywhere, where there are humans. Many people throughout the world enjoy these
delightful birds.
The sparrows are some of the few birds that engage in dust bathing. Sparrows first scratch a

hs
hole in the ground with their feet, then lie in it and fling dirt or sand over their bodies with flicks
of their wings. They also bathe in water, or in dry or melting snow. Water bathing is similar to
dust bathing, with the sparrow standing in shallow water and flicking water over its back with
its wings, also ducking its head under the water. Both activities are social, with up to a
hundred birds participating at once, and is followed by preening and sometimes group
singing.

at
SubQuestion No : 21
Q.21 What is the structure of the passage?
Ans 1. Compare and Contrast

M
2. Cause and Effect

3. Process Writing

4. Problems and Solution 


ap Question ID : 26433072780
Status : Answered
Chosen Option : 3

Comprehension:
at
Read the given passage and answer the question that follows.

A sparrow is a small bird which is found throughout the world. There are many different
species of sparrows. Sparrows are only about four to six inches in length. Many people
appreciate their beautiful song. Sparrows prefer to build their nests in low places, usually on
Pr

the ground, clumps of grass, low trees and low bushes. In cities they build their nests in
building nooks or holes. They rarely build their nests in high places. They build their nests out
of twigs, grasses and plant fibres. Their nests are usually small and well-built structures.
Female sparrows lay four to six eggs at a time. The eggs are white with reddish brown spots.
They hatch between eleven to fourteen days. Both the male and female parents care for the
young. Insects are fed to the young after hatching. The large feet of the sparrows are used for
scratching seeds. Adult sparrows mainly eat seeds. Sparrows can be found almost
an

everywhere, where there are humans. Many people throughout the world enjoy these
delightful birds.
The sparrows are some of the few birds that engage in dust bathing. Sparrows first scratch a
hole in the ground with their feet, then lie in it and fling dirt or sand over their bodies with flicks
of their wings. They also bathe in water, or in dry or melting snow. Water bathing is similar to
dust bathing, with the sparrow standing in shallow water and flicking water over its back with
its wings, also ducking its head under the water. Both activities are social, with up to a
ag

hundred birds participating at once, and is followed by preening and sometimes group
singing.

SubQuestion No : 22
Q.22 What is the central theme of the passage?
Ans 1. About the food eaten by sparrows
G

2. About the lifestyle of sparrows

3. About the nest-building style of sparrows

4. About the bathing style of sparrows

Question ID : 26433072781
Status : Answered
Chosen Option : 2
Join @MathsByGaganPratap Telegram Channel for free PDFs
Comprehension:
Read the given passage and answer the question that follows.

A sparrow is a small bird which is found throughout the world. There are many different
species of sparrows. Sparrows are only about four to six inches in length. Many people
appreciate their beautiful song. Sparrows prefer to build their nests in low places, usually on
the ground, clumps of grass, low trees and low bushes. In cities they build their nests in
building nooks or holes. They rarely build their nests in high places. They build their nests out
of twigs, grasses and plant fibres. Their nests are usually small and well-built structures.
Female sparrows lay four to six eggs at a time. The eggs are white with reddish brown spots.
They hatch between eleven to fourteen days. Both the male and female parents care for the
young. Insects are fed to the young after hatching. The large feet of the sparrows are used for
scratching seeds. Adult sparrows mainly eat seeds. Sparrows can be found almost
everywhere, where there are humans. Many people throughout the world enjoy these
delightful birds.
The sparrows are some of the few birds that engage in dust bathing. Sparrows first scratch a

hs
hole in the ground with their feet, then lie in it and fling dirt or sand over their bodies with flicks
of their wings. They also bathe in water, or in dry or melting snow. Water bathing is similar to
dust bathing, with the sparrow standing in shallow water and flicking water over its back with
its wings, also ducking its head under the water. Both activities are social, with up to a
hundred birds participating at once, and is followed by preening and sometimes group
singing.

at
SubQuestion No : 23
Q.23 How can one describe sparrows?
Ans 1. Sparrows are loving

M
2. Sparrows are good parents

3. Sparrows are found everywhere

4. Sparrows are good 


ap Question ID : 26433072782
Status : Answered
Chosen Option : 3

Comprehension:
at
Read the given passage and answer the question that follows.

A sparrow is a small bird which is found throughout the world. There are many different
species of sparrows. Sparrows are only about four to six inches in length. Many people
appreciate their beautiful song. Sparrows prefer to build their nests in low places, usually on
Pr

the ground, clumps of grass, low trees and low bushes. In cities they build their nests in
building nooks or holes. They rarely build their nests in high places. They build their nests out
of twigs, grasses and plant fibres. Their nests are usually small and well-built structures.
Female sparrows lay four to six eggs at a time. The eggs are white with reddish brown spots.
They hatch between eleven to fourteen days. Both the male and female parents care for the
young. Insects are fed to the young after hatching. The large feet of the sparrows are used for
scratching seeds. Adult sparrows mainly eat seeds. Sparrows can be found almost
an

everywhere, where there are humans. Many people throughout the world enjoy these
delightful birds.
The sparrows are some of the few birds that engage in dust bathing. Sparrows first scratch a
hole in the ground with their feet, then lie in it and fling dirt or sand over their bodies with flicks
of their wings. They also bathe in water, or in dry or melting snow. Water bathing is similar to
dust bathing, with the sparrow standing in shallow water and flicking water over its back with
its wings, also ducking its head under the water. Both activities are social, with up to a
ag

hundred birds participating at once, and is followed by preening and sometimes group
singing.

SubQuestion No : 24
Q.24 What is the tone of the passage?
Ans 1. Biased 
G

2. Speculative

3. Descriptive 

4. Apologetic 

Question ID : 26433072783
Status : Answered
Chosen Option : 3
Join @MathsByGaganPratap Telegram Channel for free PDFs
Comprehension:
Read the given passage and answer the question that follows.

A sparrow is a small bird which is found throughout the world. There are many different
species of sparrows. Sparrows are only about four to six inches in length. Many people
appreciate their beautiful song. Sparrows prefer to build their nests in low places, usually on
the ground, clumps of grass, low trees and low bushes. In cities they build their nests in
building nooks or holes. They rarely build their nests in high places. They build their nests out
of twigs, grasses and plant fibres. Their nests are usually small and well-built structures.
Female sparrows lay four to six eggs at a time. The eggs are white with reddish brown spots.
They hatch between eleven to fourteen days. Both the male and female parents care for the
young. Insects are fed to the young after hatching. The large feet of the sparrows are used for
scratching seeds. Adult sparrows mainly eat seeds. Sparrows can be found almost
everywhere, where there are humans. Many people throughout the world enjoy these
delightful birds.
The sparrows are some of the few birds that engage in dust bathing. Sparrows first scratch a

hs
hole in the ground with their feet, then lie in it and fling dirt or sand over their bodies with flicks
of their wings. They also bathe in water, or in dry or melting snow. Water bathing is similar to
dust bathing, with the sparrow standing in shallow water and flicking water over its back with
its wings, also ducking its head under the water. Both activities are social, with up to a
hundred birds participating at once, and is followed by preening and sometimes group
singing.

at
SubQuestion No : 25
Q.25 Select the most appropriate ANTONYM of the given word.

Everywhere

M
Ans 1. Little

2. Somewhere

3. Rare

4. Nowhere
ap Question ID : 26433072779
Status : Answered
Chosen Option : 4
at
Pr
an
ag
G
Join @MathsByGaganPratap Telegram Channel for free PDFs

Phase-XI/2023/Selection Posts and Selection Posts/Ladakh/2023


Roll Number Gagan Pratap Maths
Candidate Name Gagan Pratap Maths
Venue Name Sarthak Educational Trust
Exam Date 27/06/2023
Exam Time 2:30 PM - 3:30 PM
Subject Selection Post Higher Secondary Level

Section : General Intelligence

hs
Q.1 Select the combination of letters that when sequentially placed in the blanks of the
given series will complete the series.
ABZ, BCY, CDX, DEW, _ _ _
Ans 1. EFV

2. EGH

at
3. EGV

4. EFG

M
Question ID : 26433078819
Status : Answered
Chosen Option : 1

Q.2
ap
at

Ans
Pr

1.
an

2.
ag

3.
G

4.

Question ID : 264330142413
Not Attempted and
Status :
Marked For Review
Chosen Option : --
Join @MathsByGaganPratap Telegram Channel for free PDFs
Q.3 In this question, three statements are given, followed by two conclusions numbered I
and II. Assuming the statements to be true, even if they seem to be at variance with
commonly known facts, decide which of the conclusions logically follows/follow from
the statements.
Statements:
I. All planets are asteroids.
II. All orbits are asteroids.
III. No star is an asteroid.
Conclusions:
I. No planet is a star.
II. No star is an orbit.
Ans 1. Only conclusion I follows

2. Neither conclusion I nor II follows

3. Only conclusion II follows

hs
4. Both conclusions I and II follow

Question ID : 264330104060
Status : Answered
Chosen Option : 4

at
Q.4 'B + D’ means ‘B is the father-in-law of D’.
‘B & D’ means ‘B is the brother of D’.
‘B # D’ means ‘B is the wife of D’.

M
‘B $ D’ means ‘B is the son of D’.
‘B @ D’ means ‘B is the mother of D’.
If O is the only child of his parents, then which of the given options is true in reference
to the following expression?
M#L&N@O$P+R
Ans 1. R is the wife of O.

2. L and P are siblings.


ap
3. N has only one son and one daughter

4. N is the mother of R.
at
Question ID : 264330142504
Not Attempted and
Status :
Marked For Review
Chosen Option : --
Pr

Q.5 Select the correct combination of mathematical signs to sequentially replace the *
signs and balance the given equation.

15 * 5 * 16 * 15 * 2 * 61
Ans 1. ÷, ×, +, ÷, = 
an

2. ÷, ×, +, −, =

3. −, ×, +, ÷, =

4. ÷, ×, −, ÷, = 
ag

Question ID : 26433097196
Status : Answered
Chosen Option : 2

Q.6 Select the option that is related to the fifth letter cluster in the same way as the second
G

letter cluster is related to the first letter cluster and the fourth letter cluster is related to
the third letter cluster.
THEME : VFEOC: : WORLD : YMRNB : : OTHER : ?
Ans 1. PSHHP

2. PRGHP

3. QRHGP

4. QSHGQ

Question ID : 26433068312
Status : Answered
Chosen Option : 3
Join @MathsByGaganPratap Telegram Channel for free PDFs
Q.7 If,
‘C % D’ means ‘C is the brother of D’,
‘C & D’ means ‘C is the mother of D’,
'C × D’ means ‘C is the husband of D’,
‘C # D’ means ‘C is the sister of D’,
‘C $ D’ means ‘C is the son of D’,
‘C @ D’ means ‘C is the father of D’,
then how is P related to U in the following expression?
P#Q%R&D$T@U
Ans 1. Grand daughter

2. Mother’s sister

3. Sister

4. Mother-in-law

hs
Question ID : 264330142506
Status : Answered
Chosen Option : 2

at
Q.8 In a certain code language, ‘PLANT’ is written as ‘QMAMS’ and ‘TREES’ is written as
‘USEDR’. How will ‘SHRUB’ be written in that language?
Ans 1. TIRTB

2. TIRAT

M
3. TIRTA

4. TIRTV
ap Question ID : 26433068637
Status : Answered
Chosen Option : 3

Q.9
at
Pr

Ans
an

1.
ag

2.
G

3.

4.

Question ID : 264330142440
Status : Answered
Chosen Option : 1
Join @MathsByGaganPratap Telegram Channel for free PDFs
Q.10 In a certain code language, ‘FICTION’ is coded as ‘C70’ and ‘DUSTILY’ is coded as
‘A106’. How will ‘ENDGAME’ be coded in that language?
Ans 1. B44 

2. B32

3. Y78

4. U63

Question ID : 264330118646
Not Attempted and
Status :
Marked For Review
Chosen Option : --

hs
Q.11 Select the option that is related to the fifth letter-cluster in the same way as the second
letter-cluster is related to the first letter-cluster and the fourth letter-cluster is related
to the third letter-cluster.
ADMIRE: BCOGUB :: THANKS : UGCLNP :: RESULT : ?
Ans 1. SDUROQ

at
2. SDURPQ

3. SDUSOQ

4. SDTROQ

M
Question ID : 26433057261
Status : Answered
ap Chosen Option : 1

Q.12 Which of the following numbers will replace the question mark (?) in the given series?
9, 45, 180, ?, 1080, 1080
Ans 1. 540

2. 520
at
3. 360

4. 480
Pr

Question ID : 264330142408
Not Attempted and
Status :
Marked For Review
Chosen Option : --

Q.13 Select the option that represents the correct order of the given words as they would
an

appear in an English dictionary.


1. Patent
2. Paternalism
3. Patchwork
4. Paternity
5. Paternal
ag

Ans 1. 4, 1, 5, 2, 3

2. 2, 4, 5, 1, 3

3. 3, 5, 1, 4, 2

4. 3, 1, 5, 2, 4
G

Question ID : 26433057886
Status : Answered
Chosen Option : 4
Join @MathsByGaganPratap Telegram Channel for free PDFs
Q.14 Select the option that is related to the fifth letter-cluster in the same way as the second
letter-cluster is related to the first letter-cluster and the fourth letter-cluster is related
to the third letter-cluster.
CART : UTDG :: DREW : XGUH :: ENDS : ?
Ans 1. TGRJ

2. TFQI

3. TGQI

4. TFQJ

Question ID : 26433056190
Status : Answered
Chosen Option : 2

hs
Q.15

at
Ans
1.

M
2.

3.

4.
ap
Question ID : 264330142419
Status : Answered
Chosen Option : 2
at

Q.16 ‘P % Q’ means ‘P is the brother of Q’


‘P + Q’ means ‘P is the daughter of Q’.
‘P = Q’ means ‘P is the son of Q’.
Pr

‘P # Q’ means ‘P is the wife of Q’.


If L % M + N # R = S, how is L related to S?
Ans 1. Son’s son

2. Son

3. Brother
an

4. Daughter’s son

Question ID : 264330122205
Status : Answered
Chosen Option : 1
ag

Q.17 Arrange the following words in a logical and meaningful order.


1.Tree
2.Sprout
3.Sapling
4.Seed
G

5.Seedling
Ans 1. 2, 4, 3, 5, 1

2. 2, 4, 5, 3, 1

3. 4, 5, 2, 3, 1

4. 4, 2, 5, 3, 1

Question ID : 26433091417
Status : Answered
Chosen Option : 3
Join @MathsByGaganPratap Telegram Channel for free PDFs
Q.18 Select the option that is related to the third word in the same way as the second word
is related to the first word. (The words must be considered as meaningful English
words and must NOT be related to each other based on the number of letters/number
of consonants/vowels in the word)
Time : Second :: Mass : ?
Ans 1. Measurement

2. Kilogram

3. Scale

4. Commodity

Question ID : 264330142463
Status : Answered
Chosen Option : 1

hs
Q.19 Select the combination of letters that when sequentially placed from left to right in the
blanks of the given series will complete the letter series.
PQ__T__RSTP_R_T_Q_ST
Ans 1. S R P Q Q S R P

at
2. S R P Q Q S P R

3. R S P Q Q S P R

M
4. R S P Q S Q R P

Question ID : 264330142488
Status : Answered
Chosen Option : 3
ap
Q.20 In a certain code language, 'COIN' is written as '615928' and 'MINT' is written as
'2692840'.How will ‘HERO' be written in that language?
Ans 1. 81018 
at
2. 8101830 

3. 1653615 

4. 16536 
Pr

Question ID : 264330142448
Status : Answered
Chosen Option : 3

Q.21 In a certain code language, ‘CACTUS’ is written as ‘ACTCSU’ and ‘DESERT’ is written
an

as ‘EDESTR’. How will ‘SAFARI’ be written in that language?


Ans 1. ASAFRI

2. ARAIFS

3. AARFIS
ag

4. ASAFIR

Question ID : 264330103791
Status : Answered
Chosen Option : 4
G
Join @MathsByGaganPratap Telegram Channel for free PDFs
Q.22 Which of the following numbers will replace the question mark (?) in the given series?
136, 204, 306, ?, 688.5
Ans 1. 459

2. 419

3. 446

4. 476

Question ID : 264330142409
Not Attempted and
Status :
Marked For Review
Chosen Option : --

hs
Q.23 Read the given statements and conclusions carefully. Assuming that the information
given in the statements is true, even if it appears to be at variance with commonly
known facts, decide which of the given conclusions logically follow(s) from the
statements.

Statements:

at
Some gems are stones.
Some gems are rocks.
All rocks are diamonds.

Conclusions:

M
I. Some diamonds are rocks.
II. Some stones are rocks.
III. Some gems are diamonds.
Ans 1. Only conclusion I follows

2. Only conclusions I and III follow

3. Only conclusions I and II follow


ap
4. Only conclusions II and III follow

Question ID : 264330142509
at
Status : Answered
Chosen Option : 2

Q.24 Select the correct option that indicates the arrangement of the given words in the
Pr

order in which they appear in an English dictionary.


1. Lenient
2. Lessons
3. Liberalization
4. Literal
5. License
Ans 1. 5,2,3,4,1
an

2. 2,1,3,5,4

3. 1,2,3,5,4

4. 4,1,3,2,5
ag

Question ID : 26433057963
Status : Answered
Chosen Option : 3

Q.25 In a certain code language, ‘INDIA’ is written as ‘KPFKC’ and ‘CHINA’ is written as
G

‘EJKPC’. How will ‘JAPAN’ be written in that language?


Ans 1. LCRQP

2. LCREP

3. LCRCO

4. LCRCP

Question ID : 26433068638
Status : Answered
Chosen Option : 4
Join @MathsByGaganPratap Telegram Channel for free PDFs
Section : General Awareness

Q.1 How many Directive Principles of the original list in the Indian Constitution were
amended by the 44th Amendment Act of 1978?
Ans 1. Three

2. One

3. Two

4. Four

Question ID : 264330113404
Status : Not Answered
Chosen Option : --

hs
Q.2 Jyoti Prasad Agarwala was a music composer popularly called ‘Rupkonwar’ in the
state of ___________.
Ans 1. Madhya Pradesh

2.  Rajasthan

at
3. Gujarat

4.  Assam

M
Question ID : 264330141603
Status : Not Answered
Chosen Option : --

Ans 1. To provide food grains to children


ap
Q.3 What is the aim of the Annapurna Yojana of the Government of India?

2. To provide food grains to women

3. To provide food to all


at
4. To provide food grains to senior citizens

Question ID : 264330110662
Status : Answered
Pr

Chosen Option : 2

Q.4 Which of the following architects designed the Gateway of India?


Ans 1. Herbert Baker

2. George Wittet
an

3. Henry Irwin

4. Lauri Baker

Question ID : 264330130885
ag

Status : Not Answered


Chosen Option : --

Q.5 Which of the following texts lays down minute details of the administrative and
military organization during the Mauryan empire?
G

Ans 1. Brihat Samhita

2. Nitisara

3. Shulba Sutra

4. Arthashastra

Question ID : 264330111367
Status : Answered
Chosen Option : 4
Join @MathsByGaganPratap Telegram Channel for free PDFs
Q.6 The Bahmani kingdom was founded by Alauddin Bahman Shah in ______.
Ans 1. 1345

2. 1346

3. 1347

4. 1336

Question ID : 26433081943
Status : Not Answered
Chosen Option : --

Q.7 Which of the following Articles of the Constitution of India provides that there shall be
a Legislative Assembly for the National Capital Territory of Delhi?

hs
Ans 1. Article 231AA

2. Article 239AA

3. Article 233AA

at
4. Article 237AA

Question ID : 264330113422
Status : Not Answered

M
Chosen Option : --

Q.8 Raja and Radha Reddy received the Sangeet Natak Akademi Award for their
contribution to ____________ dance.
Ans 1. Sattriya

2. Kathak
ap
3. Kathakali

4. Kuchipudi
at
Question ID : 264330110266
Status : Not Answered
Chosen Option : --
Pr

Q.9 Kumudini Lakhia is an exponent of which Indian classical dance form? 


Ans 1.  Kathak 

2.  Kuchipudi 

3.  Odissi 
an

4.  Sattriya 

Question ID : 264330141651
Status : Not Answered
ag

Chosen Option : --

Q.10 Who among the following returned his/her medal of Kaisar-i-Hind in 1920?
Ans 1. S Subramania Iyer

2. Sarojini Naidu
G

3. Rabindranath Tagore

4. Mahatma Gandhi

Question ID : 26433071416
Status : Answered
Chosen Option : 4
Join @MathsByGaganPratap Telegram Channel for free PDFs
Q.11 What is another name for Bardoli Chheerha?
Ans 1. Mango shower 

2. Nor Westers

3. Blossom shower

4. Loo  

Question ID : 264330141932
Status : Not Answered
Chosen Option : --

Q.12 ‘Moonwalk’, a memoir is written by which famous music personality?

hs
Ans 1. Michael Jackson

2. Stevie Wonder

3. Bob Dylan

4. Kurt Cobain

at
Question ID : 26433064621
Status : Answered
Chosen Option : 1

M
Q.13 Plants which are grown under shade are known as:
Ans 1. Monocots

2. Helophytes

3. psamophytes
ap
4. Sciophytes

Question ID : 26433066642
at
Status : Not Answered
Chosen Option : --

Q.14 Which of the following states won the Vijay Hazare Trophy 2021 in December at
Pr

Jaipur?
Ans 1. Haryana

2. Tamil Nadu

3. Himachal Pradesh
an

4. Punjab

Question ID : 26433090652
Status : Answered
Chosen Option : 2
ag

Q.15 The Taxation Laws (Amendment) Act, 2021, amends the Income Tax Act of _______.
Ans 1. 1961

2. 1974
G

3. 1995

4. 1988

Question ID : 26433090630
Status : Not Answered
Chosen Option : --
Join @MathsByGaganPratap Telegram Channel for free PDFs
Q.16 The average male literacy rate in India as per the 2011 census was _____.
Ans 1.

2.

3.

4.

Question ID : 264330109511
Status : Answered
Chosen Option : 4

Q.17 On which day of the Hindu calendar is the festival of Holi celebrated?

hs
Ans 1. Chaitra Poornima

2. Chaitra Saptami

3. Falgun Poornima 

4. Falgun Amawasya

at
Question ID : 264330141649
Status : Not Answered
Chosen Option : --

M
Q.18

ap
at

Ans 1. i - a, ii - b, iii - d, iv - c  


Pr

2. i - a, ii - b, iii - c, iv - d 

3. i - b, ii - a, iii - d, iv - c 

4. i - b, ii - a, iii - c, iv - d  
an

Question ID : 26433054031
Status : Not Answered
Chosen Option : --

Q.19 In December 2022, who among the following became the first woman President of the
ag

Indian Olympic Association?


Ans 1. Saina Nehwal

2. Kunjarani Devi

3. Mary Kom
G

4. PT Usha

Question ID : 264330132595
Status : Answered
Chosen Option : 3
Join @MathsByGaganPratap Telegram Channel for free PDFs
Q.20 In India, during a meeting of the Legislative Assembly or Council, if there is no
quorum, it shall be the duty of the _________ to adjourn the House or suspend the
meeting.
Ans 1. Chairman/Governor  

2. Speaker/Chairman 

3. Governor/Chief Minister 

4. Speaker/Chief Minister  

Question ID : 264330110133
Status : Answered
Chosen Option : 2

hs
Q.21 In which of the following years did the Indian team win the ODI cricket world cup?
Ans 1.  2015

2.  2007

3.  2011

at
4.  2019

Question ID : 264330103808

M
Status : Answered
Chosen Option : 4

Q.22 Select the natural port of India from the following.


Ans 1. Mumbai Port

2. Chennai Port
ap
3. Deendayal Port

4. Kolkata Port
at
Question ID : 26433082184
Status : Answered
Chosen Option : 4
Pr

Q.23 What is the ratio by mass of hydrogen and oxygen in water?


Ans 1. 1 : 8

2. 1 : 4

3. 2 : 5
an

4. 2 : 3

Question ID : 26433055756
Status : Not Answered
ag

Chosen Option : --

Q.24 Who among the following sportspersons has been awarded the Major Dhyan Chand
Khel Ratna Award 2022?
Ans 1. Shri Raj Singh
G

2. Shri Bimal Prafulla Ghosh

3. Shri Sharath Kamal Achanta

4. Shri Dinesh Jawahar Lad

Question ID : 264330130855
Status : Not Answered
Chosen Option : --
Join @MathsByGaganPratap Telegram Channel for free PDFs
Q.25 Which of the following characteristics is INCORRECT about the tropical evergreen
forests of India?
Ans 1. These forests are well-stratified.  

2. In these forests, trees reach great heights of up to 60 m or above. 

3. They are found in warm and humid areas.

4. These forests receive annual precipitation of below 150 cm.

Question ID : 264330142266
Status : Answered
Chosen Option : 4

hs
Section : Quantitative Aptitude

Q.1

at
M
ap
Ans 1. 155 

2. 45
at
3. 150

4. 40
Pr

Question ID : 26433075524
Status : Not Answered
Chosen Option : --

Q.2
an
ag

Ans 1. 8

2. 12
G

3. 16

4. 18

Question ID : 264330141224
Status : Answered
Chosen Option : 4
Join @MathsByGaganPratap Telegram Channel for free PDFs
Q.3 Abhay can do a work in 22 days. Nidhi is 28% more efficient than Abhay. The number
of days Nidhi will take to do the same piece of work is:
Ans
1.

2.

3.

4.

hs
Question ID : 26433081478
Status : Answered
Chosen Option : 1

Q.4 A varies directly with B. When A = 52, B = 91. Find the value of A when B = 126.

at
Ans 1. 76

2. 84

3. 72

M
4. 70

Question ID : 264330103275
Not Attempted and
ap Status :
Marked For Review
Chosen Option : --

Q.5 What is the selling price of a calculator if a shopkeeper allows two successive
discounts of 7% each on its marked price of Rs.500?
at
Ans 1. Rs.430.25

2. Rs.440.85

3. Rs.435.00
Pr

4. Rs.432.45

Question ID : 26433073085
Status : Answered
Chosen Option : 4
an

Q.6 The average score in a test for 50 students is 44. Out of the total students, 40% of the
students are girls and the rest are boys. The average score of boys is 20% less than
that of girls. What is the average score of girls?
Ans 1. 50
ag

2. 54

3. 48

4. 52

Question ID : 264330104156
G

Status : Answered
Chosen Option : 1
Join @MathsByGaganPratap Telegram Channel for free PDFs
Q.7 Mohan divides 18935 by a certain number. If the quotient and the remainder he gets
are 102 and 65, respectively, then the divisor is:
Ans 1. 155

2. 165

3. 175

4. 185

Question ID : 26433090295
Status : Not Answered
Chosen Option : --

Q.8 If 4 tan A = 3, then cos2A – sin2A equals:

hs
Ans
1.

at
2.

M
3.

4. ap Question ID : 264330141580
Status : Answered
Chosen Option : 2
at
Q.9 Mohan purchases a table at an MRP of ₹800, at a discount of 10%. If he sells it at ₹900,
then the profit percentage is:
Ans 1. 20% profit
Pr

2. 25% profit

3. 15% profit

4. 30% profit

Question ID : 26433099598
an

Status : Answered
Chosen Option : 2

Q.10 What would be the simple interest on principle amount of ₹2 lakh for 3 years at a rate
of 8% per annum?
ag

Ans
1.

2.

3.
G

4.

Question ID : 26433074291
Status : Answered
Chosen Option : 2
Join @MathsByGaganPratap Telegram Channel for free PDFs
Q.11 Simplify 52 × 52 - 48 × 48.
Ans 1. 386

2. 400

3. 424

4. 326

Question ID : 264330141411
Status : Answered
Chosen Option : 2

Q.12 A sum of Rs.84,000 is deposited in a bank on simple interest at the rate of 15% for the
period of 3 years. How much Simple Interest (in Rs.) will be received?

hs
Ans 1. Rs.37,800

2. Rs.38,700

3. Rs.37,200

at
4. Rs.39,800

Question ID : 26433075070
Status : Answered

M
Chosen Option : 1

Q.13 If 36 litres of 20% milk solution is mixed with 48 litres of 10% milk solution, then what
will be the percentage concentration of milk in the final solution?
Ans 1. 14.28%

2. 15.48%
ap
3. 12.38%

4. 13.36%
at
Question ID : 264330126558
Status : Answered
Chosen Option : 1
Pr

Q.14 A silver puja thali set marked at ₹10,000 is sold with two successive discounts of 10%
and 5%. An additional 5% discount is offered if the payment is made in cash mode.
Find the selling price of the article on cash payment.
Ans 1. ₹8,212.50

2. ₹8,122.50
an

3. ₹8,100.50

4. ₹8,200.50

Question ID : 264330103407
ag

Status : Answered
Chosen Option : 2

Q.15 The distance between two places is 1200 km. To cover this distance, a person X takes
5 hours lesser than another person Y, whose average speed is 40 km/h lesser than that
G

of X. The time taken by Y to complete the travel is:


Ans 1. 16 hours

2. 14 hours

3. 18 hours

4. 15 hours

Question ID : 26433061315
Status : Not Answered
Chosen Option : --
Join @MathsByGaganPratap Telegram Channel for free PDFs
Q.16 A sells an article to B at a gain of 16%, B sells it to C at a loss of 15%, and C sells it to
D at a gain of 20%. If the difference between the profits earned by C and A is ₹248, then
the loss (in ₹) incurred by B is:
Ans 1. 1250

2. 1075

3. 1160

4. 1025

Question ID : 264330104178
Not Attempted and
Status :
Marked For Review
Chosen Option : --

hs
Q.17 Melting a hemisphere of radius 7 cm, four small identical spheres are made. What is
the radius of these spheres?
Ans 1. 3 cm

2. 3.5 cm

at
3. 4 cm

4. 4.5 cm

M
Question ID : 26433081740
Status : Not Answered
Chosen Option : --

Q.18
ap
at
Pr

Ans 1. 2
an

2. 0

3. 1

4. 3
ag

Question ID : 26433076126
Status : Not Answered
Chosen Option : --

Q.19 Which of the following is NOT divisible by 4?


G

Ans 1. 67547172 

2. 56783294

3. 73568916

4. 80936292

Question ID : 264330122063
Status : Answered
Chosen Option : 2
Join @MathsByGaganPratap Telegram Channel for free PDFs
Q.20 Let A,B,C be the mid-points of sides XY,YZ and XZ, respectively of ΔXYZ. If the area of
ΔXYZ is 8464 "cm2", then find the area (in "cm2" ) of ΔABC.
Ans 1. 2116

2. 1812

3. 1516

4. 3112

Question ID : 264330126537
Not Attempted and
Status :
Marked For Review
Chosen Option : --

hs
Q.21 A goods train, travelling at constant speed, crossed two persons walking in the same
direction (as that of the train) in 11.6 seconds and 11.8 seconds, respectively. The first
person was walking at 5.85 km/h, while the second was walking at 6.3 km/h. What was
the speed of the train (in km/h)?
Ans 1. 32.5

at
2. 32.6

3. 32.4

4. 32.2

M
Question ID : 264330103282
Status : Not Answered
ap Chosen Option : --

Q.22

Ans
1.
at
2.

3.
Pr

4.

Question ID : 26433080300
an

Not Attempted and


Status :
Marked For Review
Chosen Option : --

Q.23 A, B and C can do a piece of work in 20 days, 30 days and 60 days, respectively,
working alone. How soon can the work be done if A is assisted by B and C each on
ag

every alternate day?


Ans
1.
G

2.

3.

4.

Question ID : 26433076111
Not Attempted and
Status :
Marked For Review
Chosen Option : --
Join @MathsByGaganPratap Telegram Channel for free PDFs
Q.24 In ∆ABC, P and Q are the middle points of the sides AB and AC, respectively. R is a
point on the segment PQ such that PR : RQ = 1 : 4. If PR = 5 cm, then BC = ?
Ans 1. 46 cm

2. 50 cm

3. 48 cm

4. 44 cm

Question ID : 264330141578
Status : Not Answered
Chosen Option : --

Q.25 The ratio of Rahul’s age and his son Aman’s age is 9 : 1 and the product of their ages

hs
is 144. What will be the sum of their ages after four years?
Ans 1. 40 

2. 44 

3. 48 

at
4. 46 

Question ID : 264330124002

M
Not Attempted and
Status :
Marked For Review
Chosen Option : --

Section : English Language


ap
Q.1 The following sentence has been split into four segments. Identify the segment that
contains a grammatical error.
We must / take care / of one other / in times of crisis.
Ans 1. take care
at
2. in times of crisis

3. We must

4. of one other
Pr

Question ID : 264330142378
Not Attempted and
Status :
Marked For Review
Chosen Option : --
an

Q.2 Select the option that expresses the given sentence in direct speech.
Bibiya asked the doctor if she could get the discharge in the morning.
Ans 1. Bibiya asked the doctor, “Can I get the discharge in the morning?”

2. Bibiya asked the doctor, “How can I get the discharge in the morning?”
ag

3. Bibiya asked the doctor, “Could I have got the discharge in the morning?”

4. Bibiya asked the doctor, “If I can get the discharge in the morning?”

Question ID : 264330136455
Status : Answered
G

Chosen Option : 1
Join @MathsByGaganPratap Telegram Channel for free PDFs
Q.3 Select the option that will improve the underlined part of the given sentence.
It is wrong to make accusations without will have any proof.
Ans 1. without has any proof

2. without being have any proof

3. without to be have any proof

4. without having any proof

Question ID : 264330142374
Status : Answered
Chosen Option : 4

Q.4 Select the option that can be used as a one-word substitute for the given group of

hs
words.

Social organisation where males are heads of families


Ans 1. Matriarchy

2. Patriarchy

at
3. Polygamy

4. Hierarchy

M
Question ID : 26433073907
Status : Answered
Chosen Option : 2
ap
Q.5 Select the option that expresses the given sentence in passive voice.

He ordered the police to pursue the robber and his gang.


Ans 1. The police were ordered to pursue the robber and his gang. 

2. Order the police to pursue the robber gang. 


at
3. Pursue the robber and gang were the orders by police. 

4. The robber and gang were pursued. 


Pr

Question ID : 26433093197
Status : Answered
Chosen Option : 1

Q.6 Select the most appropriate synonym of the given word.


Instant
an

Ans 1. Immediate

2. Precise

3. Anticipated

4. Gradual
ag

Question ID : 26433063114
Status : Answered
Chosen Option : 1
G
Join @MathsByGaganPratap Telegram Channel for free PDFs
Q.7 Select the most appropriate ANTONYM of the underlined word.
The old man was not only infirm but also blind and was unable to move around by
himself.
Ans 1. Robust

2. Diffident 

3. Anaemic

4. Hospitable 

Question ID : 26433073370
Status : Answered
Chosen Option : 1

hs
Q.8 Select the most appropriate meaning of the given idiom.

I don’t buy it
Ans 1. Purchasing should be controlled 

2. Only good decisions can be agreed on

at
3. I am not convinced 

4. I do not like to buy 

M
Question ID : 26433089005
Status : Answered
Chosen Option : 3
ap
Q.9 Select the most appropriate option to fill in the blank.
Plastic pollution is _________ the oceans’ ecosystem, but humanity doesn’t take
serious action against it.
Ans 1. deciding

2. aiming
at
3. making

4. affecting
Pr

Question ID : 26433097028
Status : Answered
Chosen Option : 4

Q.10 Select the most appropriate antonym of the given word.


an

Keen
Ans 1. Insensitive

2. Accurate

3. Perceptive
ag

4. Subtle

Question ID : 26433060978
Status : Answered
Chosen Option : 1
G
Join @MathsByGaganPratap Telegram Channel for free PDFs
Q.11 Parts of the following sentence have been underlined and given as options. Select the
option that contains a spelling error.

How simple for the English businesman to word a cable that would be intelligible to
Italian, Turkish and Chinese firms? If statesmen could discuss directly and accurately
the problems.
Ans 1. statesmen

2. intelligible

3. businesman

4. firms

Question ID : 26433088430
Status : Answered

hs
Chosen Option : 3

Q.12 Select the most appropriate ANTONYM of the given word from the following sentence.
Artificial
My uncle, who is a native of Australia, is a logical thinker and says that natural and

at
unnatural stones both look beautiful in gold jewellery.
Ans 1. Unnatural

2. Native

M
3. Natural

4. Logical

ap Question ID : 26433072812
Status : Answered
Chosen Option : 3

Q.13 Select the most appropriate option to fill in the blank.


Every cloud has a ______ lining.
at
Ans 1. diffused

2. silver

3. blue
Pr

4. grey

Question ID : 26433098873
Status : Answered
Chosen Option : 2
an

Q.14 Parts of the following sentence have been underlined and given as options. Select the
option that contains a spelling error.

It remains for us to apply the tools of selection, rejection or compresion in order to


understand.
ag

Ans 1. compresion

2. rejection

3. selection

4. understand
G

Question ID : 26433088427
Status : Answered
Chosen Option : 1
Join @MathsByGaganPratap Telegram Channel for free PDFs
Q.15 Select the option that expresses the given sentence in direct speech.
Liam asked if we could proceed with the award ceremony.
Ans 1. Liam said, “Can we proceed with the award ceremony?” 

2. Liam said, “Will we proceed with the award ceremony?” 

3. Liam said, “Should we proceed with the award ceremony?” 

4. Liam said, “Could we please proceed with the award ceremony?” 

Question ID : 264330133423
Status : Answered
Chosen Option : 1

Comprehension:

hs
In the following passage, some words have been deleted. Read the passage carefully and
select the most appropriate option to fill in each blank.
Bottled water demonstrates how much humans (1)___________ nature, reflecting the
tremendous value they place on it. On the other hand, bottled water indicates
(2)______________ for nature's complexities, displaying the irony that may lead to the mutual
annihilation of humanity and the (3)___________. The finest example is the (4)__________

at
of bottled water, which is the commercialisation of natural resources. Holding bottled water, it
is clear to see a gorgeous image of a lake and a small (5)________ of blue and green plastic
around its neck.

SubQuestion No : 16

M
Q.16 Select the most appropriate option to fill in blank number 1.
Ans 1. commodify

2. satisfy

3. glorify

4. beautify
ap
Question ID : 26433099006
Status : Answered
at
Chosen Option : 3

Comprehension:
In the following passage, some words have been deleted. Read the passage carefully and
Pr

select the most appropriate option to fill in each blank.


Bottled water demonstrates how much humans (1)___________ nature, reflecting the
tremendous value they place on it. On the other hand, bottled water indicates
(2)______________ for nature's complexities, displaying the irony that may lead to the mutual
annihilation of humanity and the (3)___________. The finest example is the (4)__________
of bottled water, which is the commercialisation of natural resources. Holding bottled water, it
is clear to see a gorgeous image of a lake and a small (5)________ of blue and green plastic
an

around its neck.

SubQuestion No : 17
Q.17 Select the most appropriate option to fill in blank number 2.
Ans 1. admiration
ag

2. contempt

3. acceptance

4. affectation

Question ID : 26433099007
G

Status : Answered
Chosen Option : 1
Join @MathsByGaganPratap Telegram Channel for free PDFs
Comprehension:
In the following passage, some words have been deleted. Read the passage carefully and
select the most appropriate option to fill in each blank.
Bottled water demonstrates how much humans (1)___________ nature, reflecting the
tremendous value they place on it. On the other hand, bottled water indicates
(2)______________ for nature's complexities, displaying the irony that may lead to the mutual
annihilation of humanity and the (3)___________. The finest example is the (4)__________
of bottled water, which is the commercialisation of natural resources. Holding bottled water, it
is clear to see a gorgeous image of a lake and a small (5)________ of blue and green plastic
around its neck.

SubQuestion No : 18
Q.18 Select the most appropriate option to fill in blank number 3.
Ans 1. collections

2. humans

hs
3. trash

4. environment

Question ID : 26433099008

at
Status : Answered
Chosen Option : 4

M
Comprehension:
In the following passage, some words have been deleted. Read the passage carefully and
select the most appropriate option to fill in each blank.
Bottled water demonstrates how much humans (1)___________ nature, reflecting the
tremendous value they place on it. On the other hand, bottled water indicates
(2)______________ for nature's complexities, displaying the irony that may lead to the mutual
ap
annihilation of humanity and the (3)___________. The finest example is the (4)__________
of bottled water, which is the commercialisation of natural resources. Holding bottled water, it
is clear to see a gorgeous image of a lake and a small (5)________ of blue and green plastic
around its neck.

SubQuestion No : 19
at
Q.19 Select the most appropriate option to fill in the blank number 4.
Ans 1. corruption

2. recycling

3. recollecting
Pr

4. sale

Question ID : 26433099009
Status : Answered
Chosen Option : 2
an

Comprehension:
In the following passage, some words have been deleted. Read the passage carefully and
select the most appropriate option to fill in each blank.
Bottled water demonstrates how much humans (1)___________ nature, reflecting the
ag

tremendous value they place on it. On the other hand, bottled water indicates
(2)______________ for nature's complexities, displaying the irony that may lead to the mutual
annihilation of humanity and the (3)___________. The finest example is the (4)__________
of bottled water, which is the commercialisation of natural resources. Holding bottled water, it
is clear to see a gorgeous image of a lake and a small (5)________ of blue and green plastic
around its neck.
G

SubQuestion No : 20
Q.20 Select the most appropriate option to fill in blank number 5.
Ans 1. beam

2. bunch

3. group

4. piece

Question ID : 26433099010
Status : Answered
Chosen Option : 4
Join @MathsByGaganPratap Telegram Channel for free PDFs
Comprehension:
Read the given passage and answer the questions that follow.
Virtual learning refers to an environment where students study a digital-based curriculum
taught by instructors that lecture online via video or audio. This instruction can take place
either in a self-paced (asynchronous) environment or in a real-time (synchronous)
environment. There are many reasons why people choose virtual learning over instruction in a
traditional classroom. For example, the flexibility of virtual learning makes it an appropriate
choice for full-time workers who would like to continue their education or cannot attend in-
person classes.
The virtual learning environment is also a good choice for people who enjoy self-learning.
Some virtual learning options have classes to attend at scheduled times, while others allow
you to learn at your own pace. With this self-paced option, you have a lot of autonomy with
the intensity and overall cadence of your learning while still enjoying the benefits of guided
lessons.
Virtual learning gives students access to coursework from anywhere, at any time, making
learning super convenient. Asynchronous classes also give an immense amount of flexibility

hs
for lectures and studying. Even with the flexibility of the course, online courses are associated
with higher retention rates and graduation rates. A study conducted by Arizona State
University found that students taking virtual classes benefit from increased access and cost
savings of as much as 50 percent.

SubQuestion No : 21

at
Q.21 What is the most appropriate ANTONYM of the word ‘autonomy’ from the passage?
Ans 1. Dependence 

2. Preference 

3. Liberty 

M
4. Availability

Question ID : 264330121017
Status : Answered
ap Chosen Option : 3

Comprehension:
Read the given passage and answer the questions that follow.
Virtual learning refers to an environment where students study a digital-based curriculum
at
taught by instructors that lecture online via video or audio. This instruction can take place
either in a self-paced (asynchronous) environment or in a real-time (synchronous)
environment. There are many reasons why people choose virtual learning over instruction in a
traditional classroom. For example, the flexibility of virtual learning makes it an appropriate
choice for full-time workers who would like to continue their education or cannot attend in-
Pr

person classes.
The virtual learning environment is also a good choice for people who enjoy self-learning.
Some virtual learning options have classes to attend at scheduled times, while others allow
you to learn at your own pace. With this self-paced option, you have a lot of autonomy with
the intensity and overall cadence of your learning while still enjoying the benefits of guided
lessons.
Virtual learning gives students access to coursework from anywhere, at any time, making
an

learning super convenient. Asynchronous classes also give an immense amount of flexibility
for lectures and studying. Even with the flexibility of the course, online courses are associated
with higher retention rates and graduation rates. A study conducted by Arizona State
University found that students taking virtual classes benefit from increased access and cost
savings of as much as 50 percent.

SubQuestion No : 22
ag

Q.22 What is the most appropriate ANTONYM of the word ‘retention’ from the passage?
Ans 1. Perception 

2. Preventive 

3. Retaining 
G

4. Relinquishment 

Question ID : 264330121019
Status : Answered
Chosen Option : 2
Join @MathsByGaganPratap Telegram Channel for free PDFs
Comprehension:
Read the given passage and answer the questions that follow.
Virtual learning refers to an environment where students study a digital-based curriculum
taught by instructors that lecture online via video or audio. This instruction can take place
either in a self-paced (asynchronous) environment or in a real-time (synchronous)
environment. There are many reasons why people choose virtual learning over instruction in a
traditional classroom. For example, the flexibility of virtual learning makes it an appropriate
choice for full-time workers who would like to continue their education or cannot attend in-
person classes.
The virtual learning environment is also a good choice for people who enjoy self-learning.
Some virtual learning options have classes to attend at scheduled times, while others allow
you to learn at your own pace. With this self-paced option, you have a lot of autonomy with
the intensity and overall cadence of your learning while still enjoying the benefits of guided
lessons.
Virtual learning gives students access to coursework from anywhere, at any time, making
learning super convenient. Asynchronous classes also give an immense amount of flexibility

hs
for lectures and studying. Even with the flexibility of the course, online courses are associated
with higher retention rates and graduation rates. A study conducted by Arizona State
University found that students taking virtual classes benefit from increased access and cost
savings of as much as 50 percent.

SubQuestion No : 23

at
Q.23 Which of the following options represent the structure of the passage?
Ans 1. Compare and Contrast 

2. Chronological

3. Descriptive

M
4. Spatial 

Question ID : 264330121018
Status : Answered
ap Chosen Option : 2

Comprehension:
Read the given passage and answer the questions that follow.
Virtual learning refers to an environment where students study a digital-based curriculum
at
taught by instructors that lecture online via video or audio. This instruction can take place
either in a self-paced (asynchronous) environment or in a real-time (synchronous)
environment. There are many reasons why people choose virtual learning over instruction in a
traditional classroom. For example, the flexibility of virtual learning makes it an appropriate
choice for full-time workers who would like to continue their education or cannot attend in-
Pr

person classes.
The virtual learning environment is also a good choice for people who enjoy self-learning.
Some virtual learning options have classes to attend at scheduled times, while others allow
you to learn at your own pace. With this self-paced option, you have a lot of autonomy with
the intensity and overall cadence of your learning while still enjoying the benefits of guided
lessons.
Virtual learning gives students access to coursework from anywhere, at any time, making
an

learning super convenient. Asynchronous classes also give an immense amount of flexibility
for lectures and studying. Even with the flexibility of the course, online courses are associated
with higher retention rates and graduation rates. A study conducted by Arizona State
University found that students taking virtual classes benefit from increased access and cost
savings of as much as 50 percent.

SubQuestion No : 24
ag

Q.24 Which of the following most accurately states the central idea of the passage?
Ans 1. Self-paced learning 

2. Flexible learning environment 

3. Guided learning environment 


G

4. Virtual learning and its advantages 

Question ID : 264330121015
Status : Answered
Chosen Option : 4
Join @MathsByGaganPratap Telegram Channel for free PDFs
Comprehension:
Read the given passage and answer the questions that follow.
Virtual learning refers to an environment where students study a digital-based curriculum
taught by instructors that lecture online via video or audio. This instruction can take place
either in a self-paced (asynchronous) environment or in a real-time (synchronous)
environment. There are many reasons why people choose virtual learning over instruction in a
traditional classroom. For example, the flexibility of virtual learning makes it an appropriate
choice for full-time workers who would like to continue their education or cannot attend in-
person classes.
The virtual learning environment is also a good choice for people who enjoy self-learning.
Some virtual learning options have classes to attend at scheduled times, while others allow
you to learn at your own pace. With this self-paced option, you have a lot of autonomy with
the intensity and overall cadence of your learning while still enjoying the benefits of guided
lessons.
Virtual learning gives students access to coursework from anywhere, at any time, making
learning super convenient. Asynchronous classes also give an immense amount of flexibility

hs
for lectures and studying. Even with the flexibility of the course, online courses are associated
with higher retention rates and graduation rates. A study conducted by Arizona State
University found that students taking virtual classes benefit from increased access and cost
savings of as much as 50 percent.

SubQuestion No : 25

at
Q.25 Which of the following is the best description of the tone of the passage?
Ans 1. Biased 

2. Technical 

3. Cynical 

M
4. Dogmatic 

Question ID : 264330121016
Status : Answered
ap Chosen Option : 4
at
Pr
an
ag
G
Join @MathsByGaganPratap Telegram Channel for free PDFs

Phase-XI/2023/Selection Posts and Selection Posts/Ladakh/2023


Roll Number Gagan Pratap Maths
Candidate Name Gagan Pratap Maths
Venue Name iON Digital Zone iDZ 2 Sector 62
Exam Date 27/06/2023
Exam Time 5:15 PM - 6:15 PM
Subject Selection Post Graduate Level

Section : General Intelligence

hs
Q.1 Select the option that represents the letters that, when placed from left to right in the
blanks below, will complete the letter-series.
N_M_N_ _MANNA_A_
Ans 1. AANAMN

2.  ANNMMA

at
3.  ANAMAA

4.  ANNANM

M
Question ID : 26433080454
Status : Answered
Chosen Option : 1
ap
Q.2 Select the option that is related to the fifth term in the same way as the second term is
related to the first term and the fourth term is related to the third term.
YEN10 : CWN50 :: USD8 : GIX32 :: GDP12 : ?
Ans 1. UWL72

2. UXL72
at
3. UXJ72

4. TXL72
Pr

Question ID : 26433067398
Status : Not Answered
Chosen Option : --

Q.3 In a certain code language, 'DEFECT' is written as '2747118', 'DEVICE' is written as


'27201117', how will 'DOMAIN' be written in that language?
an

Ans 1. 2171131112

2. 217113119

3. 417111912

4. 217111912
ag

Question ID : 264330143379
Status : Not Answered
Chosen Option : --
G
Join @MathsByGaganPratap Telegram Channel for free PDFs
Q.4 Select the correct mirror image of the given combination when the mirror is
placed at MN as shown.

Ans
1.

2.

hs
3.

4.

at
Question ID : 264330143447
Status : Answered

M
Chosen Option : 4

Q.5 In this question, three statements are given, followed by two conclusions numbered I
and II. Assuming the statements to be true, even if they seem to be at variance with
commonly known facts, decide which of the conclusions logically follows/follow from
the statements.

Statements:
ap
Some pears are mangoes.
All mangoes are grapes.
All pears are fruits.
at
Conclusions:
I. Some pears are grapes.
II. Some mangoes are fruits.
Ans 1. Only conclusion I follows.
Pr

2. Both conclusions I and II follow.

3. Only conclusion II follows.

4. Neither conclusion I nor II follows.

Question ID : 264330143348
an

Status : Answered
Chosen Option : 2

Q.6 Select the option that is related to the third word in the same way as the second word
is related to the first word. (The words must be considered as meaningful English
ag

words and must not be related to each other based on the number of letters/number of
consonants/vowels in the word)
Waive : Impose :: Spurious : ?
Ans 1. Counterfeit

2. Abjure
G

3. Original

4. Sporadic

Question ID : 264330143383
Status : Answered
Chosen Option : 3
Join @MathsByGaganPratap Telegram Channel for free PDFs
Q.7

Ans

1.

hs
at
2.

M
3.
ap
at
Pr

4.
an

Question ID : 26433078908
Status : Answered
Chosen Option : 2

Q.8 In a certain code language, 'CAST’ is written as '311920', and 'FEED' is written as
ag

'6554'.How will 'GLAD' be written in that language?


Ans 1. 71218 

2. 71514  

3. 71314  
G

4. 71214  

Question ID : 26433069563
Status : Answered
Chosen Option : 4
Join @MathsByGaganPratap Telegram Channel for free PDFs
Q.9 Select the option that is related to the third letter-cluster in the same way as the
second letter-cluster is related to the first letter-cluster.
NOTICE :ONUHDD :: DULCET : ?
Ans 1. ETMBFS

2. CVKDDU

3. QSYPRT

4. WFOYUH

Question ID : 26433068805
Status : Answered
Chosen Option : 1

hs
Q.10 Select the option that is related to the third word in the same way as the second word
is related to the first word.
(The words must be considered as meaningful English words and must not be related
to each other based on the number of letters/number of consonants/vowels in the
word)
Amateur : Beginner : : Dangerous : ?

at
Ans 1. Precarious

2. Enduring

3. Peculiar

M
4. Tolerant

Question ID : 26433068412
Status : Answered
ap Chosen Option : 1

Q.11 Select the correct option that indicates the arrangement of the following words in a
logical and meaningful order.
1. Finger
2. Elbow
at
3. Wrist
4. Nail
5. Palm
Ans 1. 4, 5, 1, 3, 2 
Pr

2. 4, 1, 5, 3, 2

3. 4, 1, 5, 2, 3 

4. 4, 1, 3, 5, 2 

Question ID : 264330129558
an

Status : Answered
Chosen Option : 2

Q.12 ‘Q+R’ means ‘Q is the father of R’


‘Q–R’ means ‘Q is the wife of R’
ag

‘Q×R’ means ‘Q is the brother of R’


‘Q÷R’ means ‘Q is the daughter of R’
If ‘S÷ Q+ M × P − K’, then how is Q related to K?
Ans 1. Father-in-law

2. Son-in-law
G

3. Son

4. Father

Question ID : 264330143371
Status : Answered
Chosen Option : 1
Join @MathsByGaganPratap Telegram Channel for free PDFs
Q.13

Ans

1.

hs
2.

at
3.

M
4.
ap
Question ID : 26433057009
at
Status : Answered
Chosen Option : 3

Q.14 Which of the following numbers will replace the question mark (?) in the given series?
Pr

23, 25, 28, 33, ?, 51, 64


Ans 1. 39 

2. 40 

3.  41

4. 38 
an

Question ID : 26433056429
Status : Answered
Chosen Option : 3
ag
G
Join @MathsByGaganPratap Telegram Channel for free PDFs
Q.15

Ans 1. G

2. D

3. F

hs
4. B

Question ID : 264330143438
Status : Answered

at
Chosen Option : 3

Q.16 In a certain code language, ‘MANAGE’ is written as ‘LYMYFC’ and ‘CREATE’ is written
as ‘BQCYSC’. How will ‘BUILD’ be written in that language?

M
Ans 1. CKGSA

2. ASGKC

3. AKSCG

4. GAKSC
ap
Question ID : 264330143358
Status : Answered
Chosen Option : 2
at
Q.17 Select the correct combination of mathematical signs to replace the * signs and
balance the given equation.

8 * 2 * 1 * 12 * 18 * 33
Pr

Ans 1. ÷, −, −, +, =

2. ×, −, +, +, =

3. ÷, −, +, +, =

4. ÷, −, ×, +, =
an

Question ID : 26433097206
Status : Answered
Chosen Option : 3
ag
G
Join @MathsByGaganPratap Telegram Channel for free PDFs
Q.18 In this question, three statements are given, followed by two conclusions numbered I
and II. Assuming the statements to be true, even if they seem to be at variance with
commonly known facts, decide which of the conclusions logically follows/follow from
the statements.

Statements:
I. Some gates are ways.
II. All ways are routes.
III. No route is a door.

Conclusions:
I. No door is a way.
II. Some gates are routes.
Ans 1. Neither conclusion I nor II follows

2. Only conclusion I follows

hs
3. Both conclusions I and II follow

4. Only conclusion II follows

Question ID : 264330104070

at
Status : Answered
Chosen Option : 3

Q.19 In a certain code language, ‘LASER’ is written as ‘80’ and ‘WHITE’ is written as ‘70’.

M
How will ‘GLOVES’ be written in that language?
Ans 1. 92

2. 88

3. 82

4. 78
ap
Question ID : 26433069611
Status : Answered
Chosen Option : 3
at

Q.20 Which of the following numbers will replace the question mark (?) in the given series?
0.04, 0.12, 0.42, 1.68, 2.08, 10.4, ?
Pr

Ans 1. 12.3

2. 12.9

3. 11.8

4. 10.9
an

Question ID : 264330143335
Status : Answered
Chosen Option : 4

Q.21 Select the option that represents the correct order of the given words as they would
ag

appear in an English dictionary?


1. Enthusiasm
2. Enrollment
3. Enterprise
4. Ensure
5. Entangled
6. Entail
G

Ans 1. 2, 4, 6, 5, 1, 3

2. 2, 4, 6, 5, 3, 1

3. 4, 2, 6, 5, 3, 1

4. 2, 4, 5, 6, 3, 1

Question ID : 264330117382
Status : Answered
Chosen Option : 2
Join @MathsByGaganPratap Telegram Channel for free PDFs
Q.22 In a certain code language, STATION is written as URCRKMP and BRING is written as
DPKLI. How will TOYS be written in that language?
Ans 1. UNBP

2. BPNU

3. QMAV

4. VMAQ

Question ID : 264330120697
Status : Answered
Chosen Option : 4

Q.23 Select the option that represents the correct order of the given words as they would

hs
appear in an English dictionary?
1. Emergency
2. Electrician
3. Embassy
4. Elegance
5. Embargo

at
6. Element
Ans 1. 4, 2, 6, 5, 3, 1

2. 2, 4, 5, 6, 3, 1

M
3. 2, 4, 6, 5, 3, 1

4. 2, 4, 6, 5, 1, 3

ap Question ID : 264330117381
Status : Answered
Chosen Option : 3

Q.24 In a code language, 'MISSION' is written as ‘N64O’ and ‘TEASE’ is written as ‘U56F’.
How will 'NASTY' be written in that language?
at
Ans 1. Z41O

2. O14Z 

3. Z14O
Pr

4. O41Z 

Question ID : 264330120822
Status : Answered
Chosen Option : 4
an

Q.25 तस्वीर में दिख रहे एक व्यक्ति की ओर इंगित करते हुए प्रीति ने कहा, "वह मेरे पति के ससुर, गोपाल की
इकलौती पुत्री का पुत्र है।" तस्वीर में दिख रहा व्यक्ति प्रीति से किस प्रकार संबंधित है?
Ans 1. पुत्र

2. पिता
ag

3. भाई

4. चाचा

Question ID : 26433068449
G

Status : Answered
Chosen Option : 1

Section : General Awareness


Join @MathsByGaganPratap Telegram Channel for free PDFs
Q.1 Who among the following was regarded as the Father of Modern Dance, in India?
Ans 1. Pankaj Charan Das

2. Uday Shankar

3. Kunchu Kurup

4. Raja Reddy

Question ID : 26433074078
Status : Not Answered
Chosen Option : --

Q.2 Which of the following religious festivals is known as Peerla Panduga in Telangana?

hs
Ans 1. Muharram

2. Christmas

3. Eid-ul-Fitr

4. Navroz

at
Question ID : 26433064676
Status : Answered
Chosen Option : 4

M
Q.3 For contribution to which dance was Tanjore Balasaraswati awarded the title of
Sangita Kalanidhi by Madras Music Academy in 1973?
Ans 1. Kathakali ap
2. Bharatanatyam

3. Mohiniyattam

4. Kathak
at
Question ID : 264330110267
Status : Not Answered
Chosen Option : --
Pr

Q.4 Which Article of the Indian Constitution deals with the removal and suspension of a
member of Public Service Commission?
Ans 1. Article 320

2. Article 324

3. Article 317
an

4. Article 315

Question ID : 264330111534
Status : Not Answered
ag

Chosen Option : --

Q.5 Identify the correct statement about cell membranes.


Ans 1. Cell membranes are made up of carbohydrates, lipids and proteins.

2. Cell membranes are made up of fats and fibre only.


G

3. Cell membranes are made up of lipids only.

4. Cell membranes are made up of roughage.

Question ID : 26433064757
Status : Not Answered
Chosen Option : --
Join @MathsByGaganPratap Telegram Channel for free PDFs
Q.6 Which is the only mangrove habitat in the world for the species Panthera Tigris?
Ans 1. Saranda forest

2. Molai forest

3. Sundarbans

4. Abujmarh forest

Question ID : 26433092206
Status : Answered
Chosen Option : 3

Q.7 A long, winding ridge of stratified sand and gravel is known as:

hs
Ans 1. whorl

2. esker

3. lop

4. arch

at
Question ID : 26433064188
Status : Not Answered
Chosen Option : --

M
Q.8 In which of the following sports events is the Syed Mushtaq Ali trophy given?
Ans 1. Football

2. Cricket

3. Hockey
ap
4. Badminton

Question ID : 264330109666
at
Status : Answered
Chosen Option : 3

Q.9 Who among the following is NOT associated with Brahmo Samaj?
Pr

Ans 1. Atmaram Panduranga

2. Keshab Chandra Sen

3. Debendranath Tagore

4. Raja Ram Mohan Roy


an

Question ID : 264330130613
Status : Not Answered
Chosen Option : --
ag

Q.10 Which year was designated by UNESCO as the International Year of the Periodic Table
(IYPT) to mark the 150th anniversary of the Mendeleev Periodic Table?
Ans 1. 2018

2. 2020
G

3. 2017

4. 2019

Question ID : 264330143316
Status : Not Answered
Chosen Option : --
Join @MathsByGaganPratap Telegram Channel for free PDFs
Q.11 The Pradhan Mantri Mudra Yojana (PMMY) was implemented by the Government of
India in 2015 to provide collateral free loans up to ______ for facilitating self-
employment.
Ans 1. ₹5 lakh

2. ₹10 lakh

3. ₹20 lakh

4. ₹15 lakh

Question ID : 264330113301
Status : Answered
Chosen Option : 1

hs
Q.12 In 2021, India opener Rohit Sharma became the third batsman to complete 3000 runs
in T20 international cricket during a match against:
Ans 1. Australia

2. New Zealand

at
3. Namibia

4. Pakistan

M
Question ID : 26433072905
Status : Not Answered
Chosen Option : --

Q.13 To which of the following Gharanas did Hindustani Classical Vocalist, Bhimsen Joshi

Ans
belong?
1. Rampur Sahaswan Gharana
ap
2. Kirana Gharana

3. Agra Gharana
at
4. Gwalior Gharana

Question ID : 264330143402
Status : Not Answered
Pr

Chosen Option : --

Q.14 Which of the following states used EVMs for the first time in the general elections held
in May 1982?
Ans 1. Kerala
an

2. Andhra Pradesh

3. Tamil Nadu

4. Maharashtra
ag

Question ID : 264330113423
Status : Answered
Chosen Option : 1

Q.15 As on March 2021, which of the following states does NOT have an export-processing
G

zone?
Ans 1. Bihar

2. Gujarat

3. West Bengal

4. Tamil Nadu

Question ID : 26433089220
Status : Answered
Chosen Option : 1
Join @MathsByGaganPratap Telegram Channel for free PDFs
Q.16 While historians use the term Vijayanagara Empire, contemporaries described it as the
___________Samrajyamu.
Ans 1. Kerala

2. Tamil

3. Karnataka

4. Andhra

Question ID : 26433085245
Status : Not Answered
Chosen Option : --

Q.17 Who among the following is the author of the book titled ‘Fasting, Feasting’?

hs
Ans 1. Jhumpa Lahiri 

2. Arundhati Roy

3. Anita Desai

at
4. Shashi Deshpande

Question ID : 26433054845
Status : Not Answered

M
Chosen Option : --

Q.18 Which of the following States hosted the three-day Civil Air Navigation Services
Organisation (CANSO) Conference from 1st to 3rd November 2022?
Ans 1. Maharashtra

2. Goa
ap
3. Tripura

4. Madhya Pradesh
at
Question ID : 264330132602
Status : Answered
Chosen Option : 2
Pr

Q.19 Which of the following is associated with the origin of the Sutlej river?
Ans 1. Gaumukh

2. Milam

3. Panchnad
an

4. Rakshas Tal

Question ID : 264330109866
Status : Not Answered
ag

Chosen Option : --

Q.20 The Veda Samaj of Madras was inspired by the____________.


Ans 1. Brahmo Samaj 

2. Arya Samaj 
G

3. Deoband Movement

4. Wahabi Movement

Question ID : 26433085310
Status : Not Answered
Chosen Option : --
Join @MathsByGaganPratap Telegram Channel for free PDFs
Q.21 President Smt. Droupadi Murmu launched ‘Pradhan Mantri TB Mukt Bharat Abhiyaan’
in Sep 2022 to eliminate TB from India by which of the following years?
Ans 1. 2026

2. 2024

3. 2025

4. 2027

Question ID : 264330130834
Status : Answered
Chosen Option : 3

Q.22 Which union territory has the highest male literacy rate in the 2011 census?

hs
Ans 1. Andaman and Nicobar

2. Lakshadweep

3. Puducherry

at
4. Delhi

Question ID : 264330109507
Status : Answered

M
Chosen Option : 4

Q.23 The eleventh century structure of Bhojshala in the state of Madhya Pradesh is
constructed under the patronage of which dynasty?
Ans 1. Chandela

2. Paramara
ap
3. Nanda

4. Gurjara-Pratihara 
at
Question ID : 264330132429
Status : Not Answered
Chosen Option : --
Pr

Q.24 The famous source of Gupta empire, the Prayaga Prashasti is also known as the
__________.
Ans 1. Allahabad pillar inscription

2. Ahmedabad pillar inscription


an

3. Bithur pillar inscription

4. Lucknow pillar inscription

Question ID : 264330111368
Status : Answered
ag

Chosen Option : 1
G
Join @MathsByGaganPratap Telegram Channel for free PDFs
Q.25 Which of the following statements are true regarding the Family Courts (Amendment)
Bill, 2022?

1. The Family Courts (Amendment) Bill was introduced in Lok Sabha on 18 July 2022.
2. The Bill amends the Family Courts Act, 1984.
3. The Act allows state governments to establish Family Courts.
4. The Act is not applicable in Punjab and Tamil Nadu.
Ans 1. 1, 2 and 3

2. 1, 3 and 4

3. 1, 2 and 4

4. 2, 3 and 4

Question ID : 264330130835

hs
Status : Not Answered
Chosen Option : --

Section : Quantitative Aptitude

at
Q.1 At present, Ritika’s age is 3 times the age of Vipul. After 7 years, Ritika will be 2 times
as old as Vipul. What is the present age of Ritika (in years)?
Ans 1. 42

M
2. 35

3. 53

4. 21
ap Question ID : 264330126544
Status : Answered
Chosen Option : 4

Q.2 The cost price of milk is increased to ₹62 from ₹60. The milkman sold milk at 10%
at
profit earlier. What is his profit% now if he sells milk at the same rate?
Ans 1.

2.
Pr

3.

4.

Question ID : 264330102066
Status : Answered
an

Chosen Option : 3

Q.3 In 46.5 weeks, Ram raised Rs.2,170 for science research. How much money will he
raise in 30 weeks?
Ans 1. Rs.1,500
ag

2. Rs.1,700

3. Rs.1,600

4. Rs.1,400
G

Question ID : 26433061264
Status : Answered
Chosen Option : 4
Join @MathsByGaganPratap Telegram Channel for free PDFs
Q.4 The average weight of 30 students in a class is 60 kg. If the weight of the class teacher
is included, the average increases by 0.5 kg. The weight (in kg) of the teacher is:
Ans 1. 76.5

2. 75.5

3. 72.5

4. 74.5

Question ID : 264330129389
Status : Answered
Chosen Option : 2

Q.5 Find the least value of K for which 7864K3 is divisible by 7.

hs
Ans 1. 4

2. 5

3. 2

at
4. 1

Question ID : 264330130058
Status : Answered

M
Chosen Option : 1

Q.6 Ram sold two bags at ₹1,260 each. On one, he gained 20% and on the other, he lost
20%. Find his gain or loss percentage in the whole transaction.
Ans 1. Gain 4%

2. Gain 5%
ap
3. Loss 4%

4. Loss 5%
at
Question ID : 264330130082
Status : Answered
Chosen Option : 3
Pr

Q.7
an
ag

Ans 1. 60 Percent
G

2. 70 Percent

3. 68 Percent

4. 75 Percent

Question ID : 26433080201
Status : Answered
Chosen Option : 4
Join @MathsByGaganPratap Telegram Channel for free PDFs
Q.8

Ans 1. 3

2. -6

3. 6

4. -3

Question ID : 26433061340
Status : Not Answered
Chosen Option : --

hs
Q.9

Ans 1. 20

at
2. 32

3. 24

4. 12

M
Question ID : 26433083072
Status : Answered
ap Chosen Option : 4

Q.10 In an examination, a student scores 4 marks for every correct answer and losses 1
mark for every wrong answer. A student attempted all 200 questions and scored 500
marks. What is the number of questions, he answered correctly?
Ans 1. 140

2. 130
at
3. 180

4. 160
Pr

Question ID : 264330123004
Status : Answered
Chosen Option : 1

Q.11 In ΔABC, D and F are the middle points of the sides AB and AC, respectively. E is a
an

point on the segment DF such that DE : EF = 1 : 2. If DE = 4 cm, then BC is equal to:


Ans 1. 20 cm

2. 26 cm

3. 22 cm
ag

4. 24 cm

Question ID : 26433083000
Status : Answered
Chosen Option : 4
G

Q.12 Find the remainder when 117+9 is divided by 10.


Ans 1. 1

2. 0

3. 3

4. 2

Question ID : 264330126498
Status : Answered
Chosen Option : 2
Join @MathsByGaganPratap Telegram Channel for free PDFs
Q.13

Ans 1. 5

2. 9

3. 7

4. 11

Question ID : 264330126225
Status : Answered
Chosen Option : 2

hs
Q.14

Ans 1. 52

at
2. 76

3. 80

4. 64

M
Question ID : 264330101337
Status : Answered
Chosen Option : 2
ap
Q.15 A sum of ₹16,875 was lent out at simple interest, and at the end of 1 year 8 months, the
total amount was ₹18,000. Find the rate of interest per annum.
Ans
1.
at
2.

3.
Pr

4.

Question ID : 26433076190
Status : Answered
Chosen Option : 1
an

Q.16 Anita and Anand can complete a work in 12 days and 18 days respectively. They
undertook to complete that work for ₹48,000. With the help of Arun, they completed the
work in 6 days. Find the sum of the shares of Anita and Anand (in ₹).
Ans 1. 40000
ag

2. 8000

3. 16000

4. 24000
G

Question ID : 26433079962
Status : Answered
Chosen Option : 1
Join @MathsByGaganPratap Telegram Channel for free PDFs
Q.17 R and S are the mid points of the sides XY and XZ, respectively, of ∆XYZ. Also, XR = 15
cm, XY = 25 cm, XS = 12 cm and XZ = 20 cm. RS is equal to:
Ans
1.

2.

3.

4.

hs
Question ID : 264330130310
Status : Answered
Chosen Option : 3

at
Q.18 Sonam purchased a mobile phone for ₹27,000 and sold it at a loss of 19%. With that
money she purchased a Tablet and sold it at a gain of 20%. What is her overall gain or
loss?

M
Ans 1. gain of ₹870

2. loss of ₹756

3. loss of ₹856

4. gain of ₹756
ap
Question ID : 264330126515
Status : Answered
Chosen Option : 2
at
Q.19 20 kg of rice costing ₹80 per kg is mixed with y kg of rice costing ₹100 per kg. If the
cost of this mixed rice is ₹84 per kg, what is the value of y?
Ans 1. 6
Pr

2. 3

3. 4

4. 5

Question ID : 264330103239
an

Status : Answered
Chosen Option : 4

Q.20 The ratio of cheese to sauce for a single pizza is 1 cup to 1/2 cup. If Rama used 15
cups of sauce to make a number of pizzas, then how many cups of cheese did she use
ag

on those pizzas?
Ans 1. 20 cups

2. 10 cups

3. 40 cups
G

4. 30 cups

Question ID : 26433061209
Status : Answered
Chosen Option : 4
Join @MathsByGaganPratap Telegram Channel for free PDFs
Q.21 What would be the simple interest (in ₹) on principal amount of ₹ 1 lakh for 2 years at
the rate of 12 percent per annum?
Ans
1.

2.

3.

4.

Question ID : 26433076011
Status : Answered

hs
Chosen Option : 4

Q.22 Out of the total income, a family saves 15% and the rest of the amount is spent on
three types of expenditure – health, education and food in the ratio of 1 : 2 : 2. The
income is Rs.300 less than thrice the expenditure incurred on education. What is the

at
difference between the savings and the expenditure incurred on health?
Ans 1. Rs.300 

2. Rs.350

M
3. Rs.2,250

4. Rs.500

ap Question ID : 26433058208
Status : Answered
Chosen Option : 1

Q.23
at
Pr

Ans 1. 15

2. 16
an

3. 14

4. 17

Question ID : 26433079965
ag

Status : Answered
Chosen Option : 4

Q.24 A trader marked a table at ₹37,062, and sold it allowing a 33% discount. If his profit
was 42%, then the cost price (in ₹) of the table was:
G

Ans 1. 15,487 

2. 16,487 

3. 17,487 

4. 13,487 

Question ID : 264330103482
Status : Not Answered
Chosen Option : --
Join @MathsByGaganPratap Telegram Channel for free PDFs
Q.25 If the volume of a cube is given as 12167 cm3, then the surface area of the cube will
be:
Ans 1. 3475 cm2

2. 4574 cm2

3. 3174 cm2

4. 5413 cm2

Question ID : 26433076090
Status : Answered
Chosen Option : 3

hs
Section : English Language

Q.1 Select the most appropriate meaning of the given idiom.

On cloud nine
Ans 1. To be extremely cautious

at
2. To be extremely happy

3. To be extremely shaky

4. To be extremely sad 

M
Question ID : 26433089390
Status : Answered
Chosen Option : 2
ap
Q.2 Select the most appropriate ANTONYM of the given word.

Prudent
Ans 1. Wise 
at
2. Atrocious

3. Stupid 

4. Crafty
Pr

Question ID : 26433073991
Status : Answered
Chosen Option : 3
an

Q.3 Select the most appropriate option that can substitute the underlined segment in the
given sentence.
The rebels laid up their arms.
Ans 1. laid down 

2. laid out 
ag

3. laid on 

4. laid by 

Question ID : 26433059679
Status : Answered
G

Chosen Option : 1
Join @MathsByGaganPratap Telegram Channel for free PDFs
Q.4 Select the most appropriate option to fill in the blank.

___________ UK government continually reiterates its commitment to halting the brain


drain.
Ans 1. The

2. A

3. An

4. Any

Question ID : 26433066081
Status : Answered
Chosen Option : 1

hs
Q.5 The following sentence has been split into four segments. Identify the segment that
contains a grammatical error.

I caught / a white cat / rustling / beneath my room.


Ans 1. a white cat

at
2. I caught

3. rustling

M
4. beneath my room.

Question ID : 26433096145
Status : Answered
Chosen Option : 4
ap
Q.6 Select the option that expresses the given sentence in reported speech.

"We will have a thorough search before opening the room,” the inspector told his
subordinates.
at
Ans 1. The inspector told his subordinates that they would be having a thorough search
before opening the room.
2. The inspector told his subordinates that they can have a thorough search before
opening the room.
Pr

3. The inspector told his subordinates that they will had a thorough search before
opening the room.
4. The inspector told his subordinates that they would have a thorough search before
opening the room.

Question ID : 26433091960
an

Status : Answered
Chosen Option : 4

Q.7 Select the option that expresses the given sentence in reported speech.
The teacher said, “She had started the fight”.
ag

Ans 1. The teacher complained that she has started the fight.

2. The teacher said that she had started the fight.

3. The teacher ordered that she had started the fight.

4. The teacher said that she has started the fight.


G

Question ID : 264330120518
Status : Answered
Chosen Option : 2
Join @MathsByGaganPratap Telegram Channel for free PDFs
Q.8 Select the option that can be used as a one-word substitute for the underlined group
of words in the given sentence.

The war resulted in ruthless killing of many people.


Ans 1. sacrilege

2. fratricide

3. carnage 

4. murder

Question ID : 26433073857
Status : Answered
Chosen Option : 3

hs
Q.9 Select the option that expresses the given sentence in passive voice.
The storm destroyed the entire village.
Ans 1. The entire village was destroyed by the storm.

2. The storm was destroying the entire village.

at
3. The village was being destroyed entirely by the storm.

4. The village was entirely destroyed by the storm.

M
Question ID : 264330133340
Status : Answered
Chosen Option : 1
ap
Q.10 Select the most appropriate synonym for the highlighted word.
The hapless incident was a result of sheer negligence by the authorities.
Ans 1. joyful

2. triumphant 

3. ill- fated 
at
4. grateful

Question ID : 264330120722
Pr

Status : Answered
Chosen Option : 3

Q.11 Select the most appropriate option to substitute the underlined segment in the given
sentence.
an

She has gained a lot of fame but still she is very realistic and sensible.
Ans 1. down to earth

2. down to it

3. down to bottom
ag

4. down to drown

Question ID : 26433089836
Status : Answered
Chosen Option : 1
G

Q.12 Select the INCORRECTLY spelt word.


Ans 1. Publicly

2. Punctual

3. Protien

4. Pursuit

Question ID : 26433088414
Status : Answered
Chosen Option : 3
Join @MathsByGaganPratap Telegram Channel for free PDFs
Q.13 Select the most appropriate ANTONYM of the given word.

Hasten
Ans 1. Triumph

2. Spooky

3. Present 

4. Dawdle 

Question ID : 26433073992
Status : Answered
Chosen Option : 4

hs
Q.14 Select the most appropriate option to fill in the blank.

Conditions such as stress, depression and anxiety can all affect mental health and
_________ a person’s routine.
Ans 1. fragment

at
2. crush 

3. dishevel

4. disrupt

M
Question ID : 26433060953
Status : Answered
ap Chosen Option : 4

Q.15 Select the most appropriate option that can substitute the underlined segment in the
given sentence. If there is no need to substitute it, select ‘No substitution’.

He was advised for walking alone in the neighbourhood.


Ans 1. No substitution  
at
2. advised about  

3. advised against  

4. advised on 
Pr

Question ID : 26433073167
Status : Answered
Chosen Option : 1
an

Comprehension:
In the following passage, some words have been deleted. Read the passage carefully and
select the most appropriate option to fill in each blank.
The Publications Division (PD), one of the leading publishing houses in the country and the
largest in the public sector, functions (1)________ the Ministry of Information and
Broadcasting. It attempts to promote National Integration by (2)________ information and
ag

creating awareness about the various activities, regions, people and the (3)________ cultures
and beliefs of the Nation. The Division publishes and sells books at affordable prices through
its outlets in important cities and also (4)________ agents. It also participates in exhibitions
and books fairs to promote the sale of books and journals. Till date, 7,600 titles (5)________
published, of which 1,500 are live today. Besides books, the Publication Division has also
brought out 21 journals about various issues of national and social significance.
G

SubQuestion No : 16
Q.16 Select the most appropriate option to fill in blank no. 1.
Ans 1. of

2. in

3. under

4. on

Question ID : 264330132883
Status : Answered
Chosen Option : 3
Join @MathsByGaganPratap Telegram Channel for free PDFs
Comprehension:
In the following passage, some words have been deleted. Read the passage carefully and
select the most appropriate option to fill in each blank.
The Publications Division (PD), one of the leading publishing houses in the country and the
largest in the public sector, functions (1)________ the Ministry of Information and
Broadcasting. It attempts to promote National Integration by (2)________ information and
creating awareness about the various activities, regions, people and the (3)________ cultures
and beliefs of the Nation. The Division publishes and sells books at affordable prices through
its outlets in important cities and also (4)________ agents. It also participates in exhibitions
and books fairs to promote the sale of books and journals. Till date, 7,600 titles (5)________
published, of which 1,500 are live today. Besides books, the Publication Division has also
brought out 21 journals about various issues of national and social significance.

SubQuestion No : 17
Q.17 Select the most appropriate option to fill in blank no. 2.
Ans 1. concealing

hs
2. disseminating

3. pouring

4. assembling

at
Question ID : 264330132884
Status : Answered
Chosen Option : 4

M
Comprehension:
In the following passage, some words have been deleted. Read the passage carefully and
select the most appropriate option to fill in each blank.
The Publications Division (PD), one of the leading publishing houses in the country and the
ap
largest in the public sector, functions (1)________ the Ministry of Information and
Broadcasting. It attempts to promote National Integration by (2)________ information and
creating awareness about the various activities, regions, people and the (3)________ cultures
and beliefs of the Nation. The Division publishes and sells books at affordable prices through
its outlets in important cities and also (4)________ agents. It also participates in exhibitions
and books fairs to promote the sale of books and journals. Till date, 7,600 titles (5)________
published, of which 1,500 are live today. Besides books, the Publication Division has also
at
brought out 21 journals about various issues of national and social significance.

SubQuestion No : 18
Q.18 Select the most appropriate option to fill in blank no. 3.
Pr

Ans 1. same

2. myriad

3. clear

4. absolute
an

Question ID : 264330132885
Status : Answered
Chosen Option : 2
ag
G
Join @MathsByGaganPratap Telegram Channel for free PDFs
Comprehension:
In the following passage, some words have been deleted. Read the passage carefully and
select the most appropriate option to fill in each blank.
The Publications Division (PD), one of the leading publishing houses in the country and the
largest in the public sector, functions (1)________ the Ministry of Information and
Broadcasting. It attempts to promote National Integration by (2)________ information and
creating awareness about the various activities, regions, people and the (3)________ cultures
and beliefs of the Nation. The Division publishes and sells books at affordable prices through
its outlets in important cities and also (4)________ agents. It also participates in exhibitions
and books fairs to promote the sale of books and journals. Till date, 7,600 titles (5)________
published, of which 1,500 are live today. Besides books, the Publication Division has also
brought out 21 journals about various issues of national and social significance.

SubQuestion No : 19
Q.19 Select the most appropriate option to fill in blank no. 4.
Ans 1. through

hs
2. as

3. about

4. in

at
Question ID : 264330132886
Status : Answered
Chosen Option : 1

M
Comprehension:
In the following passage, some words have been deleted. Read the passage carefully and
select the most appropriate option to fill in each blank.
The Publications Division (PD), one of the leading publishing houses in the country and the
ap
largest in the public sector, functions (1)________ the Ministry of Information and
Broadcasting. It attempts to promote National Integration by (2)________ information and
creating awareness about the various activities, regions, people and the (3)________ cultures
and beliefs of the Nation. The Division publishes and sells books at affordable prices through
its outlets in important cities and also (4)________ agents. It also participates in exhibitions
and books fairs to promote the sale of books and journals. Till date, 7,600 titles (5)________
published, of which 1,500 are live today. Besides books, the Publication Division has also
at
brought out 21 journals about various issues of national and social significance.

SubQuestion No : 20
Q.20 Select the most appropriate option to fill in blank no. 5.
Pr

Ans 1. have being

2. have been

3. have be

4. had being
an

Question ID : 264330132887
Status : Answered
Chosen Option : 2
ag
G
Join @MathsByGaganPratap Telegram Channel for free PDFs
Comprehension:
Read the given passage and answer the question that follows.

When the dance had finished, Odysseus stood, his voice raised for all to hear. “We are truly
honoured by your performance; not everyone can say that they have seen the dancers of
Scyros. As tokens of our admiration we have brought gifts for you and your king.”
A murmur of excitement. Luxuries did not come often to Scyros; no one here had the money
to buy them.
“You are too kind.” Lycomedes’ face was flushed with genuine pleasure; he had not expected
this generosity. The servants brought trunks forth at Odysseus’ signal and began unloading
them on the long tables. I saw the glitter of silver, the shine of glass and gems. All of us, men
and women both, leaned towards them, eager to see.
‘Please, take what you like,’ Odysseus said. The girls moved swiftly to the tables, and I
watched them fingering the bright trinkets: perfumes in delicate glass bottles stoppered with a
bit of wax; mirrors with carved ivory for handles; bracelets of twisted gold; ribbons, dyed deep
in purples and reds. Among these were a few things I assumed were meant for Lycomedes

hs
and his counsellors: leather-bound shields, carved spear hafts, and silvered swords with
supple kidskin sheathes. Lycomedes’ eyes had caught on one of these, like a fish snagged by
a line. Odysseus stood near, presiding benevolently.
Achilles kept to the back, drifting slowly along the table. He paused to dab some perfume on
his slender wrists, stroke the smooth handle of a mirror. He lingered a moment over a pair of
earrings, blue stones set in silver wire.
A movement at the far end of the hall caught my eye. Diomedes had crossed the chamber

at
and was speaking with one of his servants, who nodded and left through the large double
doors. Whatever it was could not be important; Diomedes seemed half-asleep, his eyes
heavy-lidded and bored.
I looked back to Achilles. He was holding the earrings up to his ears now, turning them this
way and that, pursing his lips, playing at girlishness. It amused him, and the corner of his

M
mouth curved up. His eyes flicked around the hall, catching for a moment on my face. I could
not help myself. I smiled.

SubQuestion No : 21
Q.21 Which word in the passage is OPPOSITE in meaning to ‘advanced’?
ap
Ans 1. Flushed 

2. Lingered

3. Crossed

4. Snagged
at
Question ID : 264330132869
Status : Answered
Chosen Option : 2
Pr
an
ag
G
Join @MathsByGaganPratap Telegram Channel for free PDFs
Comprehension:
Read the given passage and answer the question that follows.

When the dance had finished, Odysseus stood, his voice raised for all to hear. “We are truly
honoured by your performance; not everyone can say that they have seen the dancers of
Scyros. As tokens of our admiration we have brought gifts for you and your king.”
A murmur of excitement. Luxuries did not come often to Scyros; no one here had the money
to buy them.
“You are too kind.” Lycomedes’ face was flushed with genuine pleasure; he had not expected
this generosity. The servants brought trunks forth at Odysseus’ signal and began unloading
them on the long tables. I saw the glitter of silver, the shine of glass and gems. All of us, men
and women both, leaned towards them, eager to see.
‘Please, take what you like,’ Odysseus said. The girls moved swiftly to the tables, and I
watched them fingering the bright trinkets: perfumes in delicate glass bottles stoppered with a
bit of wax; mirrors with carved ivory for handles; bracelets of twisted gold; ribbons, dyed deep
in purples and reds. Among these were a few things I assumed were meant for Lycomedes

hs
and his counsellors: leather-bound shields, carved spear hafts, and silvered swords with
supple kidskin sheathes. Lycomedes’ eyes had caught on one of these, like a fish snagged by
a line. Odysseus stood near, presiding benevolently.
Achilles kept to the back, drifting slowly along the table. He paused to dab some perfume on
his slender wrists, stroke the smooth handle of a mirror. He lingered a moment over a pair of
earrings, blue stones set in silver wire.
A movement at the far end of the hall caught my eye. Diomedes had crossed the chamber

at
and was speaking with one of his servants, who nodded and left through the large double
doors. Whatever it was could not be important; Diomedes seemed half-asleep, his eyes
heavy-lidded and bored.
I looked back to Achilles. He was holding the earrings up to his ears now, turning them this
way and that, pursing his lips, playing at girlishness. It amused him, and the corner of his

M
mouth curved up. His eyes flicked around the hall, catching for a moment on my face. I could
not help myself. I smiled.

SubQuestion No : 22
Q.22 “He was holding the earrings up to his ears now, turning them this way and that,
pursing his lips, playing at girlishness.”
ap
What do you infer as the most probable reason for Achilles’ behaviour?
Ans 1. Achilles was selecting an ideal gift for Lycomedes. 

2. Achilles was in charge of selecting the best accessories for war.


at
3. Achilles was in a woman’s disguise and was playing his part. 

4. Achilles was playing the part of an ideal host.

Question ID : 264330132870
Pr

Status : Not Answered


Chosen Option : --
an
ag
G
Join @MathsByGaganPratap Telegram Channel for free PDFs
Comprehension:
Read the given passage and answer the question that follows.

When the dance had finished, Odysseus stood, his voice raised for all to hear. “We are truly
honoured by your performance; not everyone can say that they have seen the dancers of
Scyros. As tokens of our admiration we have brought gifts for you and your king.”
A murmur of excitement. Luxuries did not come often to Scyros; no one here had the money
to buy them.
“You are too kind.” Lycomedes’ face was flushed with genuine pleasure; he had not expected
this generosity. The servants brought trunks forth at Odysseus’ signal and began unloading
them on the long tables. I saw the glitter of silver, the shine of glass and gems. All of us, men
and women both, leaned towards them, eager to see.
‘Please, take what you like,’ Odysseus said. The girls moved swiftly to the tables, and I
watched them fingering the bright trinkets: perfumes in delicate glass bottles stoppered with a
bit of wax; mirrors with carved ivory for handles; bracelets of twisted gold; ribbons, dyed deep
in purples and reds. Among these were a few things I assumed were meant for Lycomedes

hs
and his counsellors: leather-bound shields, carved spear hafts, and silvered swords with
supple kidskin sheathes. Lycomedes’ eyes had caught on one of these, like a fish snagged by
a line. Odysseus stood near, presiding benevolently.
Achilles kept to the back, drifting slowly along the table. He paused to dab some perfume on
his slender wrists, stroke the smooth handle of a mirror. He lingered a moment over a pair of
earrings, blue stones set in silver wire.
A movement at the far end of the hall caught my eye. Diomedes had crossed the chamber

at
and was speaking with one of his servants, who nodded and left through the large double
doors. Whatever it was could not be important; Diomedes seemed half-asleep, his eyes
heavy-lidded and bored.
I looked back to Achilles. He was holding the earrings up to his ears now, turning them this
way and that, pursing his lips, playing at girlishness. It amused him, and the corner of his

M
mouth curved up. His eyes flicked around the hall, catching for a moment on my face. I could
not help myself. I smiled.

SubQuestion No : 23
Q.23 Why was Odysseus distributing the gifts? ap
Ans 1. Because he was very generous and wanted to help the people of Scyros who could
not afford such gifts.
2. Because he knew Achilles’ fascination for jewellery and wanted to gift him earrings
and other ornaments.
3. Because he was pleased after witnessing the performance of the dancers of Scyros.
at
4. Because he wanted to take Lycomedes to war and had come to gift him swords and
shields.

Question ID : 264330132867
Pr

Status : Not Answered


Chosen Option : --
an
ag
G
Join @MathsByGaganPratap Telegram Channel for free PDFs
Comprehension:
Read the given passage and answer the question that follows.

When the dance had finished, Odysseus stood, his voice raised for all to hear. “We are truly
honoured by your performance; not everyone can say that they have seen the dancers of
Scyros. As tokens of our admiration we have brought gifts for you and your king.”
A murmur of excitement. Luxuries did not come often to Scyros; no one here had the money
to buy them.
“You are too kind.” Lycomedes’ face was flushed with genuine pleasure; he had not expected
this generosity. The servants brought trunks forth at Odysseus’ signal and began unloading
them on the long tables. I saw the glitter of silver, the shine of glass and gems. All of us, men
and women both, leaned towards them, eager to see.
‘Please, take what you like,’ Odysseus said. The girls moved swiftly to the tables, and I
watched them fingering the bright trinkets: perfumes in delicate glass bottles stoppered with a
bit of wax; mirrors with carved ivory for handles; bracelets of twisted gold; ribbons, dyed deep
in purples and reds. Among these were a few things I assumed were meant for Lycomedes

hs
and his counsellors: leather-bound shields, carved spear hafts, and silvered swords with
supple kidskin sheathes. Lycomedes’ eyes had caught on one of these, like a fish snagged by
a line. Odysseus stood near, presiding benevolently.
Achilles kept to the back, drifting slowly along the table. He paused to dab some perfume on
his slender wrists, stroke the smooth handle of a mirror. He lingered a moment over a pair of
earrings, blue stones set in silver wire.
A movement at the far end of the hall caught my eye. Diomedes had crossed the chamber

at
and was speaking with one of his servants, who nodded and left through the large double
doors. Whatever it was could not be important; Diomedes seemed half-asleep, his eyes
heavy-lidded and bored.
I looked back to Achilles. He was holding the earrings up to his ears now, turning them this
way and that, pursing his lips, playing at girlishness. It amused him, and the corner of his

M
mouth curved up. His eyes flicked around the hall, catching for a moment on my face. I could
not help myself. I smiled.

SubQuestion No : 24
Q.24 “Lycomedes’ eyes had caught on one of these, like a fish snagged by a line.” What can
be inferred from this statement?
Ans
ap
1. He was attracted to the unique looking sword which looked like a fishing line with a
hook.
2. Being a warrior, he was attracted to the sword and all the more since he could not
afford one like that on his own.
3. He was very cunning and wanted to take the shields and swords in order to fight
at
Odysseus later on.
4. He was a greedy man and wanted to take the best gifts for himself without sharing
anything with others.
Pr

Question ID : 264330132871
Status : Not Answered
Chosen Option : --
an
ag
G
Join @MathsByGaganPratap Telegram Channel for free PDFs
Comprehension:
Read the given passage and answer the question that follows.

When the dance had finished, Odysseus stood, his voice raised for all to hear. “We are truly
honoured by your performance; not everyone can say that they have seen the dancers of
Scyros. As tokens of our admiration we have brought gifts for you and your king.”
A murmur of excitement. Luxuries did not come often to Scyros; no one here had the money
to buy them.
“You are too kind.” Lycomedes’ face was flushed with genuine pleasure; he had not expected
this generosity. The servants brought trunks forth at Odysseus’ signal and began unloading
them on the long tables. I saw the glitter of silver, the shine of glass and gems. All of us, men
and women both, leaned towards them, eager to see.
‘Please, take what you like,’ Odysseus said. The girls moved swiftly to the tables, and I
watched them fingering the bright trinkets: perfumes in delicate glass bottles stoppered with a
bit of wax; mirrors with carved ivory for handles; bracelets of twisted gold; ribbons, dyed deep
in purples and reds. Among these were a few things I assumed were meant for Lycomedes

hs
and his counsellors: leather-bound shields, carved spear hafts, and silvered swords with
supple kidskin sheathes. Lycomedes’ eyes had caught on one of these, like a fish snagged by
a line. Odysseus stood near, presiding benevolently.
Achilles kept to the back, drifting slowly along the table. He paused to dab some perfume on
his slender wrists, stroke the smooth handle of a mirror. He lingered a moment over a pair of
earrings, blue stones set in silver wire.
A movement at the far end of the hall caught my eye. Diomedes had crossed the chamber

at
and was speaking with one of his servants, who nodded and left through the large double
doors. Whatever it was could not be important; Diomedes seemed half-asleep, his eyes
heavy-lidded and bored.
I looked back to Achilles. He was holding the earrings up to his ears now, turning them this
way and that, pursing his lips, playing at girlishness. It amused him, and the corner of his

M
mouth curved up. His eyes flicked around the hall, catching for a moment on my face. I could
not help myself. I smiled.

SubQuestion No : 25
Q.25 What is the passage based upon? ap
Ans 1. A palace scene where the king and his counsellors are deciding to give gifts to the
poor people who had just completed their dance performance in front of everyone.
2. A palace scene where the king is distributing gifts to the performers who are very poor
and cannot afford to buy such gifts on their own.
3. A palace scene where a dance performance is taking place and thereafter, gifts are
distributed to the performers and their selecting the trinkets are described.
at
4. A palace scene where gifts are being distributed after a dance performance and
detailed descriptions of the gifts and the way that people are choosing them are presented.

Question ID : 264330132868
Pr

Status : Not Answered


Chosen Option : --
an
ag
G
Join @MathsByGaganPratap Telegram Channel for free PDFs

Phase-XI/2023/Selection Posts and Selection Posts/Ladakh/2023


Roll Number Gagan Pratap Maths
Candidate Name Gagan Pratap Maths
Venue Name TCS Gito Bitan
Exam Date 28/06/2023
Exam Time 9:00 AM - 10:00 AM
Subject Selection Post Matriculation Level

Section : General Intelligence

hs
Q.1 Select the option that is related to the third term in the same way as the second term is
related to the first term and the sixth term is related to the fifth term.
FEDERAL : LAREDEF :: ECONOMY : ? :: GENERAL : LARENEG
Ans 1. MONEYCO

2. YMONOCE

at
3. OMYNECO

4. COMONEY

M
Question ID : 264330142836
Status : Answered
Chosen Option : 2
ap
Q.2 Select the option that is related to the third word in the same way as the second word
is related to the first word. (The words must be considered as meaningful English
words and must NOT be related to each other based on the number of letters/number
of consonants/vowels in the word)

Pupils : Class :: Flowers : ?


at
Ans 1. Brood

2. Bouquet

3. Rose
Pr

4. Band

Question ID : 264330142861
Status : Answered
Chosen Option : 2
an

Q.3
ag

Ans 1.

2.
G

3.

4.

Question ID : 264330143278
Status : Answered
Chosen Option : 2
Join @MathsByGaganPratap Telegram Channel for free PDFs
Q.4 Select the option that is related to the third word in the same way as the second word
is related to the first word. (The words must be considered as meaningful English
words and must NOT be related to each other based on the number of letters/number
of consonants/vowels in the word)

Escape : Abscond :: Idea : ?


Ans 1. Squander

2. Empty

3. Notion

4. Artificial

Question ID : 264330142859
Status : Answered

hs
Chosen Option : 3

Q.5 Three Statements are given followed by Three conclusions numbered I, II and III.
Assuming the statements to be true, even if they seem to be at variance with
commonly known facts, decide which of the conclusions logically follow(s) from the

at
statements.

Statements:
All keys are windows.
No window is a door.

M
Some doors are locks.

Conclusions:
I. Some locks are keys.
II. No lock is a window.
III. No door is a key.
Ans 1. Both conclusions I and II follow.

2. Only conclusion II follows.


ap
3. Only conclusion I follows.

4. Only conclusion III follows.


at
Question ID : 264330142877
Status : Answered
Chosen Option : 4
Pr

Q.6 Three statements are followed by three conclusions numbered I, II and III. You have to
consider these statements to be true, even if they seem to be at variance with
commonly known facts. Decide which of the given conclusions logically follow(s) from
the given statements.
an

Statements:
All bottles are plastics.
Some plastics are fibres.
Some fibres are glasses.

Conclusions:
(I) Some fibres are plastics.
ag

(II) All plastics are bottles.


(III) Some glasses are fibres.
Ans 1. None of the conclusions follows

2. Only conclusions I and III follow

3. Only conclusion II follows


G

4. Only conclusion I follows

Question ID : 264330142973
Status : Answered
Chosen Option : 2
Join @MathsByGaganPratap Telegram Channel for free PDFs
Q.7 Which of the following numbers will replace the question mark (?) in the given series?
73, 57, 43, 31, ?, 13
Ans 1. 27

2. 25

3. 29

4. 21

Question ID : 26433056436
Status : Answered
Chosen Option : 4

Q.8 Select the option that indicates the correct arrangement of the given words in a logical

hs
and meaningful order.
1. Wheat
2. Dough
3. Bake
4. Grind
5. Bread

at
Ans 1. 4, 2, 1, 3, 5

2. 1, 4, 2, 3, 5

3. 3, 1, 5, 2, 4

M
4. 1, 4, 3, 5, 2

Question ID : 264330125998
Status : Answered
ap Chosen Option : 4

Q.9 Select the option that is related to the fourth term in the same way as the first term is
related to the second term and the fifth term is related to the sixth term
9 : 126 :: ? : 238 :: 21 : 294
at
Ans 1.  18

2.  28

3.  17
Pr

4.  22

Question ID : 26433040812
Status : Answered
Chosen Option : 3
an

Q.10 ‘A # B’ means ‘A is the daughter B’


‘A @ B’ means ‘A is the only son of B’.
‘A & B’ means ‘A is the mother of B’.
‘A % B’ means ‘A is the father-in-law of B’.
If P # Q @ R % S & T @ Q , then how is P related to T?
ag

Ans 1. Sister

2. Mother’s sister

3. Brother  

4. Father’s brother
G

Question ID : 26433091636
Status : Answered
Chosen Option : 3
Join @MathsByGaganPratap Telegram Channel for free PDFs
Q.11 Select the option that is related to the fifth term in the same way as the second term is
related to the first term and the fourth term is related to the third term.
FLIGHT : GHIFLT :: PERIOD : IORPED :: GARLIC : ?
Ans 1. LIRGCA

2. LIRGAC

3. RILACG

4. RILAGC

Question ID : 264330142835
Status : Answered
Chosen Option : 2

hs
Q.12 Select the option that indicates the correct arrangement of the given words in a logical
and meaningful order.
1. Country
2. Continent
3. City
4. Street

at
5. State
Ans 1. 1, 2, 5, 3, 4

2. 1, 2, 3, 5, 4

M
3. 2, 1, 5, 3, 4

4. 3, 2, 1, 5, 4

ap Question ID : 264330125993
Status : Answered
Chosen Option : 1

Q.13 Which letter-cluster will replace the question mark (?) and complete the given series?
UTSR, XWVU, ?, DCBA, GFED
at
Ans 1. BZYW

2. BZYX

3. AZWY
Pr

4. AZYX

Question ID : 264330142915
Status : Answered
Chosen Option : 4
an

Q.14 Which letter cluster will replace the question mark (?) to complete the given series?
TUBE, SXEH, ?, QDKN, PGNQ
Ans 1. RAHK 

2. RADG 
ag

3. RAKH

4. RAGD

Question ID : 26433056232
Status : Answered
G

Chosen Option : 1
Join @MathsByGaganPratap Telegram Channel for free PDFs
Q.15

Ans

1.

hs
at
2.

M
3.
ap
at
4.
Pr

Question ID : 264330143263
Status : Answered
Chosen Option : 1

Q.16 A − B means ‘A is the wife of B’;


an

A + B means ‘A is the sister of B’ and


A * B means ‘A is the father of B’.
If Q − T * Y + M − K, then how is Q related to K?
Ans 1. Mother-in-law

2. Mother
ag

3. Sister

4. Grandmother

Question ID : 26433092469
Status : Answered
G

Chosen Option : 1
Join @MathsByGaganPratap Telegram Channel for free PDFs
Q.17 Select the correct combination of mathematical signs to sequentially replace the *
signs and to balance the given equation.
83*16*4*57*3
Ans 1. =,÷,+,−

2. −,×,=,÷

3. −,+, =,−

4. −,÷,=,+

Question ID : 264330142882
Status : Answered
Chosen Option : 2

hs
Q.18 Select the figure that will replace the question mark (?) in the following figure series.

at
Ans

M
1.
ap
2.
at
Pr

3.
an

4.
ag

Question ID : 264330143087
Status : Answered
Chosen Option : 1
G
Join @MathsByGaganPratap Telegram Channel for free PDFs
Q.19 Three statements are given, followed by three conclusions numbered I, II and III.
Assuming the statements to be true, even if they seem to be at variance with
commonly known facts, decide which of the conclusions logically follow(s) from the
statements.

Statements:
Some diamonds are stones.
Some stones are hard.
All hard are rocks.

Conclusions:
I. Some stones are diamonds.
II. All stones are hard.
III. Some hard are rocks.
Ans 1. All the conclusions follow

2. Only conclusions I and II follow

hs
3. Only conclusions II and III follow

4. Only conclusions I and III follow

at
Question ID : 26433092644
Status : Answered
Chosen Option : 4

M
Q.20 Select the option that represents the correct order of the given words as they would
appear in an English dictionary.

1. Cope
2. Cosmetic
3. Courage
4. Cough
5. Course
6. Courier
ap
Ans 1. 1, 2, 4, 3, 6, 5

2. 1, 2, 4, 3, 5, 6
at
3. 1, 2, 4, 6, 3, 5

4. 2, 1, 4, 3, 6, 5
Pr

Question ID : 264330142926
Status : Answered
Chosen Option : 1

Q.21 Select the combination of letters that when sequentially placed in the blanks of the
given series, will complete the series.
an

PB_EP_UF_BU_PBUH
Ans 1. UBPG

2. RUPE

3. URGH
ag

4. EPRU

Question ID : 26433057811
Status : Answered
Chosen Option : 1
G
Join @MathsByGaganPratap Telegram Channel for free PDFs
Q.22 In a code language, 'chores are hobbies' is written as 'tpl esr nil', 'hobbies keep me
busy' is written as 'cha esr lon ane', 'busy with chores' is written as 'sia ane nil'. What
is the code for the word 'with' in this language?
Ans 1. sia

2. ane

3. cha

4. tpl

Question ID : 26433056771
Status : Answered
Chosen Option : 1

hs
Q.23 Read the given statements and conclusions carefully. Assuming that the information
given in the statements is true, even if it appears to be at variance with commonly
known facts, decide which of the given conclusions logically follow(s) from the
statements.

Statements:

at
All pigeons are ants.
Some ants are birds.
All birds are cats.

Conclusions:

M
I. Some cats are pigeons.
II. Some birds are pigeons.
III. Some cats are ants.
Ans 1. Only conclusion III follows

2. Only conclusion II follows

3. All conclusions follow


ap
4. Only conclusion I follows

Question ID : 26433068546
at
Status : Answered
Chosen Option : 1

Q.24 Select the option that is related to the fourth term in the same way as the first term is
Pr

related to the second term and the fifth term is related to the sixth term
22 : 676 :: ? : 361 :: 2 : 36
Ans 1. 18 

2. 15 

3. 17
an

4. 13

Question ID : 26433040803
Status : Answered
Chosen Option : 2
ag

Q.25 ‘A # B’ means ‘A is the brother of B’.


‘A @ B’ means ‘A is the daughter of B’.
‘A & B’ means ‘A is the husband of B’.
‘A % B’ means ‘A is the wife of B’.
If S % D # F @ G & H @ J, then how is F related to H?
G

Ans 1. Daughter

2. Son’s wife

3. Daughter’s daughter

4. Mother

Question ID : 264330142902
Status : Answered
Chosen Option : 4

Section : General Awareness


Join @MathsByGaganPratap Telegram Channel for free PDFs
Q.1 In September 2021, the ‘SAATH’ initiative was launched in ________, for women
associated with Self Help Groups (SHGs).
Ans 1. Tamil Nadu

2. Jammu and Kashmir

3. Kerala

4. Andaman and Nicobar Islands

Question ID : 26433082809
Status : Not Answered
Chosen Option : --

Q.2 Rajya Sabha has to return the money bill to Lok Sabha with or without the

hs
recommendation within _________ .
Ans 1. 10 days  

2. 30 days   

3. 14 days   

at
4. 21 days   

Question ID : 264330112758

M
Status : Answered
Chosen Option : 3

Q.3 In January 2022, TS Tirumurti was in news due to:


Ans 1. being nominated as the president of FIH
ap
2. assuming chair of the Counter-Terrorism Committee of the United Nations

3. becoming FIFA player of the year

4. becoming the first Indian to reach the north pole


at
Question ID : 26433082814
Status : Answered
Chosen Option : 2
Pr

Q.4 The 2021, U-19 Asia Cup Cricket was hosted by which of the following countries?
Ans 1. Qatar

2. Bangladesh

3. Pakistan
an

4. UAE

Question ID : 26433054675
Status : Answered
ag

Chosen Option : 2

Q.5 Identify the odd option in relation to the oceanic layers.


Ans 1. The Oceanic Deeps

2. The Oceanic Slope


G

3. The Continental Shelf

4. The Continental Slope

Question ID : 264330142699
Status : Answered
Chosen Option : 3
Join @MathsByGaganPratap Telegram Channel for free PDFs
Q.6 According to standard census definition, who among the following can be categorised
as a main worker in India?
Ans 1. A person who works for at least 160 days 

2. A person who works for at least 183 days 

3. A person who works for at least 200 days 

4. A person who works for at least 100 days 

Question ID : 264330109871
Status : Not Answered
Chosen Option : --

Q.7 Who is the 50th Chief Justice of India?

hs
Ans 1. UU Lalit

2. DY Chandrachud

3. NV Ramana

at
4. Sharad Arvind Bobde

Question ID : 264330113426
Status : Answered

M
Chosen Option : 2

Q.8 India lifted a record-extending _________ ICC U19 World Cup title after defeating
England in the final at the Sir Vivian Richards Stadium in North Sound, Antigua in
February 2022.
Ans 1. fifth
ap
2. third 

3. sixth

4. eighth
at

Question ID : 26433054451
Status : Not Answered
Pr

Chosen Option : --

Q.9 Triple point can be defined as:


Ans 1. the condition of temperature under which the gaseous and liquid phases of a
substance can exist in equilibrium
2. the temperature at which solid is converted into liquid 
an

3. the condition of pressure under which the solid and liquid phases of a substance can
exist in equilibrium 
4. the condition of temperature and pressure under which the gaseous, liquid and solid
phases of a substance can exist in equilibrium 
ag

Question ID : 26433054913
Status : Not Answered
Chosen Option : --

Q.10 In which year, the Koya rebellion took place in the eastern Godavari tract?
G

Ans 1. 1882-1883

2. 1887-1888

3. 1879-1880

4. 1874-1875

Question ID : 264330142807
Status : Answered
Chosen Option : 1
Join @MathsByGaganPratap Telegram Channel for free PDFs
Q.11 For how many days did the Government of India launch the ‘Garib Kalyan Rojgar
Abhiyan’ (GKRA), in June 2020?
Ans 1. 100 days

2. 150 days

3. 125 days

4. 200 days

Question ID : 264330113299
Status : Not Answered
Chosen Option : --

Q.12 Devdas Smriti Panthi Dance Award is related to which state?

hs
Ans 1. Rajasthan

2. Bihar

3. Jharkhand

at
4. Chhattisgarh

Question ID : 26433074137
Status : Answered

M
Chosen Option : 4

Q.13 Fazilka Heritage Festival is an annual art, cultural and food festival celebrated in which
of the following states?
Ans 1. Punjab

2. Maharashtra
ap
3. Haryana

4. Uttar Pradesh
at
Question ID : 264330141232
Status : Not Answered
Chosen Option : --
Pr

Q.14 Identify an algae that undergoes anisogamous fusion of gametes.


Ans 1. Spirogyra

2. Eudorina

3. Volvox
an

4. Ulothrix

Question ID : 26433064745
Status : Answered
ag

Chosen Option : 1

Q.15 The correct sequence in descending order of the following states in terms of the
proved coal reserves (as of the Ministry of Coal, Government of India, 2018) is ______.
Ans 1. Jharkhand, Odisha, Chhattisgarh, West Bengal
G

2. Odisha, Chhattisgarh, Madhya Pradesh, Jharkhand

3. Jharkhand, Chhattisgarh, Odisha, Madhya Pradesh

4. Chhattisgarh, Odisha, Jharkhand, Madhya Pradesh

Question ID : 26433097094
Status : Not Answered
Chosen Option : --
Join @MathsByGaganPratap Telegram Channel for free PDFs
Q.16 Zakir Hussain is renowned for playing the ____.
Ans 1. santoor

2. sitar

3. shehnai

4. tabla

Question ID : 264330141391
Status : Answered
Chosen Option : 4

Q.17 In North Indian temple architecture, which element denotes the superstructure or
tower above the sanctum and the pillared mandapas?

hs
Ans 1. Kalasha

2. Amalaka

3. Shikhara

at
4. Antarala

Question ID : 26433090747
Status : Answered

M
Chosen Option : 3

Q.18 Lai Haraoba is the earliest form of dance that forms the basis of all stylist dances of
_______.
Ans 1. Manipur

2. Madhya Pradesh
ap
3. Punjab

4. Uttar Pradesh
at
Question ID : 264330141226
Status : Answered
Chosen Option : 1
Pr

Q.19 Which of the following autobiographies was written by the Indian female poet Kamala
Das, who was nominated for Nobel Prize for literature in 1984?
Ans 1. My Story

2. A Life Less Ordinary


an

3. A Life Apart

4. Smritichitre: The Memoire of a Spirited Wife

Question ID : 264330141403
Status : Not Answered
ag

Chosen Option : --

Q.20 ‘Vikramarjuna -Vijaya’ composed by Pampa was written in which of the following
languages?
Ans 1. Sanskrit
G

2. Tamil

3. Telugu

4. Kannada

Question ID : 26433081937
Status : Not Answered
Chosen Option : --
Join @MathsByGaganPratap Telegram Channel for free PDFs
Q.21 Which of the following National Waterways is located in Tamil Nadu?
Ans 1. National Waterway-97

2. National Waterway-93

3. National Waterway-98

4. National Waterway-99

Question ID : 26433067188
Status : Not Answered
Chosen Option : --

Q.22 Pushyamitra, who was the commander of Brihadratha, the last Mauryan emperor,
killed the king and established a new dynasty. Which of the following was his dynasty?

hs
Ans 1. Shunga

2. Satavahana

3. Chedi

at
4. Kanva

Question ID : 264330142802
Status : Answered

M
Chosen Option : 1

Q.23 Who among the following founded the ‘Forward Bloc’?


Ans 1. Motilal Nehru ap
2. Dadabhai Naoroji

3. Subhas Chandra Bose 

4. Surendranath Banerjee 
at
Question ID : 26433055015
Status : Answered
Chosen Option : 3
Pr

Q.24 Who became the first Indian to win a silver medal in an individual section of the Para
Archery World championship in February 2022?
Ans 1. Pooja Jatyan

2. Rakesh Kumar

3. Jyothi Surekha Vennam


an

4. Harvinder Singh

Question ID : 26433081842
Status : Not Answered
ag

Chosen Option : --

Q.25 Identify the type of cell that is long and branched.


Ans 1. Nerve cell

2. Columnar epithelial cell


G

3. White blood cell

4. Red blood cell

Question ID : 264330142696
Status : Answered
Chosen Option : 1

Section : Quantitative Aptitude


Join @MathsByGaganPratap Telegram Channel for free PDFs
Q.1 The largest number of four digits that is divisible by 12,15 and 18 is ________.
Ans 1. 9450

2. 9900

3. 9000

4. 9750

Question ID : 264330142680
Status : Answered
Chosen Option : 2

Q.2 Find the time taken by a 150 m long train, running at a speed of 63 km/h, to cross
another train of length 100 m, running at a speed of 45 km/h in the same direction.

hs
Ans 1. 40 seconds

2. 45 seconds

3. 50 seconds

at
4. 60 seconds

Question ID : 264330142540
Status : Answered

M
Chosen Option : 3

Q.3 A can complete a piece of work in 12 days while B can the same work in 18 days. If
they both work together, then how many days will be required to finish the work?
Ans 1.  6.5

2. 6
ap
3.  7

4.  7.2
at
Question ID : 26433063961
Status : Answered
Chosen Option : 4
Pr

Q.4 A vehicle travels 200 metres in 5 sec and then another 250 metres in 5 sec. What is its
average speed (in meters/sec)?
Ans 1. 45

2. 50
an

3. 55

4. 40

Question ID : 26433075935
Status : Answered
ag

Chosen Option : 1

Q.5 In a company, the total customer calls on Monday, Wednesday and Friday are 15,500
and the total calls on Tuesday and Thursday are 12,500. The average calls on Saturday
and Sunday are 7,000. What are the average calls per day in a week?
G

Ans 1. 6500

2. 7500

3. 7000

4. 6000

Question ID : 264330126095
Status : Answered
Chosen Option : 4
Join @MathsByGaganPratap Telegram Channel for free PDFs
Q.6 In a street there are 20 dogs and cats.If the ratio between dogs and cats is 3:2, then the
number of cats is ______.
Ans 1. 16

2. 10

3. 8

4. 22

Question ID : 264330142681
Status : Answered
Chosen Option : 3

Q.7 X,Y and Z can do a piece of work in 4 days, 5 days and 7 days, respectively. They get

hs
₹415 for completing the work. If X,Y and Z have worked together to complete the work,
what is X's share?
Ans 1. ₹275

2. ₹175

at
3. ₹200

4. ₹225

M
Question ID : 264330142541
Status : Answered
Chosen Option : 2

Q.8 The average of 48, 39 and y is 40. Find the value of y.


Ans 1. 33
ap
2. 32

3. 35

4. 31
at

Question ID : 264330103236
Status : Answered
Chosen Option : 1
Pr

Q.9 The annual incomes of Anand and Bharath are in the ratio 3 : 5 and their annual
expenses are in the ratio 1 : 3. If each of them saves ₹10,000 at the end of the year,
then the annual income of Bharath is:
Ans 1. ₹25,000
an

2. ₹12,000

3. ₹15,000

4. ₹30,000
ag

Question ID : 264330142258
Status : Answered
Chosen Option : 1

Q.10 Mukesh sells almonds at the cost price but uses a false weight and thus gains a 23%
G

profit. How many grams of almonds is he giving for 3.075 kg?


Ans 1. 2500

2. 2700

3. 2400

4. 2600

Question ID : 26433091990
Status : Answered
Chosen Option : 1
Join @MathsByGaganPratap Telegram Channel for free PDFs
Q.11 A dishonest shopkeeper professes to sell grains at cost price, but he uses a weight of
935 g for 1 kg weight. Find his gain percentage.
Ans
1.

2.

3.

4.

Question ID : 26433090323
Status : Answered

hs
Chosen Option : 4

Q.12 In an election between two candidates, one candidate got 60% of the valid votes. If the
total number of votes polled were 8850, and 10% of the votes were invalid, then find
the number of valid votes that the other candidate got.

at
Ans 1. 3816

2. 3186

3. 3681

M
4. 3618

Question ID : 26433081555
Status : Answered
Chosen Option : 2
ap
Q.13 A person completes 80 km of a journey at 10 km/h and the remaining 70 km in 7 hours.
His average speed for the whole journey is:
Ans 1. 10.5 km/h
at
2. 9 km/h

3. 10 km/h

4. 9.5 km/h
Pr

Question ID : 264330142261
Status : Answered
Chosen Option : 3

Q.14 By working 12 hours a day, 90 people can complete a work in 16 days. How much work
an

will be left after 24 days if 70 people work for 8 hours per day?
Ans
1.

2.
ag

3.

4.
G

Question ID : 26433073115
Status : Answered
Chosen Option : 2
Join @MathsByGaganPratap Telegram Channel for free PDFs
Q.15 To buy a car, Sahil borrowed some amount at 10% simple interest rate. He has paid
some loan amount but ₹1,81,500 is due which he has to pay in 3 years. What will be
the annual installment (in ₹)?
Ans 1. 65,000

2. 47,000

3. 55,000

4. 54,500

Question ID : 264330103335
Status : Answered
Chosen Option : 3

hs
Q.16 A book is sold at a discount of 32% and an additional discount of 25% is allowed on
cash payment. If Ramesh purchased the book by paying Rs.2,550 in cash, find the
marked price of the book.
Ans 1. Rs.5,000

2. Rs.5,050

at
3. Rs.4,950

4. Rs.5,560

M
Question ID : 26433064981
Status : Answered
Chosen Option : 1

Q.17
ap
Ans 1. 8%
at
2. 10%

3. 5%

4. 12%
Pr

Question ID : 264330142675
Status : Answered
Chosen Option : 1

Q.18 In an examination, the ratio of practical to theory marks is 3∶ 7. If 80% of the marks are
an

scored in the practical and 60% of marks are scored in the theory, what is the ratio of
marks scored in the theory and the practical?
Ans 1. 6 : 7

2. 5 : 7

3. 7 : 4
ag

4. 3 : 7

Question ID : 26433085453
Status : Answered
G

Chosen Option : 3
Join @MathsByGaganPratap Telegram Channel for free PDFs
Q.19 What annual instalment will discharge a debt of Rs.35,455 due in three years at 12%
simple interest per annum (Installments will be paid at the end of the years)(Round off
to nearest Rupee)?
Ans 1. Rs.10,552

2. Rs.14,652

3. Rs.11,804

4. Rs.12,548

Question ID : 26433075411
Status : Not Answered
Chosen Option : --

hs
Q.20 The perimeter of an isosceles right-angled triangle having an area of 200 cm2 is:
Ans 1. 68.3 cm

2. 78.2 cm

3. 70.6 cm

at
4. 58.6 cm

Question ID : 26433082729

M
Status : Answered
Chosen Option : 1

Q.21 The height of Ankit was 160 cm last year. In a year, his height increased by 8%. What is
his height now?
Ans 1. 173 cm
ap
2. 172.2 cm

3. 172.8 cm

4. 173.2 cm
at

Question ID : 26433063299
Status : Answered
Pr

Chosen Option : 3

Q.22 A solid sphere has a surface area of 616 cm2. This sphere is now cut into two
hemispheres. What is the total surface area of one of the hemispheres?
Ans 1. 440 cm2

2. 462 cm2
an

3. 452 cm2

4. 390 cm2

Question ID : 264330142762
ag

Status : Not Answered


Chosen Option : --

Q.23 If a shopkeeper sold one-third of his goods at a loss of 20%, then at what gain %
should the remainder be sold to gain 25% on the whole transaction?
G

Ans 1. 32.5 Percent

2. 47.5 Percent

3. 20 Percent

4. 5 Percent

Question ID : 26433081564
Status : Answered
Chosen Option : 2
Join @MathsByGaganPratap Telegram Channel for free PDFs
Q.24 In an election between two candidates, 84 votes were declared as invalid. The winning
candidate secures 62% of the valid votes and wins by 96 votes. The number of votes
polled is:
Ans 1. 424

2. 543

3. 641

4. 484

Question ID : 26433080009
Status : Answered
Chosen Option : 4

hs
Q.25 After giving two successive discounts of 30% and 40% on the marked price, an article
is sold for Rs.420. Find the marked price of the article.
Ans 1. ₹900

2. ₹800

at
3. ₹1,000

4. ₹1,100

M
Question ID : 264330132900
Status : Answered
Chosen Option : 3

Section : English Language


ap
Q.1 Select the idiom that gives the most appropriate meaning of the underlined phrase in
the following sentence.
Chanda always annoys everyone by praising herself all the time.
Ans 1. Crying over the spilt milk
at
2. Catching one’s eyes

3. Blowing her own trumpet

4. Facing the music


Pr

Question ID : 26433086440
Status : Answered
Chosen Option : 1

Q.2 Select the most appropriate ANTONYM of the given word.


an

Despair
Ans 1. Distant

2. Sorrow
ag

3. Optimism

4. Distinct

Question ID : 264330135890
Status : Answered
G

Chosen Option : 3
Join @MathsByGaganPratap Telegram Channel for free PDFs
Q.3 Select the option that expresses the given sentence in indirect speech.
He said, “I will see you now”.
Ans 1. He said he will see you now. 

2. He said he will see me now. 

3. He said he would see me then. 

4. He said I will see you now.

Question ID : 264330133305
Status : Answered
Chosen Option : 3

Q.4 Select the most appropriate option to fill in the blank.

hs
Tropical forests have _________ greenery.
Ans 1. exciting

2. lush

3. thorny

at
4. withered

Question ID : 26433060378

M
Status : Not Answered
Chosen Option : --

Q.5 From among the words given in bold, select the INCORRECTLY spelt word in the
following sentence.
ap
Women’s vulnarability due to the denial of ownership of productive resources has
been focused on in the analysis of how progressive laws shaped gender relations.
Ans 1. analysis

2. vulnarability

3. progressive
at
4. focused

Question ID : 264330143252
Pr

Status : Answered
Chosen Option : 2

Q.6 Select the most appropriate ANTONYM to replace the italicised word.

“Can I borrow your cell?” She asked.


an

Ans 1.  bite

2.  rent

3.  hire

4. lend 
ag

Question ID : 26433059651
Status : Answered
Chosen Option : 2
G
Join @MathsByGaganPratap Telegram Channel for free PDFs
Q.7 Select the most appropriate meaning of the given idiom.

Get out of hand


Ans 1.  Keep control of hand

2. Go out of control 

3. Hands should be crossed

4.  Fall from hand

Question ID : 26433060411
Status : Answered
Chosen Option : 2

hs
Q.8 Select the option that expresses the given sentence in active voice.
The sweater is knitted by Suman.
Ans 1. Suman knits the sweater.

2. Suman be knitted the sweater.

at
3. Suman have knitting the sweater.

4. Suman been knit the sweater.

M
Question ID : 264330132872
Status : Answered
Chosen Option : 1

Q.9 Choose the option that rectifies the grammatical and spelling errors in the given
sentence.
ap
The secret to tightrope walking is to graps the rope between the twos and distribute
one's weight evenly on the soles of one's feet.
Ans 1. to grasp the rope between the toes
at
2. evenly on the soul of one's feet

3. and distribute ones weight

4. The secrte of tightrope walk is 


Pr

Question ID : 26433088850
Status : Answered
Chosen Option : 1

Q.10 Select the most appropriate option to fill in the blank.


an

Time management is essential for a ________ life and career.


Ans 1. failed

2. successful  

3. scrupulous
ag

4. mere

Question ID : 26433060790
Status : Answered
G

Chosen Option : 2
Join @MathsByGaganPratap Telegram Channel for free PDFs
Q.11 Select the most appropriate option to fill in the blank.

All desire wealth and some will _________ it.


Ans 1. remove 

2. acquire 

3. aspire 

4. collude

Question ID : 26433059687
Status : Answered
Chosen Option : 2

hs
Q.12 Select the most appropriate meaning of the given idiom.

Throw caution to the wind


Ans 1. To be indecisive or unsure about a decision

2. To take a big risk or be reckless

at
3. To be overly cautious or hesitant

4. To be too strict or rigid in following rules

M
Question ID : 264330133361
Status : Answered
Chosen Option : 1
ap
Q.13 Select the option that can be used as a one-word substitute for the given group of
words.

Jokes and funny remarks amongst friends.


Ans 1. Banter
at
2. Discussion

3. Meet

4. Saunter
Pr

Question ID : 26433060094
Status : Not Answered
Chosen Option : --

Q.14 Select the most appropriate meaning of the given idiom.


an

Through the grapevine


Ans 1. Through difficulties

2. Via gossip

3. Extremely ill
ag

4. Succeeded brilliantly

Question ID : 264330134217
Status : Answered
G

Chosen Option : 4
Join @MathsByGaganPratap Telegram Channel for free PDFs
Q.15 Choose the option that rectifies the incorrectly spelt underlined word.

The Swiss entrepreneure has pulled the plug on any further investment in the firm.
Ans 1. antepreneure 

2. entreperneur

3. entreprenur  

4. entrepreneur  

Question ID : 26433088847
Status : Answered
Chosen Option : 4

hs
Comprehension:
In the following passage, some words have been deleted. Read the passage carefully and
select the most appropriate option to fill in each blank.

Children are the future of any country and a country that does not take care of this
(1)_________ resource suffers later. A successful nation is that which makes its youth strong

at
enough to lift this (2)_________ responsibility on their shoulders. (3)_________ , this is not
the case in our country. Education, which is a necessity, is still a (4)__________ here. Poor
people (5) _________ to send their children to school.

SubQuestion No : 16

M
Q.16 Select the most appropriate option to fill in blank number 1.
Ans 1. valued

2. extravagant

3. valuable

4. worthless
ap
Question ID : 26433073412
Status : Answered
at
Chosen Option : 3

Comprehension:
In the following passage, some words have been deleted. Read the passage carefully and
Pr

select the most appropriate option to fill in each blank.

Children are the future of any country and a country that does not take care of this
(1)_________ resource suffers later. A successful nation is that which makes its youth strong
enough to lift this (2)_________ responsibility on their shoulders. (3)_________ , this is not
the case in our country. Education, which is a necessity, is still a (4)__________ here. Poor
people (5) _________ to send their children to school.
an

SubQuestion No : 17
Q.17 Select the most appropriate option to fill in blank number 2.
Ans 1. impotent

2. mighty
ag

3. bulky

4. frail

Question ID : 26433073413
G

Status : Answered
Chosen Option : 1
Join @MathsByGaganPratap Telegram Channel for free PDFs
Comprehension:
In the following passage, some words have been deleted. Read the passage carefully and
select the most appropriate option to fill in each blank.

Children are the future of any country and a country that does not take care of this
(1)_________ resource suffers later. A successful nation is that which makes its youth strong
enough to lift this (2)_________ responsibility on their shoulders. (3)_________ , this is not
the case in our country. Education, which is a necessity, is still a (4)__________ here. Poor
people (5) _________ to send their children to school.

SubQuestion No : 18
Q.18 Select the most appropriate option to fill in blank number 3.
Ans 1. Certainly

2. Necessarily

hs
3. Unfortunately

4. Partially

Question ID : 26433073414

at
Status : Answered
Chosen Option : 3

Comprehension:

M
In the following passage, some words have been deleted. Read the passage carefully and
select the most appropriate option to fill in each blank.

Children are the future of any country and a country that does not take care of this
(1)_________ resource suffers later. A successful nation is that which makes its youth strong
enough to lift this (2)_________ responsibility on their shoulders. (3)_________ , this is not
ap
the case in our country. Education, which is a necessity, is still a (4)__________ here. Poor
people (5) _________ to send their children to school.

SubQuestion No : 19
Q.19 Select the most appropriate option to fill in blank number 4.
Ans 1. theory
at
2. delight

3. luxury

4. study
Pr

Question ID : 26433073415
Status : Answered
Chosen Option : 3
an

Comprehension:
In the following passage, some words have been deleted. Read the passage carefully and
select the most appropriate option to fill in each blank.

Children are the future of any country and a country that does not take care of this
(1)_________ resource suffers later. A successful nation is that which makes its youth strong
ag

enough to lift this (2)_________ responsibility on their shoulders. (3)_________ , this is not
the case in our country. Education, which is a necessity, is still a (4)__________ here. Poor
people (5) _________ to send their children to school.

SubQuestion No : 20
Q.20 Select the most appropriate option to fill in blank number 5.
G

Ans 1. explicate

2. agitate

3. fluctuate

4. hesitate

Question ID : 26433073416
Status : Answered
Chosen Option : 2
Join @MathsByGaganPratap Telegram Channel for free PDFs
Comprehension:
Read the given passage and answer the questions that follows.
Sparrows are little birds that can be found all throughout the world. Sparrows come in a wide
variety of varieties. The length of a sparrow ranges from four to six inches. Their wonderful
melody is well-liked and appreciated by a large number of people. On the ground, grass, low
trees and bushes are all good sites for sparrows to make their homes. They nest in the cracks
and crevices of buildings in urban areas. Nests are rarely constructed at great heights. Nests
are constructed from twigs, grasses and other plant fibres. Their nests are usually modest,
but well-built.
Four to six eggs are laid by female sparrows at a time. Reddish brown blotches appear on the
eggs' white shells. Between eleven and fourteen days, they'll hatch. The young are taken care
of by both, male and female parents. After hatching, the young are fed insects. Using their big
feet, sparrows scrape seeds. Seeds are the primary food source for adult sparrows. Birds like
sparrows can be found nearly anywhere there are people. These lovely birds are beloved by
a large number of people all around the world.

SubQuestion No : 21

hs
Q.21 Identify the central theme of the passage.
Ans 1. Description of lifestyle of sparrows

2. Description of sparrows’ nest

at
3. Different shortcomings of sparrows

4. Availability of sparrows

Question ID : 264330102441

M
Status : Answered
Chosen Option : 1

Comprehension: ap
Read the given passage and answer the questions that follows.
Sparrows are little birds that can be found all throughout the world. Sparrows come in a wide
variety of varieties. The length of a sparrow ranges from four to six inches. Their wonderful
melody is well-liked and appreciated by a large number of people. On the ground, grass, low
trees and bushes are all good sites for sparrows to make their homes. They nest in the cracks
and crevices of buildings in urban areas. Nests are rarely constructed at great heights. Nests
are constructed from twigs, grasses and other plant fibres. Their nests are usually modest,
at
but well-built.
Four to six eggs are laid by female sparrows at a time. Reddish brown blotches appear on the
eggs' white shells. Between eleven and fourteen days, they'll hatch. The young are taken care
of by both, male and female parents. After hatching, the young are fed insects. Using their big
feet, sparrows scrape seeds. Seeds are the primary food source for adult sparrows. Birds like
Pr

sparrows can be found nearly anywhere there are people. These lovely birds are beloved by
a large number of people all around the world.

SubQuestion No : 22
Q.22 How many eggs are laid by female sparrows at a time?
Ans 1. Four to six
an

2. Two to four

3. Six to eight

4. Eight to ten
ag

Question ID : 264330102442
Status : Answered
Chosen Option : 1
G
Join @MathsByGaganPratap Telegram Channel for free PDFs
Comprehension:
Read the given passage and answer the questions that follows.
Sparrows are little birds that can be found all throughout the world. Sparrows come in a wide
variety of varieties. The length of a sparrow ranges from four to six inches. Their wonderful
melody is well-liked and appreciated by a large number of people. On the ground, grass, low
trees and bushes are all good sites for sparrows to make their homes. They nest in the cracks
and crevices of buildings in urban areas. Nests are rarely constructed at great heights. Nests
are constructed from twigs, grasses and other plant fibres. Their nests are usually modest,
but well-built.
Four to six eggs are laid by female sparrows at a time. Reddish brown blotches appear on the
eggs' white shells. Between eleven and fourteen days, they'll hatch. The young are taken care
of by both, male and female parents. After hatching, the young are fed insects. Using their big
feet, sparrows scrape seeds. Seeds are the primary food source for adult sparrows. Birds like
sparrows can be found nearly anywhere there are people. These lovely birds are beloved by
a large number of people all around the world.

SubQuestion No : 23

hs
Q.23 Select the most appropriate ANTONYM of the given word.
Melody
Ans 1. Musicality

2. Symphony

at
3. Disharmony

4. Ballad

M
Question ID : 264330102444
Status : Answered
Chosen Option : 3

Comprehension:
ap
Read the given passage and answer the questions that follows.
Sparrows are little birds that can be found all throughout the world. Sparrows come in a wide
variety of varieties. The length of a sparrow ranges from four to six inches. Their wonderful
melody is well-liked and appreciated by a large number of people. On the ground, grass, low
trees and bushes are all good sites for sparrows to make their homes. They nest in the cracks
and crevices of buildings in urban areas. Nests are rarely constructed at great heights. Nests
at
are constructed from twigs, grasses and other plant fibres. Their nests are usually modest,
but well-built.
Four to six eggs are laid by female sparrows at a time. Reddish brown blotches appear on the
eggs' white shells. Between eleven and fourteen days, they'll hatch. The young are taken care
of by both, male and female parents. After hatching, the young are fed insects. Using their big
Pr

feet, sparrows scrape seeds. Seeds are the primary food source for adult sparrows. Birds like
sparrows can be found nearly anywhere there are people. These lovely birds are beloved by
a large number of people all around the world.

SubQuestion No : 24
Q.24 Identify a suitable title for the passage.
Ans 1. Song of Sparrows
an

2. How Sparrows Build Nests

3. What Do The Sparrows Eat?

4. The Lives Of Sparrows


ag

Question ID : 264330102440
Status : Answered
Chosen Option : 4
G
Join @MathsByGaganPratap Telegram Channel for free PDFs
Comprehension:
Read the given passage and answer the questions that follows.
Sparrows are little birds that can be found all throughout the world. Sparrows come in a wide
variety of varieties. The length of a sparrow ranges from four to six inches. Their wonderful
melody is well-liked and appreciated by a large number of people. On the ground, grass, low
trees and bushes are all good sites for sparrows to make their homes. They nest in the cracks
and crevices of buildings in urban areas. Nests are rarely constructed at great heights. Nests
are constructed from twigs, grasses and other plant fibres. Their nests are usually modest,
but well-built.
Four to six eggs are laid by female sparrows at a time. Reddish brown blotches appear on the
eggs' white shells. Between eleven and fourteen days, they'll hatch. The young are taken care
of by both, male and female parents. After hatching, the young are fed insects. Using their big
feet, sparrows scrape seeds. Seeds are the primary food source for adult sparrows. Birds like
sparrows can be found nearly anywhere there are people. These lovely birds are beloved by
a large number of people all around the world.

SubQuestion No : 25

hs
Q.25 Identify the tone of the passage.
Ans 1. Analytical

2. Descriptive

at
3. Satiric

4. Humorous

Question ID : 264330102443

M
Status : Answered
Chosen Option : 1

ap
at
Pr
an
ag
G
Join @MathsByGaganPratap Telegram Channel for free PDFs

Phase-XI/2023/Selection Posts and Selection Posts/Ladakh/2023


Roll Number Gagan Pratap Maths
Candidate Name Gagan Pratap Maths
Venue Name iON Digital Zone iDZ Tirunelveli
Exam Date 28/06/2023
Exam Time 11:45 AM - 12:45 PM
Subject Selection Post Higher Secondary Level

Section : General Intelligence

hs
Q.1 Select the option that is related to the third word in the same way as the second word
is related to the first word. (The words must be considered as meaningful English
words and must NOT be related to each other based on the number of letters/number
of consonants/vowels in the word)
Uproar : Peace :: Gloom : ?
Ans 1. Delight

at
2. Obscurity

3. Shade

4. Murk

M
Question ID : 264330142462
Status : Answered
Chosen Option : 1

Q.2
ap
at
Ans

1.
Pr

2.
an
ag

3.
G

4.

Question ID : 26433055941
Status : Answered
Chosen Option : 4
Join @MathsByGaganPratap Telegram Channel for free PDFs
Q.3 Read the given statements and conclusions carefully. Assuming that the information
given in the statements is true, even if it appears to be at variance with commonly
known facts, decide which of the given conclusions logically follow(s) from the
statements.

Statements:

All dinosaurs are extinct.


No tigers are extinct.
Some tigers are omnivores.

Conclusions:

I. Some omnivores are dinosaurs.


II. No tigers are dinosaurs.
III. Some tigers are dinosaurs.

hs
Ans 1. Only conclusions II and III follow

2. Only conclusion II follows

3. Only conclusions I and III follow

4. Only conclusions I and II follow

at
Question ID : 264330142510
Status : Answered
Chosen Option : 2

M
Q.4 Select the correct combination of mathematical signs to sequentially replace the *
signs and balance the given equation.

11 * 4 * 12 * 14 * 7 * 530
Ans 1. ×, ×, +, ÷, =

2. ×, −, +, ÷, =
ap
3. −, ×, +, ÷, = 

4. +, ×, +, ÷, =
at
Question ID : 26433097195
Status : Answered
Chosen Option : 1
Pr

Q.5 If,
‘B % D’ means ‘B is the sister of D’,
‘B & D’ means ‘B is the father of D’,
'B × D’ means ‘B is the husband of D’,
‘B # D’ means ‘B is the brother of D’,
an

‘B $ D’ means ‘B is the wife of D’,


‘B @ D’ means ‘B is the mother-in-law of D’,
then how is A related to K in the following expression?
A×B@C$D#E&K
Ans 1. Paternal uncle

2. Sibling
ag

3. Grandfather

4. Brother

Question ID : 264330142505
G

Status : Answered
Chosen Option : 3
Join @MathsByGaganPratap Telegram Channel for free PDFs
Q.6 Select the option that represents the correct order of the given words as they would
appear in an English dictionary.
1. Goodness
2. Gooseberry
3. Goodbye
4. Goodwill
5. Goodies
Ans 1. 4, 1, 5, 2, 3

2. 2, 4, 5, 1, 3

3. 3, 5, 1, 4, 2

4. 1, 2, 3, 4, 5

Question ID : 26433057884

hs
Status : Answered
Chosen Option : 3

Q.7 ‘A + B’ means ‘A is the brother of B’


‘A – B’ means ‘A is the mother of B’

at
‘A × B’ means ‘A is the husband of B’
‘A ÷ B’ means ‘A is the sister of B’
‘A @ B’ means ‘A is the son of B’
If P – Q + R @ S + T, then how is P related to T?
Ans 1. Husband’s brother

M
2. Husband

3. Son

4. Brother’s wife ap Question ID : 264330122204


Status : Answered
Chosen Option : 4
at
Q.8 Select the correct option that indicates the correct arrangement of the following
human body parts from top to bottom.
1. Waist
2. Neck
3. Legs
Pr

4. Chest
5. Eyes
Ans 1. 2, 1, 4, 5, 3

2. 2, 5, 1, 4, 3

3. 5, 4, 2, 1, 3
an

4. 5, 2, 4, 1, 3

Question ID : 264330129473
Status : Answered
Chosen Option : 4
ag

Q.9 Select the combination of letters that when sequentially placed from left to right in the
blanks of the given series will complete the letter series.
A_E_IAC_G___EGIACE__
Ans 1. C E G I A C I G
G

2. C G I E A C I G

3. C G E I A I G C

4. C G E I A C G I

Question ID : 264330142490
Status : Answered
Chosen Option : 4
Join @MathsByGaganPratap Telegram Channel for free PDFs
Q.10 Select the option that is related to the third term in the same way as the second term is
related to the first term and the sixth term is related to the fifth term.
22 : 52 :: 17 : ? :: 42 : 92
Ans 1. 48

2. 44

3. 42

4. 34

Question ID : 264330142400
Status : Answered
Chosen Option : 3

hs
Q.11 Select the option that represents the letters that, when sequentially placed from left to
right in the blanks below, will complete the letter series.
J_K_ON_LKM_N_ L_MO_
Ans 1. LMOJJNK

2. LMOKNKJ

at
3. LMJOJKN

4. LMJLMJO

M
Question ID : 26433069626
Status : Answered
Chosen Option : 3
ap
Q.12 In a certain code language, ‘DISHRAG’ is coded as ‘132’ and ‘ENVELOPE’ is coded as
‘188’. How will ‘IMMINENT’ be coded in that language?
Ans 1. 139

2. 167 

3. 107
at
4. 194 

Question ID : 264330118645
Pr

Status : Answered
Chosen Option : 4

Q.13 In a certain code language, ‘VIOLENT’ is coded as ‘104’ and ‘UPDATED’ is coded as
‘78’. How will ‘TOUCHED’ be coded in that language?
Ans 1. 65 
an

2. 93

3. 83

4. 26 
ag

Question ID : 264330118650
Status : Answered
Chosen Option : 3
G
Join @MathsByGaganPratap Telegram Channel for free PDFs
Q.14 Select the option that is related to the third word in the same way as the second word
is related to the first word.
(The words must be considered as meaningful English words and must NOT be related
to each other based on the number of letters/number of consonants/vowels in the
word)
Son : Daughter :: Fox : ?
Ans 1. Witch

2. Doe

3. Vixen

4. Filly

Question ID : 264330142482
Status : Answered

hs
Chosen Option : 3

Q.15

at
M
Ans 1.

2. ap
3.

4.

Question ID : 264330142430
at
Status : Answered
Chosen Option : 2

Q.16
Pr
an

Ans 1. T

2. S

3. R

4. U
ag

Question ID : 264330142441
Status : Answered
Chosen Option : 2
G

Q.17 In a certain code language, ‘PAWS’ is written as ‘59’ and ‘CLAW’ is written as ‘39’.How
will ‘TOES’ be written in that language?
Ans 1. 69

2. 59

3. 39

4. 29

Question ID : 26433068639
Status : Answered
Chosen Option : 2
Join @MathsByGaganPratap Telegram Channel for free PDFs
Q.18 ‘M+O’ means ‘M is the father of O’
‘M–O’ means ‘M is the mother of O’
‘M×O’ means ‘M is the brother of O’
‘M÷O’ means ‘M is the sister of O’

What does ‘W+T+S’ mean?


Ans 1. W is the father of S

2. W is the sister of S

3. W is the father’s father of S

4. W is the brother of S

Question ID : 264330142501
Status : Answered

hs
Chosen Option : 3

Q.19 Select the combination of letters that when sequentially placed in the blanks of the
given series will complete the series.
BA, DC, _ E, H _ , JI, _K

at
Ans 1. FGL

2. LGF

3. GGL

M
4. GFL

Question ID : 26433078820
Status : Answered
ap Chosen Option : 1

Q.20 Select the correct combination of mathematical signs to sequentially replace the *
signs and balance the given equation.

2 * 16 * 11 * 12 * 4 * 355
at
Ans 1. ×, +, +, ÷, =  

2. ×, ×, +, ÷, =

3. −, ×, +, ÷, = 
Pr

4. ×, −, +, ÷, =  

Question ID : 26433097197
Status : Answered
Chosen Option : 2
an

Q.21 Three statements are given, followed by three conclusions numbered I, II and III.
Assuming the statements to be true, even if they seem to be at variance with
commonly known facts, decide which of the conclusions logically follow(s) from the
statements.
Statements:
ag

Some pages are words.


All pages are books.
All words are registers.
Conclusions:
I. Some books are registers.
II. Some books are words.
III. All pages are registers.
G

Ans 1.  Only conclusions I and II follow.

2. All conclusions follow. 

3. Only conclusions II and III follow. 

4. Only conclusions I and III follow. 

Question ID : 26433095998
Status : Answered
Chosen Option : 1
Join @MathsByGaganPratap Telegram Channel for free PDFs
Q.22 Select the option that is related to the fifth letter-cluster in the same way as the second
letter-cluster is related to the first letter-cluster and the fourth letter-cluster is related
to the third letter-cluster.
GROUNDED : UORGDEDN :: SPECIALS : CEPSSLAI :: BOOKENDS : ?
Ans 1. KOBOSDNE

2. KOOBSDNE

3. KOBODSNE

4. KOOBDSNE

Question ID : 26433056184
Status : Answered
Chosen Option : 2

hs
Q.23 In a certain code language, ‘HAPPY’ is written as ‘SZKKB’ and ‘SOUND’ is written as
‘HLFMW’. How will ‘SNORE’ be written in that language?
Ans 1. MHLIV

2. HMLEV

at
3. HMLIV

4. HMLUV

M
Question ID : 26433068635
Status : Answered
Chosen Option : 3
ap
Q.24 In a certain code language, 'SUPER' is coded as '61' and 'COLOR' is coded as '77'. How
will 'CENTER' be coded in that language?
Ans 1. 88

2. 98

3. 65
at
4. 103

Question ID : 264330142483
Pr

Status : Answered
Chosen Option : 4

Q.25 Select the option that is related to the fifth term in the same way as the second term is
related to the first term and the fourth term is related to the third term.
LOGO : PHPM :: MOCK : LDPN :: DEAR : ?
an

Ans 1. EFBS

2. SBFE

3. SCFE

4. SBEF
ag

Question ID : 26433067392
Status : Answered
Chosen Option : 2
G

Section : General Awareness


Join @MathsByGaganPratap Telegram Channel for free PDFs
Q.1 Which of the following food items can be used as natural food preservatives?
Ans 1. Vinegar, ginger, apple and banana 

2. Garlic, lemon, sugar and vinegar 

3. Garlic, apple, salt and tamarind 

4. Ginger, garlic, banana and tamarind 

Question ID : 26433054933
Status : Not Answered
Chosen Option : --

Q.2 Who among the following won the 2022 JCB Prize for Literature for his book ‘The
Paradise of Food’?

hs
Ans 1. AS Panneerselvan

2. Amitabha Bagchi

3. Khalid Jawed

at
4. Ramchandra Guha

Question ID : 264330132596
Status : Not Answered

M
Chosen Option : --

Q.3 Nagarjuni Caves were donated to which of the following sects by Mauryans?
Ans 1. Ajivika  ap
2. Buddhism 

3. Lokayat 

4. Jainism 
at
Question ID : 264330110200
Status : Not Answered
Chosen Option : --
Pr

Q.4 Which of the following is NOT a Constitutional Body in India?


Ans 1. Election Commission of India

2. State Human Rights Commission

3. State Public Service Commission


an

4. Union Public Service Commission of India

Question ID : 264330120509
Status : Answered
Chosen Option : 2
ag

Q.5 Which Kabaddi player received the defender of the season award of the Pro Kabaddi
League Season 8?
Ans 1. Mohit Goyat

2. Mohammadreza Chiyaneh shadloui


G

3. Naveen Kumar 

4.  Pradeep Narwal

Question ID : 26433061861
Status : Answered
Chosen Option : 2
Join @MathsByGaganPratap Telegram Channel for free PDFs
Q.6 Name the hot, dry oppressing winds that blow in between Delhi and Patna.
Ans 1. Loo  

2. Mango shower 

3.  Nor westers

4.  Blossom shower

Question ID : 264330141931
Status : Answered
Chosen Option : 1

Q.7 Which festival celebrated by Muslims is also known as Barawafat?

hs
Ans 1. Muharram

2. Milad-un-Nabi 

3.  Id-ul-Zuha

4. Shab-e-Barat 

at
Question ID : 26433054262
Status : Not Answered
Chosen Option : --

M
Q.8 The noted danseuse Yamini Krishnamurthy opened Yamini School of Dance in
_________, in the year 1990.
Ans 1. Mumbai ap
2. Delhi

3. Chennai

4. Hyderabad
at
Question ID : 264330110265
Status : Not Answered
Chosen Option : --
Pr

Q.9 The name of Bhajan Sopori is associated with which musical instrument?
Ans 1.  Sitar

2.  Surbahar

3.  Santoor
an

4.  Sarod

Question ID : 264330141607
Status : Not Answered
Chosen Option : --
ag

Q.10 Which stone could convert all baser metals like iron into gold?
Ans 1. Philosopher’s stone

2. Emerald
G

3. Jade

4. Copper stone

Question ID : 26433064766
Status : Not Answered
Chosen Option : --
Join @MathsByGaganPratap Telegram Channel for free PDFs
Q.11 Which of the following rulers is credited with the construction of Purana Quila in
Delhi?
Ans 1. Babar and Akbar

2. Qutub ud din and Illtutmish

3. ShahJahan and Jahangir

4. Humayun and Sher Shah

Question ID : 264330130883
Status : Answered
Chosen Option : 4

Q.12 The capacity of Vishakhapatnam, one of the oil refinery of Hindustan Petroleum

hs
Corporation Ltd, is:
Ans 1. 6.5 million metric tonnes per annum

2. 9.69 million metric tonnes per annum

3. 8.3 million metric tonnes per annum

at
4. 5.5 million metric tonnes per annum

Question ID : 26433089208

M
Status : Not Answered
Chosen Option : --

Q.13 Who became India’s second highest wicket taker in Tests after Anil Kumble by taking
his 435th test wicket in March 2022?
Ans 1. Yuzvendra Chahal
ap
2. Ravichandran Ashwin

3. Ishant Sharma

4. Mohammed Shami
at

Question ID : 264330122349
Status : Answered
Pr

Chosen Option : 2

Q.14 The Constitution (One Hundred and Twenty Seventh Amendment) Bill, 2021, was
introduced in the Lok Sabha by the ______ in August 2021.
Ans 1. Minister of Social Justice and Empowerment

2. Minister of Minority Affairs


an

3. Minister of Tribal Affairs

4. Minister of Rural Development

Question ID : 26433090631
ag

Status : Not Answered


Chosen Option : --

Q.15 Which of the following correctly define the way the unemployment rate is calculated in
India?
G

Ans 1. The percentage of persons unemployed among the persons in the labour force

2. The percentage of persons unemployed among the total population

3. The percentage of persons in labour force

4. The percentage of employed persons in the population

Question ID : 264330109518
Status : Answered
Chosen Option : 2
Join @MathsByGaganPratap Telegram Channel for free PDFs
Q.16 The Election Commission of India was established in ________.
Ans 1. 1953

2. 1952

3. 1950

4. 1951

Question ID : 264330110807
Status : Answered
Chosen Option : 2

Q.17 The famous tripartite struggle was fought for control over ________ in early medieval
period.

hs
Ans 1. Mudgagiri  

2. Kannauj 

3. Pataliputra  

at
4. Manyakheta 

Question ID : 26433085232
Status : Answered

M
Chosen Option : 2

Q.18 Vasanta Ritu falls in which months, according to the Gregorian Calendar?
Ans 1. January-February ap
2. July-August

3. March-April

4. May-June
at
Question ID : 264330110646
Status : Not Answered
Chosen Option : --
Pr

Q.19 How many Directive Principles were added or amended in the Indian Constitution by
the 42nd Amendment Act of 1976?
Ans 1. Five

2. Three

3. Two
an

4. Four

Question ID : 264330113403
Status : Not Answered
ag

Chosen Option : --

Q.20 When was the Duckworth-Lewis formula used for the first time in international cricket?
Ans 1. 1 January 1998

2. 1 January 1999
G

3. 1 January 1996

4. 1 January 1997

Question ID : 26433059368
Status : Not Answered
Chosen Option : --
Join @MathsByGaganPratap Telegram Channel for free PDFs
Q.21 Who among the following founded the Depressed Classes Association, in 1930?
Ans 1. Swami Sahajanand Saraswati

2. Mahatma Gandhi

3. Dr. BR Ambedkar

4. Jayaprakash Narayan

Question ID : 264330106581
Status : Answered
Chosen Option : 3

Q.22 Choose from the following the novel written by Salman Rushdie.

hs
Ans 1.  In a Free State

2. A Bend in the River 

3. An Area of Darkness

4. The Satanic Verses 

at
Question ID : 26433054256
Status : Not Answered
Chosen Option : --

M
Q.23 In which Five-year plan was the following statement quoted?

“The urge to bring economic and social change under present conditions comes from
the fact of poverty and inequalities in income, wealth and opportunity”.
Ans 1. Second Five-year plan

2. Fourth Five-year plan


ap
3. First Five-year plan

4. Third Five-year plan


at

Question ID : 264330109877
Status : Not Answered
Chosen Option : --
Pr

Q.24 Which of the following traditional dances is mainly performed during Navaratri?
Ans 1. Ras leela

2. Nati
an

3. Dumhal

4. Garba

Question ID : 264330141615
Status : Answered
ag

Chosen Option : 4

Q.25 What will be the focal length of a convex lens with the power of a +2.5 Diopter?
Ans 1. 20 cm
G

2. 10 cm

3. 40 cm

4. 5 cm

Question ID : 264330109883
Status : Not Answered
Chosen Option : --

Section : Quantitative Aptitude


Join @MathsByGaganPratap Telegram Channel for free PDFs
Q.1 The speed of a boat when travelling downstream is 48 km/h, whereas when travelling
upstream it is 32 km/h. What is the speed of the boat in still water?
Ans 1. 80 km/h

2. 30 km/h

3. 40 km/h

4. 20 km/h

Question ID : 264330141222
Status : Answered
Chosen Option : 3

Q.2 △ABC ≈ △ LMN and their perimeters are 72 cm and 48 cm, respectively. If LM = 8 cm,

hs
then what is the length of AB (in cm)?
Ans 1. 14 

2. 12 

3. 10

at
4. 8 

Question ID : 264330126613

M
Status : Answered
Chosen Option : 2

Q.3 ap
Ans
1.

2.
at

3.
Pr

4.

Question ID : 26433096378
an

Status : Answered
Chosen Option : 1

Q.4
ag

Ans 1. 2625.5

2. 2255.5

3. 1925.5
G

4. 2425.5

Question ID : 264330103346
Status : Answered
Chosen Option : 4
Join @MathsByGaganPratap Telegram Channel for free PDFs
Q.5 10 men can complete a work in 12 days, and 10 women can complete the same work in
6 days. How many days will it take to complete the work if they work together?
Ans 1. 4 days

2. 9 days

3. 12 days

4. 6 days

Question ID : 26433091941
Status : Answered
Chosen Option : 1

Q.6 There was a mock test conducted in a class. The following are the scores of the

hs
students who gave all the tests. What is the average score of the subject that has the
highest average score for all the students?

at
Ans 1. 131

M
2. 129.33

3. 133.33

4. 126
ap Question ID : 264330131409
Status : Answered
Chosen Option : 1

Q.7 For what value of q does the system of equations 38x + qy + 171 = 0 and 46x + 414y +
at
207 = 0 have infinite number of solutions?
Ans 1. 380

2. 345
Pr

3. 342

4. 350

Question ID : 264330126547
Status : Answered
an

Chosen Option : 3

Q.8 Raman buys 18 kg of rice at ₹35 per kg and 30 kg of rice at ₹39 per kg. Find the
average price (in ₹) per kg of the total rice.
Ans 1. 39
ag

2. 37.5

3. 38.5

4. 38
G

Question ID : 264330102321
Status : Answered
Chosen Option : 2
Join @MathsByGaganPratap Telegram Channel for free PDFs
Q.9

Ans 1. D

hs
2. C

3. A

4. B

at
Question ID : 264330130411
Status : Answered
Chosen Option : 3

M
Q.10

Ans 1. 50

2. 54
ap
3. 56

4. 52
at
Question ID : 26433093249
Status : Answered
Chosen Option : 2
Pr

Q.11 A trader offers to sell a commodity under any of the two schemes mentioned below:
Scheme 1: The trader offers two successive discounts of 20% and 12.5% on the sale of
each unit.
Scheme 2: The trader offers the buyer an option to ‘pay for y units, get z units free’.
The buyer realizes that if he needs to buy (y + z) units, it does not matter to him which
of the two schemes he opts for.
an

Which of the following options gives a correct ordered pair of (y, z)?
Ans 1. (3, 7)

2. (7, 10)

3. (10, 7)
ag

4. (7, 3)

Question ID : 264330103266
Status : Answered
Chosen Option : 1
G

Q.12 If tan 15θ = cot 15 θ (0° < θ < 10°) , then the value of θ is:
Ans 1. 3°

2. 1°

3. 9°

4. 6°

Question ID : 264330141583
Status : Answered
Chosen Option : 1
Join @MathsByGaganPratap Telegram Channel for free PDFs
Q.13 If x = 14, y = 15 and z = 17, then the value of x3 + y3 + z3 - 3xyz is:
Ans 1. 333

2. 322

3. 312

4. 222

Question ID : 264330141412
Status : Answered
Chosen Option : 2

Q.14 A policeman follows a thief, who is 1 km ahead of him. The thief and the policeman run

hs
at speeds of 8 km/h and 10.5 km/h, respectively. What distance (in km) is run by the
thief before he is nabbed by the policeman?
Ans 1. 4.4 

2. 3.2 

3. 3.6 

at
4. 2.5 

Question ID : 26433063238

M
Status : Answered
Chosen Option : 2

Q.15 Find the least value of ‘@’ to make the number 7@5471 perfectly divisible by 9.
ap
Ans 1. 6

2. 1

3. 3

4. 4
at
Question ID : 264330129982
Status : Answered
Chosen Option : 3
Pr

Q.16

Ans 1. Rs.1,250

2. Rs.1,400
an

3. Rs.1,200

4. Rs.1,350

Question ID : 26433073088
ag

Status : Answered
Chosen Option : 2

Q.17 Two-thirds of a certain distance was covered at the speed of 15 km/h, one-fifth at the
speed of 25 km/h and the rest at the speed of 50 km/h. Find the average speed (in
km/h) for the whole journey (rounded off to two decimal places).
G

Ans 1. 22.65

2. 25.25

3. 18.15

4. 20.25

Question ID : 264330126505
Status : Answered
Chosen Option : 3
Join @MathsByGaganPratap Telegram Channel for free PDFs
Q.18 What is the least value of * so that the number 457643*4 is divisible by 18?
Ans 1. 9

2. 3

3. 4

4. 5

Question ID : 26433092167
Status : Answered
Chosen Option : 2

Q.19 The marked price of a commodity was given as ₹y per kg. After two successive
discounts of 8% and 12.5%, respectively, the retailer purchased it for ₹3,059 per kg.

hs
What was the value of y?
Ans 1. 3840

2. 3800

3. 3900

at
4. 3750

Question ID : 264330103265

M
Status : Answered
Chosen Option : 2

Q.20 A shopkeeper keeps the marked price of his goods 50% more than the cost price. If he
gives successive discounts of 5%, 10% and 20% on marked price, then what is his

Ans
profit percentage?
ap
1.
at
2.

3.
Pr

4.

Question ID : 264330126184
an

Status : Answered
Chosen Option : 1

Q.21 A dealer sells rice at a profit of 15% and uses a weight that is 15% less than the actual
weight. Find his percentage gain.
ag

Ans 1.

2.

3.
G

4.

Question ID : 26433092143
Status : Answered
Chosen Option : 3
Join @MathsByGaganPratap Telegram Channel for free PDFs
Q.22

Ans
1.

2.

3.

4.

Question ID : 26433058419

hs
Status : Answered
Chosen Option : 2

Q.23

at
M
ap
Ans 1. 1973
at
2. 1972

3. 1974

4. 1975
Pr

Question ID : 26433096367
Status : Answered
Chosen Option : 1
an

Q.24 On reducing the entry fee by 25% in a museum, the number of people coming to the
museum increased by 50%, Find the percentage increase or decrease in the collection
of the entry fee.
Ans 1. 10% decrease

2. 12.5% decrease
ag

3. 10% increase

4. 12.5% increase

Question ID : 26433081552
G

Status : Answered
Chosen Option : 4
Join @MathsByGaganPratap Telegram Channel for free PDFs
Q.25 If x ∶ y = 2 ∶ 1, find the value of (2x + 3y) ∶ (4x + 7y).
Ans 1. 3 : 5

2. 7 : 5

3. 7 : 15

4. 3 : 15

Question ID : 264330141380
Status : Answered
Chosen Option : 3

Section : English Language

hs
Q.1 Select the most appropriate ANTONYM of the underlined word.

Her collection consists of rare stamps.


Ans 1. Disposed

at
2. Essential

3. Unique

4. Common

M
Question ID : 26433073444
Status : Answered
Chosen Option : 4
ap
Q.2 Select the option that can be used as a one-word substitute for the given group of
words.

A building containing interesting, rare, and old objects


Ans 1. Auditorium  
at
2. Museum 

3. Stockroom  

4. Mansion 
Pr

Question ID : 26433073905
Status : Answered
Chosen Option : 2

Q.3 Sentences of a paragraph are given below in jumbled order. Arrange the sentences in
an

the correct order to form a meaningful and coherent paragraph.

A.I was taking a stroll along the beach, when I came across a heap of plastic waste.
B.Realising the importance of removing the waste, I joined them
C.Some children came to clean the beach, seeing the dumped waste.
D.The adjacent industry had dumped the waste, which was floating, into the water,
ag

therefore polluting the sea water.


Ans 1. DCAB 

2. BADC 

3. ADBC 
G

4. ADCB 

Question ID : 26433072377
Status : Answered
Chosen Option : 4
Join @MathsByGaganPratap Telegram Channel for free PDFs
Q.4 Select the most appropriate ANTONYM of the given word.
Humble
Ans 1. Pompous

2. Brilliant

3. Meek

4. Compliant

Question ID : 264330135892
Status : Answered
Chosen Option : 1

Q.5 Select the INCORRECTLY spelt word.

hs
Ans 1. Unconscious

2. Unnatural

3. Vaccant

at
4. Vengeance

Question ID : 26433088408
Status : Answered

M
Chosen Option : 3

Q.6 Select the most appropriate meaning of the given idiom.

Pull yourself together


Ans 1. Fight violently
ap
2. Work overtime

3. Help the needy

4. Calm down
at

Question ID : 26433089347
Status : Answered
Chosen Option : 4
Pr

Q.7 Select the INCORRECTLY spelt word.


Ans 1. Inflammatory

2. Whether
an

3. Zodiac

4. Communiqe

Question ID : 26433088407
Status : Answered
ag

Chosen Option : 4

Q.8 Select the most appropriate option that can substitute the underlined segment in the
given sentence.
G

The old man now cherishes only broken memories.


Ans 1. unified

2. compact

3. rectified

4. fragmented 

Question ID : 26433089345
Status : Answered
Chosen Option : 4
Join @MathsByGaganPratap Telegram Channel for free PDFs
Q.9 Select the most appropriate option that can substitute the underlined segment in the
given sentence.

The policemen thoroughly examined the room where the theft had happened.
Ans 1. restricted 

2. scrambled

3. groped

4. inspected

Question ID : 26433089346
Status : Answered
Chosen Option : 4

hs
Q.10 Select the option that expresses the given sentence in indirect speech.
She asked, “Where do you live?”
Ans 1. She asked me where I had lived.

2. She told me where I lived.

at
3. She asked me where I had been living.

4. She asked me where I lived.

M
Question ID : 264330133426
Status : Answered
Chosen Option : 4
ap
Q.11 Select the most appropriate option that can substitute the words in the brackets in the
given sentence.

Ten years have passed since my father (is dead).


Ans 1. had died 
at
2. died 

3. was dead 

4. had been dead


Pr

Question ID : 26433061028
Status : Answered
Chosen Option : 3

Q.12 Select the most appropriate option to fill in the blank.


an

One is sure ________ what one sees.


Ans 1. about  

2. with  

3. around  
ag

4. without  

Question ID : 26433093481
Status : Answered
G

Chosen Option : 2
Join @MathsByGaganPratap Telegram Channel for free PDFs
Q.13 The following sentence has been split into four segments. Identify the segment that
contains a grammatical error.
He is as concerned as me / regarding / the renewed / examination schedule.
Ans 1. the renewed

2. examination schedule

3. regarding

4. He is as concerned as me

Question ID : 264330142377
Status : Answered
Chosen Option : 4

hs
Q.14 Select the option that expresses the given sentence in active voice.

Ice-cream is liked by her.


Ans 1. She likes ice-cream.

2. She like to eat ice-cream.

at
3. She like ice-cream.

4. She loves ice-cream.

M
Question ID : 26433088475
Status : Answered
Chosen Option : 1
ap
Q.15 Select the option that expresses the given sentence in reported speech.
I said, “They never help anybody”.
Ans 1. I said nobody helped them.

2. I said they never help anybody.

3. I said that they never helped anybody.


at
4. I said them never helped anybody.

Question ID : 264330120581
Pr

Status : Answered
Chosen Option : 3

Comprehension:
In the following passage, some words have been deleted. Read the passage carefully and
select the most appropriate option to fill in each blank.
an

A healthy diet is (1) _________ in fibre, whole grains, fresh fruits and vegetables, "good" or
unsaturated fats, and omega-3 fatty acids. These dietary components turn down
inflammation, which can (2)__________ tissue, joints, artery walls and organs. Going easy on
processed foods is another (3)_________ of healthy eating. Sweets, foods made with highly
refined grains, and sugar-sweetened beverages can (4)_________ spikes in blood sugar that
can lead to early hunger. High blood sugar is (5)_________ the development of diabetes,
ag

obesity, heart disease and even dementia.

SubQuestion No : 16
Q.16 Select the most appropriate option to fill in blank number 1.
Ans 1. easy

2. rich
G

3. wealthy

4. prosperous

Question ID : 26433098983
Status : Answered
Chosen Option : 2
Join @MathsByGaganPratap Telegram Channel for free PDFs
Comprehension:
In the following passage, some words have been deleted. Read the passage carefully and
select the most appropriate option to fill in each blank.
A healthy diet is (1) _________ in fibre, whole grains, fresh fruits and vegetables, "good" or
unsaturated fats, and omega-3 fatty acids. These dietary components turn down
inflammation, which can (2)__________ tissue, joints, artery walls and organs. Going easy on
processed foods is another (3)_________ of healthy eating. Sweets, foods made with highly
refined grains, and sugar-sweetened beverages can (4)_________ spikes in blood sugar that
can lead to early hunger. High blood sugar is (5)_________ the development of diabetes,
obesity, heart disease and even dementia.

SubQuestion No : 17
Q.17 Select the most appropriate option to fill in blank number 2.
Ans 1. resuscitate

2. damage

hs
3. revive

4. casualty

Question ID : 26433098984

at
Status : Answered
Chosen Option : 1

M
Comprehension:
In the following passage, some words have been deleted. Read the passage carefully and
select the most appropriate option to fill in each blank.
A healthy diet is (1) _________ in fibre, whole grains, fresh fruits and vegetables, "good" or
unsaturated fats, and omega-3 fatty acids. These dietary components turn down
inflammation, which can (2)__________ tissue, joints, artery walls and organs. Going easy on
ap
processed foods is another (3)_________ of healthy eating. Sweets, foods made with highly
refined grains, and sugar-sweetened beverages can (4)_________ spikes in blood sugar that
can lead to early hunger. High blood sugar is (5)_________ the development of diabetes,
obesity, heart disease and even dementia.

SubQuestion No : 18
at
Q.18 Select the most appropriate option to fill in blank number 3.
Ans 1. ground

2. element

3. cause
Pr

4. compartment

Question ID : 26433098985
Status : Answered
Chosen Option : 2
an

Comprehension:
In the following passage, some words have been deleted. Read the passage carefully and
select the most appropriate option to fill in each blank.
A healthy diet is (1) _________ in fibre, whole grains, fresh fruits and vegetables, "good" or
ag

unsaturated fats, and omega-3 fatty acids. These dietary components turn down
inflammation, which can (2)__________ tissue, joints, artery walls and organs. Going easy on
processed foods is another (3)_________ of healthy eating. Sweets, foods made with highly
refined grains, and sugar-sweetened beverages can (4)_________ spikes in blood sugar that
can lead to early hunger. High blood sugar is (5)_________ the development of diabetes,
obesity, heart disease and even dementia.
G

SubQuestion No : 19
Q.19 Select the most appropriate option to fill in blank number 4.
Ans 1. reason

2. amend

3. cause

4. provision

Question ID : 26433098986
Status : Answered
Chosen Option : 3
Join @MathsByGaganPratap Telegram Channel for free PDFs
Comprehension:
In the following passage, some words have been deleted. Read the passage carefully and
select the most appropriate option to fill in each blank.
A healthy diet is (1) _________ in fibre, whole grains, fresh fruits and vegetables, "good" or
unsaturated fats, and omega-3 fatty acids. These dietary components turn down
inflammation, which can (2)__________ tissue, joints, artery walls and organs. Going easy on
processed foods is another (3)_________ of healthy eating. Sweets, foods made with highly
refined grains, and sugar-sweetened beverages can (4)_________ spikes in blood sugar that
can lead to early hunger. High blood sugar is (5)_________ the development of diabetes,
obesity, heart disease and even dementia.

SubQuestion No : 20
Q.20 Select the most appropriate option to fill in blank number 5.
Ans 1. source of

2. made of

hs
3. linked to

4. affiliated to

Question ID : 26433098987

at
Status : Answered
Chosen Option : 3

M
Comprehension:
Read the given passage and answer the questions that follow.
Like we pollute the earth, we pollute the water; we also pollute the subtle environment
through our negative feelings and emotions. We have become a victim of our environment.
We are not in control of our mind. How to handle our mind? How to be in the present
moment? How to be happy and grateful? This we have not learnt. This is the most
ap
unfortunate thing. Then what is the solution? This is where we miss a very fundamental
principle that governs our environment, our mind, our emotions and our life in general.
Our body has the capacity to sustain much longer, the vibration of bliss and peace than it
does negative emotions because positivity is in the centre of our existence. Just like in the
structure of atom, protons and neutrons are in the centre of the atom and electrons are only
the periphery, same is with our lives; the centre core of our existence is bliss, positivity and
joy but it is surrounded by a cloud of negative ions. Through the help of the breath we can
at
easily get over our negative emotions in a short period of time.
This life has so much to offer to you. You can see this once you take some time off,
rejuvenating the soul. Your soul is hungry for a smile from you.
Everyone wants to be successful in life. But without knowing what is success, you want to be
successful. What is the sign of success? Because money gives you freedom so that you can
Pr

do whatever you want. You may have a big bank balance, but, you have stomach aches,
ulcers, you may have to go for the bypass surgery; can’t eat this, can’t do this, can’t do that.
Look at all those who claim to be successful-are they successful? No, they are miserable.
Then, what is sign of success? It is confidence, compassion, generosity and a smile that none
can snatch away, being really happy and being able to be more free. These are the signs of a
successful person.
Take some time off to look a little deep into yourself and calm the mind down. Thus, erasing
an

all the impressions that we are carrying in our minds and experience the presence, the divine
that is very core of our existence.

SubQuestion No : 21
Q.21 Select the most appropriate option that depicts the central theme of the passage.
Ans 1. Being happy and content are the keys to a successful life.
ag

2. Positive and negative emotions are a part of life.

3. Man has polluted life and environment both.

4. God is the supreme power.


G

Question ID : 264330121863
Status : Answered
Chosen Option : 1
Join @MathsByGaganPratap Telegram Channel for free PDFs
Comprehension:
Read the given passage and answer the questions that follow.
Like we pollute the earth, we pollute the water; we also pollute the subtle environment
through our negative feelings and emotions. We have become a victim of our environment.
We are not in control of our mind. How to handle our mind? How to be in the present
moment? How to be happy and grateful? This we have not learnt. This is the most
unfortunate thing. Then what is the solution? This is where we miss a very fundamental
principle that governs our environment, our mind, our emotions and our life in general.
Our body has the capacity to sustain much longer, the vibration of bliss and peace than it
does negative emotions because positivity is in the centre of our existence. Just like in the
structure of atom, protons and neutrons are in the centre of the atom and electrons are only
the periphery, same is with our lives; the centre core of our existence is bliss, positivity and
joy but it is surrounded by a cloud of negative ions. Through the help of the breath we can
easily get over our negative emotions in a short period of time.
This life has so much to offer to you. You can see this once you take some time off,
rejuvenating the soul. Your soul is hungry for a smile from you.

hs
Everyone wants to be successful in life. But without knowing what is success, you want to be
successful. What is the sign of success? Because money gives you freedom so that you can
do whatever you want. You may have a big bank balance, but, you have stomach aches,
ulcers, you may have to go for the bypass surgery; can’t eat this, can’t do this, can’t do that.
Look at all those who claim to be successful-are they successful? No, they are miserable.
Then, what is sign of success? It is confidence, compassion, generosity and a smile that none
can snatch away, being really happy and being able to be more free. These are the signs of a

at
successful person.
Take some time off to look a little deep into yourself and calm the mind down. Thus, erasing
all the impressions that we are carrying in our minds and experience the presence, the divine
that is very core of our existence.

M
SubQuestion No : 22
Q.22 Select the most appropriate ANTONYM for ‘Periphery’.
Ans 1. Troops

2. Centre ap
3. Clatter

4. Divine

Question ID : 264330121865
at
Status : Answered
Chosen Option : 2
Pr
an
ag
G
Join @MathsByGaganPratap Telegram Channel for free PDFs
Comprehension:
Read the given passage and answer the questions that follow.
Like we pollute the earth, we pollute the water; we also pollute the subtle environment
through our negative feelings and emotions. We have become a victim of our environment.
We are not in control of our mind. How to handle our mind? How to be in the present
moment? How to be happy and grateful? This we have not learnt. This is the most
unfortunate thing. Then what is the solution? This is where we miss a very fundamental
principle that governs our environment, our mind, our emotions and our life in general.
Our body has the capacity to sustain much longer, the vibration of bliss and peace than it
does negative emotions because positivity is in the centre of our existence. Just like in the
structure of atom, protons and neutrons are in the centre of the atom and electrons are only
the periphery, same is with our lives; the centre core of our existence is bliss, positivity and
joy but it is surrounded by a cloud of negative ions. Through the help of the breath we can
easily get over our negative emotions in a short period of time.
This life has so much to offer to you. You can see this once you take some time off,
rejuvenating the soul. Your soul is hungry for a smile from you.

hs
Everyone wants to be successful in life. But without knowing what is success, you want to be
successful. What is the sign of success? Because money gives you freedom so that you can
do whatever you want. You may have a big bank balance, but, you have stomach aches,
ulcers, you may have to go for the bypass surgery; can’t eat this, can’t do this, can’t do that.
Look at all those who claim to be successful-are they successful? No, they are miserable.
Then, what is sign of success? It is confidence, compassion, generosity and a smile that none
can snatch away, being really happy and being able to be more free. These are the signs of a

at
successful person.
Take some time off to look a little deep into yourself and calm the mind down. Thus, erasing
all the impressions that we are carrying in our minds and experience the presence, the divine
that is very core of our existence.

M
SubQuestion No : 23
Q.23 According to the passage, what is the human soul hungry for?
Ans 1. More wealth

2. Success ap
3. A positive feeling

4. A smile

Question ID : 264330121864
at
Status : Answered
Chosen Option : 4
Pr
an
ag
G
Join @MathsByGaganPratap Telegram Channel for free PDFs
Comprehension:
Read the given passage and answer the questions that follow.
Like we pollute the earth, we pollute the water; we also pollute the subtle environment
through our negative feelings and emotions. We have become a victim of our environment.
We are not in control of our mind. How to handle our mind? How to be in the present
moment? How to be happy and grateful? This we have not learnt. This is the most
unfortunate thing. Then what is the solution? This is where we miss a very fundamental
principle that governs our environment, our mind, our emotions and our life in general.
Our body has the capacity to sustain much longer, the vibration of bliss and peace than it
does negative emotions because positivity is in the centre of our existence. Just like in the
structure of atom, protons and neutrons are in the centre of the atom and electrons are only
the periphery, same is with our lives; the centre core of our existence is bliss, positivity and
joy but it is surrounded by a cloud of negative ions. Through the help of the breath we can
easily get over our negative emotions in a short period of time.
This life has so much to offer to you. You can see this once you take some time off,
rejuvenating the soul. Your soul is hungry for a smile from you.

hs
Everyone wants to be successful in life. But without knowing what is success, you want to be
successful. What is the sign of success? Because money gives you freedom so that you can
do whatever you want. You may have a big bank balance, but, you have stomach aches,
ulcers, you may have to go for the bypass surgery; can’t eat this, can’t do this, can’t do that.
Look at all those who claim to be successful-are they successful? No, they are miserable.
Then, what is sign of success? It is confidence, compassion, generosity and a smile that none
can snatch away, being really happy and being able to be more free. These are the signs of a

at
successful person.
Take some time off to look a little deep into yourself and calm the mind down. Thus, erasing
all the impressions that we are carrying in our minds and experience the presence, the divine
that is very core of our existence.

M
SubQuestion No : 24
Q.24 Select the option that is NOT true, according to the passage.
Ans 1. One should take out time to look deep into oneself.

2. Life has so much to offer. ap


3. Human beings are in control of their mind.

4. Money gives freedom to do whatever we want to do.

Question ID : 264330121866
at
Status : Answered
Chosen Option : 4
Pr
an
ag
G
Join @MathsByGaganPratap Telegram Channel for free PDFs
Comprehension:
Read the given passage and answer the questions that follow.
Like we pollute the earth, we pollute the water; we also pollute the subtle environment
through our negative feelings and emotions. We have become a victim of our environment.
We are not in control of our mind. How to handle our mind? How to be in the present
moment? How to be happy and grateful? This we have not learnt. This is the most
unfortunate thing. Then what is the solution? This is where we miss a very fundamental
principle that governs our environment, our mind, our emotions and our life in general.
Our body has the capacity to sustain much longer, the vibration of bliss and peace than it
does negative emotions because positivity is in the centre of our existence. Just like in the
structure of atom, protons and neutrons are in the centre of the atom and electrons are only
the periphery, same is with our lives; the centre core of our existence is bliss, positivity and
joy but it is surrounded by a cloud of negative ions. Through the help of the breath we can
easily get over our negative emotions in a short period of time.
This life has so much to offer to you. You can see this once you take some time off,
rejuvenating the soul. Your soul is hungry for a smile from you.

hs
Everyone wants to be successful in life. But without knowing what is success, you want to be
successful. What is the sign of success? Because money gives you freedom so that you can
do whatever you want. You may have a big bank balance, but, you have stomach aches,
ulcers, you may have to go for the bypass surgery; can’t eat this, can’t do this, can’t do that.
Look at all those who claim to be successful-are they successful? No, they are miserable.
Then, what is sign of success? It is confidence, compassion, generosity and a smile that none
can snatch away, being really happy and being able to be more free. These are the signs of a

at
successful person.
Take some time off to look a little deep into yourself and calm the mind down. Thus, erasing
all the impressions that we are carrying in our minds and experience the presence, the divine
that is very core of our existence.

M
SubQuestion No : 25
Q.25 Select the most suitable title for the passage.
Ans 1. Earth and Environment

2. Environment Pollution ap
3. Core of Human Existence

4. How to be Successful?

Question ID : 264330121862
at
Status : Answered
Chosen Option : 3
Pr
an
ag
G
Join @MathsByGaganPratap Telegram Channel for free PDFs

Phase-XI/2023/Selection Posts and Selection Posts/Ladakh/2023


Roll Number Gagan Pratap Maths
Candidate Name Gagan Pratap Maths
Venue Name iON Digital Zone iDZ Sirsakala Bhilai-3
Exam Date 28/06/2023
Exam Time 2:30 PM - 3:30 PM
Subject Selection Post Graduate Level

Section : General Intelligence

hs
Q.1 Which of the following numbers will replace the question mark (?) in the given series?
5, 8, 19, 36, 75, ?
Ans 1. 148

2. 146

at
3. 164

4. 184

Question ID : 264330143333

M
Status : Not Answered
Chosen Option : --

Q.2 In a certain code language, ‘FANG’ is written as ‘24’ and ‘CRAZY’ is written as ‘69’.How

Ans
will ‘PLAY’ be written in that language?
1. 52
ap
2. 50

3. 48
at
4. 45

Question ID : 264330143369
Status : Marked For Review
Pr

Chosen Option : 2

Q.3 Pointing to a man in a picture, Sunny said, “He is the brother of the husband of my
sister’s mother”. How is Sunny related to the man in the picture?
Ans 1. Grandfather
an

2. Nephew

3. Brother

4. Son
ag

Question ID : 26433070275
Status : Answered
Chosen Option : 2

Q.4 In a certain code language, ‘CLIMATE’ is written as ‘763’, ‘GLOBAL’ is written as ‘649’.
G

How will ‘CREEP’ be written as in that language?


Ans 1. 74

2. 475

3. 47

4. 547

Question ID : 264330122371
Status : Answered
Chosen Option : 4
Join @MathsByGaganPratap Telegram Channel for free PDFs
Q.5

Ans

1.

2.

hs
3.

at
M
4.

Question ID : 26433056949
ap Status : Answered
Chosen Option : 2

Q.6 ‘A # B’ means ‘A is the brother of B’


‘A @ B’ means ‘A is the father of B’
at
‘A & B’ means ‘A is the mother of B’
‘A % B’ means ‘A is the wife of B’
If ‘P & Q # R % S @ T # U’, then how is P related to U?
Ans 1. Mother
Pr

2. Mother’s mother

3. Father’s mother

4. Son’s daughter

Question ID : 264330143357
an

Status : Answered
Chosen Option : 2

Q.7 In a certain code language, ‘COLDER’ is written as ‘105’ and ‘MOUSE’ is written as ‘62’.
How will ‘PLIGHT’ be written in that language?
ag

Ans 1. 82

2. 90

3. 80

4. 102
G

Question ID : 26433069610
Status : Answered
Chosen Option : 2
Join @MathsByGaganPratap Telegram Channel for free PDFs
Q.8 Select the option that is related to the third word in the same way as the second word
is related to the first word.
(The words must be considered as meaningful English words and must not be related
to each other based on the number of letters/number of consonants/vowels in the
word)
Exclude : Include : : Elementary : ?
Ans 1. Beginners

2. Preliminary

3. Basic

4. Advanced

Question ID : 26433068409
Status : Answered

hs
Chosen Option : 4

Q.9 In this question, three statements are given, followed by two conclusions numbered I
and II. Assuming the statements to be true, even if they seem to be at variance with
commonly known facts, decide which of the conclusion(s) logically follows/follow

at
from the statements.
Statements:
Some flowers are fruits.
All fruits are vegetables.
Some vegetables are healthy.

M
Conclusions:
I. All fruits are healthy.
II. Some vegetables are flowers.
Ans 1. Only conclusion II follows.

2. Neither conclusion I nor II follows.

3. Both conclusions I and II follow.


ap
4. Only conclusion I follows.

Question ID : 264330143342
at
Status : Answered
Chosen Option : 1

Q.10 Select the option that represents the letters that, when sequentially placed from left to
Pr

right in the blanks below, will complete the letter series.


_AA_ F _AN FA_ _ _AAN
Ans 1. F A A F N A

2. A A F F N A

3. F A N A N F
an

4. F N A A N F

Question ID : 26433067729
Status : Answered
Chosen Option : 4
ag

Q.11 Select the option that is related to the third word in the same way as the second word
is related to the first word.(The words must be considered as meaningful English
words and must not be related to each other based on the number of letters/number of
consonants/vowels in the word)
Horror : Fright :: Joy : ?
G

Ans 1. Peace

2. Calm

3. Delight

4. Sadness

Question ID : 26433067985
Status : Answered
Chosen Option : 3
Join @MathsByGaganPratap Telegram Channel for free PDFs
Q.12 Select the option that represents the correct order of the given words as they would
appear in an English dictionary?
1. Dash
2. Damage
3. Darkness
4. Damp
5. Danger
6. Dancer
Ans 1. 2, 4, 6, 5, 3, 1

2. 4, 2, 6, 5, 3, 1

3. 2, 4, 1, 5, 3, 6

4. 2, 4, 5, 6, 3, 1

hs
Question ID : 264330117380
Status : Answered
Chosen Option : 1

Q.13 Select the correct mirror image of the given combination when the mirror is placed at

at
MN as shown.

M
Ans
1.
ap
2.
at

3.
Pr

4.

Question ID : 264330143446
Status : Answered
an

Chosen Option : 3

Q.14 Select the correct option that indicates the arrangement of the following words in a
logical and meaningful order.
1. Primary
2. Lower secondary
ag

3. Kindergarten
4. University
5. Upper Secondary
Ans 1. 3, 1, 2, 4, 5

2. 3, 2, 1, 5, 4
G

3. 3, 1, 2, 5, 4

4. 3, 1, 5, 2, 4

Question ID : 264330129559
Status : Answered
Chosen Option : 3
Join @MathsByGaganPratap Telegram Channel for free PDFs
Q.15 Select the correct combination of mathematical signs to sequentially replace the *
signs and balance the given equation.

5 * 14 * 11 * 9 * 10 * 82
Ans 1. ×, −, −, +, =

2. ×, ×, −, +, =

3. ×, +, −, +, =

4. ×, +, −, ×, =

Question ID : 26433097201
Status : Answered
Chosen Option : 3

hs
Q.16 Select the combination of letters that when sequentially placed in the blanks of the
given series will complete the series.
AC_E_A_DEF_CD_F
Ans 1. DCFAE

at
2. DFCAE

3. ADFCE

4. DCAEF

M
Question ID : 26433078822
Status : Answered
ap Chosen Option : 2

Q.17 Select the option that is related to the fifth letter-cluster in the same way as the second
letter-cluster is related to the first letter-cluster and the fourth letter-cluster is related
to the third letter-cluster.
CAMERA : MACARE :: BEYOND : YEBDNO :: DEBATE : ?
Ans 1. BEDEAT
at
2. BDETEA

3. BEDETA

4. BEDTEA
Pr

Question ID : 26433068518
Status : Answered
Chosen Option : 3
an

Q.18 In this question, three statements are given, followed by two conclusions numbered I
and II. Assuming the statements to be true, even if they seem to be at variance with
commonly known facts, decide which of the conclusions logically follows/follow from
the statements.

Statements:
I. No forest is a jungle.
ag

II. All hills are jungles.


III. All orchards are hills.

Conclusions:
I. All orchards are jungles.
II. No forest is a hill.
G

Ans 1. Only conclusion II follows

2. Both conclusions I and II follow

3. Neither conclusion I nor II follows

4. Only conclusion I follows

Question ID : 264330104067
Status : Answered
Chosen Option : 2
Join @MathsByGaganPratap Telegram Channel for free PDFs
Q.19

Ans

1.

hs
2.

at
M
3.

ap
4.
at

Question ID : 264330143437
Status : Answered
Pr

Chosen Option : 3

Q.20 Select the option that represents the letters that, when sequentially placed from left to
right in the blanks below, will complete the letter series.
F_LORFWL_R_W_ORF_LO_
Ans 1. F W O L R F
an

2. O W F L R W

3. L F O W R W

4. W O F L W R
ag

Question ID : 26433067733
Status : Answered
Chosen Option : 4

Q.21 Which of the following numbers will replace the question mark (?) in the given series?
G

4, 7, 17, 38, 72, 123, 193, 286, 408, ?


Ans 1. 521

2. 561

3. 512

4. 516

Question ID : 264330143334
Status : Answered
Chosen Option : 2
Join @MathsByGaganPratap Telegram Channel for free PDFs
Q.22

Ans

1.

hs
2.

at
3.

M
4.
ap
Question ID : 26433080448
Status : Answered
Chosen Option : 1
at

Q.23 In a certain code language, ‘SPECIAL’ is written as ‘KBJBFOR’ and 'PENSION' is


written as 'MPJRMFO'. How will ‘RELEASE’ be written as in that language?
Ans 1. FTBFKFQ
Pr

2. QFKFBRF

3. FRZFKFQ

4. FRBFKFQ
an

Question ID : 264330122202
Status : Marked For Review
Chosen Option : 4

Q.24 Select the option that represents the correct order of the given words as they would
ag

appear in an English dictionary.


1. Emigrant
2. Emissary
3. Embolden
4. Embezzle
5. Eminence
6. Empathize
G

7. Emphatic
Ans 1. 4,3,2,5,1,7,6

2. 4,3,2,1,6,7,5

3. 4,3,1,5,2,6,7

4. 4,2,3,1,5,6,7

Question ID : 26433097399
Status : Answered
Chosen Option : 3
Join @MathsByGaganPratap Telegram Channel for free PDFs
Q.25 In a certain code language, 'SINGER' is coded as '96' and 'CRACK' is coded as
'104'.How will 'JACKAL' be coded in that language?
Ans 1. 128

2. 122

3. 111

4. 130

Question ID : 264330143391
Status : Answered
Chosen Option : 4

hs
Section : General Awareness

Q.1 The Principal of a Sanskrit College who represented a blend of Indian and Western
culture was __________.
Ans 1. Raja Ram Mohan Roy 

at
2. Keshab Chandra Sen 

3. Ishwar Chandra Vidyasagar 

4. Debendranath Tagore 

M
Question ID : 264330132691
Status : Answered
Chosen Option : 3

Ans 1. Red sandstone


ap
Q.2 The Tomb of Razia Sultan in the state of Haryana is made of which building material?

2. Granite

3. Marble 
at
4. Baked bricks 

Question ID : 264330132428
Pr

Status : Answered
Chosen Option : 4

Q.3 Initially who among the following were subordinate to the Chalukyas of Karnataka?
Ans 1. Rashtrakutas
an

2. Satavahanas

3. Pratiharas

4. Palas
ag

Question ID : 26433089865
Status : Answered
Chosen Option : 1

Q.4 Shahid Parvez Khan, Budhaditya Mukherjee, Anushka Shankar and Hara Shankar
Bhattacharya are well known players of which of the following instruments?
G

Ans 1. Veena

2. Sitar 

3. Mohan veena

4. Surbahar

Question ID : 264330143394
Status : Not Answered
Chosen Option : --
Join @MathsByGaganPratap Telegram Channel for free PDFs
Q.5 What is the name of the evacuation operation conducted by the Government of India
to evacuate the Indian citizens amidst the 2022 Russian invasion of Ukraine?
Ans 1. Operation Ganga

2. Operation Eagle

3. Operation Tabas

4. Operation Aquarius

Question ID : 26433055032
Status : Answered
Chosen Option : 2

Q.6 Forest Research Institute, Dehradun was established as Imperial Forest Research

hs
Institute. In which year was it established as “Imperial Forest Research Institute”?
Ans 1. 1905

2. 1908

3. 1907

at
4. 1906

Question ID : 26433089209

M
Status : Not Answered
Chosen Option : --

Q.7 Which of the following players is associated with billiards?


Ans 1. Sourabh Chaudhari

2. Manika Batra
ap
3. Sankalp Gupta

4. Pankaj Advani
at
Question ID : 264330109667
Status : Answered
Chosen Option : 4
Pr

Q.8 Which of the following is NOT a constitutional body in India?


Ans 1. National human rights commission

2. Finance commission

3. State public service commission


an

4. National commission for SCs

Question ID : 264330111535
Status : Answered
ag

Chosen Option : 1

Q.9 Makaravilakku festival is celebrated in which of the following Indian states?


Ans 1. Andhra Pradesh

2. Tamil Nadu
G

3. Kerala

4. Karnataka

Question ID : 26433064690
Status : Not Answered
Chosen Option : --
Join @MathsByGaganPratap Telegram Channel for free PDFs
Q.10 According to Human Development Index (HDI) report 2021-22, out of all the
neighbouring countries of India, how many countries have got ranking in the ‘Very
High Human Development’ category of HDI?
Ans 1. 3

2. 1

3. 2

4. 0

Question ID : 264330109522
Status : Not Answered
Chosen Option : --

hs
Q.11 What is the strength of a hydrogen bond determined by?
Ans 1. Interaction between the lone-pair electrons of the electronegative atom of one
molecule and the hydrogen atom of another molecule
2. Interaction between the lone-pair electrons of the electropositive atom of one
molecule and the hydrogen atom of another molecule

at
3. Interaction between the lone-pair electrons of the electronegative atom of one
molecule and the oxygen atom of another molecule
4. Interaction between the lone-pair electrons of the polar molecule and the hydrogen
atom of another molecule

M
Question ID : 264330143313
Status : Answered
ap Chosen Option : 4

Q.12 Which disease is caused by excessive iron deposition within the reticuloendothelial
cells of the liver, spleen and bone marrow?
Ans 1. Paraesthesia

2. Hemosiderosis
at
3. Ariboflavinosis

4. Dermatitis 
Pr

Question ID : 26433092196
Status : Answered
Chosen Option : 2

Q.13 As of January 2023, who among the following is the Chairman of Khadi & Village
Industries Commission (KVIC) of India?
an

Ans 1. Manoj Kumar Singh

2. Dr. Shireesh Kedare

3. Manoj Kumar

4. Nagendra Raghuvanshi
ag

Question ID : 264330132604
Status : Not Answered
Chosen Option : --
G

Q.14 ‘Long Walk to Freedom’ is written by:


Ans 1. Aung San Suu Kyi

2. Mahatma Gandhi

3. Dalai Lama

4. Nelson Mandela

Question ID : 26433064622
Status : Answered
Chosen Option : 4
Join @MathsByGaganPratap Telegram Channel for free PDFs
Q.15 The Purva Mimansa school of philosophy was founded by __________ in ancient India.
Ans 1. Vyasa

2. Kapila

3. Patanjali

4. Jaimini

Question ID : 264330111366
Status : Answered
Chosen Option : 4

Q.16 Which danseuse among the following was called to perform for Queen Elizabeth-II’s
coronation festivities in 1953?

hs
Ans 1. Mrinalini Sarabhai

2. Sitara Devi

3. Rukmini Devi Arundale

at
4. Kamala Lakshmi Narayanan

Question ID : 264330110268
Status : Not Answered

M
Chosen Option : --

Q.17 The Padma Bhushan award in the field of sports was bestowed on ________ for the
year 2022.
Ans 1. Devendra Jhajharia

2. Sankarnarayana Menon Chundayil


ap
3. Brahmanand Sankhwalkar

4. Sumit Antil
at
Question ID : 26433086888
Status : Not Answered
Chosen Option : --
Pr

Q.18 Which of the following parts of India was affected by the Kuka movement?
Ans 1. Bihar

2. Jharkhand

3. Punjab
an

4. Bengal

Question ID : 26433089143
Status : Answered
ag

Chosen Option : 3

Q.19 The total female literacy in India as per the 2011 census was ______.
Ans 1.

2.
G

3.

4.

Question ID : 264330109512
Status : Answered
Chosen Option : 1
Join @MathsByGaganPratap Telegram Channel for free PDFs
Q.20 The High Court and Supreme Court Judges (Salaries and Conditions of Service)
Amendment Bill, 2021 was introduced in the Lok Sabha on ______________by the
Minister of Law and Justice, Kiren Rijiju.
Ans 1. November 2021

2. September 2021

3. October 2021

4. December 2021

Question ID : 264330122340
Status : Not Answered
Chosen Option : --

hs
Q.21 Which of the following ritual dances is performed by the Kamar tribe of Madhya
Pradesh?
Ans 1. Ghapal

2. Munari

at
3. Painka

4. Tertali

M
Question ID : 26433072630
Status : Not Answered
Chosen Option : --

Q.22 In which year was the Andhra Pradesh Reorganisation Act passed?
Ans 1. 2013
ap
2. 2012

3. 2014

4. 2015
at

Question ID : 264330113439
Status : Answered
Chosen Option : 3
Pr

Q.23 Which of the following rivers is NOT a tributary of the Indus river?
Ans 1. The Zaskar 

2. The Hunza 
an

3. The Rind

4. The Gilgit 

Question ID : 264330109865
Status : Not Answered
ag

Chosen Option : --

Q.24 Which water reservoir has been declared a ‘Ramsar site’ on the list of wetlands of
international importance in 2020?
Ans 1. Chilika Lake 
G

2. Sur Sarovar

3. Pong Dam Lake  

4. Sarsai Nawar Jheel 

Question ID : 264330143305
Status : Not Answered
Chosen Option : --
Join @MathsByGaganPratap Telegram Channel for free PDFs
Q.25 Plants that are adapted to live under plenty of sunlight are called:
Ans 1. xerophytes

2. heliophytes

3. neophytes

4. protophytes

Question ID : 26433066710
Status : Not Answered
Chosen Option : --

Section : Quantitative Aptitude

hs
Q.1 What is the side of a cube (in metres) in which an iron rod of maximum length 10
metres can be put?
Ans 1. 5.77

2. 7.57

at
3. 5.57

4. 7.75

M
Question ID : 26433081751
Status : Answered
Chosen Option : 1

Ans
ap
Q.2 Let M denote the mean proportional between 16 and 9 and let N denote the third
proportional of 9 and 15. Find the value of 5M +3N.
1. 300

2. 125

3. 135
at
4. 225

Question ID : 264330143417
Pr

Status : Answered
Chosen Option : 3

Q.3 The average weight of 10 parcels is 28.6 kg. The addition of a new parcel reduces the
average weight by 0.6 kg. What is the weight (in kg) of the new parcel?
Ans 1. 23
an

2. 21

3. 22

4. 20
ag

Question ID : 264330129390
Status : Answered
Chosen Option : 3

Q.4 Ramu can build a wall alone in 15 days, while Ravan can build it alone in 10 days.
G

Ramu worked for 5 days. Then Ravan joined Ramu for the next 3 days. How many
more days could Ramu take to build the remaining wall?
Ans 1. 7.5

2. 10.5

3. 2.5

4. 5.5

Question ID : 26433081608
Status : Answered
Chosen Option : 3
Join @MathsByGaganPratap Telegram Channel for free PDFs
Q.5 A car travels the first 180 km at a speed of 20 km/h. It covers the next 210 km at a
speed of 35 km/h. Find the average speed of the car (in km/h).
Ans 1. 15

2. 55

3. 27.5

4. 26

Question ID : 264330102322
Status : Answered
Chosen Option : 4

Q.6 A shopkeeper is incurring a loss of cost price of 4 pens while selling 20 pens to Ajay. If

hs
the shopkeeper had purchased 20 pens at 10% less price and had sold 20 pens to Ajay
at 25% more than the selling price, then how much percentage would he have gained
(rounded off to 1 decimal place)?
Ans 1. 8.0

2. 11.1

at
3. 10.0

4. 12.5

M
Question ID : 264330130299
Status : Not Answered
Chosen Option : --
ap
Q.7 A certain sum of money lent out at simple interest amounts ₹13,000 in 3 years and
₹15,000 in 5 years. Find the rate per cent per annum.
Ans 1.

2.
at
3.

4.
Pr

Question ID : 26433081034
Status : Answered
Chosen Option : 4

Q.8 If tan θ + sec θ = a , then what is sec θ?


an

Ans
1.

2.
ag

3.
G

4.

Question ID : 264330126224
Status : Answered
Chosen Option : 2
Join @MathsByGaganPratap Telegram Channel for free PDFs
Q.9 The simple interest on ₹2,460 will be ₹162 less than the simple interest on ₹3,000 at the
rate of 5% per annum simple interest. Find the time taken in years.
Ans 1. 4

2. 6

3. 5

4. 3

Question ID : 26433080187
Status : Answered
Chosen Option : 2

Q.10 Study the given bar-graph and answer the question that follows.

hs
A company in India exports ready-made clothes. The bar-graph indicates the export of
clothes (in crores of rupees) over 6 years from 2015 to 2020.

at
M
ap
Find the average export of clothes over the years 2015 to 2020 (in crores of rupees).
Ans 1. 5.4
at
2. 6.4

3. 5.2 

4. 6.2
Pr

Question ID : 264330129951
Status : Answered
Chosen Option : 2
an

Q.11 Arun is driving a car to cover a distance of 225.5 km in 660 minutes. What is his speed,
in km/h?
Ans 1. 21.5

2. 20.2

3. 22
ag

4. 20.5

Question ID : 264330103337
Status : Answered
Chosen Option : 4
G
Join @MathsByGaganPratap Telegram Channel for free PDFs
Q.12 If M is the mid-point of the side BC of ABC, and the area of ABM is 19 cm2, then the
area of ABC is:
Ans 1. 36 cm2

2. 42 cm2

3. 40 cm2

4. 38 cm2

Question ID : 26433083001
Status : Answered
Chosen Option : 4

hs
Q.13 During the previous year, the price of an electronic item increased by 10%, and this
year, the price of the same item decreased by 10%. If the price of the electronic item at
present is ₹66,000, then what was the price of the discussed electronic item in the
beginning of the previous year? (Rounded off to one decimal place)
Ans 1.

at
2.

3.

M
4.

Question ID : 26433080007
ap Status : Answered
Chosen Option : 3

Q.14 If 2P = 3Q = 4R = 5S, the find P : Q : R : S.


Ans 1. 30 : 20 : 12 : 15

2. 12 : 20 : 30 : 15
at
3. 20 : 30 : 15 : 12

4. 30 : 20 : 15 : 12
Pr

Question ID : 26433061667
Status : Answered
Chosen Option : 4
an
ag
G
Join @MathsByGaganPratap Telegram Channel for free PDFs
Q.15

hs
at
Ans 1. 1

2. 2

M
3. 4

4. 3

Question ID : 264330122700
ap Status : Answered
Chosen Option : 4

Q.16 A piece of land was sold to a person through two middlemen, each gaining 25%. If the
person purchased the land for ₹6,25,000, the original cost (in ₹) of the land was:
at
Ans 1. 4,25,000

2. 4,00,000

3. 3,50,000
Pr

4. 3,75,000

Question ID : 264330129412
Status : Answered
Chosen Option : 2
an

Q.17 If the seven digit number 965x475 is divisible by 9, then the value of x is:
Ans 1. 0

2. 6
ag

3. 2

4. 3

Question ID : 264330126166
Status : Answered
G

Chosen Option : 1
Join @MathsByGaganPratap Telegram Channel for free PDFs
Q.18 In ΔPQR, S and T are points on the sides PQ and PR, respectively, such that ΔPST is
similar to ΔPRQ. If m∠PQR = 47°, then find m∠STR.
Ans 1. 130°

2. 133°

3. 120°

4. 123°

Question ID : 264330131413
Status : Marked For Review
Chosen Option : 2

Q.19 A company is manufacturing three different products – Glass, Plate and Bowl. The

hs
number of Plates is double the number of Glasses. The number of Glasses
manufactured is twice the number of Bowls. What is the difference between the
number of Plates and the number of Bowls if the total units manufactured by the
company is 2380?
Ans 1. 1360

at
2. 680

3. 1120

4. 1020

M
Question ID : 26433058210
Status : Answered
ap Chosen Option : 4

Q.20
at

Ans
1.
Pr

2.

3.
an

4.
ag

Question ID : 264330126543
Status : Marked For Review
Chosen Option : 2

Q.21 If 2a + b = 10 and 2ab = 9, then one of the values of 2a − b is:


G

Ans 1. 8

2. 4

3. 6

4. 10

Question ID : 26433083019
Status : Answered
Chosen Option : 1
Join @MathsByGaganPratap Telegram Channel for free PDFs
Q.22 In a sale, a shop is giving discount as per the following scheme:
10% discount on clothes, 12% on groceries, 15% on stationery and 20% on footwear.
Anjali bought a packet of tea, a notebook and bathroom sleepers whose marked prices
are ₹450, ₹100 and ₹850, respectively. How much amount (in ₹) does she have to pay?
Ans 1. 1116

2. 1261

3. 1161

4. 1471

Question ID : 264330103326
Status : Answered
Chosen Option : 3

hs
Q.23

Ans
1.

at
2.

3.

M
4.

Question ID : 264330101341
ap Status : Answered
Chosen Option : 3

Q.24
at
Ans 1.

2.
Pr

3.

4.
an

Question ID : 26433080387
Status : Answered
Chosen Option : 1
ag

Q.25 The present age of a husband and wife is in the ratio 5 : 4. After 6 years their ages will
be in the ratio 6 : 5. At the time of their marriage the ratio of their ages was 4 : 3. In how
many years ago they were married?
Ans 1. 8 years

2. 10 years 
G

3. 6 years

4. 4 years

Question ID : 264330129977
Status : Answered
Chosen Option : 3

Section : English Language


Join @MathsByGaganPratap Telegram Channel for free PDFs
Q.1 Select the most appropriate option that can substitute the underlined segment in the
given sentence. If there is no need to substitute it, select ‘No substitution required’.

Ria sits right besides Shimona in the craft class.


Ans 1. beside

2. be side

3. besides to

4. No substitution required

Question ID : 26433060073
Status : Answered
Chosen Option : 4

hs
Q.2 Select the most appropriate ANTONYM of the underlined word in the given sentence.

The library sessions were conducive for healthy discussions.


Ans 1. Exclusive

at
2. Integrated

3. Invaluable 

M
4. Unfavourable

Question ID : 26433073872
Status : Answered
Chosen Option : 4
ap
Q.3 Select the option that can be used as a one-word substitute for the underlined part of
the given sentence.

Dogs are known for possession of the quality of being faithful.


at
Ans 1. honesty

2. genuineness

3. creditability
Pr

4. fidelity

Question ID : 26433072479
Status : Answered
Chosen Option : 1
an

Q.4 Select the sentence that has the most appropriate meaning of the given idiom.

Kill two birds with one stone


Ans 1. Today’s fishing session was a success as both the fishermen were working together.

2. He was so unlucky that he never won a single coin toss in the entire tournament.
ag

3. Maya decided to clean the house but at the end of the day, she realised it was a
great workout.
4. He ruined his chances of getting into college as he got distracted while studying for
two different entrance tests and not perfecting even a single one.
G

Question ID : 26433089734
Status : Answered
Chosen Option : 3
Join @MathsByGaganPratap Telegram Channel for free PDFs
Q.5 Select the most appropriate option to fill in the blank.

________ being an officer, he lives in a very tiny apartment.


Ans 1. Except  

2. Despite 

3. Instead  

4. Unlike  

Question ID : 26433060982
Status : Answered
Chosen Option : 2

hs
Q.6 Select the option that expresses the given sentence in direct speech.
The ticket collector asked the passengers where their tickets were.
Ans 1. The ticket collector said to the passengers, “Where your tickets were?”

2. The ticket collector said to the passengers, “Where were your tickets?”

at
3. The ticket collector demanded the passengers, “Show me your tickets”

4. The ticket collector said to the passengers, “Where are your tickets?”

M
Question ID : 264330132827
Status : Answered
Chosen Option : 4

Q.7 Select the correct indirect narration of the given sentence.

Ans
He said to me, “My phone is switched off.”
ap
1. He told me that his phone was switched off.

2. He told me that our phone had switch off.

3. He told me that his phone was switch off.


at
4. He told me that my phone was switched off.

Question ID : 264330132786
Status : Answered
Pr

Chosen Option : 1

Q.8 Select the most appropriate synonym for the highlighted word.
There were a plethora of options available to the applicant.
Ans 1. destructive
an

2. recess

3. excess

4. insufficient
ag

Question ID : 264330120719
Status : Answered
Chosen Option : 3

Q.9 The following sentence has been split into four segments. Identify the segment that
G

contains a grammatical error.

Kids learn / a lot by / their / school activities and homework.


Ans 1. Kids learn

2. school activities and homework.

3. their

4. a lot by

Question ID : 26433096146
Status : Answered
Chosen Option : 4
Join @MathsByGaganPratap Telegram Channel for free PDFs
Q.10 Select the most appropriate ANTONYM of the underlined word in the following
sentence.

Suraj has always shown a willingness to accept and respect the differences in others'
lives with impartiality and avoid prejudice.
Ans 1. Willingness

2. Impartiality

3. Avoid

4. Differences

Question ID : 26433073642
Status : Answered
Chosen Option : 2

hs
Q.11 Select the grammatically correct sentence.

A.I would like to spend an evening near pool with a cup of coffee.
B.I would like to spend an evening near pool with cup of coffee.

at
C.I would like to spend a evening near a pool with a cup of coffee.
D.I would like to spend an evening near a pool with a cup of coffee.
Ans 1. C

2. D

M
3. B

4. A
ap Question ID : 26433073536
Status : Answered
Chosen Option : 2

Q.12 Select the most appropriate option to fill in the blank.


That night the dinner was special, and the _________ himself came out to read out the
at
menu list.
Ans 1. servant

2. public
Pr

3. guest

4. chef

Question ID : 26433098880
Status : Answered
an

Chosen Option : 4

Q.13 Select the most appropriate meaning of the given idiom.

A cock and bull story


ag

Ans 1. A story of animals

2. A foolish and concocted story

3. A true story

4. A children’s story
G

Question ID : 26433088140
Status : Marked For Review
Chosen Option : 2
Join @MathsByGaganPratap Telegram Channel for free PDFs
Q.14 Select the INCORRECTLY spelt word.
Ans 1. Psychic

2. Though

3. Truthfully

4. Suddeness

Question ID : 26433088413
Status : Answered
Chosen Option : 4

Q.15 Select the option that expresses the given sentence in active voice.
Why was such a letter written by your brother?

hs
Ans 1. Why is your brother writing such a letter? 

2. Why has your brother written such a letter? 

3. Why did your brother write such a letter? 

at
4. Why your brother wrote such a letter?

Question ID : 264330133302
Status : Answered

M
Chosen Option : 3

Comprehension:
In the following passage, some words have been deleted. Read the passage carefully and
select the most appropriate option to fill in each blank.
ap
The earliest kind of print technology was developed in China. This was a system of hand
(1)_____. From AD 594 onwards, (2)______ in China were printed by (3)______ paper,
against the inked surface (4)______ woodblocks. As both sides of (5)______ thin, porous
sheet could not be printed, the traditional Chinese ‘accordion book’ was folded and stitched at
the side.
at
SubQuestion No : 16
Q.16 Select the most appropriate option for blank number 1.
Ans 1. writing

2. inking
Pr

3. reading

4. printing

Question ID : 264330132855
Status : Answered
an

Chosen Option : 2

Comprehension:
In the following passage, some words have been deleted. Read the passage carefully and
select the most appropriate option to fill in each blank.
ag

The earliest kind of print technology was developed in China. This was a system of hand
(1)_____. From AD 594 onwards, (2)______ in China were printed by (3)______ paper,
against the inked surface (4)______ woodblocks. As both sides of (5)______ thin, porous
sheet could not be printed, the traditional Chinese ‘accordion book’ was folded and stitched at
the side.

SubQuestion No : 17
G

Q.17 Select the most appropriate option for blank number 2.


Ans 1. page

2. books

3. paper

4. book

Question ID : 264330132856
Status : Answered
Chosen Option : 2
Join @MathsByGaganPratap Telegram Channel for free PDFs
Comprehension:
In the following passage, some words have been deleted. Read the passage carefully and
select the most appropriate option to fill in each blank.
The earliest kind of print technology was developed in China. This was a system of hand
(1)_____. From AD 594 onwards, (2)______ in China were printed by (3)______ paper,
against the inked surface (4)______ woodblocks. As both sides of (5)______ thin, porous
sheet could not be printed, the traditional Chinese ‘accordion book’ was folded and stitched at
the side.

SubQuestion No : 18
Q.18 Select the most appropriate option for blank number 3.
Ans 1. write

2. rub

3. rubbing

hs
4. writing

Question ID : 264330132857
Status : Answered

at
Chosen Option : 3

Comprehension:
In the following passage, some words have been deleted. Read the passage carefully and

M
select the most appropriate option to fill in each blank.
The earliest kind of print technology was developed in China. This was a system of hand
(1)_____. From AD 594 onwards, (2)______ in China were printed by (3)______ paper,
against the inked surface (4)______ woodblocks. As both sides of (5)______ thin, porous
sheet could not be printed, the traditional Chinese ‘accordion book’ was folded and stitched at
the side.

SubQuestion No : 19
ap
Q.19 Select the most appropriate option for blank number 4.
Ans 1. of

2. in
at
3. by

4. from
Pr

Question ID : 264330132858
Status : Answered
Chosen Option : 1

Comprehension:
In the following passage, some words have been deleted. Read the passage carefully and
an

select the most appropriate option to fill in each blank.


The earliest kind of print technology was developed in China. This was a system of hand
(1)_____. From AD 594 onwards, (2)______ in China were printed by (3)______ paper,
against the inked surface (4)______ woodblocks. As both sides of (5)______ thin, porous
sheet could not be printed, the traditional Chinese ‘accordion book’ was folded and stitched at
the side.
ag

SubQuestion No : 20
Q.20 Select the most appropriate option for blank number 5.
Ans 1. from

2. by
G

3. a

4. the

Question ID : 264330132859
Status : Answered
Chosen Option : 4
Join @MathsByGaganPratap Telegram Channel for free PDFs
Comprehension:
Read the given passage and answer the questions that follow.
Satyajit Ray's films are rich in sociological elements which portray themes like faulty social
structures, childhood experiences, poverty-stricken India, rural life, complexities in
relationships and politics, religion, gender and on. Films like Devi or The Goddess and
Mahapurush or The Godman show how religion becomes a breeding ground of superstitions
and how the blind faith of people in ‘Godmen’, without analysing the reality, leave people
trapped in the end. In Pather Panchali or Song of the Road, Ray shows how Apu and Durga
enjoy their childhood despite their poverty. The innocence and mirth of childhood offers a
sharp contrast with the disconsolate faces of other adults in the family who are affected by the
present harsh situations. A critique of autocracy is found in his film Hirak Rajar Deshe or In
the Country of the Diamond King which is an overall political satire. Although Satyajit Ray's
films are deeply rooted in Bengali culture, they have a universal significance, and much of it is
perhaps because his film making is heavily inspired by neo-realist principles.

SubQuestion No : 21

hs
Q.21 Which of the following is NOT a theme of any of Satyajit Ray’s movies, according to
the passage?
Ans 1. Motivational life stories

2. Rural life

at
3. Faulty social structures 

4. Complexities in relationships

Question ID : 264330134176

M
Status : Not Answered
Chosen Option : --

Comprehension: ap
Read the given passage and answer the questions that follow.
Satyajit Ray's films are rich in sociological elements which portray themes like faulty social
structures, childhood experiences, poverty-stricken India, rural life, complexities in
relationships and politics, religion, gender and on. Films like Devi or The Goddess and
Mahapurush or The Godman show how religion becomes a breeding ground of superstitions
and how the blind faith of people in ‘Godmen’, without analysing the reality, leave people
trapped in the end. In Pather Panchali or Song of the Road, Ray shows how Apu and Durga
at
enjoy their childhood despite their poverty. The innocence and mirth of childhood offers a
sharp contrast with the disconsolate faces of other adults in the family who are affected by the
present harsh situations. A critique of autocracy is found in his film Hirak Rajar Deshe or In
the Country of the Diamond King which is an overall political satire. Although Satyajit Ray's
films are deeply rooted in Bengali culture, they have a universal significance, and much of it is
Pr

perhaps because his film making is heavily inspired by neo-realist principles.

SubQuestion No : 22
Q.22 Which film by Satyajit Ray deals with the theme of autocracy?
Ans 1. Pather Panchali

2. Devi 
an

3. Hirak Rajar Deshe

4. Mahapurush

Question ID : 264330134173
ag

Status : Answered
Chosen Option : 3
G
Join @MathsByGaganPratap Telegram Channel for free PDFs
Comprehension:
Read the given passage and answer the questions that follow.
Satyajit Ray's films are rich in sociological elements which portray themes like faulty social
structures, childhood experiences, poverty-stricken India, rural life, complexities in
relationships and politics, religion, gender and on. Films like Devi or The Goddess and
Mahapurush or The Godman show how religion becomes a breeding ground of superstitions
and how the blind faith of people in ‘Godmen’, without analysing the reality, leave people
trapped in the end. In Pather Panchali or Song of the Road, Ray shows how Apu and Durga
enjoy their childhood despite their poverty. The innocence and mirth of childhood offers a
sharp contrast with the disconsolate faces of other adults in the family who are affected by the
present harsh situations. A critique of autocracy is found in his film Hirak Rajar Deshe or In
the Country of the Diamond King which is an overall political satire. Although Satyajit Ray's
films are deeply rooted in Bengali culture, they have a universal significance, and much of it is
perhaps because his film making is heavily inspired by neo-realist principles.

SubQuestion No : 23

hs
Q.23 According to the passage, which of the following was a major inspiration for Satyajit
Ray?
Ans 1. Neo realism

2. The society

at
3. Comics

4. Early Bengali cinema

Question ID : 264330134174

M
Status : Answered
Chosen Option : 1

Comprehension: ap
Read the given passage and answer the questions that follow.
Satyajit Ray's films are rich in sociological elements which portray themes like faulty social
structures, childhood experiences, poverty-stricken India, rural life, complexities in
relationships and politics, religion, gender and on. Films like Devi or The Goddess and
Mahapurush or The Godman show how religion becomes a breeding ground of superstitions
and how the blind faith of people in ‘Godmen’, without analysing the reality, leave people
trapped in the end. In Pather Panchali or Song of the Road, Ray shows how Apu and Durga
at
enjoy their childhood despite their poverty. The innocence and mirth of childhood offers a
sharp contrast with the disconsolate faces of other adults in the family who are affected by the
present harsh situations. A critique of autocracy is found in his film Hirak Rajar Deshe or In
the Country of the Diamond King which is an overall political satire. Although Satyajit Ray's
films are deeply rooted in Bengali culture, they have a universal significance, and much of it is
Pr

perhaps because his film making is heavily inspired by neo-realist principles.

SubQuestion No : 24
Q.24 Which film by Satyajit Ray deals with the portrayal of childhood?
Ans 1. Pather Panchali

2. Hirak Rajar Deshe


an

3. Mahapurush

4. Devi

Question ID : 264330134175
ag

Status : Answered
Chosen Option : 1
G
Join @MathsByGaganPratap Telegram Channel for free PDFs
Comprehension:
Read the given passage and answer the questions that follow.
Satyajit Ray's films are rich in sociological elements which portray themes like faulty social
structures, childhood experiences, poverty-stricken India, rural life, complexities in
relationships and politics, religion, gender and on. Films like Devi or The Goddess and
Mahapurush or The Godman show how religion becomes a breeding ground of superstitions
and how the blind faith of people in ‘Godmen’, without analysing the reality, leave people
trapped in the end. In Pather Panchali or Song of the Road, Ray shows how Apu and Durga
enjoy their childhood despite their poverty. The innocence and mirth of childhood offers a
sharp contrast with the disconsolate faces of other adults in the family who are affected by the
present harsh situations. A critique of autocracy is found in his film Hirak Rajar Deshe or In
the Country of the Diamond King which is an overall political satire. Although Satyajit Ray's
films are deeply rooted in Bengali culture, they have a universal significance, and much of it is
perhaps because his film making is heavily inspired by neo-realist principles.

SubQuestion No : 25

hs
Q.25 According to the passage, how has Mahapurush been translated?
Ans 1. Great man

2. The one man

3. Godman

at
4. Great being

Question ID : 264330134172
Status : Answered

M
Chosen Option : 3

ap
at
Pr
an
ag
G
Join @MathsByGaganPratap Telegram Channel for free PDFs

Phase-XI/2023/Selection Posts and Selection Posts/Ladakh/2023


Roll Number Gagan Pratap Maths
Candidate Name Gagan Pratap Maths
Venue Name Oriental Association For Education And Research
Exam Date 28/06/2023
Exam Time 5:15 PM - 6:15 PM
Subject Selection Post Matriculation Level

Section : General Intelligence

hs
Q.1

at
Ans

M
1.
ap
2.
at
Pr

3.
an

4.
ag

Question ID : 264330143267
Status : Answered
Chosen Option : 2
G
Join @MathsByGaganPratap Telegram Channel for free PDFs
Q.2

Ans

1.

hs
at
M
2.

ap
at
3.
Pr
an

4.
ag

Question ID : 264330143273
Status : Answered
G

Chosen Option : 4
Join @MathsByGaganPratap Telegram Channel for free PDFs
Q.3

Ans 1.

2.

3.

4.

hs
Question ID : 264330143262
Status : Answered
Chosen Option : 3

at
Q.4 Select the option that indicates the correct arrangement of the given words in the
order in which they appear in an English dictionary.
1. Reduce
2. Refer
3. Redress
4. Reedy

M
5. Redeem
Ans 1. 5, 1, 4, 3, 2

2. 4, 3, 5, 2, 1

3. 5, 3, 1, 4, 2

4. 4, 5, 1, 2, 3
ap
Question ID : 264330126647
Status : Answered
at
Chosen Option : 3

Q.5 In this question, three statements are given, followed by two conclusions numbered I
and II. Assuming the statements to be true, even if they seem to be at variance with
Pr

commonly known facts, decide which of the conclusions logically follows/follow from
the statements.

Statements:
All candies are gems.
All gems are cakes.
Some cakes are sweets.
an

Conclusions:
I. All candies are cakes.
II. Some gems are sweets.
Ans 1. Neither conclusion I nor II follows.

2. Both conclusions I and II follow.


ag

3. Only conclusion II follows.

4. Only conclusion I follows.

Question ID : 26433070377
G

Status : Answered
Chosen Option : 4
Join @MathsByGaganPratap Telegram Channel for free PDFs
Q.6 ‘A # B’ means ‘A is the father of B’.
‘A @ B’ means ‘A is the brother of B’.
‘A & B’ means ‘A is the husband of B’.
‘A % B’ means ‘A is the mother of B’.
If D # L @ M & N % P @ T, then how is L related to P?
Ans 1. Brother’s son

2. Father’s brother

3. Father’s brother’s wife

4. Wife’s brother 

Question ID : 26433091635
Status : Answered
Chosen Option : 2

hs
Q.7 Select the correct combination of mathematical signs to sequentially replace the *
signs and to balance the given equation.
59 * 11 * 11 * 48 * 1 * 12
Ans 1. –, ÷, +, ×, =

at
2. –, +, ÷, ×, =

3. +, ÷, –, ×, =

M
4. ×, –, +, ÷, =

Question ID : 264330142885
Status : Answered
Chosen Option : 3
ap
Q.8 Select the option that represents the correct order of the given words as they would
appear in an English dictionary.

1. Base
2. Barrier
at
3. Basin
4. Basic
5. Batter
6. Battle
Pr

Ans 1. 2, 1, 4, 3, 6, 5

2. 2, 1, 3, 4, 6, 5

3. 2, 4, 3, 6, 5, 1

4. 2, 1, 4, 3, 5, 6
an

Question ID : 264330142923
Status : Answered
Chosen Option : 4

Q.9 ‘A + B’ means ‘A is the sister of B’.


ag

‘A – B’ means ‘A is the mother of B’.


‘A × B’ means ‘A is the husband of B’.
‘A ÷ B’ means ‘A is the father-in-law of B’.
if U ÷ V – W + X × Y, then which of the following statements is NOT correct?
Ans 1. W is the granddaughter of U.
G

2.  Y is the sister of W . 

3.  V is the mother of X.

4. W is the sister-in-law of Y. 

Question ID : 26433095990
Status : Answered
Chosen Option : 2
Join @MathsByGaganPratap Telegram Channel for free PDFs
Q.10 Select the option that is related to the third word in the same way as the second word
is related to the first word. (The words must be considered as meaningful English
words and must NOT be related to each other based on the number of letters/number
of consonants/vowels in the word)

Warrior : Battlefield :: Gambler : ?


Ans 1. Field

2. Casino

3. Factory

4. Garage

Question ID : 264330142867
Status : Answered

hs
Chosen Option : 2

Q.11 In a certain code language, ‘CARRY’ is written as ‘DBSSZ’ and ‘DELAY’ is written as
‘EFMBZ’. How will ‘FAITH’ be written in that language?
Ans 1. GCKVJ 

at
2. HBKUJ 

3. GBJUI 

M
4. KCJVI 

Question ID : 26433056559
Status : Answered
Chosen Option : 3
ap
Q.12 Select the option that indicates the correct arrangement of the given words in a logical
and meaningful order.
1. Fortnight
2. Year
3. Month
at
4. Decade
5. Week
Ans 1. 2, 4, 5, 3, 1

2. 4, 2, 3, 1, 5
Pr

3. 2, 4, 3, 5, 1

4. 4, 2, 3, 5, 1

Question ID : 264330125996
an

Status : Answered
Chosen Option : 2

Q.13 Select the option that is related to the fifth term in the same way as the second term is
related to the first term and the fourth term is related to the third term.
CORRECT : ROCRTCE :: WRONGLY : ORWNYLG :: NEUTRAL : ?
ag

Ans 1. RALTNEW

2. UENTLAR

3. LARTUEN

4. ENUTRLA
G

Question ID : 264330142845
Status : Answered
Chosen Option : 2
Join @MathsByGaganPratap Telegram Channel for free PDFs
Q.14 Select the option that is related to the fourth term in the same way as the first term is
related to the second term and the fifth term is related to the sixth term
7 : 58 :: ? : 370 :: 12 : 153
Ans 1. 18

2. 21

3. 20

4. 19 

Question ID : 26433040802
Status : Answered
Chosen Option : 4

hs
Q.15 Three statements are given, followed by three conclusions numbered I, II and III.
Assuming the statements to be true, even if they seem to be at variance with
commonly known facts, decide which of the conclusions logically follow(s) from the
statements.

Statements:

at
All bottles are buildings.
Some buildings are short.
Some short are girls.

Conclusions:

M
I. Some buildings are bottles.
II. All buildings are short.
III. Some girls are short.
Ans 1. Only conclusions II and III follow

2. Only conclusions I and III follow

3. All the conclusions follow


ap
4. Only conclusions I and II follow

Question ID : 26433092645
at
Status : Answered
Chosen Option : 2

Q.16 Select the option that is related to the fourth term in the same way as the first term is
Pr

related to the second term and the fifth term is related to the sixth term
28 : 841 :: ? : 324 :: 2 : 9
Ans 1.  22

2.  7

3.  19
an

4.  17

Question ID : 26433040800
Status : Answered
Chosen Option : 4
ag

Q.17 Which letter-cluster will replace the question mark (?) and complete the given series?
BIRD, DMTG, ?, HUXM, JYZP
Ans 1. FRVJ
G

2. FQWJ

3. FQVJ

4. FRWJ

Question ID : 264330142910
Status : Answered
Chosen Option : 3
Join @MathsByGaganPratap Telegram Channel for free PDFs
Q.18 Which of the following letter-clusters will replace the question mark (?) in the given
series?
BW, DV, ET, GS, HQ, JP, ?
Ans 1. JN

2. MN

3. LN

4. KN

Question ID : 26433057204
Status : Answered
Chosen Option : 4

hs
Q.19 Select the option that represents the letters that, when sequentially placed from left to
right in the blanks below, will complete the letter series.
_BC_C_A_CDC_AB_D_B
Ans 1. ADBBBCC

2. ADBBBCA

at
3. DBDBBCC

4. AABBCCD

M
Question ID : 26433069624
Status : Answered
Chosen Option : 1
ap
Q.20 Select the option that is related to the fifth term in the same way as the second term is
related to the first term and the fourth term is related to the third term.
HISTORY : HIOTSRY :: FEATURE : FEUTARE :: JOURNAL : ?
Ans 1. JORUNAL

2. JORNUAL
at
3. JONRUAL

4. JOUNRAL
Pr

Question ID : 264330142838
Status : Answered
Chosen Option : 3

Q.21
an

Ans 1. ÷, +, =, −, ×, +, ÷

2. ×, +, −, +, ×, =, ÷

3. +, −, +, ÷, =, ×, +
ag

4. ×, +, =, +, ×, −, ÷

Question ID : 26433057656
Status : Answered
Chosen Option : 3
G
Join @MathsByGaganPratap Telegram Channel for free PDFs
Q.22 ‘A # B’ means ‘A is the brother of B’.
‘A @ B’ means ‘A is the daughter of B’.
‘A & B’ means ‘A is the husband of B’.
‘A % B’ means ‘A is the wife of B’.
If D @ N @ H & Y @ F % V, then how is Y related to D?
Ans 1. Father’s sister

2. Mother

3. Husband’s mother

4. Mother’s mother

Question ID : 264330142906
Status : Answered
Chosen Option : 4

hs
Q.23 Select the option that is related to the third word in the same way as the second word
is related to the first word. (The words must be considered as meaningful English
words and must NOT be related to each other based on the number of letters/number
of consonants/vowels in the word)

at
Pen : Write :: Shovel : ?
Ans 1. Scoop

2. Feed

M
3. Amplify

4. View
ap Question ID : 264330142868
Status : Answered
Chosen Option : 4

Q.24 Three Statements are given followed by Three conclusions numbered I, II and III.
Assuming the statements to be true, even if they seem to be at variance with
at
commonly known facts, decide which of the conclusions logically follow(s) from the
statements.

Statements:
All dogs are cats.
Pr

No bat is a cat.
Some rats are bats.

Conclusions:
I. Some cats are rats.
II. Some dogs are bats.
III. No dog is a bat.
an

Ans 1. Both conclusions I and II follow.

2. Both conclusions I and III follow.

3. Only conclusion I follows.

4. Only conclusion III follows.


ag

Question ID : 264330142875
Status : Answered
Chosen Option : 4
G
Join @MathsByGaganPratap Telegram Channel for free PDFs
Q.25 Select the option that represents the letters that, when sequentially placed from left to
right in the blanks below, will complete the letter series.

_ T _ OE _ U _ E _ _ O_ _UO
Ans 1. E U T O T U E U

2. E U T U T U E T

3. E U E O T U E T

4. E U T O T U E T

Question ID : 264330142852
Status : Answered
Chosen Option : 4

hs
Section : General Awareness

Q.1 Which of the following statements is INCORRECT about the characteristic features of
the Indian Desert?

at
Ans 1. It is also known as Marusthali.

2. It lies northwest of the Aravalli hills.

3. It receives 250 mm rainfall per year.

M
4. It has sand dunes.

Question ID : 264330142781
ap Status : Answered
Chosen Option : 3

Q.2 In December 2021, who became the 3rd fastest Indian fast bowler to pick up 200 Test
wickets?
Ans 1. Ishant Sharma
at
2. Jasprit Bumrah

3. Bhuvneshwar Kumar

4. Mohammed Shami
Pr

Question ID : 264330142689
Status : Answered
Chosen Option : 4
an

Q.3 In which of the following states is the Tamu Lachar festival celebrated as the New Year
by the Gurung Community?
Ans 1. Goa

2. Assam

3. Sikkim
ag

4. Madhya Pradesh

Question ID : 264330141231
Status : Answered
G

Chosen Option : 3
Join @MathsByGaganPratap Telegram Channel for free PDFs
Q.4 In 1948, Ernest Mackay mentioned that in the Harappan city of Lothal, the drains for
the drainage system were made of _______ bricks.
Ans 1. red

2. burnt

3. mud

4. sandstone

Question ID : 26433072652
Status : Answered
Chosen Option : 2

Q.5 In hockey, if the umpire indicates the direction with one arm raised horizontally, it

hs
means _______.
Ans 1. Goal scored

2. Bully

3. Timing

at
4. Free hit

Question ID : 26433054701

M
Status : Answered
Chosen Option : 4

Q.6 Who among the following social reformers of British India is known as Lokhitwadi?
Ans 1. Balshastri Jambhekar

2. Daboda Pandurang
ap
3. Gopal Ganesh Agarkar

4. Gopal Hari Deshmukh


at
Question ID : 26433065570
Status : Not Answered
Chosen Option : --
Pr

Q.7 The ________ is an embryonic midline structure common to all members of the
phylum Chordata that serves as a source of midline signals to surrounding tissues
and as a major skeletal element of the developing embryo.
Ans 1. notochord

2. nerve cord
an

3. tail fin

4. anus

Question ID : 264330142713
ag

Status : Not Answered


Chosen Option : --

Q.8 Which of the following is NOT an example of a minor industrial region of India?
Ans 1. Ambala-Amritsar region
G

2. Durg-Raipur region

3. Hugli region

4. Northern Malabar region

Question ID : 264330142798
Status : Answered
Chosen Option : 2
Join @MathsByGaganPratap Telegram Channel for free PDFs
Q.9 Who among the following was conferred with the Major Dhyan Chand Khel Ratna
Award 2021?
Ans 1. Pramod Bhagat

2. Sharad Kumar

3. Harvinder Singh

4. Amit Rohidas

Question ID : 26433072906
Status : Answered
Chosen Option : 1

Q.10 Hideki Yukawa, who received the Nobel Prize in 1949, is well known for which

hs
discovery?
Ans 1. Thermal ionization

2. Cascade process of cosmic radiation

3. Theory of nuclear forces

at
4. Measurement of electronic charge

Question ID : 264330142737

M
Status : Not Answered
Chosen Option : --

Q.11 Which Indian state launched the CLAP Mission on 2nd October 2021?
ap
Ans 1. Arunachal Pradesh

2. Uttar Pradesh

3. Andhra Pradesh

4. Himachal Pradesh
at
Question ID : 26433086870
Status : Not Answered
Chosen Option : --
Pr

Q.12 Who among the following won the ‘World Choreography Award 2020’ ?
Ans 1. Geeta Kapoor

2. Prabhu Deva

3. Suresh Mukund
an

4. Farah Khan

Question ID : 26433097102
Status : Answered
ag

Chosen Option : 3

Q.13 Who among the following has been credited with taking the shehnai from the marriage
mandap to the concert hall?
Ans 1. Ali Ahmed Hussain Khan
G

2. Ustad Bismillah Khan

3. Rajendra Prasanna

4. Anant Lal

Question ID : 264330141394
Status : Answered
Chosen Option : 2
Join @MathsByGaganPratap Telegram Channel for free PDFs
Q.14 In which year did the Election Commission of India for the first time in its history
become multimember body?
Ans 1.  2004

2.  2014

3.  1995

4. 1989 

Question ID : 264330114492
Status : Not Answered
Chosen Option : --

Q.15 Raichur doab, the land between Krishna and Tungabhadra was a reason of conflict

hs
between the kings of Vijayanagar and ________.
Ans 1. Malwa

2. Golconda

3. Bahamani

at
4. Bengal

Question ID : 26433081942

M
Status : Answered
Chosen Option : 3

Q.16 Who among the following announced the ‘Sushma Swaraj Award’ for women in the
state budget in 2022?
Ans 1.  Ashok Gehlot
ap
2.  Yogi Adityanath

3.  Manohar Lal Khattar

4.  Amarinder Singh
at

Question ID : 26433061838
Status : Not Answered
Chosen Option : --
Pr

Q.17 Pottery was first traceable to which period of ancient Indian history?
Ans 1. Chalcolithic

2. Palaeolithic
an

3. Neolithic

4. Mesolithic

Question ID : 264330108991
Status : Answered
ag

Chosen Option : 1

Q.18 As per the 2011 Census, what percentage of the total population in India lives in rural
areas?
Ans 1. 68.8%
G

2. 58.8%

3. 66.8%

4. 67.8%

Question ID : 264330142775
Status : Not Answered
Chosen Option : --
Join @MathsByGaganPratap Telegram Channel for free PDFs
Q.19 Which is a multicellular filamentous green alga consisting of thin unbranched chains
of cylindrical cells and found in floating masses near the surface of streams and
ponds?
Ans 1. Ectocarpus

2. Laminaria

3. Spirogyra

4. Chlorella

Question ID : 264330142711
Not Attempted and
Status :
Marked For Review
Chosen Option : --

hs
Q.20 Surupa Sen is a famous _________ dancer.
Ans 1. Kathak

2. Kathakali

at
3. Odissi

4. Bharatanatyam

M
Question ID : 264330141228
Status : Not Answered
Chosen Option : --

Q.21 ‘How I Became a Hindu’ is an autobiography of _______.


Ans 1. Paramahansa Yogananda
ap
2. Kiran Bedi

3. Kamala Surayya

4. Sita Ram Goel


at

Question ID : 264330141362
Status : Answered
Chosen Option : 4
Pr

Q.22 ‘Dzongkha’ is the official language of which country?


Ans 1. Myanmar

2. Maldives
an

3. Bangladesh

4. Bhutan

Question ID : 264330113310
Status : Not Answered
ag

Chosen Option : --

Q.23 Some of Swami Dayanand’s followers started a network of schools and colleges called
D.A.V. What does ‘A’ stand for in D.A.V.?
Ans 1. Author
G

2. Angel

3. Anglo

4. Accept

Question ID : 26433082080
Status : Answered
Chosen Option : 3
Join @MathsByGaganPratap Telegram Channel for free PDFs
Q.24 Which form of government is established by the Constitution of India both, at the
centre and states?
Ans 1. Parliamentary

2. Socialist

3. Confederal

4. Presidential

Question ID : 26433086772
Status : Answered
Chosen Option : 2

Q.25 What type of climate occurs along the west coast of continents in subtropical latitudes

hs
between 30°-40° latitudes, covering Central California, Central Chile along the coast in
South East and South Western Australia?
Ans 1. Subtropical steppe climate

2. Humid subtropical climate

at
3. Mediterranean climate

4. Marine west coast climate

M
Question ID : 264330142720
Status : Not Answered
Chosen Option : --

Section : Quantitative Aptitude

Q.1 The third proportional to 12 and 36 is:


ap
Ans 1. 96

2. 36
at
3. 12

4. 108

Question ID : 26433073021
Pr

Status : Answered
Chosen Option : 4

Q.2 A man sells a bicycle for ₹990 at a loss of 10%. At what price (in ₹) should he sell the
bicycle to earn a profit of 24%?
an

Ans 1. 1,640

2. 1,664

3. 1,364

4. 1,380
ag

Question ID : 264330142588
Status : Answered
Chosen Option : 3
G

Q.3 What annual payment (in ₹) will discharge a debt of ₹1,500 in 10 years at 5% per
annum simple interest? (Correct to two decimal places)
Ans 1. 111.44

2. 122.44

3. 123.44

4. 125.44

Question ID : 264330102357
Status : Answered
Chosen Option : 3
Join @MathsByGaganPratap Telegram Channel for free PDFs
Q.4 If by selling 12 textbooks, a seller earns profit equal to the selling price of 4 textbooks,
what is his percentage profit?
Ans
1.

2.

3.

4.

Question ID : 26433082678
Status : Answered

hs
Chosen Option : 2

Q.5 Total savings of X and Y is 40% of their total income. Their average expenditure is
₹21,000. What is the total salary of X and Y?

at
Ans
1.

2.

M
3.

4.
ap Question ID : 26433081557
Status : Answered
Chosen Option : 4
at
Q.6
Pr

Ans 1. ₹451.92

2. ₹492.92

3. ₹803.84

4. ₹401.92
an

Question ID : 26433081652
Status : Answered
Chosen Option : 3
ag

Q.7 The monthly income of Anvita was ₹28,500 and her monthly expenditure was ₹22,500.
Next year her income increased by 16% and her expenditure increased by 10%. Find
the percentage increase in her savings.
Ans 1. 35.5 Percent

2. 35.8 Percent
G

3. 38.5 Percent

4. 21.05 Percent

Question ID : 26433074334
Status : Answered
Chosen Option : 3
Join @MathsByGaganPratap Telegram Channel for free PDFs
Q.8 A shopkeeper sold an article at 25% profit. On selling it for ₹225 more he would get a
profit of 40%. The cost price of the article was:
Ans
1.

2.

3.

4.

Question ID : 26433087359
Status : Answered
Chosen Option : 3

hs
Q.9 Find the average of first 38 natural numbers.
Ans 1. 16.5

2. 19.5

at
3. 20.5

4. 19

M
Question ID : 264330102319
Status : Answered
Chosen Option : 2
ap
Q.10 The average of four numbers was 44. With the inclusion of a fifth number, y, the new
average dropped to 42. What is the value of y?
Ans 1. 36

2. 34

3. 32
at
4. 40

Question ID : 264330103237
Pr

Status : Answered
Chosen Option : 2

Q.11 A solid iron sphere of radius 21 cm is melted down into a cone of height 84 cm. What
will be the radius of the cone ?
Ans 1. 24 cm
an

2. 28 cm

3. 32 cm

4. 21 cm
ag

Question ID : 264330142754
Status : Answered
Chosen Option : 4
G
Join @MathsByGaganPratap Telegram Channel for free PDFs
Q.12

Ans
1.

2.

3.

4.

hs
Question ID : 264330103409
Status : Answered

at
Chosen Option : 3

Q.13 It takes 4 hours to go from Delhi to Chandigarh at an average speed of 60 km/h. How
long it will take a person to go from Delhi to Chandigarh at an average speed of 50
km/h?

M
Ans 1. 4 hours 40 minutes

2. 4 hours 30 minutes

3. 4 hours 48 minutes ap
4. 5 hours

Question ID : 264330142277
Status : Answered
Chosen Option : 3
at
Q.14 If a : b : c = 4 : 7 : 9 and b : c : d = 28 : 36 : 21, then a : b : c : d is:
Ans 1. 12 : 21 : 27 : 17
Pr

2. 16 : 28 : 36 : 21

3. 8 : 14 : 18 : 21

4. 4 : 7 : 9 : 11

Question ID : 264330142256
an

Status : Answered
Chosen Option : 2

Q.15 What is the net discount for two successive discounts of 15% and 35%?
Ans 1. 50%
ag

2. 44.75%

3. 44.57%

4. 25%
G

Question ID : 264330122074
Status : Answered
Chosen Option : 2
Join @MathsByGaganPratap Telegram Channel for free PDFs
Q.16 Medha covers 81 km at a speed of 27 km/h by bike, 12 km at a speed of 6 km/h by
bicycle, and another 270 km at a speed of 45 km/h by car. Find her average speed for
the whole journey.
Ans 1. 33 km/h

2. 36 km/h

3. 34 km/h

4. 35 km/h

Question ID : 264330142246
Status : Answered
Chosen Option : 1

hs
Q.17 The number 974581297426 is divisible by:
Ans 1. 6

2. 11

3. 4

at
4. 9

Question ID : 264330126162

M
Status : Answered
Chosen Option : 2

Q.18 ap
Ans 1. ₹54,575

2. ₹54,675

3. ₹54,475
at
4. ₹54,875

Question ID : 264330142244
Status : Answered
Pr

Chosen Option : 2

Q.19 15 men can complete a work in 15 days, whereas it takes 18 women to complete the
work in 15 days. In how many days will 20 men and 9 women, together, complete the
work? (Rounded off to complete number of days.)
Ans 1. 8
an

2. 15

3. 12

4. 5
ag

Question ID : 26433092029
Status : Answered
Chosen Option : 1
G

Q.20 Rajesh marks his goods 30% above the cost price but allows a 12% discount for cash
payment. If he sells the goods for ₹3,500, find his cost price (up to one place of
decimal).
Ans 1. ₹3,090.5

2. ₹3,095.9

3. ₹3,054.9

4. ₹3,059.4

Question ID : 26433099035
Status : Answered
Chosen Option : 4
Join @MathsByGaganPratap Telegram Channel for free PDFs
Q.21 A person covers 11 km at a speed of 3 km/h, 21 km at a speed of 5 km/h and 37 km at a
speed of 10 km/h. Find the average speed for the entire journey.
Ans
1.

2.

3.

hs
4.

Question ID : 26433074221
Status : Answered

at
Chosen Option : 1

Q.22 Ravi borrowed ₹1,380 from a bank which he repaid in 6 years at the rate of 6% per
annum simple interest. If payment was made in six equal instalment, then each

M
instalment was:
Ans 1. ₹220

2. ₹200

3. ₹180

4. ₹190
ap
Question ID : 26433099599
Status : Answered
at
Chosen Option : 2

Q.23 Which of the following numbers is divisible by 120?


Ans 1. 170280
Pr

2. 140240

3. 156200

4. 170360
an

Question ID : 264330142591
Status : Answered
Chosen Option : 1

Q.24 If in an examination, the marks obtained by Rohan is 36% less than that of Pawan,
ag

then marks obtained by Pawan is how much percentage more than the marks obtained
by Rohan?
Ans 1. 26.47%

2. 63.25%

3. 56.25%
G

4. 52.47%

Question ID : 26433063681
Status : Answered
Chosen Option : 3
Join @MathsByGaganPratap Telegram Channel for free PDFs
Q.25 13 men can complete a work in 7 days. After 3 days, 4 more men joined them. How
many days will they now take to complete the remaining work?
Ans
1.

2.

3.

4.

hs
Question ID : 264330142250
Status : Answered
Chosen Option : 4

at
Section : English Language

Q.1 Select the most appropriate option to fill in the blank.

M
A fire ______ out in the basement of the house.
Ans 1. broke

2. breaking ap
3. burned

4. burnt

Question ID : 26433060084
at
Status : Answered
Chosen Option : 1

Q.2 Select the option that expresses the given sentence in active voice.
Pr

The gardening is done by Anil.


Ans 1. Anil had been done the gardening

2. Anil does the gardening

3. Anil is be done the gardening

4. Anil been done the gardening


an

Question ID : 264330132873
Status : Answered
Chosen Option : 2
ag

Q.3 Select the most appropriate option to fill in the blank.

The weather report shows the _____ of rainfall.


Ans 1. tranquillity

2. mobility
G

3. validity

4. probability

Question ID : 26433060086
Status : Answered
Chosen Option : 4
Join @MathsByGaganPratap Telegram Channel for free PDFs
Q.4 Select the correct direct narration of the given sentence.
They said that they would go the next day.
Ans 1. They said, “We will go tomorrow.”

2. They said, “I will have went tomorrow.”

3. They said, “I would go tomorrow.”

4. They said, “We will be go tomorrow.”

Question ID : 264330132788
Status : Answered
Chosen Option : 1

Q.5 Select the most appropriate option that can substitute the underlined segment in the

hs
given sentence. If there is no need to substitute it, select ‘No substitution’.

For a little while Binod was the media's blue-eyed boy.


Ans 1. No substitution

2. blue-shirt boy

at
3. green-eyed boy

4. big-name boy

M
Question ID : 26433087931
Status : Answered
Chosen Option : 1
ap
Q.6 Choose the most appropriate ANTONYM of the underlined word in the given sentence.

The melancholy song brought tears to my eyes.


Ans 1. Seclusion

2. Ecstasy
at
3. Tremendousness

4. Aesthetic
Pr

Question ID : 26433071838
Status : Answered
Chosen Option : 3

Q.7 Select the option that can be used as a one-word substitute for the given group of
words.
an

The supervisor in the examination hall.


Ans 1. Teacher

2. Monitor

3. Principal
ag

4. Invigilator

Question ID : 26433060102
Status : Answered
G

Chosen Option : 4
Join @MathsByGaganPratap Telegram Channel for free PDFs
Q.8 Select the INCORRECTLY spelt word.
Ans 1. Corporeale 

2.  Diminutives

3. Credulous 

4.  Sagacious

Question ID : 26433060196
Status : Answered
Chosen Option : 1

Q.9 Select the most appropriate meaning of the given idiom.

hs
To go down in flames
Ans 1. To burn completely

2. To take risks whole heartedly

3. To fail miserably at something

at
4. To mend permanently

Question ID : 26433091742

M
Status : Answered
Chosen Option : 3

Q.10 Select the most appropriate ANTONYM of the given word.


ap
Humane
Ans 1. cordial 

2. merciless 

3. forgiving 
at
4. generous

Question ID : 26433059804
Status : Answered
Pr

Chosen Option : 2

Q.11 Select the most appropriate meaning of the given idiom.

Ruffle someone’s feathers


Ans 1. Be relaxed and in control
an

2. Do something very easily

3. Make more of an effort

4. Make someone annoyed


ag

Question ID : 264330132896
Status : Answered
Chosen Option : 4
G

Q.12 Select the most appropriate meaning of the underlined idiom.

Those were only crocodile tears.


Ans 1. Mild regret

2. Very gloomy

3. Pretended sadness 

4. A weeping sign

Question ID : 26433088143
Status : Answered
Chosen Option : 3
Join @MathsByGaganPratap Telegram Channel for free PDFs
Q.13 Select the INCORRECTLY spelt word.
Ans 1. Impious

2. Renonce

3. Admirable

4. Cautious

Question ID : 26433087935
Status : Answered
Chosen Option : 2

Q.14 Select the INCORRECTLY spelt word.

hs
Ans 1. Longevity

2. Leftinant

3. Ludicrous

4. Location

at
Question ID : 26433088537
Not Attempted and
Status :
Marked For Review

M
Chosen Option : --

Q.15 Select the most appropriate option to fill in the blank.

Rome grew from a small town on the Tiber River in central Italy into a _______ empire

Ans
that ultimately embraced England.
1. vast
ap
2. voluminous 

3. bountiful
at
4. lofty

Question ID : 26433060958
Status : Answered
Pr

Chosen Option : 1

Comprehension:
In the following passage, some words have been deleted. Read the passage carefully and
select the most appropriate option to fill in each blank.
an

Shark Tank has struck a chord with the aspiring, hardworking people who have a
(1)__________ and a stake in India's future in a very short period since its (2)__________.
What is particularly exciting to observe are the stories from across India (3)__________centre
stage to carve out their individuality via entrepreneurial (4)___________. The Sharks have
made incredible (5)_________ with new ventures ranging from gadget start-ups to food firms.
ag

SubQuestion No : 16
Q.16 Select the most appropriate option to fill in blank number 1.
Ans 1. vision

2. priority

3. perception  
G

4. consistency

Question ID : 26433096168
Status : Answered
Chosen Option : 1
Join @MathsByGaganPratap Telegram Channel for free PDFs
Comprehension:
In the following passage, some words have been deleted. Read the passage carefully and
select the most appropriate option to fill in each blank.

Shark Tank has struck a chord with the aspiring, hardworking people who have a
(1)__________ and a stake in India's future in a very short period since its (2)__________.
What is particularly exciting to observe are the stories from across India (3)__________centre
stage to carve out their individuality via entrepreneurial (4)___________. The Sharks have
made incredible (5)_________ with new ventures ranging from gadget start-ups to food firms.

SubQuestion No : 17
Q.17 Select the most appropriate option to fill in blank number 2.
Ans 1. birth

2. debut  

hs
3. departure

4. formation

Question ID : 26433096169

at
Status : Answered
Chosen Option : 4

Comprehension:

M
In the following passage, some words have been deleted. Read the passage carefully and
select the most appropriate option to fill in each blank.

Shark Tank has struck a chord with the aspiring, hardworking people who have a
(1)__________ and a stake in India's future in a very short period since its (2)__________.
What is particularly exciting to observe are the stories from across India (3)__________centre
ap
stage to carve out their individuality via entrepreneurial (4)___________. The Sharks have
made incredible (5)_________ with new ventures ranging from gadget start-ups to food firms.

SubQuestion No : 18
Q.18 Select the most appropriate option to fill in blank number 3.
Ans 1. gratifying
at
2. measuring

3. taking

4. reinventing
Pr

Question ID : 26433096170
Status : Answered
Chosen Option : 3
an

Comprehension:
In the following passage, some words have been deleted. Read the passage carefully and
select the most appropriate option to fill in each blank.

Shark Tank has struck a chord with the aspiring, hardworking people who have a
(1)__________ and a stake in India's future in a very short period since its (2)__________.
ag

What is particularly exciting to observe are the stories from across India (3)__________centre
stage to carve out their individuality via entrepreneurial (4)___________. The Sharks have
made incredible (5)_________ with new ventures ranging from gadget start-ups to food firms.

SubQuestion No : 19
Q.19 Select the most appropriate option to fill in blank number 4.
G

Ans 1. integrity

2. enthusiasm

3. collaboration

4. limitation  

Question ID : 26433096171
Status : Answered
Chosen Option : 2
Join @MathsByGaganPratap Telegram Channel for free PDFs
Comprehension:
In the following passage, some words have been deleted. Read the passage carefully and
select the most appropriate option to fill in each blank.

Shark Tank has struck a chord with the aspiring, hardworking people who have a
(1)__________ and a stake in India's future in a very short period since its (2)__________.
What is particularly exciting to observe are the stories from across India (3)__________centre
stage to carve out their individuality via entrepreneurial (4)___________. The Sharks have
made incredible (5)_________ with new ventures ranging from gadget start-ups to food firms.

SubQuestion No : 20
Q.20 Select the most appropriate option to fill in blank number 5.
Ans 1. agreements  

2. promises

hs
3. accounts

4. association

Question ID : 26433096172

at
Status : Answered
Chosen Option : 1

Comprehension:

M
Read the given passage and answer the questions that follow.
When it comes to nutrition, ‘power foods’ are those with a high concentration of nutrients,
such fibre and potassium. A growing number of fitness trainers advise their customers to
include these items in their regular diets in order to boost muscle growth. Power foods can be
incorporated into your diet in a variety of ways. Of course, careful preparation, the use of
season-fresh items and identifying your preferred flavour among power meals are the keys to
enjoying these dishes.
ap
Even if you're not a gourmet cook, you may find some of the best power food combos in your
own pantry. Onions and chickpeas are an excellent pairing. In order for the body to deliver
oxygen to all of its numerous parts, it needs iron. The symptoms of an iron deficiency include
anaemia, fatigue, mental fog and exhaustion.
Bananas and yoghurt are another popular power food pairing. After a long football game, this
is the ideal post-game snack. Exercising reduces blood sugar by reducing the amount of
at
glucose in the bloodstream. Yoghurt is rich in protein and bananas are rich in carbohydrates,
both of which aid in refuelling and preventing muscular stiffness after a workout.
Green tea is the best source of catechins, antioxidants that can protect cells from oxidative
damage, in the beverage market. Adding a squeeze of lemon to green tea, say Purdue
University researchers, increases the body's ability to absorb catechins. So, the next time you
Pr

have friends around, serve them iced green tea with mint and lemon
juice in a pitcher.

SubQuestion No : 21
Q.21 Who says that adding a squeeze of lemon to green tea increases the body's ability to
absorb catechins?
Ans 1. Western Cape University researchers
an

2. Purdue University researchers

3. Harvard University researchers

4. Kentucky University researchers


ag

Question ID : 264330102436
Status : Answered
Chosen Option : 2
G
Join @MathsByGaganPratap Telegram Channel for free PDFs
Comprehension:
Read the given passage and answer the questions that follow.
When it comes to nutrition, ‘power foods’ are those with a high concentration of nutrients,
such fibre and potassium. A growing number of fitness trainers advise their customers to
include these items in their regular diets in order to boost muscle growth. Power foods can be
incorporated into your diet in a variety of ways. Of course, careful preparation, the use of
season-fresh items and identifying your preferred flavour among power meals are the keys to
enjoying these dishes.
Even if you're not a gourmet cook, you may find some of the best power food combos in your
own pantry. Onions and chickpeas are an excellent pairing. In order for the body to deliver
oxygen to all of its numerous parts, it needs iron. The symptoms of an iron deficiency include
anaemia, fatigue, mental fog and exhaustion.
Bananas and yoghurt are another popular power food pairing. After a long football game, this
is the ideal post-game snack. Exercising reduces blood sugar by reducing the amount of
glucose in the bloodstream. Yoghurt is rich in protein and bananas are rich in carbohydrates,
both of which aid in refuelling and preventing muscular stiffness after a workout.

hs
Green tea is the best source of catechins, antioxidants that can protect cells from oxidative
damage, in the beverage market. Adding a squeeze of lemon to green tea, say Purdue
University researchers, increases the body's ability to absorb catechins. So, the next time you
have friends around, serve them iced green tea with mint and lemon
juice in a pitcher.

SubQuestion No : 22

at
Q.22 Identify the central theme of the passage.
Ans 1. Food and lifestyle

2. Impact of consuming green tea

M
3. Maintenance of unhealthy food habits

4. Usefulness of bananas and yoghurt

Question ID : 264330102435
ap Status : Answered
Chosen Option : 1

Comprehension:
Read the given passage and answer the questions that follow.
at
When it comes to nutrition, ‘power foods’ are those with a high concentration of nutrients,
such fibre and potassium. A growing number of fitness trainers advise their customers to
include these items in their regular diets in order to boost muscle growth. Power foods can be
incorporated into your diet in a variety of ways. Of course, careful preparation, the use of
season-fresh items and identifying your preferred flavour among power meals are the keys to
Pr

enjoying these dishes.


Even if you're not a gourmet cook, you may find some of the best power food combos in your
own pantry. Onions and chickpeas are an excellent pairing. In order for the body to deliver
oxygen to all of its numerous parts, it needs iron. The symptoms of an iron deficiency include
anaemia, fatigue, mental fog and exhaustion.
Bananas and yoghurt are another popular power food pairing. After a long football game, this
is the ideal post-game snack. Exercising reduces blood sugar by reducing the amount of
an

glucose in the bloodstream. Yoghurt is rich in protein and bananas are rich in carbohydrates,
both of which aid in refuelling and preventing muscular stiffness after a workout.
Green tea is the best source of catechins, antioxidants that can protect cells from oxidative
damage, in the beverage market. Adding a squeeze of lemon to green tea, say Purdue
University researchers, increases the body's ability to absorb catechins. So, the next time you
have friends around, serve them iced green tea with mint and lemon
juice in a pitcher.
ag

SubQuestion No : 23
Q.23 Identify a suitable title for the passage.
Ans 1. Food Choices

2. Foods for Enjoyment


G

3. Easy to Prepare Snacks

4. Foods for Fitness

Question ID : 264330102434
Status : Answered
Chosen Option : 4
Join @MathsByGaganPratap Telegram Channel for free PDFs
Comprehension:
Read the given passage and answer the questions that follow.
When it comes to nutrition, ‘power foods’ are those with a high concentration of nutrients,
such fibre and potassium. A growing number of fitness trainers advise their customers to
include these items in their regular diets in order to boost muscle growth. Power foods can be
incorporated into your diet in a variety of ways. Of course, careful preparation, the use of
season-fresh items and identifying your preferred flavour among power meals are the keys to
enjoying these dishes.
Even if you're not a gourmet cook, you may find some of the best power food combos in your
own pantry. Onions and chickpeas are an excellent pairing. In order for the body to deliver
oxygen to all of its numerous parts, it needs iron. The symptoms of an iron deficiency include
anaemia, fatigue, mental fog and exhaustion.
Bananas and yoghurt are another popular power food pairing. After a long football game, this
is the ideal post-game snack. Exercising reduces blood sugar by reducing the amount of
glucose in the bloodstream. Yoghurt is rich in protein and bananas are rich in carbohydrates,
both of which aid in refuelling and preventing muscular stiffness after a workout.

hs
Green tea is the best source of catechins, antioxidants that can protect cells from oxidative
damage, in the beverage market. Adding a squeeze of lemon to green tea, say Purdue
University researchers, increases the body's ability to absorb catechins. So, the next time you
have friends around, serve them iced green tea with mint and lemon
juice in a pitcher.

SubQuestion No : 24

at
Q.24 Identify the tone of the passage.
Ans 1. Deductive

2. Informative

M
3. Imperative

4. Assertive

Question ID : 264330102437
ap Status : Answered
Chosen Option : 2

Comprehension:
Read the given passage and answer the questions that follow.
at
When it comes to nutrition, ‘power foods’ are those with a high concentration of nutrients,
such fibre and potassium. A growing number of fitness trainers advise their customers to
include these items in their regular diets in order to boost muscle growth. Power foods can be
incorporated into your diet in a variety of ways. Of course, careful preparation, the use of
season-fresh items and identifying your preferred flavour among power meals are the keys to
Pr

enjoying these dishes.


Even if you're not a gourmet cook, you may find some of the best power food combos in your
own pantry. Onions and chickpeas are an excellent pairing. In order for the body to deliver
oxygen to all of its numerous parts, it needs iron. The symptoms of an iron deficiency include
anaemia, fatigue, mental fog and exhaustion.
Bananas and yoghurt are another popular power food pairing. After a long football game, this
is the ideal post-game snack. Exercising reduces blood sugar by reducing the amount of
an

glucose in the bloodstream. Yoghurt is rich in protein and bananas are rich in carbohydrates,
both of which aid in refuelling and preventing muscular stiffness after a workout.
Green tea is the best source of catechins, antioxidants that can protect cells from oxidative
damage, in the beverage market. Adding a squeeze of lemon to green tea, say Purdue
University researchers, increases the body's ability to absorb catechins. So, the next time you
have friends around, serve them iced green tea with mint and lemon
juice in a pitcher.
ag

SubQuestion No : 25
Q.25 Select the most appropriate ANTONYM of the given word.
Absorb
Ans 1. Ingest
G

2. Dissipate

3. Blot

4. Emit

Question ID : 264330102438
Status : Answered
Chosen Option : 2
Join @MathsByGaganPratap Telegram Channel for free PDFs

Phase-XI/2023/Selection Posts and Selection Posts/Ladakh/2023


Roll Number Gagan Pratap Maths
Candidate Name Gagan Pratap Maths
Venue Name iON Digital Zone iDZ Ramtekdi 2
Exam Date 30/06/2023
Exam Time 9:00 AM - 10:00 AM
Subject Selection Post Graduate Level

Section : General Intelligence

hs
Q.1 Select the option that represents the correct order of the given words as they would
appear in an English dictionary.
1- Labial
2- Labefaction
3- Laboratory
4- Label

at
5- Labile
Ans 1. 4, 2, 5, 1, 3

2. 4, 2, 1, 5, 3

M
3. 2, 4, 1, 5, 3

4. 2, 4, 5, 1, 3

ap Question ID : 26433079446
Status : Answered
Chosen Option : 3

Q.2 In a certain code language, ‘free prison’ is coded as ‘float-away microphone’, ‘prison
and’ is coded as ‘microphone cucumber’ and ‘free and’ is coded as ‘float-away
cucumber’. What is the code word for ‘free’?
at
Ans 1. cucumber

2. microphone

3. float
Pr

4. float-away

Question ID : 264330118657
Status : Answered
Chosen Option : 4
an

Q.3 In a code language, 'MUMBAI' is written as ‘N46’ and ‘KOLKATA’ is written as ‘L60’.
How will 'LUCKNOW' be written in that language?
Ans 1. K85

2. M87
ag

3. M82

4. Q97

Question ID : 264330120821
G

Status : Answered
Chosen Option : 2
Join @MathsByGaganPratap Telegram Channel for free PDFs
Q.4 Select the option that represents the letters that, when placed from left to right in the
blanks below, will complete the letter-series.
x_y y _ y x x _ y yy _ x y _ y _
Ans 1. xyyxyy 

2. xyxxyx 

3. xxxxyy 

4. xyxxxy 

Question ID : 26433080455
Status : Answered
Chosen Option : 1

hs
Q.5 Which of the following numbers will replace the question mark (?) in the given series?
11, 21, 16, 26, ?, 31
Ans 1. 29

2. 27

at
3. 25

4. 21

M
Question ID : 26433056430
Status : Answered
Chosen Option : 4

Q.6 Three statements are given followed by three conclusions numbered I, II and III.
ap
Assuming the statements to be true, even if they seem to be at variance with
commonly known facts, decide which of the conclusions logically follow(s) from the
statements.

Statements:
Some gifts are presents.
Some presents are boxes.
at
All presents are tables.

Conclusions:
I. Some gifts are tables.
II. Some tables are boxes.
Pr

III. All gifts are boxes.


Ans 1. Only I and II follow

2. Only I follows

3. Only III follows

4. Only II and III follow


an

Question ID : 264330143352
Status : Answered
Chosen Option : 1
ag

Q.7 Select the option that is related to the third word in the same way as the second word
is related to the first word.
(The words must be considered as meaningful English words and must not be related
to each other based on the number of letters/number of consonants/vowels in the
word)
Chair : Stool :: Ladder : ?
G

Ans 1. Table

2. Cupboard

3. Shelf

4. Staircase

Question ID : 26433067984
Status : Answered
Chosen Option : 4
Join @MathsByGaganPratap Telegram Channel for free PDFs
Q.8 Select the correct option that indicates the arrangement of the following words in a
logical and meaningful order.
1. Knee
2. Neck
3. Thigh
4. Head
5. Toe
6. Waist
Ans 1. 5, 1, 3, 6, 4, 2 

2. 5, 3, 1, 6, 2, 4

3. 5, 1, 3, 6, 2, 4 

4. 5, 1, 6, 3, 2, 4 

hs
Question ID : 264330129560
Status : Answered
Chosen Option : 3

Q.9 Select the option that is related to the third word in the same way as the second word

at
is related to the first word.
(The words must be considered as meaningful English words and must not be related
to each other based on the number of letters/number of consonants/vowels in the
word)
Extrovert : Introvert : : Religious : ?

M
Ans 1. Pessimist

2. Optimist

3. Narcissist ap
4. Atheist

Question ID : 26433068408
Status : Answered
Chosen Option : 4
at
Q.10 Select the correct combination of mathematical signs to sequentially replace the *
signs and balance the given equation.

9 * 13 * 4 * 15 * 5 * 64
Pr

Ans 1. +, ×, ×, ÷, =

2. −, ×, +, ÷, =

3. +, ×, −, ÷, =

4. +, ×, +, ÷, =
an

Question ID : 26433097203
Status : Answered
Chosen Option : 4
ag
G
Join @MathsByGaganPratap Telegram Channel for free PDFs
Q.11

Ans

1.

hs
at
2.

M
3.
ap
at
Pr

4.
an

Question ID : 26433078910
Status : Answered
Chosen Option : 1

Q.12 In a certain code language, 'DIFFRENT' is written as 'EHGESDOS' and 'COMBINE' is


ag

written as 'DNNAJMF'. How will 'LETTER' be written in that language?


Ans 1. MDUSFQ

2. MUDFSQ

3. NDFSUS
G

4. NEVTGR

Question ID : 26433070265
Status : Answered
Chosen Option : 1
Join @MathsByGaganPratap Telegram Channel for free PDFs
Q.13 Select the option that is related to the third term in the same way as the second term is
related to the first term and the sixth term is related to the fifth term.
7 : 51 :: 12 : ? :: 14 : 198
Ans 1. 146

2. 148

3. 144

4. 141

Question ID : 26433057064
Status : Answered
Chosen Option : 1

hs
Q.14 In a certain code language, 'Broken' is written as '1171410413', 'Bottle' is written as
'1141919114', how will 'Sample' be written in that language?
Ans 1. 1711215114

2. 1801215114

at
3. 1801214114

4. 1811114103

M
Question ID : 264330143380
Status : Answered
Chosen Option : 2

Q.15
ap
Ans
at
1.
Pr

2.
an

3.
ag

4.
G

Question ID : 26433056948
Status : Answered
Chosen Option : 4
Join @MathsByGaganPratap Telegram Channel for free PDFs
Q.16 In this question, three statements are given, followed by two conclusions numbered I
and II. Assuming the statements to be true, even if they seem to be at variance with
commonly known facts, decide which of the conclusions logically follows/follow from
the statements.

Statements:
I. Some pins are needles.
II. All pins are nails.
III. All needles are screws.

Conclusions:
I. No nail is a screw.
II. Some needles are nails.
Ans 1. Only conclusion I follows

2. Both conclusions I and II follow

hs
3. Neither conclusion I nor II follows

4. Only conclusion II follows

Question ID : 264330104069

at
Status : Answered
Chosen Option : 4

Q.17 Select the option that is related to the third word in the same way as the second word

M
is related to the first word. (The words must be considered as meaningful English
words and must not be related to each other based on the number of letters/number of
consonants/vowels in the word)
Mettle : Diffidence :: Noxious : ?
Ans 1. Harmless ap
2. Disastrous

3. Pernicious

4. Profitable
at
Question ID : 264330143389
Status : Answered
Chosen Option : 4
Pr

Q.18 Select the correct mirror image of the given combination when the mirror is placed at
MN as shown.
an

Ans
1.
ag

2.

3.
G

4.

Question ID : 264330143449
Status : Answered
Chosen Option : 3
Join @MathsByGaganPratap Telegram Channel for free PDFs
Q.19 Select the option that represents the correct order of the given words as they would
appear in an English dictionary.
1- Refine
2- Reflect
3- Referendum
4- Refill
5- Refinery
Ans 1. 4, 2, 1, 3, 5

2. 3, 4, 1, 5, 2

3. 4, 3, 5, 1, 2

4. 4, 1, 2, 5, 3

Question ID : 26433079448

hs
Status : Answered
Chosen Option : 2

Q.20 ‘Z + Q’ means ‘Z is the brother of Q’,


‘Z × Q’ means ‘Z is the father of Q’,

at
‘Z ÷ Q’ means ‘Z is the mother of Q’.
Which of the following would mean ‘A is the son of B’?
Ans 1. B × I × A

2. B + A × I

M
3. B ÷ A ÷ I

4. B × A + I
ap Question ID : 264330143370
Status : Answered
Chosen Option : 4
at
Pr
an
ag
G
Join @MathsByGaganPratap Telegram Channel for free PDFs
Q.21

Ans

1.

hs
at
2.

M
3.

ap
4.
at

Question ID : 264330143436
Pr

Status : Answered
Chosen Option : 2

Q.22 Select the option that represents the letters that, when sequentially placed from left to
right in the blanks below, will complete the letter series.
C _ L L A_ _ O L _ A R CO_ L _ R
an

Ans 1. C O L L A R

2. O R C L L A

3. C L L R O L

4. R A L O C R
ag

Question ID : 26433067728
Status : Answered
Chosen Option : 2
G

Q.23 In a certain code language, ‘SOUTH’ is written as ‘83’ and ‘CONVOY’ is written as ‘94’.
How will ‘SYSTEM’ be written in that language?
Ans 1. 93

2. 101

3. 98

4. 108

Question ID : 26433069612
Status : Answered
Chosen Option : 2
Join @MathsByGaganPratap Telegram Channel for free PDFs
Q.24 Pointing to the photograph of a man, Muskan said, “He is the only grandson of my
father’s grandfather”. How is Muskan related to the man in the photograph?
Ans 1. Sister-in-law

2. Daughter

3. Mother

4. Sister

Question ID : 26433070274
Status : Answered
Chosen Option : 2

Q.25 Select the option that represents the letters that, when sequentially placed from left to

hs
right in the blanks below, will complete the letter series.
DK_YD_N__KNYD_NY_KN_
Ans 1. N K Y D K D Y

2. D N Y D K D D

at
3. K D Y N K Y D

4. K K D N Y K D

M
Question ID : 26433067732
Status : Answered
Chosen Option : 1

Section : General Awareness


ap
Q.1 Which is a saltwater lake located in the Tibet Autonomous Region, China, to the west
of Lake Mansarovar and to the south of Mount Kailash?
Ans 1. Lake Yamdrok Tso

2. Lake Rawok
at
3. Lake Rakshastal

4. Lake Basum Tso


Pr

Question ID : 264330143306
Status : Answered
Chosen Option : 1

Q.2 The Sun Temple of Modhera was built by the king of which dynasty?
an

Ans 1. Rashtrakuta

2. Paramara

3. Tomar

4. Solanki
ag

Question ID : 26433089131
Not Attempted and
Status :
Marked For Review
Chosen Option : --
G

Q.3 Which state/union territory has the highest percentage of literacy rate in India
according to the 2011 census?
Ans 1. Delhi

2. Lakshadweep

3. Kerala

4. Maharashtra

Question ID : 264330109508
Status : Answered
Chosen Option : 3
Join @MathsByGaganPratap Telegram Channel for free PDFs
Q.4 In 2022, under which of the following schemes was it announced that more than 14,500
schools across the country will be developed to showcase all the components of NEP
2020?
Ans 1. PM Schools for Rising India

2. Rashtriya Madhyamik Shiksha Abhiyan

3. Sharva Shiksha Abhiyan

4. Beti Bachao Beti Padhao

Question ID : 264330130833
Status : Answered
Chosen Option : 1

hs
Q.5 Eugene Odum is well known for:
Ans 1. contribution in ecology

2. contribution in animal hormonal balance

3. contribution in plant stress regulation

at
4. contribution in animal physiology

Question ID : 26433082213

M
Status : Not Answered
Chosen Option : --

Q.6 Who among the following said that “English education has enslaved us”?
Ans 1. Raja Rammohan Roy

2. Mahatma Gandhi  
ap
3. Muhammad Ali Jinnah

4. Muhammad Iqbal
at
Question ID : 26433085293
Status : Answered
Chosen Option : 1
Pr

Q.7 Vasundhara Doraswamy was awarded the Sangeet Natak Akademi Award (Akademi
Puraskar) for the year 2019 for her contribution to which of the following?
Ans 1. Bharatanatyam

2. Kuchipudi
an

3. Odissi

4. Kathak

Question ID : 264330130893
Status : Not Answered
ag

Chosen Option : --

Q.8 ________________won the National Award for the Best Music Director for his music
album in the movie 'Viswasam' at the 67th National Award.
Ans 1. Mani Sharma
G

2. MM Keeravani

3. Anirudh Ravi Chander

4. D Imman

Question ID : 264330143403
Not Attempted and
Status :
Marked For Review
Chosen Option : --
Join @MathsByGaganPratap Telegram Channel for free PDFs
Q.9 _____ lead India at the Women’s Hockey Asia Cup 2022 in Muscat.
Ans 1. Deep Grace Ekka

2. Savita Punia

3. Nikki Pradhan

4. Rani Rampal

Question ID : 26433081882
Status : Answered
Chosen Option : 4

Q.10 In the Winter Session of Parliament, which was held from 29 November 2021 to 22
December 2021, ___________ bills were passed (excluding an Appropriation Bill).

hs
Ans 1. 6

2. 16

3. 10

at
4. 3

Question ID : 264330122341
Status : Not Answered

M
Chosen Option : --

Q.11 ‘Six Machine: I Don't Like Cricket ... I Love It’ is a book written by:
Ans 1. Andre Russell ap
2. Kieron Pollard

3. Chris Gayle

4. MS Dhoni
at
Question ID : 26433064624
Status : Answered
Chosen Option : 3
Pr

Q.12 Which part of the Indian Constitution deals with the organisation, composition and
powers of state legislature?
Ans 1. Part IV

2. Part VI

3. Part III
an

4. Part V

Question ID : 264330111546
Status : Answered
ag

Chosen Option : 2

Q.13 Which of the following states is associated with the tropical thorn forests?
Ans 1. Andaman and Nicobar Islands 

2. West Bengal
G

3. Madhya Pradesh 

4. Mizoram

Question ID : 264330109869
Status : Answered
Chosen Option : 3
Join @MathsByGaganPratap Telegram Channel for free PDFs
Q.14 In November 2022, who among the following from India was elected as a Member of
International Committee for Weight and Measures (CIPM)?
Ans 1. CR Rao

2. Amartya Sen

3. Jean Dreeze

4. Venu Gopal Achanta

Question ID : 264330132605
Status : Answered
Chosen Option : 1

Q.15 Who were the first to make land grants to Brahmins?

hs
Ans 1. Sakas

2. Satavahanas

3. Guptas

at
4. Kushans

Question ID : 264330110201
Status : Not Answered

M
Chosen Option : --

Q.16 Which dancer was the first person from Odisha to get Padma Vibhushan in 2000?
Ans 1. Guru Shyama Charan Pati ap
2. Guru Bipin Singh

3. Guru Mayadhar Rout

4. Guru Kelucharan Mohapatra


at
Question ID : 264330112685
Status : Answered
Chosen Option : 4
Pr

Q.17 ‘Moksha’ is an item of which of the following classical dance forms of India?
Ans 1. Odissi

2. Sattriya

3. Bharatanatyam
an

4. Kathak

Question ID : 26433086630
Status : Not Answered
Chosen Option : --
ag

Q.18 The East West Corridor connects ______ in India.


Ans 1. Silchar to Porbandar

2. Udaipur to Aurangabad
G

3. Jorhat to Dawki

4. Kolkata to Delhi

Question ID : 26433089225
Status : Answered
Chosen Option : 4
Join @MathsByGaganPratap Telegram Channel for free PDFs
Q.19 In which of the following countries is the Hindu religious place ‘Chandranath Hill’
located?
Ans 1. Bhutan

2. Nepal

3. Bangladesh

4. Sri Lanka

Question ID : 264330113313
Status : Answered
Chosen Option : 2

Q.20 In which of the following sports is the term ‘Leg bye’ used?

hs
Ans 1. Volleyball

2. Hockey

3. Cricket

at
4. Football

Question ID : 26433071563
Status : Answered

M
Chosen Option : 3

Q.21 Hindustan Republcan Association took to socialist ideas, under whose leadership, in
1928?
Ans 1. Ram Prasad Bismil 

2. Rajguru 
ap
3. Bhagat Singh 

4. Chandra Sekhar Azad 


at
Question ID : 264330132676
Status : Answered
Chosen Option : 4
Pr

Q.22 Easter is celebrated on which of the following days of the week?


Ans 1. Wednesday

2. Monday

3. Sunday
an

4. Tuesday

Question ID : 26433064682
Status : Not Answered
ag

Chosen Option : --

Q.23 When will you find the magnetic field stronger?


Ans 1. When field lines are crowded

2. When the lines are together


G

3. When the field lines are not seen

4. When the poles are together

Question ID : 264330143315
Status : Answered
Chosen Option : 4
Join @MathsByGaganPratap Telegram Channel for free PDFs
Q.24 In which of the following places is the Tomb of Ahmad Shah Al Wali Bahmani built?
Ans 1. Fatehpur Sikri

2. Agra

3. Ashtoor 

4. Ajmer

Question ID : 264330132435
Status : Not Answered
Chosen Option : --

Q.25 Which of the following is NOT a part of the electoral college of the president of India?

hs
Ans 1. Governor of States

2. Elected members of legislative assemblies of states

3. Elected members of Rajya Sabha

4. Elected members of Lok Sabha

at
Question ID : 264330109841
Status : Answered
Chosen Option : 1

M
Section : Quantitative Aptitude

Q.1 ap
Ans
1.

2.
at
3.

4.
Pr

Question ID : 264330101340
Status : Answered
Chosen Option : 2
an

Q.2 A shopkeeper bought articles A and B for a total ₹2,540. He sold A at a gain of 15% and
sold B at a loss of 25%. The selling price of A was ₹109 more than that of B. His profit
or loss percentage in the entire transaction was (rounded off to one decimal place):
Ans 1. Loss, 8.3%

2. Profit, 8.3%
ag

3. Profit, 9.5%

4. Loss, 9.5%

Question ID : 264330109337
G

Not Attempted and


Status :
Marked For Review
Chosen Option : --
Join @MathsByGaganPratap Telegram Channel for free PDFs
Q.3 If x + y = √13 and x - y = √11, then the value of xy (x2 + y2) is:
Ans 1. 10

2. 6

3. √6

4. √10

Question ID : 26433058452
Status : Answered
Chosen Option : 2

Q.4

hs
Ans 1. Rs.15

2. Rs.14

3. Rs.12

at
4. Rs.11

Question ID : 26433075674

M
Status : Answered
Chosen Option : 3

Q.5 ap
at
Pr

Ans 1. 1973
an

2. 1975

3. 1974

4. 1972
ag

Question ID : 264330122571
Status : Answered
Chosen Option : 3

Q.6 Two different kinds of rice of quantities R1 and R2, costing ₹60 per kg and ₹75 per kg,
G

respectively, are mixed to get a mixture that costs ₹65 per kg. What is the ratio in
which R1 is mixed with R2?
Ans 1. 3 : 4

2. 4 : 3

3. 2 : 1

4. 3 : 2

Question ID : 264330126559
Status : Answered
Chosen Option : 3
Join @MathsByGaganPratap Telegram Channel for free PDFs
Q.7

Ans
1.

2.

3.

4.

hs
Question ID : 26433058937
Status : Answered
Chosen Option : 2

Q.8

at
M
ap
at
Ans 1. 160

2. 90
Pr

3. 80

4. 180

Question ID : 264330122701
Status : Answered
an

Chosen Option : 2

Q.9
ag

Ans 1. 24

2. 20

3. 30

4. 26
G

Question ID : 264330143416
Status : Answered
Chosen Option : 1
Join @MathsByGaganPratap Telegram Channel for free PDFs
Q.10

Ans 1. 7.5 m

2. 14 m

3. 21 m

4. 2.1 m

Question ID : 26433063909
Status : Answered
Chosen Option : 3

Q.11 Which of the following is a prime number?

hs
Ans 1. 161

2. 171

3. 193

4. 177

at
Question ID : 264330129984
Status : Answered

M
Chosen Option : 3

Q.12 In ΔABC, a line is drawn parallel to BC, intersecting sides AB and AC at


Points S and T, where AB = 8.3 cm, BC = 16.6 cm and
BS = 5.3 cm. What is the length of ST (in cm)?
Ans 1. 6
ap
2. 12

3. 24
at
4. 18

Question ID : 264330130422
Status : Answered
Pr

Chosen Option : 1

Q.13

Ans 1.
an

2.

3.

4.
ag

Question ID : 26433082107
Status : Answered
Chosen Option : 1
G
Join @MathsByGaganPratap Telegram Channel for free PDFs
Q.14

Ans 1.

2.

3.

4.

Question ID : 26433061774
Status : Answered

hs
Chosen Option : 1

Q.15 The cost price of 10 goods is equal to the selling price of 8 goods. Find the profit
percentage.
Ans 1.

at
2.

3.

M
4.

Question ID : 26433092025
ap Status : Answered
Chosen Option : 1

Q.16 S sold his phone for ₹15,500, which he had bought for ₹12,000. What was his
percentage profit?
Ans 1. 26.67%
at
2. 29.17%

3. 32.67%

4. 22.67%
Pr

Question ID : 264330126108
Status : Answered
Chosen Option : 2
an

Q.17 A train takes 7 seconds to pass man standing on a platform and another train whose
length is double that of the first train, and moving in the opposite direction, takes 10
seconds to pass him. The time taken (in seconds, to the nearest integer) by the trains
to pass each other will be:
Ans 1. 8

2. 9
ag

3. 10

4. 12

Question ID : 264330103562
G

Status : Not Answered


Chosen Option : --
Join @MathsByGaganPratap Telegram Channel for free PDFs
Q.18 A money lender finds that due to the fall in the annual rate of interest from 18% to 15%,
his yearly income diminishes by Rs.750. His capital is:
Ans 1. Rs.25,500

2. Rs.25,000

3. Rs.5,000

4. Rs.2,500

Question ID : 26433081035
Status : Answered
Chosen Option : 2

Q.19 Gori and Gopal alone can do a piece of work in 9 and 21 days respectively. In how

hs
many days will the work be completed, if they both work on alternate days starting
with Gopal?
Ans
1.

at
2.

3.

M
4.

ap Question ID : 26433080055
Status : Answered
Chosen Option : 4

Q.20

Ans
at
1.

2.
Pr

3.

4.

Question ID : 26433061675
Status : Answered
an

Chosen Option : 4

Q.21 Rakhi is 20 years older than her daughter Shikha. In 6 years, Rakhi will be twice as old
as Shikha. Therefore, Rakhi’s present age is ____ years.
Ans 1. 42 
ag

2. 37 

3. 34 

4. 36 
G

Question ID : 264330126622
Status : Answered
Chosen Option : 3
Join @MathsByGaganPratap Telegram Channel for free PDFs
Q.22 A number is increased by 25%, and subsequently decreased by 15%. Find the
percentage of net increase or decrease.
Ans 1. 7.93% decrease

2. 6.25% increase

3. 7.23% increase

4. 9.18% decrease

Question ID : 26433080006
Status : Answered
Chosen Option : 2

Q.23 A football team won 10 games from the total they played. This was 40% of the total.

hs
How many games were played in all?
Ans 1. 25

2. 15

3. 45

at
4. 35

Question ID : 26433061199

M
Status : Answered
Chosen Option : 1

Q.24 The marked price of every item being sold by a wholesaler was ₹400. The wholesaler
was offering a stock-clearance sale, under which for every four items paid for, one
ap
item was being given free. In addition to that, a further 5% discount on the amount
payable on the ‘Buy 4, Get 1 free’ scheme price was being offered to anyone making
purchases worth more than ₹15,000. Rameshwar made purchases for which the
amount payable, before applying the 5% discount, was ₹16,000. What was the effective
discount percentage that was finally offered to Rameshwar during this transaction?
Ans
1.
at
2.

3.
Pr

4.

Question ID : 264330103269
Status : Answered
Chosen Option : 4
an

Q.25

Ans
ag

1.

2.

3.
G

4.

Question ID : 26433079984
Status : Answered
Chosen Option : 3

Section : English Language


Join @MathsByGaganPratap Telegram Channel for free PDFs
Q.1 Select the option that expresses the given sentence in active voice.
By whom was this done?
Ans 1. Who has been doing this?

2. Who was doing this?

3. Who has done this? 

4. Who did this? 

Question ID : 264330133301
Status : Answered
Chosen Option : 4

Q.2 Select the option that expresses the given sentence in reported speech.

hs
Rohit said, “The board game is mine.”
Ans 1. Rohit said that the board game was his.  

2. Rohit said that the board game was me.  

3. Rohit said that he was the board game.  

at
4. Rohit said that the board game were hers.

Question ID : 264330137235

M
Status : Answered
Chosen Option : 1

Q.3 Select the most appropriate synonym for the highlighted word.
The moral values in this nation were at their nadir during the autocrat’s rule.
Ans 1. highest point 
ap
2. lowest point 

3. equatorial

4. medial point 
at

Question ID : 264330120720
Status : Answered
Chosen Option : 2
Pr

Q.4 The following sentence has been split into four segments. Identify the segment that
contains a grammatical error.
If you live / in a foreign country, / you should / try and learn language.
Ans 1. If you live
an

2. you should

3. try and learn language

4. in a foreign country
ag

Question ID : 264330120576
Status : Answered
Chosen Option : 3
G
Join @MathsByGaganPratap Telegram Channel for free PDFs
Q.5 Select the option that expresses the given sentence in direct speech.
Mrs. Mane said that she regretted having supplied us with an inferior brand of TV set
and was ready to apologise for that.
Ans 1. Mrs. Mane said, “I regret supplying you with an inferior brand of TV set and I am
ready to apologise for that.”
2. Mrs. Mane said, “I am regretting having supplied you with an inferior brand of TV set
and I am ready to apologise for that.”
3. Mrs. Mane said, “I regret having supplied you with an inferior brand of TV set and I
am ready to apologise for that.”
4. Mrs. Mane said, “I regret to supply you with an inferior brand of TV set and I am ready
to apologise for that.”

Question ID : 264330117440
Status : Answered

hs
Chosen Option : 3

Q.6 Select the most appropriate ANTONYM of the underlined word in the following
sentence.

at
It is a fact that Pradip’s bigotry results out of his lack of appreciation for another
group's activities, and disregard of others’ belief systems.
Ans 1. System

2. Disregard 

M
3. Lack 

4. Bigotry
ap Question ID : 26433073644
Status : Answered
Chosen Option : 2

Q.7 Select the most appropriate option that can substitute the underlined segment in the
given sentence. If there is no need to substitute it, select ‘No substitution’.
at
The patient was suggested to cut off on the daily intake of carbs.
Ans 1. cut for 

2. No substitution
Pr

3. cut down 

4. cut about

Question ID : 26433073166
an

Status : Answered
Chosen Option : 3

Q.8 Select the most appropriate option to fill in the blank.

Their conviction and determination helped Kimirica overcome ________ odds and
ag

become India’s largest manufacturer of luxury hotel toiletries and guest room
amenities.
Ans 1. undefeated

2. inviolable
G

3. insurmountable 

4. unstoppable

Question ID : 26433060951
Status : Answered
Chosen Option : 3
Join @MathsByGaganPratap Telegram Channel for free PDFs
Q.9 The following sentence has been split into four segments. Identify the segment that
contains a grammatical error.

There are more than two million / job-seekers who are already / completed their fitness
test / for the post since 2020.
Ans 1. There are more than two million

2. job-seekers who are already

3. completed their fitness test

4. for the post since 2020

Question ID : 26433091954
Status : Answered
Chosen Option : 2

hs
Q.10 Select the INCORRECTLY spelt word.
Ans 1. Professor

2. Mosquitose

at
3. Systematically

4. Prophet

M
Question ID : 26433088411
Status : Answered
Chosen Option : 2
ap
Q.11 Select the most appropriate meaning of the underlined phrase.

Julia has the habit of comparing apples to oranges.


Ans 1. Comparing two things that cannot be compared 

2. Blaming others for her mistakes 


at
3. Choosing things after careful scrutiny 

4. Purchasing different fruits for different purposes  


Pr

Question ID : 26433089349
Status : Answered
Chosen Option : 1

Q.12 Select the option that can be used as a one-word substitute for the underlined group
of words in the given sentence.
an

The two women chatted in a friendly and peaceable manner, as if they'd known one
another for a lifetime.
Ans 1. apparently 

2. assuredly
ag

3. absurdly

4. amicably 

Question ID : 26433073858
Status : Answered
G

Chosen Option : 4
Join @MathsByGaganPratap Telegram Channel for free PDFs
Q.13 Select the most appropriate option that can substitute the underlined segment in the
given sentence.

The volunteers of the organisation refused point blank of do what was requested.
Ans 1. refused point blank to do

2. refused point blank doing

3. refused point blankedly to do

4. refused in point blank of doing

Question ID : 26433091969
Status : Answered
Chosen Option : 1

hs
Q.14 Select the most appropriate ANTONYM of the underlined word in the following
sentence.

Rather than being deceitful, we need to be truthful and work together to stop the
unrestrained growth of corruption in all areas of our society.

at
Ans 1. Deceitful

2. Growth

3. Unrestrained 

M
4. Stop 

Question ID : 26433073643
Status : Answered
ap Chosen Option : 1

Q.15 Select the sentence that has the most appropriate meaning of the given idiom.

Die a dog’s death


at
Ans 1. The old businessman died in peace after meeting his whole family on his deathbed.

2. Yesterday, a poor vegetable seller died in an unfortunate accident.

3. The doctor who used illegal methods to conduct experiments on humans must die a
shameful death.
Pr

4. Wars compel people to starve to death and they are helpless.

Question ID : 26433089733
Status : Answered
Chosen Option : 2
an

Comprehension:
In the following passage, some words have been deleted. Read the passage carefully and
select the most appropriate option to fill in each blank.
Agriculture, with its (1)________ sectors, is unquestionably the largest livelihood provider in
India, more so in the vast rural areas. It also (2)________ a significant figure to the Gross
ag

Domestic Product (GDP). Sustainable agriculture, in (3)________ food security, rural


employment and environmentally sustainable technologies, such as soil conservation,
sustainable natural resource management and biodiversity protection, are essential
(4)________ holistic rural development. Indian agriculture and allied activities (5)________
witnessed a green revolution, a white revolution, a yellow revolution and a blue revolution.
G

SubQuestion No : 16
Q.16 Select the most appropriate option to fill in blank no. 1.
Ans 1. social

2. another

3. secured

4. allied

Question ID : 264330132889
Status : Answered
Chosen Option : 4
Join @MathsByGaganPratap Telegram Channel for free PDFs
Comprehension:
In the following passage, some words have been deleted. Read the passage carefully and
select the most appropriate option to fill in each blank.
Agriculture, with its (1)________ sectors, is unquestionably the largest livelihood provider in
India, more so in the vast rural areas. It also (2)________ a significant figure to the Gross
Domestic Product (GDP). Sustainable agriculture, in (3)________ food security, rural
employment and environmentally sustainable technologies, such as soil conservation,
sustainable natural resource management and biodiversity protection, are essential
(4)________ holistic rural development. Indian agriculture and allied activities (5)________
witnessed a green revolution, a white revolution, a yellow revolution and a blue revolution.

SubQuestion No : 17
Q.17 Select the most appropriate option to fill in blank no. 2.
Ans 1. assume

2. insist

hs
3. contributes

4. insight

Question ID : 264330132890

at
Status : Answered
Chosen Option : 3

M
Comprehension:
In the following passage, some words have been deleted. Read the passage carefully and
select the most appropriate option to fill in each blank.
Agriculture, with its (1)________ sectors, is unquestionably the largest livelihood provider in
India, more so in the vast rural areas. It also (2)________ a significant figure to the Gross
Domestic Product (GDP). Sustainable agriculture, in (3)________ food security, rural
ap
employment and environmentally sustainable technologies, such as soil conservation,
sustainable natural resource management and biodiversity protection, are essential
(4)________ holistic rural development. Indian agriculture and allied activities (5)________
witnessed a green revolution, a white revolution, a yellow revolution and a blue revolution.

SubQuestion No : 18
at
Q.18 Select the most appropriate option to fill in blank no. 3.
Ans 1. condition of

2. time of

3. spite of
Pr

4. terms of

Question ID : 264330132891
Status : Answered
Chosen Option : 4
an

Comprehension:
In the following passage, some words have been deleted. Read the passage carefully and
select the most appropriate option to fill in each blank.
Agriculture, with its (1)________ sectors, is unquestionably the largest livelihood provider in
ag

India, more so in the vast rural areas. It also (2)________ a significant figure to the Gross
Domestic Product (GDP). Sustainable agriculture, in (3)________ food security, rural
employment and environmentally sustainable technologies, such as soil conservation,
sustainable natural resource management and biodiversity protection, are essential
(4)________ holistic rural development. Indian agriculture and allied activities (5)________
witnessed a green revolution, a white revolution, a yellow revolution and a blue revolution.
G

SubQuestion No : 19
Q.19 Select the most appropriate option to fill in blank no. 4.
Ans 1. in

2. on

3. for

4. as

Question ID : 264330132892
Status : Answered
Chosen Option : 3
Join @MathsByGaganPratap Telegram Channel for free PDFs
Comprehension:
In the following passage, some words have been deleted. Read the passage carefully and
select the most appropriate option to fill in each blank.
Agriculture, with its (1)________ sectors, is unquestionably the largest livelihood provider in
India, more so in the vast rural areas. It also (2)________ a significant figure to the Gross
Domestic Product (GDP). Sustainable agriculture, in (3)________ food security, rural
employment and environmentally sustainable technologies, such as soil conservation,
sustainable natural resource management and biodiversity protection, are essential
(4)________ holistic rural development. Indian agriculture and allied activities (5)________
witnessed a green revolution, a white revolution, a yellow revolution and a blue revolution.

SubQuestion No : 20
Q.20 Select the most appropriate option to fill in blank no. 5.
Ans 1. being

2. will

hs
3. have

4. be

Question ID : 264330132893

at
Status : Answered
Chosen Option : 3

M
Comprehension:
Read the given passage and answer the questions that follow.
People often become helpless while dealing with anxiety. Anxiety can be caused by several
factors, some of which can emerge from hypertension, the inability to share one's thoughts,
sudden contextual differences, broken relationships, work stress, and so on. The fear of
missing out, also known as FOMO, leading to anxiety issues is mostly prevalent among
ap
youngsters nowadays. Meditation, physical exercises, and proper sleep cycles, among other
such productive habits, can help cure anxiety. Most importantly, one must respect and love
oneself, which in turn will help in boosting self-confidence and positive energy. A person can
be one's own therapist if he/she sits back for a while contemplating and meditating. A deep
breath, a calm mind, and a healthy body helps to inculcate positivity, which otherwise gets
cancelled out due to several blocks of negativity created by limited mindsets.
at
SubQuestion No : 21
Q.21 Which of the following is NOT a productive habit, according to the passage?
Ans 1. Brushing teeth
Pr

2. Proper sleep cycles

3. Physical exercise

4. Meditation 

Question ID : 264330134168
an

Status : Answered
Chosen Option : 1
ag
G
Join @MathsByGaganPratap Telegram Channel for free PDFs
Comprehension:
Read the given passage and answer the questions that follow.
People often become helpless while dealing with anxiety. Anxiety can be caused by several
factors, some of which can emerge from hypertension, the inability to share one's thoughts,
sudden contextual differences, broken relationships, work stress, and so on. The fear of
missing out, also known as FOMO, leading to anxiety issues is mostly prevalent among
youngsters nowadays. Meditation, physical exercises, and proper sleep cycles, among other
such productive habits, can help cure anxiety. Most importantly, one must respect and love
oneself, which in turn will help in boosting self-confidence and positive energy. A person can
be one's own therapist if he/she sits back for a while contemplating and meditating. A deep
breath, a calm mind, and a healthy body helps to inculcate positivity, which otherwise gets
cancelled out due to several blocks of negativity created by limited mindsets.

SubQuestion No : 22
Q.22 The given passage is mainly based _________.
Ans 1. On Positivity 

hs
2. On Anxiety 

3. On FOMO 

4. On Negativity

at
Question ID : 264330134166
Status : Answered
Chosen Option : 2

M
Comprehension:
Read the given passage and answer the questions that follow.
People often become helpless while dealing with anxiety. Anxiety can be caused by several
factors, some of which can emerge from hypertension, the inability to share one's thoughts,
ap
sudden contextual differences, broken relationships, work stress, and so on. The fear of
missing out, also known as FOMO, leading to anxiety issues is mostly prevalent among
youngsters nowadays. Meditation, physical exercises, and proper sleep cycles, among other
such productive habits, can help cure anxiety. Most importantly, one must respect and love
oneself, which in turn will help in boosting self-confidence and positive energy. A person can
be one's own therapist if he/she sits back for a while contemplating and meditating. A deep
breath, a calm mind, and a healthy body helps to inculcate positivity, which otherwise gets
at
cancelled out due to several blocks of negativity created by limited mindsets.

SubQuestion No : 23
Q.23 What is the primary inference that can be made by reading this passage?
Pr

Ans 1. Anxiety can be defeated by a positive attitude and productive practices.

2. There is no clinical cure to anxiety and one must always live with it. 

3. FOMO is the main reason why people experience anxiety. 

4. Anxiety is very harmful for our emotional as well as physical health and cannot be
dealt with individually.
an

Question ID : 264330134170
Status : Answered
Chosen Option : 1
ag
G
Join @MathsByGaganPratap Telegram Channel for free PDFs
Comprehension:
Read the given passage and answer the questions that follow.
People often become helpless while dealing with anxiety. Anxiety can be caused by several
factors, some of which can emerge from hypertension, the inability to share one's thoughts,
sudden contextual differences, broken relationships, work stress, and so on. The fear of
missing out, also known as FOMO, leading to anxiety issues is mostly prevalent among
youngsters nowadays. Meditation, physical exercises, and proper sleep cycles, among other
such productive habits, can help cure anxiety. Most importantly, one must respect and love
oneself, which in turn will help in boosting self-confidence and positive energy. A person can
be one's own therapist if he/she sits back for a while contemplating and meditating. A deep
breath, a calm mind, and a healthy body helps to inculcate positivity, which otherwise gets
cancelled out due to several blocks of negativity created by limited mindsets.

SubQuestion No : 24
Q.24 What is the full form of FOMO?
Ans 1. Fear of Moving Out

hs
2. Fear of Moping on

3. Fear of Missing Out

4. Fear of Managing Overstress

at
Question ID : 264330134167
Status : Answered
Chosen Option : 3

M
Comprehension:
Read the given passage and answer the questions that follow.
People often become helpless while dealing with anxiety. Anxiety can be caused by several
factors, some of which can emerge from hypertension, the inability to share one's thoughts,
ap
sudden contextual differences, broken relationships, work stress, and so on. The fear of
missing out, also known as FOMO, leading to anxiety issues is mostly prevalent among
youngsters nowadays. Meditation, physical exercises, and proper sleep cycles, among other
such productive habits, can help cure anxiety. Most importantly, one must respect and love
oneself, which in turn will help in boosting self-confidence and positive energy. A person can
be one's own therapist if he/she sits back for a while contemplating and meditating. A deep
breath, a calm mind, and a healthy body helps to inculcate positivity, which otherwise gets
at
cancelled out due to several blocks of negativity created by limited mindsets.

SubQuestion No : 25
Q.25 Which of the following best defines the word ‘inculcate’ as used in the passage?
Pr

Ans 1. Cancel 

2. Scream

3. Develop

4. Break
an

Question ID : 264330134169
Status : Answered
Chosen Option : 3
ag
G
Join @MathsByGaganPratap Telegram Channel for free PDFs

Phase-XI/2023/Selection Posts and Selection Posts/Ladakh/2023


Roll Number Gagan Pratap Maths
Candidate Name Gagan Pratap Maths
Venue Name iON Digital Zone iDZ 1 Mathura Road
Exam Date 30/06/2023
Exam Time 11:45 AM - 12:45 PM
Subject Selection Post Higher Secondary Level

Section : General Intelligence

hs
Q.1 Which of the following numbers will replace the question mark (?) in the given series?
18, 54, 29, 87, 62, 186, ?
Ans 1. 174

2. 154

at
3. 161

4. 156

Question ID : 264330142401

M
Status : Answered
Chosen Option : 3

Q.2 Which of the following interchanges of signs would make the given equation correct?

Ans
288 × 72 + 36 ÷ 12 - 6 = 20770
1. × and −
ap
2. − and ÷

3. + and ×
at
4. ÷ and +

Question ID : 26433057930
Status : Answered
Pr

Chosen Option : 2

Q.3 Read the given statements and conclusions carefully. Assuming that the information
given in the statements is true, even if it appears to be at variance with commonly
known facts, decide which of the given conclusions logically follow from the
statements.
an

Statements:
All streams are rivers.
All ponds are rivers.
All rivers are oceans.

Conclusions:
ag

I. Some oceans are streams.


II. All ponds are oceans.
III. Some oceans are not rivers.
Ans 1. Only conclusions I and II follow

2. Only conclusions II and III follow


G

3. Only conclusions I and III follow

4. Conclusions I, II and III follow

Question ID : 264330142511
Status : Answered
Chosen Option : 4
Join @MathsByGaganPratap Telegram Channel for free PDFs
Q.4 Select the correct option that indicates the arrangement of the given words in the
order in which they appear in an English dictionary.
1. Glacier
2. Gloomy
3. Glossy
4. Glorious
5. Glitter
Ans 1. 1,5,2,4,3

2. 1,2,4,3,5

3. 1,3,2,4,5

4. 1,5,4,2,3

Question ID : 26433057964

hs
Status : Answered
Chosen Option : 1

Q.5 In a certain code language, ‘OFFCUTS’ is written as ‘LUUXDEF’ and ‘PARADOX’ is


written as ‘KZIZUJA’. How will ‘NAILING’ be written in that language?

at
Ans 1.  CSAWQMI

2.  VTGDESF

3.  BJMUROL

M
4. MZROPKR

Question ID : 264330118648
Status : Answered
ap Chosen Option : 4

Q.6 Select the option that represents the correct order of the given words as they would
appear in an English dictionary.
1.Bachelor
2.Backdoor
at
3.Bacteria
4.Backbone
5.Badgered
Ans 1. 1,3,2,4,5
Pr

2. 1,2,3,4,5

3. 2,1,3,4,5

4. 1,4,2,3,5

Question ID : 264330142447
an

Status : Answered
Chosen Option : 4

Q.7 Select the option that is related to the third word in the same way as the second word
is related to the first word. (The words must be considered as meaningful English
ag

words and must not be related to each other based on the number of letters / number
of consonants / vowels in the word.)
Omelette : Egg :: Metal : ?
Ans 1. Wood 

2. Latex 
G

3.  Axe

4. Ore 

Question ID : 26433056587
Status : Answered
Chosen Option : 4
Join @MathsByGaganPratap Telegram Channel for free PDFs
Q.8 यदि,
'A % Z' का अर्थ है 'A, Z का पुत्र है',
‘A & Z' का अर्थ है ‘A, Z के पिता है',
'A × Z' का अर्थ है 'A, Z की बहन है',
‘A # Z' का अर्थ है ‘A, Z की माता है',
‘A $ Z' का अर्थ है ‘A, Z की पुत्री है',
‘A @ Z' का अर्थ है ‘A, Z की पुत्रवधू है',
तो दिए गए व्‍यंजक में, C का D से क्‍या संबंध है?
C$A#E&B%G@D
Ans 1. पत्नी

2. भाई

3. बहन

4. पुत्री

hs
Question ID : 264330142507
Status : Answered
Chosen Option : 4

at
Q.9 Select the option that is related to the fifth word in the same way as the second word
is related to the first word and the fourth word is related to the third word. (The words
must be considered as meaningful English words and must NOT be related to each
other based on the number of letters/number of consonants/vowels in the word)

M
APPOINT : APPOINTMENT :: ARRANGE : ARRANGEMENT :: ADJUST : ?
Ans 1. ADJUSTMENT

2. ANNOUNCEMENT

3. ACKNOWLEDGEMENT

4. ADVANCEMENT
ap
Question ID : 264330142470
Status : Answered
at
Chosen Option : 1

Q.10 In a code language, 'STATE' is written as ‘19-20-1-20-5’ and ‘HOME’ is written as ‘8-15-
13-5’. How will 'TELEVISION' be written in that language?
Pr

Ans 1. 5-20-5-12-9-22-1-19-14-15 

2. 20-5-12-5-22-9-19-9-15-14 

3. 19-4-11-4-21-8-18-0-14-13 

4.  21-6-13-6-23-10-20-2-16-15
an

Question ID : 26433068446
Status : Answered
Chosen Option : 2

Q.11 Select the option that is related to the fifth term in the same way as the second term is
ag

related to the first term and fourth term is related to third term.
SHIRT : RGISU :: KNIFE : JMIGF :: CHAIR : ?
Ans 1. CGAJS

2. BGBJS
G

3. CGBJS

4. BGAJS

Question ID : 264330142457
Status : Answered
Chosen Option : 4
Join @MathsByGaganPratap Telegram Channel for free PDFs
Q.12 Select the combination of letters that when sequentially placed from left to right in the
blanks of the given series will complete the letter series.
G__ABGHI__G_I_B_HI_B
Ans 1. H I A B H A G A

2. I H A B H A G A

3. I H A B H A A G

4. H I B A H A A G

Question ID : 264330142487
Status : Answered
Chosen Option : 1

hs
Q.13

at
Ans 1.

M
2.

3. ap
4.

Question ID : 264330142428
Status : Answered
Chosen Option : 1
at

Q.14 Select the combination of letters that when sequentially placed in the blanks of the
given series will complete the series.
CFL, EIK, GLJ, IOI, _ _ _
Pr

Ans 1. KRH

2. KQH

3. JRH

4. KRK
an

Question ID : 26433078821
Status : Answered
Chosen Option : 1
ag
G
Join @MathsByGaganPratap Telegram Channel for free PDFs
Q.15 In this question, three statements are given, followed by two conclusions numbered I
and II. Assuming the statements to be true, even if they seem to be at variance with
commonly known facts, decide which of the conclusions logically follows/follow from
the statements.
Statements:
I. All pens are sketches.
II. All sketches are books.
III. Some books are colours.
Conclusions:
I. Some books are pens.
II. Some colours are sketches.
Ans 1. Only conclusion I follows

2. Only conclusion II follows

3. Neither conclusion I nor II follows

hs
4. Both conclusions I and II follow

Question ID : 264330104059
Status : Answered
Chosen Option : 1

at
Q.16 Which of the following numbers will replace the question mark (?) in the given series?
287, 195, 286, 189, 284, 184, 281, 180, ?
Ans 1. 197

M
2. 177

3. 277

4. 265 ap Question ID : 264330142404


Status : Answered
Chosen Option : 3
at
Pr
an
ag
G
Join @MathsByGaganPratap Telegram Channel for free PDFs
Q.17

Ans

1.

hs
at
2.

M
3.
ap
at

4.
Pr

Question ID : 264330142412
Status : Answered
an

Chosen Option : 2

Q.18 'A + B' का अर्थ है 'A, B का पुत्र है'।


'A - B' का अर्थ है 'A, B की माता है'।
'A × B' का अर्थ है 'A, B का पिता है'।
ag

'A ÷ B' का अर्थ है 'A, B की बहन है'।


यदि J ÷ K × L + M – N है, तो निम्नलिखित में से कौन सा कथन सही नहीं है?
Ans 1. K, N का पिता है।

2. K, M का पति है।
G

3. J, L की माता है।

4. L, N का भाई है।

Question ID : 264330103771
Status : Answered
Chosen Option : 3
Join @MathsByGaganPratap Telegram Channel for free PDFs
Q.19 Select the correct option that indicates the arrangement of the following words in a
logical and meaningful order.
1. Month
2. Day
3. Decade
4. Year
5. Century
Ans 1. 3, 5, 4, 1, 2

2. 5, 4, 3, 1, 2

3. 5, 3, 4, 1, 2

4. 3, 5, 1, 4, 2

Question ID : 264330129474

hs
Status : Answered
Chosen Option : 3

Q.20 In a code language, 'BOARD' is written as ‘40’ and ‘TIME’ is written as ‘47’. How will
'CUDDLE' be written in that language?

at
Ans 1. 49

2. 30

3. 33

M
4. 35

Question ID : 26433068447
Status : Answered
ap Chosen Option : 1

Q.21
at

Ans 1. 8
Pr

2. 6

3. 5

4. 7
an

Question ID : 264330142442
Status : Answered
Chosen Option : 2

Q.22 ‘F + C’ का अर्थ है कि ‘F, C की बहन है’,


ag

‘F – C’ का अर्थ है कि ‘F, C का पति है’,


‘F × C’ का अर्थ है कि ‘F, C के पिता है’,
‘F ÷ C’ का अर्थ है कि ‘F, C की पुत्री है’।

यदि ‘R+ T÷P × Q’ है, तो P का R से क्‍या रिश्‍ता है?


Ans 1. भाई
G

2. बहन

3. माता

4. पिता

Question ID : 264330142500
Status : Answered
Chosen Option : 4
Join @MathsByGaganPratap Telegram Channel for free PDFs
Q.23 Select the option that is related to the fifth letter-cluster in the same way as the second
letter-cluster is related to the first letter-cluster and the fourth letter-cluster is related
to the third letter-cluster.
ISLE : GNVL :: JUMP : ROXM :: KATE : ?
Ans 1. GVDN

2. GVEM

3. GVDM

4. GVEN

Question ID : 26433056192
Status : Answered
Chosen Option : 1

hs
Q.24 In a certain code language, 'ALUMNUS' is written as 'ZOFMMFH' and ‘EMINENT’ is
written as ‘VNRNVMG’. How will 'HAPPENS' be written in that language?
Ans 1. SZKPVMH

2. SZLQVMH

at
3. SZKQVMH

4. SZLPVMH

M
Question ID : 26433097291
Status : Answered
Chosen Option : 1
ap
Q.25 In a certain code language, 'MATURE' is coded as '7 26 14 22 9 6', and 'OUTPUT' is
coded as '7 6 12 7 6 11'. How will 'REMOVE' be coded in that language?
Ans 1. 9 12 22 12 22 5

2. 22 14 9 22 12 5

3. 14 22 9 22 5 12
at
4. 12 22 9 5 12 22

Question ID : 264330142445
Pr

Status : Answered
Chosen Option : 1

Section : General Awareness

Q.1 Who among the following musicians was awarded India's highest civilian honour, the
an

Bharat Ratna, in 1999?


Ans 1. Ustad Zakir Hussain 

2. Pandit Ram Narayan 

3. Ustad Amjad Ali Khan Bangash 


ag

4. Pandit Ravi Shankar 

Question ID : 264330141606
Status : Answered
Chosen Option : 4
G

Q.2 Ganesh Chaturthi is observed in __________ month of Hindu calendar.


Ans 1. Kartika

2. Ashadha

3. Bhadrapada

4. Vaishakh

Question ID : 26433064328
Status : Answered
Chosen Option : 3
Join @MathsByGaganPratap Telegram Channel for free PDFs
Q.3 On the occasion of Azadi Ka Amrit Mahotsav, which of the following departments of
the Government of India organised ‘Nari Samagam & Spardha’ – a Women Sports Meet
in January 2023?
Ans 1. Department of Home Affairs

2. Department of Personnel and Training

3. Department of Finance

4. Department of Sports, Arts and Culture

Question ID : 264330132593
Status : Answered
Chosen Option : 4

hs
Q.4 Who among the following was the author of the book ‘Tarikh-i-Firoz Shahi’?
Ans 1. Minhas-us-Siraj

2. Ziauddin Barani

3. Abul Fazal

at
4. Badr-ud-din Muhammad

Question ID : 26433090576

M
Status : Answered
Chosen Option : 3

Q.5 Who among the following is the first Indian girl to win Jr. Squash Open in the US in
December 2021?
Ans 1. Sunayna Kuruvilla  
ap
2. Dipika Karthik  

3. Akanksha Salunkhe

4. Anahat Singh  
at

Question ID : 26433092458
Status : Answered
Chosen Option : 2
Pr

Q.6 Which Indian musician’s autobiography is titled ‘Raga Mala’?


Ans 1. Ravi Shankar

2. Vishwa Mohan Bhatt


an

3. Zakir Hussain

4. Amjad Ali Khan

Question ID : 26433054842
Status : Answered
ag

Chosen Option : 1

Q.7 Which of the following is a Rabi crop in Northern India?


Ans 1. Rice
G

2. Cotton

3. Maize

4. Mustard

Question ID : 26433092870
Status : Answered
Chosen Option : 4
Join @MathsByGaganPratap Telegram Channel for free PDFs
Q.8 Which of the following pair of dances belongs to Kerala?
Ans 1. Sattriya and Bharatanatyam

2. Kathakali and Mohiniyattam

3. Kathakali and Kuchipudi

4. Mohiniyattam and Kathak

Question ID : 264330141616
Status : Answered
Chosen Option : 2

Q.9 What is the ratio by mass of carbon and oxygen in carbon dioxide?

hs
Ans 1. 3 : 8

2. 3 : 13

3. 3 : 1

4. 3 : 2

at
Question ID : 26433059454
Status : Answered
Chosen Option : 1

M
Q.10 Who was appointed as the Union Minister of Law and Justice in July 2021?
Ans 1. Narendra Singh Tomar

2. MN Pandey

3. Rajnath Singh
ap
4. Kiren Rijiju

Question ID : 26433081855
at
Status : Answered
Chosen Option : 1

Q.11 Under _____ a new policy of “paramountcy” was initiated. In this policy the Company
Pr

claimed that its authority was paramount or supreme, hence its power was greater
than that of Indian states.
Ans 1. Charles Cornwallis

2. Thomas Munro

3. Lord Hastings
an

4. Robert Clive

Question ID : 26433082067
Status : Answered
ag

Chosen Option : 4

Q.12 Which union territory has the highest sex ratio value according to the 2011 census?
Ans 1. Daman and Diu

2. Andaman and Nicobar Islands


G

3. Puducherry

4. Delhi

Question ID : 264330109505
Status : Answered
Chosen Option : 3
Join @MathsByGaganPratap Telegram Channel for free PDFs
Q.13 The Kandariya Mahadeva temple, which is dedicated to Shiva and was constructed in
999 by the Chandela dynasty, is an example of which of the following styles of
architecture?
Ans 1. Dravida style 

2. Nagara style 

3. Ahom style 

4.  Pagoda style

Question ID : 264330141648
Status : Answered
Chosen Option : 4

hs
Q.14 Who among the following was the founder and director of the ‘Darpana Academy of
Performing Arts’, an institute that focuses on training students in dance, drama, music
and puppetry, in the city of Ahmedabad?
Ans 1. Amala Shankar

2. Sitara Devi

at
3. Padmini Chettur

4. Mrinalini Sarabhai

M
Question ID : 264330110263
Status : Answered
Chosen Option : 3

Ans 1.  Pluto
ap
Q.15 From the given alternatives, identify the dwarf planet.

2.  Mercury

3. Neptune
at
4.  Mars

Question ID : 264330141927
Status : Answered
Pr

Chosen Option : 1

Q.16 Which of the following is the oldest text of ancient India?


Ans 1. Mahabhashya

2. Arthashastra
an

3. Rig Veda

4. Manu Smriti

Question ID : 264330111364
ag

Status : Answered
Chosen Option : 2

Q.17 The Jhelum River rises from which of the following sources?
Ans 1. Chemayungdung Glacier
G

2. Mansarovar Lake

3. Spring at Verinag

4. Rakas Lake

Question ID : 26433089235
Status : Answered
Chosen Option : 2
Join @MathsByGaganPratap Telegram Channel for free PDFs
Q.18 According to recent UNFPA estimates, 68 per cent of India’s population is between
_____ years of age in 2022.
Ans 1. 10 and 19

2. 10 and 24

3. 15 and 29

4. 15 and 64

Question ID : 264330111347
Status : Answered
Chosen Option : 3

Q.19 How many Fundamental Duties are there in the Indian Constitution?

hs
Ans 1. 11

2. 9

3. 12

at
4. 10

Question ID : 264330110782
Status : Answered

M
Chosen Option : 4

Q.20 भारत में संघ की कार्यपालिका शक्ति _______ को प्राप्त है।


Ans 1. प्रधानमंत्री

2. मुख्य न्यायमूर्ति
ap
3. उप राष्ट्र पति

4. राष्ट्र पति
at
Question ID : 26433055542
Status : Answered
Chosen Option : 4
Pr

Q.21 There shall be a _______ with the Prime Minister at the head to aid and advise the
President of India.
Ans 1. Council of States

2. Council of Ministers

3. Advocate General
an

4. Attorney General for India

Question ID : 264330110596
Status : Answered
ag

Chosen Option : 2

Q.22 Which of the following does NOT belong to class Insecta?


Ans 1. Cockroach

2. Mosquito
G

3. Scorpion

4. Silverfish

Question ID : 26433064138
Status : Answered
Chosen Option : 4
Join @MathsByGaganPratap Telegram Channel for free PDFs
Q.23 What is the distance between the two goal posts in a football match?
Ans 1. 7.32 m

2. 8.32 m

3. 9.32 m

4. 6.32 m

Question ID : 26433054704
Status : Answered
Chosen Option : 2

Q.24 Peeling, cutting or crushing onion tissue releases which enzyme that makes our eyes
water?

hs
Ans 1. Xylanases

2. Bordelaises

3. Allinases

at
4. Pectinases

Question ID : 26433055146
Status : Answered

M
Chosen Option : 1

Q.25 Which of the following women personalities has the honour of being a ‘Double
Medalist’ at the Olympics?
Ans 1. Sakshi Malik

2. Saina Nehwal  
ap
3. Karnam Malleswari

4. P V Sindhu  
at
Question ID : 26433092455
Status : Answered
Chosen Option : 4
Pr

Section : Quantitative Aptitude

Q.1 The value of cosec 41° – sec 49° is:


Ans 1.
an

2.

3.
ag

4.

Question ID : 264330141579
G

Status : Answered
Chosen Option : 2
Join @MathsByGaganPratap Telegram Channel for free PDFs
Q.2 For what value of p does the system of equations 18x + 36y + 45 = 0 and px - 54y + 67 =
0 have no solution?
Ans 1. 54

2. −27

3. −36

4. 27

Question ID : 264330126546
Status : Not Answered
Chosen Option : --

Q.3 विक्टोरिया ने ऐसी वस्तुएँ खरीदीं जिनका संयुक्त अंकित मूल्य $4,00,000 था। उसे नकद में किए गए

hs
भुगतान के हिस्से पर 10% की छू ट और क्रे डिट कार्ड के माध्यम से किए गए भुगतान के हिस्से पर 2%
अधिभार की पेशकश की गई थी। यदि विक्टोरिया ने कु ल $3,78,000 का भुगतान किया, तो उसने
क्रे डिट कार्ड के माध्यम से अधिभार सहित कितना भुगतान किया?
Ans 1. $1,53,000

at
2. $1,58,100

3. $1,63,200

4. $1,47,900

M
Question ID : 264330103268
Status : Answered
Chosen Option : 2
ap
Q.4 Two numbers are in the ratio of 7 : 6. If the difference between their squares is 637,
then the numbers are:
Ans 1. 49 and 42

2. 49 and 12
at
3. 43 and 36

4. 14 and 12

Question ID : 264330141372
Pr

Status : Answered
Chosen Option : 1

Q.5 Two triangles PRS and DEF are similar whose perimeters are 36 cm and 40 cm,
respectively. If length of DE is 10 cm, then what is the length (in cm) of PR?
an

Ans 1. 15

2. 9

3. 8

4. 12
ag

Question ID : 264330130043
Status : Answered
Chosen Option : 2
G

Q.6 A car starts running at an initial speed of 60km/h, with its speed increasing every hour
by 10km/h. How many hours will it take to cover a distance of 1210km?
Ans 1. 12

2. 8

3. 10

4. 11

Question ID : 264330103425
Status : Answered
Chosen Option : 4
Join @MathsByGaganPratap Telegram Channel for free PDFs
Q.7 Muskan buys two kg of pulses for ₹400 and sells it for ₹440. She also uses the weight
of 850 gm instead of 1 kg. What is Muskan’s actual profit percentage?
Ans 1.

2.

3.

4.

hs
Question ID : 264330123976
Status : Answered
Chosen Option : 4

Q.8 13 विद्यार्थियों के औसत अंक 67 हैं। यदि पहले सात विद्यार्थियों के औसत अंक 61 हैं और अंतिम सात

at
विद्यार्थियों के औसत अंक 65 हैं, तो सातवें विद्यार्थी के अंक क्या हैं?
Ans 1. 11

2. 9

M
3. 7

4. 6

ap Question ID : 264330126172
Status : Answered
Chosen Option : 1

Q.9 Mohan divides 14817 by a certain number. If the quotient and the remainder he gets
are 98 and 117, respectively, then the divisor is:
at
Ans 1. 150

2. 145

3. 155
Pr

4. 140

Question ID : 26433090296
Status : Answered
Chosen Option : 2
an

Q.10 The driver of a car travelling at a speed of 60 km/h decides to slow down to 40 km/h
after covering half of the distance. What is the average speed of the car (in km/h)?
Ans 1. 30

2. 36
ag

3. 42

4. 48

Question ID : 26433091530
G

Status : Answered
Chosen Option : 4
Join @MathsByGaganPratap Telegram Channel for free PDFs
Q.11

hs
Ans 1. 210

2. 209

3. 211.2

at
4. 212.5

Question ID : 26433096362
Status : Answered

M
Chosen Option : 1

Q.12 A merchant purchases a smartwatch for ₹1,800 and fixes its list price in such a way
that, after allowing a discount of 10%, he earns a profit of 22%. The list price of the
ap
smartwatch (in ₹) is:
Ans 1. 2440

2. 2450

3. 2520
at
4. 2180

Question ID : 264330102351
Status : Answered
Pr

Chosen Option : 4
an
ag
G
Join @MathsByGaganPratap Telegram Channel for free PDFs
Q.13

hs
at
Ans 1. 30°

2. 60°

3. 90°

M
4. 80°

Question ID : 26433066388
Status : Answered
Chosen Option : 3
ap
Q.14

Ans 1. 10
at
2. 13

3. 7

4. 11
Pr

Question ID : 26433086268
Status : Answered
Chosen Option : 3
an
ag
G
Join @MathsByGaganPratap Telegram Channel for free PDFs
Q.15

hs
Ans 1. 1792

2. 1952

at
3. 1800

4. 1972

M
Question ID : 264330100285
Status : Answered
Chosen Option : 2

Q.16 Two numbers are, respectively, 16% and 30% less than a third number. The ratio of the

Ans
two numbers is:
1. 5 : 8
ap
2. 5 : 7

3. 8 : 5
at
4. 6 : 5

Question ID : 264330103543
Pr

Status : Answered
Chosen Option : 4

Q.17 Anita travelled a distance for 11 minutes at the speed of 35 km/h in an autorikshaw.
She travelled for 11 minutes in a taxi at 55 km/h and finally she travelled for 11 minutes
by bus at 42 km/h to reach home. Find her average speed for the whole journey.
an

Ans 1. 34 km/h

2. 54 km/h

3. 44 km/h

4. 64 km/h
ag

Question ID : 26433063889
Status : Answered
Chosen Option : 3
G
Join @MathsByGaganPratap Telegram Channel for free PDFs
Q.18 The area of the triangle whose sides are 3 cm, 5 cm, and 6 cm is:
Ans
1.

2.

3.

4.

Question ID : 26433081618
Status : Answered

hs
Chosen Option : 2

Q.19 In an election between two candidates, 80% of the voters cast their votes, out of which
10% of the votes were declared invalid. A candidate got 9,936 votes which were 60% of
the total valid votes. Find the total number of voters enrolled in that election.

at
Ans 1. 24000

2. 23000

3. 25500

M
4. 24320

Question ID : 26433083032
Status : Answered
ap Chosen Option : 4

Q.20 The monthly income of a person was Rs.80,000 and his monthly expenditure was
Rs.45,000. Next year, his income increased by 16% and his expenditure increased by
8%. Find the percentage increase in his savings (correct to 2 decimal places).
at
Ans 1. 25.35%

2. 26.29%

3. 30.25%
Pr

4. 44.36%

Question ID : 264330141378
Status : Answered
Chosen Option : 2
an

Q.21 Which of the following is perfectly divisible by 11?


Ans 1. 57464054

2. 57464044

3. 57463822
ag

4. 57463823

Question ID : 264330129983
Status : Answered
G

Chosen Option : 2
Join @MathsByGaganPratap Telegram Channel for free PDFs
Q.22 At what rate per annum will the simple interest on Rs.550 be Rs.33 in 3 years?
Ans 1. 5%

2. 2%

3. 6%

4. 3%

Question ID : 26433075498
Status : Answered
Chosen Option : 2

Q.23 The diagonal (in cm) of a cuboid with dimensions 9cm × 12cm × 20cm is:

hs
Ans 1. 9

2. 12

3. 25

4. 20

at
Question ID : 26433082930
Status : Answered
Chosen Option : 3

M
Q.24

Ans 1. 4
ap
2. 1

3. 8

4. 2
at

Question ID : 264330141410
Status : Answered
Pr

Chosen Option : 3

Q.25 What sum of money amounts to Rs. 2,600 in 2 years and to Rs. 2,700 in 3 years at a
fixed rate of simple interest annually?
Ans 1. Rs. 2,300

2. Rs. 2,200
an

3. Rs. 2,600

4. Rs. 2,400

Question ID : 26433064916
ag

Status : Answered
Chosen Option : 4

Section : English Language


G

Q.1 Select the most appropriate ANTONYM of the underlined word.


My cousin’s fanatical views made him very unpopular with his friends.
Ans 1. Apathetic 

2. Impartial 

3. Weary 

4. Moderate

Question ID : 26433073369
Status : Answered
Chosen Option : 1
Join @MathsByGaganPratap Telegram Channel for free PDFs
Q.2 The following sentence has been split into four segments. Identify the segment that
contains a grammatical error.

After that long afternoon, / when I quietly entered her room / to hand over the packet to
her, / she asked me to sit reluctant
Ans 1. she asked me to sit reluctant

2. After that long afternoon,

3. to hand over the packet to her,

4. when I quietly entered her room

Question ID : 26433055585
Status : Answered
Chosen Option : 2

hs
Q.3 Select the option that expresses the given sentence in direct speech.
Anant said that he was feeling sleepy.
Ans 1. Anant said, “He was feeling sleepy.”

at
2. Anant said, “I was feeling sleepy.”

3. “I was feeling sleepy,” says Anant.

4. Anant said, “I am feeling sleepy.” 

M
Question ID : 264330134143
Status : Answered
ap Chosen Option : 3

Q.4 Select the option that can be used as a one-word substitute for the given group of
words.

Something which lasts forever


Ans 1. Transitory 
at
2. Memorable 

3. Pertinent 

4. Eternal 
Pr

Question ID : 26433073906
Status : Answered
Chosen Option : 4
an

Q.5 Select the option that expresses the given sentence in active voice.
A diamond necklace was to be bought by my wife.
Ans 1. My wife were to buy a diamond necklace.

2. My wife was to buy a diamond necklace.

3. My wife was to buy a necklace of diamond.


ag

4. My wife was buying a diamond necklace.

Question ID : 26433084725
Status : Answered
G

Chosen Option : 2
Join @MathsByGaganPratap Telegram Channel for free PDFs
Q.6 Select the most appropriate synonym of the underlined word.

We always love to get the updated version of gadgets we use.


Ans 1. Latest

2. Refurbished

3. Corrected 

4. Old

Question ID : 26433089383
Status : Answered
Chosen Option : 1

hs
Q.7 Select the most appropriate option to fill in the blank.
______ for an hour made me lose my breath.
Ans 1. Sitting

2. Sleeping

at
3. Walking

4. Eating

M
Question ID : 26433098877
Status : Answered
Chosen Option : 3

Q.8 Select the INCORRECTLY spelt word.


Ans 1. Violence
ap
2. Tyranny

3. Valorous

4. Persuesive
at

Question ID : 26433088410
Status : Answered
Chosen Option : 1
Pr

Q.9 Select the most appropriate ANTONYM of the given word from the following sentence.
Elegant
The table cloth on my brother’s study table was classic and splendid, yet my mother
found it very awkward and odd.
Ans 1. Classic  
an

2. Odd

3. Splendid

4. Awkward
ag

Question ID : 26433072813
Status : Not Answered
Chosen Option : --
G

Q.10 Select the most appropriate option to fill in the blank.


Ravi _______ to my house yesterday.
Ans 1. left

2. came

3. come

4. go

Question ID : 26433098872
Status : Answered
Chosen Option : 2
Join @MathsByGaganPratap Telegram Channel for free PDFs
Q.11 Select the most appropriate meaning of the given idiom.
Hand and glove
Ans 1. Oppressive

2. Intimate

3. Discourage

4. Enemy

Question ID : 26433086332
Status : Answered
Chosen Option : 1

Q.12 Select the most appropriate synonym of the given word.

hs
Paranoia
Ans 1. Assurance

2. Anxiety 

at
3. Belief

4. Faith

M
Question ID : 26433060979
Status : Answered
Chosen Option : 2

Q.13 Identify the sentence that contains no spelling errors.


Ans
ap
1. Drones are imployed for a variety of purposes, including transporting commodities,
acisting in search and rescue efforts, etc. 
2. Drones are employed for a variety of purposes, including transporting commodities,
assisting in search and rescue efforts, etc. 
3. Drones are imployed for a variety of purposes, including transporting commodities,
at
assisting in search and resque efforts, etc. 
4. Drones are employed for a variety of purposes, including transporting comodities,
assisting in search and resque efforts, etc. 
Pr

Question ID : 26433088540
Status : Answered
Chosen Option : 2

Q.14 Select the most appropriate option to fill in the blank.


It is only a rope, she ______ herself, though it looked very much like a snake.
an

Ans 1. retained

2. replied

3. remembered

4. reassured
ag

Question ID : 26433098875
Status : Answered
Chosen Option : 4
G
Join @MathsByGaganPratap Telegram Channel for free PDFs
Q.15 Select the option that expresses the given sentence in direct speech.

Bidding farewell to me, my juniors wished me a bright and successful career ahead.
Ans 1. Bidding farewell to me, my juniors inquired, “Wish you a bright and successful career
ahead.” 
2. Bidding farewell to me, my juniors said to themselves, “Wish you a bright and
successful career ahead.” 
3. Bidding farewell to me, my juniors said to me, “Wish you a bright and successful
career ahead.” 
4. Bidding farewell to me, my juniors wish me, “Wish you a bright and successful career
ahead.”

Question ID : 26433099083
Status : Answered

hs
Chosen Option : 3

Comprehension:
In the following passage, some words have been deleted. Read the passage carefully and

at
select the most appropriate option to fill in each blank.

Many parents think of overweight children as 'cute' and cuddly. However, obesity must not be
(1)_________ as a healthy sign. (2)________ obese children grow up, they will be more
prone (3)____________ health problems like high blood pressure (4)________ diabetes.
Today, overweight children are becoming a common (5)___________ in many countries.

M
SubQuestion No : 16
Q.16 Select the most appropriate option to fill in blank number 1.
Ans 1. seen   ap
2. casted

3. keen

4. wasted
at
Question ID : 26433099099
Status : Answered
Chosen Option : 1
Pr

Comprehension:
In the following passage, some words have been deleted. Read the passage carefully and
select the most appropriate option to fill in each blank.

Many parents think of overweight children as 'cute' and cuddly. However, obesity must not be
(1)_________ as a healthy sign. (2)________ obese children grow up, they will be more
prone (3)____________ health problems like high blood pressure (4)________ diabetes.
an

Today, overweight children are becoming a common (5)___________ in many countries.

SubQuestion No : 17
Q.17 Select the most appropriate option to fill in blank number 2.
Ans 1. When  
ag

2. Then  

3. Here  

4. There  
G

Question ID : 26433099100
Status : Answered
Chosen Option : 1
Join @MathsByGaganPratap Telegram Channel for free PDFs
Comprehension:
In the following passage, some words have been deleted. Read the passage carefully and
select the most appropriate option to fill in each blank.

Many parents think of overweight children as 'cute' and cuddly. However, obesity must not be
(1)_________ as a healthy sign. (2)________ obese children grow up, they will be more
prone (3)____________ health problems like high blood pressure (4)________ diabetes.
Today, overweight children are becoming a common (5)___________ in many countries.

SubQuestion No : 18
Q.18 Select the most appropriate option to fill in blank number 3.
Ans 1. the  

2. on  

3. for  

hs
4. to  

Question ID : 26433099101
Status : Answered

at
Chosen Option : 4

Comprehension:
In the following passage, some words have been deleted. Read the passage carefully and

M
select the most appropriate option to fill in each blank.

Many parents think of overweight children as 'cute' and cuddly. However, obesity must not be
(1)_________ as a healthy sign. (2)________ obese children grow up, they will be more
prone (3)____________ health problems like high blood pressure (4)________ diabetes.
Today, overweight children are becoming a common (5)___________ in many countries.

SubQuestion No : 19
ap
Q.19 Select the most appropriate option to fill in blank number 4.
Ans 1. or  

2. if  
at
3. either  

4. nor  
Pr

Question ID : 26433099102
Status : Answered
Chosen Option : 1

Comprehension:
In the following passage, some words have been deleted. Read the passage carefully and
an

select the most appropriate option to fill in each blank.

Many parents think of overweight children as 'cute' and cuddly. However, obesity must not be
(1)_________ as a healthy sign. (2)________ obese children grow up, they will be more
prone (3)____________ health problems like high blood pressure (4)________ diabetes.
Today, overweight children are becoming a common (5)___________ in many countries.
ag

SubQuestion No : 20
Q.20 Select the most appropriate option to fill in blank number 5.
Ans 1. thing  

2. find  
G

3. solution  

4. sight  

Question ID : 26433099103
Status : Answered
Chosen Option : 4
Join @MathsByGaganPratap Telegram Channel for free PDFs
Comprehension:
Read the given passage and answer the question that follows.
Time is the essence of life. It is the wealth given to us by God. Everyday all of us get 24
hours; nothing less, nothing more. Now, it is up to us how we use it. People who know how to
make the best use of every moment easily open the doors of success in their lives and those
who misuse time get failures one after the other. There is a saying that, ‘‘Spent time and
spent words cannot come back.’’ Everybody is tied by the limits of time. Work wins
appreciation only when it gets completed in a requisite time frame. Beyond the time allotted,
work loses all its utility, however good may have been its quality. Just as falling of rains after
the crops have dried up have no use; likewise, when the time is past the deadline, the work
loses its importance and value. When iron is hot it can be cast into whichever shape we want.
Once it gets cold nothing can be made out of it, however much we may beat the same. In the
same way, a person who has learnt to wisely use his time and has understood how to cast
himself according to the need of the time has actually learnt the true mantra of life.
Shakespeare wrote in one of his plays, ‘‘I wasted time, and now doth time waste me.’’ It is
true that a person who wastes even a little bit of time loses wonderful opportunities that he

hs
could have otherwise availed of by utilising that time. All the great leaders have one thing in
common. They wisely utilise every moment of their time. When other people are busy in
wasting time in laziness, great men are busy in making plans for the future. There is no
person in history who wasted his time and still managed to achieve greatness. Therefore,
there is great need to manage time wisely. ‘‘Careful Time Management’’ refers to careful
planning of time and sincere execution of this plan. This is the only mantra to attain success
in any endeavour. A simple change in outlook and some modification in one’s daily routine

at
can result in a big jump forward towards greater achievements.

SubQuestion No : 21
Q.21 Select the most appropriate synonym of the word ‘endeavour’ given in the second
paragraph.

M
Ans 1. Quit

2. Idle

3. Drop ap
4. Attempt

Question ID : 264330121507
Status : Answered
Chosen Option : 4
at
Comprehension:
Read the given passage and answer the question that follows.
Time is the essence of life. It is the wealth given to us by God. Everyday all of us get 24
Pr

hours; nothing less, nothing more. Now, it is up to us how we use it. People who know how to
make the best use of every moment easily open the doors of success in their lives and those
who misuse time get failures one after the other. There is a saying that, ‘‘Spent time and
spent words cannot come back.’’ Everybody is tied by the limits of time. Work wins
appreciation only when it gets completed in a requisite time frame. Beyond the time allotted,
work loses all its utility, however good may have been its quality. Just as falling of rains after
the crops have dried up have no use; likewise, when the time is past the deadline, the work
an

loses its importance and value. When iron is hot it can be cast into whichever shape we want.
Once it gets cold nothing can be made out of it, however much we may beat the same. In the
same way, a person who has learnt to wisely use his time and has understood how to cast
himself according to the need of the time has actually learnt the true mantra of life.
Shakespeare wrote in one of his plays, ‘‘I wasted time, and now doth time waste me.’’ It is
true that a person who wastes even a little bit of time loses wonderful opportunities that he
could have otherwise availed of by utilising that time. All the great leaders have one thing in
ag

common. They wisely utilise every moment of their time. When other people are busy in
wasting time in laziness, great men are busy in making plans for the future. There is no
person in history who wasted his time and still managed to achieve greatness. Therefore,
there is great need to manage time wisely. ‘‘Careful Time Management’’ refers to careful
planning of time and sincere execution of this plan. This is the only mantra to attain success
in any endeavour. A simple change in outlook and some modification in one’s daily routine
can result in a big jump forward towards greater achievements.
G

SubQuestion No : 22
Q.22 What should be done in order to achieve success in whatever you do?
Ans 1. Doing quality work

2. Planning of one’s time

3. Keeping your routine same

4. Keeping your outlook fixed

Question ID : 264330121504
Status : Answered
Chosen Option : 1
Join @MathsByGaganPratap Telegram Channel for free PDFs
Comprehension:
Read the given passage and answer the question that follows.
Time is the essence of life. It is the wealth given to us by God. Everyday all of us get 24
hours; nothing less, nothing more. Now, it is up to us how we use it. People who know how to
make the best use of every moment easily open the doors of success in their lives and those
who misuse time get failures one after the other. There is a saying that, ‘‘Spent time and
spent words cannot come back.’’ Everybody is tied by the limits of time. Work wins
appreciation only when it gets completed in a requisite time frame. Beyond the time allotted,
work loses all its utility, however good may have been its quality. Just as falling of rains after
the crops have dried up have no use; likewise, when the time is past the deadline, the work
loses its importance and value. When iron is hot it can be cast into whichever shape we want.
Once it gets cold nothing can be made out of it, however much we may beat the same. In the
same way, a person who has learnt to wisely use his time and has understood how to cast
himself according to the need of the time has actually learnt the true mantra of life.
Shakespeare wrote in one of his plays, ‘‘I wasted time, and now doth time waste me.’’ It is
true that a person who wastes even a little bit of time loses wonderful opportunities that he

hs
could have otherwise availed of by utilising that time. All the great leaders have one thing in
common. They wisely utilise every moment of their time. When other people are busy in
wasting time in laziness, great men are busy in making plans for the future. There is no
person in history who wasted his time and still managed to achieve greatness. Therefore,
there is great need to manage time wisely. ‘‘Careful Time Management’’ refers to careful
planning of time and sincere execution of this plan. This is the only mantra to attain success
in any endeavour. A simple change in outlook and some modification in one’s daily routine

at
can result in a big jump forward towards greater achievements.

SubQuestion No : 23
Q.23 What do great leaders have in common?

M
Ans 1. They follow a fixed daily routine.

2. They are not tied by limits of time.

3. They learn from failures.

4. They plan and use their time wisely.


ap Question ID : 264330121505
Status : Answered
Chosen Option : 1
at
Comprehension:
Read the given passage and answer the question that follows.
Time is the essence of life. It is the wealth given to us by God. Everyday all of us get 24
hours; nothing less, nothing more. Now, it is up to us how we use it. People who know how to
Pr

make the best use of every moment easily open the doors of success in their lives and those
who misuse time get failures one after the other. There is a saying that, ‘‘Spent time and
spent words cannot come back.’’ Everybody is tied by the limits of time. Work wins
appreciation only when it gets completed in a requisite time frame. Beyond the time allotted,
work loses all its utility, however good may have been its quality. Just as falling of rains after
the crops have dried up have no use; likewise, when the time is past the deadline, the work
loses its importance and value. When iron is hot it can be cast into whichever shape we want.
an

Once it gets cold nothing can be made out of it, however much we may beat the same. In the
same way, a person who has learnt to wisely use his time and has understood how to cast
himself according to the need of the time has actually learnt the true mantra of life.
Shakespeare wrote in one of his plays, ‘‘I wasted time, and now doth time waste me.’’ It is
true that a person who wastes even a little bit of time loses wonderful opportunities that he
could have otherwise availed of by utilising that time. All the great leaders have one thing in
common. They wisely utilise every moment of their time. When other people are busy in
ag

wasting time in laziness, great men are busy in making plans for the future. There is no
person in history who wasted his time and still managed to achieve greatness. Therefore,
there is great need to manage time wisely. ‘‘Careful Time Management’’ refers to careful
planning of time and sincere execution of this plan. This is the only mantra to attain success
in any endeavour. A simple change in outlook and some modification in one’s daily routine
can result in a big jump forward towards greater achievements.
G

SubQuestion No : 24
Q.24 Identify the tone of the passage.
Ans 1. Realistic

2. Sarcastic

3. Impractical

4. Aggressive

Question ID : 264330121506
Status : Answered
Chosen Option : 1
Join @MathsByGaganPratap Telegram Channel for free PDFs
Comprehension:
Read the given passage and answer the question that follows.
Time is the essence of life. It is the wealth given to us by God. Everyday all of us get 24
hours; nothing less, nothing more. Now, it is up to us how we use it. People who know how to
make the best use of every moment easily open the doors of success in their lives and those
who misuse time get failures one after the other. There is a saying that, ‘‘Spent time and
spent words cannot come back.’’ Everybody is tied by the limits of time. Work wins
appreciation only when it gets completed in a requisite time frame. Beyond the time allotted,
work loses all its utility, however good may have been its quality. Just as falling of rains after
the crops have dried up have no use; likewise, when the time is past the deadline, the work
loses its importance and value. When iron is hot it can be cast into whichever shape we want.
Once it gets cold nothing can be made out of it, however much we may beat the same. In the
same way, a person who has learnt to wisely use his time and has understood how to cast
himself according to the need of the time has actually learnt the true mantra of life.
Shakespeare wrote in one of his plays, ‘‘I wasted time, and now doth time waste me.’’ It is
true that a person who wastes even a little bit of time loses wonderful opportunities that he

hs
could have otherwise availed of by utilising that time. All the great leaders have one thing in
common. They wisely utilise every moment of their time. When other people are busy in
wasting time in laziness, great men are busy in making plans for the future. There is no
person in history who wasted his time and still managed to achieve greatness. Therefore,
there is great need to manage time wisely. ‘‘Careful Time Management’’ refers to careful
planning of time and sincere execution of this plan. This is the only mantra to attain success
in any endeavour. A simple change in outlook and some modification in one’s daily routine

at
can result in a big jump forward towards greater achievements.

SubQuestion No : 25
Q.25 Select the most appropriate ANTONYM of the word ‘laziness’ given in the second
paragraph.

M
Ans 1. Indolent

2. Inactive

3. Industrious ap
4. Lethargic

Question ID : 264330121503
Status : Answered
Chosen Option : 4
at
Pr
an
ag
G
Join @MathsByGaganPratap Telegram Channel for free PDFs

Phase-XI/2023/Selection Posts and Selection Posts/Ladakh/2023


Roll Number Gagan Pratap Maths
Candidate Name Gagan Pratap Maths
Venue Name ION DIGITAL ZONE IDZ TUPUDANA
Exam Date 30/06/2023
Exam Time 2:30 PM - 3:30 PM
Subject Selection Post Matriculation Level

Section : General Intelligence

hs
Q.1 Three statements are given followed by three conclusions numbered I, II and III.
Assuming the statements to be true, even if they seem to be at variance with
commonly known facts, decide which of the conclusions logically follow(s) from the
statements.

Statements:

at
All rackets are lotions.
All lotions are wickets.
Some tables are wickets.

Conclusions:

M
I. Some wickets are rackets.
II. Some tables are lotions.
III. All lotions are rackets.
Ans 1. Only conclusions I and III follow

2. Only conclusions I and II follow

3. Only conclusion II follows


ap
4. Only conclusion I follows

Question ID : 264330142959
at
Status : Answered
Chosen Option : 4

Q.2 Select the option that indicates the correct arrangement of the given words in a logical
Pr

and meaningful order.


1. Chapter
2. Library
3. Book
4. Page
5. Sentence
Ans 1. 2, 3, 1, 4, 5
an

2. 5, 2, 3, 1, 4

3. 4, 5, 3, 1, 2

4. 3, 1, 2, 4, 5
ag

Question ID : 264330125994
Status : Answered
Chosen Option : 1
G
Join @MathsByGaganPratap Telegram Channel for free PDFs
Q.3 ‘A # B’ means ‘A is the brother of B’.
‘A @ B’ means ‘A is the daughter of B’.
‘A & B’ means ‘A is the husband of B’.
‘A % B’ means ‘A is the wife of B’.
If S % D # F @ G & H @ J, then how is F related to J?
Ans 1. Sister’s daughter

2. Daughter’s daughter

3. Son’s wife

4. Daughter

Question ID : 264330142901
Status : Answered
Chosen Option : 2

hs
Q.4 Select the option that represents the letters that, when sequentially placed from left to
right in the blanks below, will complete the letter series.

A_ M X _ Z _ X A Z _ _

at
Ans 1. Z A M M X

2. Z A X M X

3. Z A M M A

M
4. X A M M X

Question ID : 264330142850
Status : Answered
ap Chosen Option : 1

Q.5 Select the option that represents the letters which when sequentially placed from left
to right in the blanks below will complete the letter series.
B__CV_NM_VB_ME_BNM__
at
Ans 1. N M B D N W F V

2. N M B D N V F V

3. N M B C N W F V
Pr

4. N M B C N V F V

Question ID : 26433067545
Status : Answered
Chosen Option : 2
an

Q.6 Select the correct combination of mathematical signs to sequentially replace the *
signs and to balance the given equation.
193 * 2 * 2 * 56 * 107 * 142
Ans 1. ×, ÷, +, –, =

2. ×, ÷, ×, –, =
ag

3. –, +, ÷, ×, =

4. ÷, +, –, ×, =

Question ID : 264330142887
G

Not Attempted and


Status :
Marked For Review
Chosen Option : --
Join @MathsByGaganPratap Telegram Channel for free PDFs
Q.7 Three statements are followed by three conclusions numbered I, II and III. You have to
consider these statements to be true, even if they seem to be at variance with
commonly known facts. Decide which of the given conclusions logically follow(s) from
the given statements.

Statement:
All papers are pulps.
All pulps are branches.
All branches are trees.

Conclusions:
(I) Some trees are papers.
(II) All pulps are papers.
(III) No pulp is a paper.
Ans 1. Only conclusion I follows

2. Either conclusion I or conclusion III follows

hs
3. None of the conclusions follow

4. Only conclusion II follows

at
Question ID : 264330142974
Status : Answered
Chosen Option : 1

M
Q.8 Three statements are given, followed by three conclusions numbered I, II and III.
Assuming the statements to be true, even if they seem to be at variance with
commonly known facts, decide which of the conclusions logically follow(s) from the
statements.

Statements:
All trucks are buses.
Some buses are bikes.
All bikes are planes.
ap
Conclusions:
I. Some planes are buses.
II. Some buses are trucks.
at
III. All planes are trucks.
Ans 1. Only conclusions I and III follow

2. Only conclusions II and III follow


Pr

3. All the conclusions follow

4. Only conclusions I and II follow

Question ID : 26433092646
Status : Answered
an

Chosen Option : 4

Q.9 उस विकल्प का चयन कीजिए जो तीसरे शब्द से उसी प्रकार संबंधित है जिस प्रकार दू सरा शब्द पहले
शब्द से संबंधित है। (शब्दों को अर्थपूर्ण हिंदी शब्दों के रूप में माना जाना चाहिए और शब्द में अक्षरों की
संख्या/व्यंजनों/स्वरों की संख्या के आधार पर वे एक-दू सरे से संबंधित नहीं होने चाहिए।)
ag

एं थ्रोपोलॉजी (Anthropology) : मनुष्य :: कॉन्कोलॉजी (Conchology) : ?


Ans 1. शंख/शीप (Shells)

2. खोपड़ी (Skull)

3. मिट्टी (Soil)
G

4. नाम (Names)

Question ID : 264330142870
Status : Answered
Chosen Option : 2
Join @MathsByGaganPratap Telegram Channel for free PDFs
Q.10 Select the option that is related to the fifth term in the same way as the second term is
related to the first term and the fourth term is related to the third term.
HOPE : QFIP :: CUTE : UFDV :: MARS : ?
Ans 1. SRAM

2. TQLB

3. RSMA

4. STNB

Question ID : 264330142839
Status : Answered
Chosen Option : 4

hs
Q.11 Select the option that represents the letters that, when sequentially placed from left to
right in the blanks below, will complete the letter series.

_M__U_D__M_CU_DC
Ans 1. U D C M C U D M

at
2. U D C M C U D U

3. U U C M C U D M

4. U D C U C U D M

M
Question ID : 264330142854
Status : Answered
ap Chosen Option : 1

Q.12 In a certain code language, ‘CREDIT’ is written as ‘SHCFSD’ and ‘DEMAND’ is written
as ‘CMZNFE’. How will ‘FAMILY’ be written in that language?
Ans 1. XKHNBG

2.  GBNJMZ
at
3. EZLHKX

4. EBLHMX
Pr

Question ID : 26433056577
Status : Answered
Chosen Option : 1

Q.13 Select the correct combination of mathematical signs to sequentially replace the *
signs and to balance the given equation.
an

14*7*88*37*47
Ans 1. +,×,=,−

2. −,+,=,×

3. =,−,×,+
ag

4. ×,−,+,=

Question ID : 264330142884
Not Attempted and
Status :
Marked For Review
G

Chosen Option : --
Join @MathsByGaganPratap Telegram Channel for free PDFs
Q.14 ‘L & M’ means ‘L is the father of M’.
‘L = M’ means ‘L is the sister of M’.
‘L ^ M’ means ‘L is the son of M’s brother’.
‘L # M’ means ‘L is the wife of M’.
Which of the following options means, ‘B is the father-in-law of D’?
Ans 1.  B ^ A = C # D

2. A ^ B & C # D

3.  B = C # A & D

4.  D & A = C # B

Question ID : 26433086989
Status : Answered
Chosen Option : 2

hs
Q.15 Select the option that is related to the fifth term in the same way as the second term is
related to the first term and the fourth term is related to the third term.
FAST : UZHG :: SLOW : HOLD :: WALK : ?
Ans 1. ULAJ

at
2. DYSP

3. DZOP

M
4. LKWA

Question ID : 264330142843
Status : Answered
Chosen Option : 3
ap
Q.16 उस विकल्प का चयन कीजिए जो तीसरे शब्द से उसी प्रकार संबंधित है जिस प्रकार दू सरा शब्द पहले
शब्द से संबंधित है। (शब्दों को अर्थपूर्ण हिंदी शब्दों के रूप में माना जाना चाहिए और शब्द में अक्षरों की
संख्या/व्यंजनों/स्वरों की संख्या के आधार पर वे एक-दू सरे से संबंधित नहीं होने चाहिए।)
at
सुस्ती (Lethargy) : सतर्क ता (Alertness) :: क्रमिक (Gradual) : ?
Ans 1. नष्ट करना (Destroy)

2. विलाप (mourn)
Pr

3. उथला (Shallow)

4. आकस्मिक (Abrupt)

Question ID : 264330142860
Status : Answered
Chosen Option : 4
an

Q.17
ag

Ans
1.
G

2.

3.

4.

Question ID : 264330143277
Status : Answered
Chosen Option : 2
Join @MathsByGaganPratap Telegram Channel for free PDFs
Q.18 Select the option that represents the correct order of the given words as they would
appear in an English dictionary.
1. OBSCURE
2. OBLIQUE
3. OBLIGE
4. OBLIGATE
5. OBEY
6. OBJECT
Ans 1. 6, 5, 4, 3, 2, 1

2. 5, 6, 4, 3, 2, 1

3. 2, 6, 5, 4, 3, 1

4. 3, 6, 5, 4, 2, 1

hs
Question ID : 264330142856
Status : Answered
Chosen Option : 2

Q.19 Which of the following numbers will replace the question mark (?) in the given series?

at
17, 21, 28, 38, 51, 67, ?
Ans 1. 86

2. 96

M
3. 88

4. 98

ap Question ID : 264330142985
Status : Answered
Chosen Option : 1

Q.20 In a certain code language, ‘FRANK’ is written as ‘121811422’ and ‘CRIED’ is written as
‘618958’. How will ‘GREAT’ be written in that language?
at
Ans 1. 14185140

2. 14185240

3. 141810120
Pr

4. 14185120

Question ID : 26433056201
Status : Answered
Chosen Option : 1
an
ag
G
Join @MathsByGaganPratap Telegram Channel for free PDFs
Q.21

Ans

1.

hs
2.

at
M
3.

ap
4.
at

Question ID : 264330143266
Pr

Not Attempted and


Status :
Marked For Review
Chosen Option : --

Q.22 ‘A + B’ means ‘A is the father of B’.


‘A – B’ means ‘A is the sister of B’.
‘A × B’ means ‘A is the mother of B’.
an

‘A ÷ B’ means ‘A is the son of B’.


if P – Q + R ÷ S × T, then which of the following statements is NOT correct?
Ans 1.  R is the nephew of P.

2. P is the sister-in-law of S. 


ag

3.  Q is the husband of S.

4. S is the mother of Q. 

Question ID : 26433095989
Status : Answered
G

Chosen Option : 4
Join @MathsByGaganPratap Telegram Channel for free PDFs
Q.23

Ans

1.

hs
2.

at
3.

M
4.

Question ID : 264330143291
ap Status : Answered
Chosen Option : 1

Q.24 Select the option that is related to the third term in the same way as the second term is
related to the first term and the sixth term is related to the fifth term.
at
7: 1000 :: 4:? :: 9: 1728
Ans 1. 344

2. 340
Pr

3. 343

4. 348

Question ID : 264330142991
Status : Answered
an

Chosen Option : 3

Q.25 Select the option that indicates the correct arrangement of the given words in a logical
and meaningful order.
1. House
2. Wall
ag

3. Clay
4. Bricks
5. Room
Ans 1. 3, 2, 4, 5, 1

2. 3, 4, 1, 2, 5
G

3. 3, 5, 2, 4, 1

4. 3, 4, 2, 5, 1

Question ID : 264330125999
Status : Answered
Chosen Option : 4

Section : General Awareness


Join @MathsByGaganPratap Telegram Channel for free PDFs
Q.1 Which foods is a good source of vitamin B12?
Ans 1. Meats

2. Fruits

3. Pulses

4. Vegetables

Question ID : 264330142724
Status : Answered
Chosen Option : 3

Q.2 Zangtalam is the folk dance of which Indian state?

hs
Ans 1. Nagaland

2. Assam

3. Mizoram

4. West Bengal

at
Question ID : 264330141324
Status : Answered
Chosen Option : 1

M
Q.3 As per the 2011 census, what is the sex ratio of Daman & Diu?
Ans 1. 775

2. 678

3. 886
ap
4. 618

Question ID : 264330109870
at
Status : Not Answered
Chosen Option : --

Q.4 Amarsimha was one of the luminaries in the court of ________.


Pr

Ans 1. Chandragupta II

2. Kumaragupta

3. Chandragupta I

4. Samudragupta
an

Question ID : 264330108995
Status : Answered
Chosen Option : 1
ag

Q.5 Who among the following unveiled a statue of Chhatrapati Shivaji Maharaj in Pune,
Maharashtra, in March 2022?
Ans 1. Uddhav Thackeray

2. Ram Nath Kovind


G

3. Bhagat Singh Koshyari

4. Narendra Modi

Question ID : 26433090633
Not Attempted and
Status :
Marked For Review
Chosen Option : --
Join @MathsByGaganPratap Telegram Channel for free PDFs
Q.6 Which of the following is NOT a fundamental duty for Indian citizens?
Ans 1. To defend the country

2. To safeguard public property

3. To develop philosophical temper and spirit of enquiry

4. To protect and improve the natural environment

Question ID : 264330109836
Status : Answered
Chosen Option : 3

Q.7 Which of the following temples is built in the style of the Rekha Deula or the one with a
curvilinear superstructure?

hs
Ans 1. Kailasanathar Temple in Tamil Nadu

2. Sri Jagannath Temple in Odisha

3. Shree Padmanabhaswamy Temple in Kerala

at
4. Kandariya Mahadev Temple in Madhya Pradesh

Question ID : 26433090748
Status : Not Answered

M
Chosen Option : --

Q.8 What was the capital of the Bahmani kingdom at the time of its foundation?
Ans 1. Ahmednagar ap
2. Bidar

3. Kolar

4. Gulbarga
at
Question ID : 26433065576
Status : Answered
Chosen Option : 3
Pr

Q.9 ‘Moon Walk’ is an autobiographical work of which famous American dancer and
singer?
Ans 1. Debbie Reynolds

2. Michael Jackson

3. Lindsey Stirling
an

4. Donna McKechnie

Question ID : 264330141397
Status : Answered
ag

Chosen Option : 2

Q.10 How/By whom is the Attorney General of India appointed?


Ans 1. Voting by the Supreme Court judges

2. President
G

3. Prime Minister

4. UPSC

Question ID : 264330113428
Status : Answered
Chosen Option : 1
Join @MathsByGaganPratap Telegram Channel for free PDFs
Q.11 Who was the recipient of Ustad Bismillah Khan Yuva Puraskar 2017 for Kathak?
Ans 1. Jitendra Maharaj

2. Rajendra Gangani

3. Geetanjali Lal

4. Vidha Lal

Question ID : 26433074826
Status : Not Answered
Chosen Option : --

Q.12 Which is a flowerless, spore-producing plant that is usually a thin, horn-like, or needle-
like capsule with spores that develops from a flat, green leaf?

hs
Ans 1. Hornwort

2. Chara

3. Celandine

at
4. Riccia

Question ID : 264330142712
Status : Not Answered

M
Chosen Option : --

Q.13 Which Indian Grandmaster defeated World No.1 Magnus Carlsen in the eighth round of
the Airthings Masters, an online rapid chess tournament in February 2022?
Ans 1. Gukesh D

2.  Nihal Sarin
ap
3.  Raunak Sidhwani

4.  R Praggnanandhaa
at
Question ID : 26433054450
Status : Answered
Chosen Option : 4
Pr

Q.14 उड़ीसा में चावल को किस प्रकार की फसल माना जाता है?
Ans 1. व्यावसायिक

2. निर्वाह

3. बागवानी
an

4. रोपण

Question ID : 264330142769
Status : Answered
ag

Chosen Option : 1

Q.15 Which of the following statements is INCORRECT about migration in India?


Ans 1. Migration may be rural to urban.

2. Migration does not depend on social factors.


G

3. Migration has pull and push factors.

4. It may be urban to rural areas.

Question ID : 264330142799
Status : Answered
Chosen Option : 4
Join @MathsByGaganPratap Telegram Channel for free PDFs
Q.16 Who among the following founded the Asiatic Society in 1784 in Calcutta?
Ans 1. Jonathan Duncan

2. William Jones

3. Warren Hastings

4. Charles Eyre

Question ID : 26433065569
Status : Answered
Chosen Option : 2

Q.17 The Social Service League was founded in Bombay by:

hs
Ans 1. Manabendra Nath Roy

2. Gopal Krishna Gokhale

3. Narayan Malhar Joshi

4. Mahadev Govind Ranade

at
Question ID : 26433089162
Status : Not Answered
Chosen Option : --

M
Q.18 Papanasam Sivan was related to which of following music?
Ans 1. Carnatic

2. Punjabi

3. Khyal
ap
4. Dhrupad

Question ID : 264330141205
at
Status : Not Answered
Chosen Option : --

Q.19 The Padma Awardee 2021, Anshu Jamsenpa, has received the award for her illustrious
Pr

achievements in which of the following fields?


Ans 1. Athletics

2. Basketball

3. Mountaineering
an

4. Swimming

Question ID : 264330142684
Status : Not Answered
Chosen Option : --
ag

Q.20 Which of the following rivers is a tributary of the Indus river system?
Ans 1. Sutlej

2. Son
G

3. Betwa

4. Kosi

Question ID : 26433082192
Status : Answered
Chosen Option : 1
Join @MathsByGaganPratap Telegram Channel for free PDFs
Q.21 In June 2021, the Government of India extended the FAME India Scheme II that aims to
give a boost to the development of Electric Vehicles until __________.
Ans 1. March 2024

2. March 2025

3. March 2023

4. March 2026

Question ID : 26433074177
Not Attempted and
Status :
Marked For Review
Chosen Option : --

hs
Q.22 Which of the following weight lifting styles is used in power lifting?
Ans 1. Straight lift

2. Dead lift

3. Clean and jerk

at
4. Power lift

Question ID : 264330142824

M
Status : Answered
Chosen Option : 2

Q.23 Which of the following states won the best tableau award at the Republic Day parade
of 2022?
Ans 1. Gujarat
ap
2. Meghalaya

3. Uttar Pradesh

4. Karnataka
at

Question ID : 264330141332
Status : Answered
Chosen Option : 3
Pr

Q.24 What is the average temperature usually in the hills of the western ghats?
Ans 1. Above 35°C

2. Below 15°C
an

3. Below 25°C

4. Above 25°C

Question ID : 264330142725
Status : Not Answered
ag

Chosen Option : --

Q.25 In 1887, whose experiment was an attempt to find the velocity of the Earth with respect
to the hypothetical luminiferous ether?
Ans 1. James Clerk Maxwell
G

2. Michelson and Morley

3. Pieter Zeeman

4. Hendrik Antoon Lorentz

Question ID : 264330142735
Status : Not Answered
Chosen Option : --

Section : Quantitative Aptitude


Join @MathsByGaganPratap Telegram Channel for free PDFs
Q.1 Ajay offers his customers a discount of 20% on an overcoat and he still makes a profit
of 28%. What is the actual cost of the overcoat marked at ₹1,800?
Ans
1.

2.

3.

4.

Question ID : 26433063404
Status : Answered

hs
Chosen Option : 2

Q.2 The curved surface area of a cylindrical pillar is 528 m2 and its volume is 1848 m3. The
ratio between its radius and height is ________.
Ans 1. 9 : 10

at
2. 7 : 12

3. 7 : 3 

4. 9 : 5

M
Question ID : 26433063976
Status : Not Answered
Chosen Option : --
ap
Q.3 A motorboat, whose speed in 20 km/h in still water goes 75 km downstream and
comes back in a total of 8 hours. The speed of the stream is:
Ans 1. 5 km/h

2. 6 km/h
at
3. 4.5 km /h

4. 6.5 km/h
Pr

Question ID : 264330142264
Status : Not Answered
Chosen Option : --

Q.4 If 180 : y :: y : 245, and y > 0, find the value of y.


an

Ans 1. 220

2. 190

3. 200

4. 210
ag

Question ID : 264330103271
Status : Answered
Chosen Option : 4
G

Q.5 What is the fourth proportional to the numbers 8, 66 and 112?


Ans 1. 784

2. 848

3. 924

4. 729

Question ID : 26433061899
Status : Answered
Chosen Option : 3
Join @MathsByGaganPratap Telegram Channel for free PDFs
Q.6 What annual payment will discharge a debt of ₹38,700 due 4 years hence, at the rate of
5% per annum simple interest?
Ans 1. ₹9,000

2. ₹9,150

3. ₹9,100

4. ₹9,050

Question ID : 264330104194
Not Attempted and
Status :
Marked For Review
Chosen Option : --

hs
Q.7 The smallest six-digit number which is divisible by 12, 16, 24, 28 is:
Ans 1. 100128

2. 100190

3. 100180

at
4. 100160

Question ID : 26433082956

M
Not Attempted and
Status :
Marked For Review
Chosen Option : --

Q.8 Mahesh buys a water cooler for ₹28,500 and sells it for ₹24,800. What is his loss

Ans
percentage? (Correct to 2 decimal places)

1.
ap
2.
at
3.

4.
Pr

Question ID : 26433082596
Status : Answered
Chosen Option : 1

Q.9 The average age of 17 persons is 37 years. The average age increases by 1 year when
an

2 new persons are included. What is the average age of the new persons?
Ans
1.

2.
ag

3.

4.
G

Question ID : 26433061062
Status : Answered
Chosen Option : 3
Join @MathsByGaganPratap Telegram Channel for free PDFs
Q.10 A class of 40 students took a mathematics test. 18 students had an average score of
75. The other students had an average score of 62. What is the average score of the
entire class?
Ans 1. 67.85

2. 72.52

3. 68.50

4. 69.85

Question ID : 264330102320
Status : Answered
Chosen Option : 1

hs
Q.11 Jay spends 50% of his monthly income on grocery and bills, 20% of his monthly
income on buying clothes, 5% of his monthly income on medicines, and the remaining
amount of ₹12,500 he saves. What is Jay's monthly income?
Ans 1. ₹45,000

2. ₹57,500

at
3. ₹65,000

4. ₹50,000

M
Question ID : 264330142516
Status : Answered
Chosen Option : 4
ap
Q.12 The average age of 35 students in a group is 12 years. When the teacher's age is
included, the average increases by one. What is the teacher's age?
Ans 1. 42 years

2. 48 years

3. 45 years
at
4. 39 years

Question ID : 264330142515
Pr

Status : Answered
Chosen Option : 2

Q.13 If a watch that is marked at ₹1,800 is being sold for ₹1,250, then the discount
percentage at which the watch is being sold is:
Ans
an

1.

2.
ag

3.

4.
G

Question ID : 264330129416
Status : Answered
Chosen Option : 1
Join @MathsByGaganPratap Telegram Channel for free PDFs
Q.14 In a 1610 m race, Amarjeet reaches the final point in 91 seconds and Dinesh reaches
the final point in 115 seconds. By how much distance does Amarjeet beat Dinesh?
Ans 1. 326 m

2. 340 m

3. 336 m

4. 330 m

Question ID : 26433063423
Status : Not Answered
Chosen Option : --

Q.15 A reduction of 15% in the price of washing powder enables a purchaser to obtain 5 kg

hs
more for Rs.1,275. The original price of the washing powder per kg is:
Ans 1. Rs.40

2. Rs.35

3. Rs.45

at
4. Rs.50

Question ID : 26433075463

M
Status : Not Answered
Chosen Option : --

Q.16 Anand can construct a wall in 4 days, while Rajan can complete the same work in 7
days. In how many days will Anand and Rajan construct the wall if they work together?
Ans
1.
ap
2.
at
3.

4.
Pr

Question ID : 26433074567
Status : Answered
Chosen Option : 1
an

Q.17 The radius of the base of a solid right circular cone is 20 cm and its height is 21 cm.
What is its total surface area (in cm2)?
Ans 1. 760π

2. 980π
ag

3. 890π

4. 680π

Question ID : 264330142755
Status : Not Answered
G

Chosen Option : --
Join @MathsByGaganPratap Telegram Channel for free PDFs
Q.18 16 men can dig a ditch 24 m long in 18 days, working 6 hours a day. How many more
men should be engaged to dig a similar ditch 42 m long in 9 days, each man now
working 12 hours per day?
Ans 1. 14

2. 10

3. 12

4. 9

Question ID : 264330142259
Status : Answered
Chosen Option : 3

hs
Q.19 The retail price of a fan is Rs.2,530. If the manufacturer gains 10%, the wholesale
dealer gains 15% and the retailer gains 25%, then the cost of the product is:
Ans 1.

2.

at
3.

4.

M
Question ID : 26433082371
Status : Not Answered
Chosen Option : --
ap
Q.20 What is the annual payment Sunil should pay (in ₹) to clear his debt of ₹15,750, which
he borrowed from his cousin and promised to return in 3 years at 5% simple interest?
Ans 1. 4,800

2. 5,000

3. 2,500
at
4. 4,850

Question ID : 264330104258
Pr

Status : Not Answered


Chosen Option : --

Q.21 A sum becomes triple in 9 years. Find the annual rate of simple interest.
Ans
1.
an

2.
ag

3.

4.
G

Question ID : 264330142668
Status : Answered
Chosen Option : 2
Join @MathsByGaganPratap Telegram Channel for free PDFs
Q.22 In an election, 15% of the voters on the voters list did not cast their votes and 100
voters cast their ballot paper blank. There were only two candidates Ram and Shyam.
The winner, Ram, was supported by 69% of all the voters in the list and he got 630
votes more than Shyam. Find the number of voters on the list.
Ans 1. 1200

2. 1000

3. 1100

4. 1300

Question ID : 26433075527
Status : Not Answered
Chosen Option : --

hs
Q.23 Anil buys a shirt for ₹1,400 and sells it at a loss of 15%. What is the selling price of the
shirt?
Ans 1. ₹1,250

2. ₹1,190

at
3. ₹1,140

4. ₹ 1,200

M
Question ID : 264330142584
Status : Answered
Chosen Option : 2

Ans 1. 11
ap
Q.24 When 151314 is divided by 15, the remainder is:

2. 6

3. 9
at
4. 7

Question ID : 264330126161
Status : Answered
Pr

Chosen Option : 3

Q.25 Shivam can do a piece of work in 6 days and Vivek in 24 days. They started work
together but Shivam left the work 3 days before its completion. In how many days was
the work completed?
Ans
an

1.

2.
ag

3.

4.
G

Question ID : 26433074943
Status : Not Answered
Chosen Option : --

Section : English Language


Join @MathsByGaganPratap Telegram Channel for free PDFs
Q.1 Select the INCORRECTLY spelt word.
Ans 1. Bureacracy

2. Bequeath

3. Beverage

4. Beguile

Question ID : 26433073879
Status : Marked For Review
Chosen Option : 2

Q.2 Select the correct indirect narration of the given sentence.


Aravind said, “Let us wait for the cab.”

hs
Ans 1. Aravind proposed that they should wait for the cab.

2. Aravind proposed that they should has wait for the cab.

3. Aravind proposes that let we waited for the cab.

at
4. Aravind propose that they should been wait for the cab.

Question ID : 264330132877
Status : Answered

M
Chosen Option : 1

Q.3 Select the most appropriate ANTONYM of the word ‘Decipher’ from the given sentence.

Many sculptures of our history are like the unravelled path as we cannot garble and

Ans 1. Unravelled
ap
there is a lot of work that is yet to be interpreted or translated.

2. Garble  

3. Interpreted  
at
4. Translated

Question ID : 26433071887
Status : Answered
Pr

Chosen Option : 1

Q.4 Select the option that expresses the given sentence in active voice.
The shop owners were requested to close the shops by the agitators.
Ans 1. The agitators request the shop owners to close the shops.
an

2. The agitators have requested the shop owners to close the shops.

3. The agitators are requesting the shop owners to close the shops.

4. The agitators requested the shop owners to close the shops.


ag

Question ID : 264330132821
Status : Answered
Chosen Option : 4

Q.5 Select the idiom that gives the most appropriate meaning of the underlined phrase in
G

the following sentence.

Our secretary is always in the mood for an argument. He must learn to keep quiet.
Ans 1. To go through

2. To turn up

3. To take off

4. To hold his tongue

Question ID : 26433086437
Status : Answered
Chosen Option : 4
Join @MathsByGaganPratap Telegram Channel for free PDFs
Q.6 Fill in the blank with the most appropriate word that can be used here based on the
given context.

Macaulay _________ the western education system in India.


Ans 1. introduced 

2. improvising

3. accepting

4. revisiting

Question ID : 26433059830
Status : Answered
Chosen Option : 1

hs
Q.7 Select the INCORRECTLY spelt word.
Ans 1. Bourgeoisie

2. Burlesque

at
3. Burgaler

4. Beggar

M
Question ID : 26433088823
Status : Answered
Chosen Option : 1

Q.8 Select the most appropriate meaning of the underlined idiom.


ap
We must draw the line somewhere as living expenses are increasing daily.
Ans 1. Fix the limit

2. Line up expenses

3. Fix the broken line


at
4. Draw a straight line

Question ID : 26433098329
Pr

Status : Answered
Chosen Option : 1

Q.9 Select the option that can be used as a one-word substitute for the given group of
words.
an

A place where coins are made


Ans 1. Furnace

2. Mine

3. Mint
ag

4. Factory

Question ID : 26433060347
Status : Answered
Chosen Option : 3
G
Join @MathsByGaganPratap Telegram Channel for free PDFs
Q.10 Select the most appropriate meaning of the underlined segment in the given sentence.

Reghu’s been trying to keep a lid on his emotions, but every now and then his anger
erupts.
Ans 1. make things difficult for someone

2. either take less care of your appearance or relax completely and enjoy yourself 

3. leave someone at a time when they need you to stay and help them 

4. control the level of something in order to stop it increasing

Question ID : 26433059815
Status : Answered
Chosen Option : 4

hs
Q.11 Substitute one word for the italicised expression.
As a port it was notorious for its smuggling and the trade was prohibited by law.
Ans 1. licit 

2. chaste

at
3. legitimate 

4. illicit 

M
Question ID : 26433059752
Status : Answered
Chosen Option : 4
ap
Q.12 Identify the idiom that best expresses the meaning of the underlined group of words.

There are a number of things that the country needs to improve but the main part of
the problem is corruption.
Ans 1. Crux of the matter 
at
2. Last resort

3. Grasping at straws  

4. Dodged a bullet
Pr

Question ID : 264330121052
Status : Answered
Chosen Option : 4

Q.13 Choose the option that rectifies the incorrectly spelt underlined word.
an

John chanced upon a former collegeue of his at the airport.


Ans 1. collegue

2. coleageu

3. colleague
ag

4. coleague

Question ID : 26433088846
Status : Answered
G

Chosen Option : 3
Join @MathsByGaganPratap Telegram Channel for free PDFs
Q.14 Select the most appropriate synonym of the underlined word in the following
sentence.

As far as possible, solid waste should be managed at the household level so that
minimum waste is delivered for management at the community level.
Ans 1. withdrew 

2. relocated 

3. replaced 

4. deposited

Question ID : 26433059802
Status : Answered
Chosen Option : 4

hs
Q.15 Select the most appropriate option to fill in the blank.

When you apply the medicine, the wound ___________.


Ans 1. feels

at
2. kills

3. heals

M
4. deals

Question ID : 26433060081
Status : Answered
Chosen Option : 3

Comprehension:
ap
In the following passage, some words have been deleted. Read the passage carefully and
select the most appropriate option to fill in each blank.
Palaeontologists are like detectives who (1) ________ the evidence that (2) ______ animals
at
left behind. Those clues to what dinosaurs were like are found in fossils—the ancient (3)
_______ of an organism, such as teeth, bone or shell—or (4) _______ of animal activity, such
as (5) ________ and trackways.

SubQuestion No : 16
Pr

Q.16 Select the most appropriate option to fill in blank number 1.


Ans 1. examine

2. dominate

3. control

4. conduct 
an

Question ID : 26433078994
Status : Answered
Chosen Option : 1
ag
G
Join @MathsByGaganPratap Telegram Channel for free PDFs
Comprehension:
In the following passage, some words have been deleted. Read the passage carefully and
select the most appropriate option to fill in each blank.
Palaeontologists are like detectives who (1) ________ the evidence that (2) ______ animals
left behind. Those clues to what dinosaurs were like are found in fossils—the ancient (3)
_______ of an organism, such as teeth, bone or shell—or (4) _______ of animal activity, such
as (5) ________ and trackways.

SubQuestion No : 17
Q.17 Select the most appropriate option to fill in blank number 2.
Ans 1. surviving

2. vanished

3. extinct 

hs
4. inactive

Question ID : 26433078995
Status : Answered
Chosen Option : 3

at
Comprehension:
In the following passage, some words have been deleted. Read the passage carefully and
select the most appropriate option to fill in each blank.

M
Palaeontologists are like detectives who (1) ________ the evidence that (2) ______ animals
left behind. Those clues to what dinosaurs were like are found in fossils—the ancient (3)
_______ of an organism, such as teeth, bone or shell—or (4) _______ of animal activity, such
as (5) ________ and trackways.

SubQuestion No : 18

Ans 1. memories 
ap
Q.18 Select the most appropriate option to fill in blank number 3.

2. fragments 

3. remains 
at
4. leftovers 

Question ID : 26433078996
Status : Answered
Pr

Chosen Option : 3

Comprehension:
In the following passage, some words have been deleted. Read the passage carefully and
select the most appropriate option to fill in each blank.
Palaeontologists are like detectives who (1) ________ the evidence that (2) ______ animals
an

left behind. Those clues to what dinosaurs were like are found in fossils—the ancient (3)
_______ of an organism, such as teeth, bone or shell—or (4) _______ of animal activity, such
as (5) ________ and trackways.

SubQuestion No : 19
Q.19 Select the most appropriate option to fill in blank number 4.
ag

Ans 1. proposal

2. submission 

3. evidence

4. compliance
G

Question ID : 26433078997
Status : Answered
Chosen Option : 3
Join @MathsByGaganPratap Telegram Channel for free PDFs
Comprehension:
In the following passage, some words have been deleted. Read the passage carefully and
select the most appropriate option to fill in each blank.
Palaeontologists are like detectives who (1) ________ the evidence that (2) ______ animals
left behind. Those clues to what dinosaurs were like are found in fossils—the ancient (3)
_______ of an organism, such as teeth, bone or shell—or (4) _______ of animal activity, such
as (5) ________ and trackways.

SubQuestion No : 20
Q.20 Select the most appropriate option to fill in blank number 5.
Ans 1. routes

2. pathways 

3. canals

hs
4. footprints

Question ID : 26433078998
Status : Answered
Chosen Option : 4

at
Comprehension:
Read the given passage and answer the question that follows.

M
Wetlands, the most economically valuable and among the most biodiverse ecosystems in the
world, are disappearing three times faster than forests which will lead to severe
consequences. Approximately 35 per cent of the world’s wetlands were lost between 1970 -
2015 with annual rates of loss accelerating from 2000, according to the first-ever Global
Wetland Outlook of the Ramsar Convention, a global treaty ratified by 170 countries to protect
wetlands and promote their wise use. Wetlands, which include lakes, rivers, marshes and
ap
peatlands as well as coastal and marine areas such as estuaries, lagoons, mangroves and
coral reefs, are currently estimated to cover more than 12.1 million km2, an area greater than
Canada.
Wetlands contribute to climate regulation and support ecosystems and biodiversity. Plants in
swamps absorb pollutants, mangroves store carbon, lakes and underground aquifers are a
crucial source of water, and coral reefs protect coastlines from wave action and serve as
shelter for marine organisms. Wetlands are home to indigenous peoples and a natural source
at
of livelihoods for their communities. They provide drinking water, energy, fisheries, agriculture,
transport, recreation, cultural values and tourism. While human-made wetlands - largely rice
paddy and reservoirs - have almost doubled between 1970 and 2015, natural wetlands have
been progressively declining. Losses have been driven by megatrends such as climate
change, population increase, urbanisation, and changing consumption patterns that have all
Pr

fuelled changes to land and water use and to agriculture. Increasing pollution and land
overuse significantly affect wetlands’ water quality. This impacts the livelihoods of
communities living near wetlands. Untreated wastewater, industrial waste, agricultural runoff
and erosion can severely compromise the water by escalating the levels of nitrogen,
phosphorous, salinity and faecal coliform bacteria. The main international efforts to promote
the conservation and prudent use of wetlands are through the secretariat of the Convention
on Wetlands, an intergovernmental treaty adopted in Ramsar, Iran in 1971. 170 countries - or
an

almost 90 per cent of UN member states - from all regions have become Contracting Parties
of the treaty.

SubQuestion No : 21
Q.21 Select the most appropriate ANTONYM of the given word.

Fuel
ag

Ans 1. Encourage  

2. Project

3. Deter  
G

4. Associate

Question ID : 26433072745
Status : Answered
Chosen Option : 3
Join @MathsByGaganPratap Telegram Channel for free PDFs
Comprehension:
Read the given passage and answer the question that follows.

Wetlands, the most economically valuable and among the most biodiverse ecosystems in the
world, are disappearing three times faster than forests which will lead to severe
consequences. Approximately 35 per cent of the world’s wetlands were lost between 1970 -
2015 with annual rates of loss accelerating from 2000, according to the first-ever Global
Wetland Outlook of the Ramsar Convention, a global treaty ratified by 170 countries to protect
wetlands and promote their wise use. Wetlands, which include lakes, rivers, marshes and
peatlands as well as coastal and marine areas such as estuaries, lagoons, mangroves and
coral reefs, are currently estimated to cover more than 12.1 million km2, an area greater than
Canada.
Wetlands contribute to climate regulation and support ecosystems and biodiversity. Plants in
swamps absorb pollutants, mangroves store carbon, lakes and underground aquifers are a
crucial source of water, and coral reefs protect coastlines from wave action and serve as
shelter for marine organisms. Wetlands are home to indigenous peoples and a natural source

hs
of livelihoods for their communities. They provide drinking water, energy, fisheries, agriculture,
transport, recreation, cultural values and tourism. While human-made wetlands - largely rice
paddy and reservoirs - have almost doubled between 1970 and 2015, natural wetlands have
been progressively declining. Losses have been driven by megatrends such as climate
change, population increase, urbanisation, and changing consumption patterns that have all
fuelled changes to land and water use and to agriculture. Increasing pollution and land
overuse significantly affect wetlands’ water quality. This impacts the livelihoods of

at
communities living near wetlands. Untreated wastewater, industrial waste, agricultural runoff
and erosion can severely compromise the water by escalating the levels of nitrogen,
phosphorous, salinity and faecal coliform bacteria. The main international efforts to promote
the conservation and prudent use of wetlands are through the secretariat of the Convention
on Wetlands, an intergovernmental treaty adopted in Ramsar, Iran in 1971. 170 countries - or

M
almost 90 per cent of UN member states - from all regions have become Contracting Parties
of the treaty.

SubQuestion No : 22
Q.22 Select an appropriate tone of the passage. ap
Ans 1. Critical and cynical

2. Sarcastic and offensive 

3. Objective and concerned  

4. Pleading and indifferent


at
Question ID : 26433072746
Status : Answered
Chosen Option : 3
Pr
an
ag
G
Join @MathsByGaganPratap Telegram Channel for free PDFs
Comprehension:
Read the given passage and answer the question that follows.

Wetlands, the most economically valuable and among the most biodiverse ecosystems in the
world, are disappearing three times faster than forests which will lead to severe
consequences. Approximately 35 per cent of the world’s wetlands were lost between 1970 -
2015 with annual rates of loss accelerating from 2000, according to the first-ever Global
Wetland Outlook of the Ramsar Convention, a global treaty ratified by 170 countries to protect
wetlands and promote their wise use. Wetlands, which include lakes, rivers, marshes and
peatlands as well as coastal and marine areas such as estuaries, lagoons, mangroves and
coral reefs, are currently estimated to cover more than 12.1 million km2, an area greater than
Canada.
Wetlands contribute to climate regulation and support ecosystems and biodiversity. Plants in
swamps absorb pollutants, mangroves store carbon, lakes and underground aquifers are a
crucial source of water, and coral reefs protect coastlines from wave action and serve as
shelter for marine organisms. Wetlands are home to indigenous peoples and a natural source

hs
of livelihoods for their communities. They provide drinking water, energy, fisheries, agriculture,
transport, recreation, cultural values and tourism. While human-made wetlands - largely rice
paddy and reservoirs - have almost doubled between 1970 and 2015, natural wetlands have
been progressively declining. Losses have been driven by megatrends such as climate
change, population increase, urbanisation, and changing consumption patterns that have all
fuelled changes to land and water use and to agriculture. Increasing pollution and land
overuse significantly affect wetlands’ water quality. This impacts the livelihoods of

at
communities living near wetlands. Untreated wastewater, industrial waste, agricultural runoff
and erosion can severely compromise the water by escalating the levels of nitrogen,
phosphorous, salinity and faecal coliform bacteria. The main international efforts to promote
the conservation and prudent use of wetlands are through the secretariat of the Convention
on Wetlands, an intergovernmental treaty adopted in Ramsar, Iran in 1971. 170 countries - or

M
almost 90 per cent of UN member states - from all regions have become Contracting Parties
of the treaty.

SubQuestion No : 23
Q.23 Ramsar Convention was signed in the year ____________.
ap
Ans 1. 2000

2. 1969

3. 2015

4. 1971
at
Question ID : 26433072743
Status : Answered
Chosen Option : 4
Pr
an
ag
G
Join @MathsByGaganPratap Telegram Channel for free PDFs
Comprehension:
Read the given passage and answer the question that follows.

Wetlands, the most economically valuable and among the most biodiverse ecosystems in the
world, are disappearing three times faster than forests which will lead to severe
consequences. Approximately 35 per cent of the world’s wetlands were lost between 1970 -
2015 with annual rates of loss accelerating from 2000, according to the first-ever Global
Wetland Outlook of the Ramsar Convention, a global treaty ratified by 170 countries to protect
wetlands and promote their wise use. Wetlands, which include lakes, rivers, marshes and
peatlands as well as coastal and marine areas such as estuaries, lagoons, mangroves and
coral reefs, are currently estimated to cover more than 12.1 million km2, an area greater than
Canada.
Wetlands contribute to climate regulation and support ecosystems and biodiversity. Plants in
swamps absorb pollutants, mangroves store carbon, lakes and underground aquifers are a
crucial source of water, and coral reefs protect coastlines from wave action and serve as
shelter for marine organisms. Wetlands are home to indigenous peoples and a natural source

hs
of livelihoods for their communities. They provide drinking water, energy, fisheries, agriculture,
transport, recreation, cultural values and tourism. While human-made wetlands - largely rice
paddy and reservoirs - have almost doubled between 1970 and 2015, natural wetlands have
been progressively declining. Losses have been driven by megatrends such as climate
change, population increase, urbanisation, and changing consumption patterns that have all
fuelled changes to land and water use and to agriculture. Increasing pollution and land
overuse significantly affect wetlands’ water quality. This impacts the livelihoods of

at
communities living near wetlands. Untreated wastewater, industrial waste, agricultural runoff
and erosion can severely compromise the water by escalating the levels of nitrogen,
phosphorous, salinity and faecal coliform bacteria. The main international efforts to promote
the conservation and prudent use of wetlands are through the secretariat of the Convention
on Wetlands, an intergovernmental treaty adopted in Ramsar, Iran in 1971. 170 countries - or

M
almost 90 per cent of UN member states - from all regions have become Contracting Parties
of the treaty.

SubQuestion No : 24
Q.24 Select an appropriate title from the given options.
ap
Ans 1. Brief into the stretch of Canadian Wetlands  

2. Natural and human-made wetlands - A review  

3. Disappearing wetlands - A study

4. An overview of effects of water contamination


at
Question ID : 26433072747
Status : Answered
Chosen Option : 3
Pr
an
ag
G
Join @MathsByGaganPratap Telegram Channel for free PDFs
Comprehension:
Read the given passage and answer the question that follows.

Wetlands, the most economically valuable and among the most biodiverse ecosystems in the
world, are disappearing three times faster than forests which will lead to severe
consequences. Approximately 35 per cent of the world’s wetlands were lost between 1970 -
2015 with annual rates of loss accelerating from 2000, according to the first-ever Global
Wetland Outlook of the Ramsar Convention, a global treaty ratified by 170 countries to protect
wetlands and promote their wise use. Wetlands, which include lakes, rivers, marshes and
peatlands as well as coastal and marine areas such as estuaries, lagoons, mangroves and
coral reefs, are currently estimated to cover more than 12.1 million km2, an area greater than
Canada.
Wetlands contribute to climate regulation and support ecosystems and biodiversity. Plants in
swamps absorb pollutants, mangroves store carbon, lakes and underground aquifers are a
crucial source of water, and coral reefs protect coastlines from wave action and serve as
shelter for marine organisms. Wetlands are home to indigenous peoples and a natural source

hs
of livelihoods for their communities. They provide drinking water, energy, fisheries, agriculture,
transport, recreation, cultural values and tourism. While human-made wetlands - largely rice
paddy and reservoirs - have almost doubled between 1970 and 2015, natural wetlands have
been progressively declining. Losses have been driven by megatrends such as climate
change, population increase, urbanisation, and changing consumption patterns that have all
fuelled changes to land and water use and to agriculture. Increasing pollution and land
overuse significantly affect wetlands’ water quality. This impacts the livelihoods of

at
communities living near wetlands. Untreated wastewater, industrial waste, agricultural runoff
and erosion can severely compromise the water by escalating the levels of nitrogen,
phosphorous, salinity and faecal coliform bacteria. The main international efforts to promote
the conservation and prudent use of wetlands are through the secretariat of the Convention
on Wetlands, an intergovernmental treaty adopted in Ramsar, Iran in 1971. 170 countries - or

M
almost 90 per cent of UN member states - from all regions have become Contracting Parties
of the treaty.

SubQuestion No : 25
Q.25 ___________ exerts influence on the sustenance of people dwelling adjacent to the
ap
wetland.
Ans 1. Rapid coastland erosion 

2. Recreational activities  

3. Depletion of water quality 


at
4. Agricultural patterns 

Question ID : 26433072744
Status : Answered
Pr

Chosen Option : 3
an
ag
G
Join @MathsByGaganPratap Telegram Channel for free PDFs

Phase-XI/2023/Selection Posts and Selection Posts/Ladakh/2023


Roll Number Gagan Pratap Maths
Candidate Name Gagan Pratap Maths
Venue Name Sarthak Educational Trust
Exam Date 30/06/2023
Exam Time 5:15 PM - 6:15 PM
Subject Selection Post Matriculation Level

Section : General Intelligence

hs
Q.1

at
Ans
1.

M
2.

3.

4.
ap
Question ID : 264330143276
Status : Answered
Chosen Option : 2
at

Q.2 Which of the following numbers will replace the question mark (?) in the given series?
3, 12, 30, 57, 93, 138, 192, ?
Ans 1. 222
Pr

2. 255

3. 252

4. 225
an

Question ID : 264330142984
Status : Answered
Chosen Option : 2

Q.3 Select the option that is related to the fourth term in the same way as the first term is
ag

related to the second term and the fifth term is related to the sixth term
8 : 512 :: ? : 1728 :: 18 : 5832
Ans 1. 8

2. 14

3. 9
G

4. 12

Question ID : 26433040810
Status : Answered
Chosen Option : 4
Join @MathsByGaganPratap Telegram Channel for free PDFs
Q.4 A # B means ‘A is the husband of B’.
A & B means ‘A is the daughter of B’.
A @ B means ‘A is the grandmother of B’.
A % B means ‘A is the son of B’.
If E @ F % G # H & I @ J, then how is G related to J if I has only one child?
Ans 1. Father 

2. Uncle  

3. Grandfather  

4. Son  

Question ID : 26433091668
Status : Answered
Chosen Option : 2

hs
Q.5 Select the option that is related to the third word in the same way as the second word
is related to the first word. (The words must be considered as meaningful English
words and must NOT be related to each other based on the number of letters/number
of consonants/vowels in the word)

at
Termites : Colony :: Musicians : ?
Ans 1. Crowd

2. Army

M
3. Band

4. Brood

ap Question ID : 264330142862
Status : Answered
Chosen Option : 3

Q.6 Which letter-cluster will replace the question mark (?) and complete the given series?
HERO, GISM, ?, EQUI, DUVG
at
Ans 1. FMTK

2. FMUL

3. FNUL
Pr

4. FMUK

Question ID : 264330142913
Status : Answered
Chosen Option : 1
an

Q.7 ‘P # Q’ means ‘P is the wife of Q’.


‘P % Q’ means ‘P is the father of Q’.
‘P & Q’ means ‘P is the son of Q’.
‘P @ Q’ means ‘P is the mother of Q’.
If A % B # C % D & B @ E, then how is A related to E?
ag

Ans 1. Father’s father

2. Wife

3. Mother’s father

4.  Brother
G

Question ID : 26433091634
Status : Answered
Chosen Option : 3
Join @MathsByGaganPratap Telegram Channel for free PDFs
Q.8 In a certain code language, ‘DEVOUT’ is written as ‘FIXSWX’ and ‘BLAMED’ is written
as ‘DPCQGH’. How will ‘SWEET’ be written in that language?
Ans 1. UZIIV

2. UAIGV

3. UYGGV

4. UAGIV

Question ID : 26433056205
Status : Answered
Chosen Option : 4

Q.9 Select the correct combination of mathematical signs to sequentially replace the *

hs
signs and to balance the given equation.
41 * 5 * 3 * 10 * 2 = 31
Ans 1.

2.

at
3.

4.

M
Question ID : 26433057475
Status : Answered
Chosen Option : 2

Ans
ap
Q.10 In a certain code language, ‘GOLDEN’ is written as ‘OFEMPH’ and ‘INCOME’ is written
as ‘FNPDOJ’. How will ‘METHOD’ be written in that language?
1. DOHTEM 

2. EQIUFN 

3. EPIUFN 
at
4. DPUIEN 

Question ID : 26433056568
Pr

Status : Answered
Chosen Option : 3

Q.11 ‘A # B’ means ‘A is the brother of B’.


‘A @ B’ means ‘A is the daughter of B’.
‘A & B’ means ‘A is the husband of B’.
‘A % B’ means ‘A is the wife of B’.
an

If V & S @ F % H # T @ Q, then how is F related to Q?


Ans 1. Mother

2. Husband’s mother

3. Daughter
ag

4. Son’s wife

Question ID : 264330142905
Status : Answered
Chosen Option : 4
G
Join @MathsByGaganPratap Telegram Channel for free PDFs
Q.12 Select the option that represents the correct order of the given words as they would
appear in an English dictionary.

1. Earn
2. Efficient
3. Educator
4. Effort
5. Elite
6. Eloquence
Ans 1. 1, 3, 4, 2, 5, 6

2. 1, 3, 2, 4, 6, 5

3. 1, 3, 2, 4, 5, 6

4. 1, 3, 4, 5, 6, 2

hs
Question ID : 264330142925
Status : Answered
Chosen Option : 3

at
Q.13 Select the option that represents the letters that, when sequentially placed from left to
right in the blanks below, will complete the letter series.
hf__rpi__xt_pihf_tr__
Ans 1. xthfrpxi 

M
2. xthfrxpi

3. xthfxrpi

4. xthrxpfi
ap Question ID : 26433056246
Status : Answered
Chosen Option : 2

Q.14 Select the option that is related to the fourth term in the same way as the first term is
at
related to the second term and the fifth term is related to the sixth term
64 : 320 :: ? : 70 :: 22 : 110
Ans 1. 17

2. 8
Pr

3. 12

4. 14

Question ID : 26433040807
an

Status : Answered
Chosen Option : 4

Q.15 Select the option that indicates the correct arrangement of the given words in a logical
and meaningful order.
1. Shear
ag

2. Wool
3. Knit
4. Sweater
5. Sheep
Ans 1. 5, 1, 2, 3, 4
G

2. 4, 1, 3, 2, 5

3. 5, 1, 3, 2, 4

4. 3, 2, 1, 4, 5

Question ID : 264330126001
Status : Answered
Chosen Option : 3
Join @MathsByGaganPratap Telegram Channel for free PDFs
Q.16 Which letter-cluster will replace the question mark (?) and complete the given series?
STAR, PUBO, ?, JWDI, GXEF
Ans 1. MVCK

2. NVCM

3. MVCL

4. NVCL

Question ID : 264330142911
Status : Answered
Chosen Option : 3

Q.17 Three statements are followed by three conclusions numbered I, II and III. You have to

hs
consider these statements to be true, even if they seem to be at variance with
commonly known facts. Decide which of the given conclusions logically follow(s) from
the given statements.

Statements:
Some horses are donkeys.

at
Some goats are sheep.
All sheep are fishes.

Conclusions:
(I) Some goats are donkeys.

M
(II) Some fishes are goats.
(III) Some horses are sheep.
Ans 1. Either conclusion I or conclusion III follows

2. None of the conclusions follow ap


3. Only conclusion I follows

4. Only conclusion II follows

Question ID : 264330142975
Status : Answered
at
Chosen Option : 4
Pr
an
ag
G
Join @MathsByGaganPratap Telegram Channel for free PDFs
Q.18

Ans

1.

hs
2.

at
M
3.

4.
ap
at
Question ID : 264330143264
Status : Answered
Chosen Option : 1
Pr

Q.19 Three Statements are given followed by Three conclusions numbered I, II and III.
Assuming the statements to be true, even if they seem to be at variance with
commonly known facts, decide which of the conclusions logically follow(s) from the
statements.

Statements:
All beds are pillows.
an

No pillow is a chair.
Some tables are chairs.

Conclusions:
I. Some tables are pillows.
II. Some beds are tables.
ag

III. No bed is a chair.


Ans 1. Both conclusions I and II follow.

2. Both conclusions I and III follow.

3. Only conclusion III follows.


G

4. Only conclusion II follows.

Question ID : 264330142876
Status : Answered
Chosen Option : 3
Join @MathsByGaganPratap Telegram Channel for free PDFs
Q.20 Select the option that is related to the third word in the same way as the second word
is related to the first word. (The words must be considered as meaningful English
words and must NOT be related to each other based on the number of letters/number
of consonants/vowels in the word)

Sculptor : Chisel :: Woodcutter : ?


Ans 1. Scalpel

2. Anvil

3. Axe

4. Plough

Question ID : 264330142865
Status : Answered

hs
Chosen Option : 3

Q.21 Select the option that is related to the fifth term in the same way as the second term is
related to the first term and the fourth term is related to the third term.
SCENE : DMDBR :: STORY : XQNSR :: WRITE : ?

at
Ans 1. DUHSV

2. DSJSV

3. ETIRW

M
4. DSHQV

Question ID : 264330142842
Status : Answered
ap Chosen Option : 4

Q.22
at
Pr

Ans

1.
an

2.
ag

3.
G

4.

Question ID : 264330143290
Status : Answered
Chosen Option : 2
Join @MathsByGaganPratap Telegram Channel for free PDFs
Q.23 Three statements are given, followed by three conclusions numbered I, II and III.
Assuming the statements to be true, even if they seem to be at variance with
commonly known facts, decide which of the conclusions logically follow(s) from the
statements.

Statements:
Some shops are houses.
Some houses are garages.
Some garages are cameras.

Conclusions:
I. Some cameras are shops.
II. All cameras are houses.
III. Some cameras are garages.
Ans 1. Only conclusion II follows

2. All the conclusions follow

hs
3. Only conclusion III follows

4. Only conclusion I follows

at
Question ID : 26433092647
Status : Answered
Chosen Option : 3

M
Q.24 Select the option that is related to the fifth term in the same way as the second term is
related to the first term and the fourth term is related to the third term.
LARGEST : ESTGLAR :: QUALITY : ITYLQUA :: QUICKLY : ?
Ans 1. UCLKYQI

2. KLYCQUI

3. KQICUYL
ap
4. KYLCUQI

Question ID : 264330142832
at
Status : Answered
Chosen Option : 2

Q.25 Three statements are given, followed by three conclusions numbered I, II and III.
Pr

Assuming the statements to be true, even if they seem to be at variance with


commonly known facts, decide which of the conclusions logically follow(s) from the
statements.

Statements:
Some chairs are tables.
All chairs are stairs.
an

All tables are roofs.

Conclusions:
I. Some roofs are stairs.
II. Some roofs are chairs.
III. All tables are stairs.
Ans 1. Only conclusions II and III follow. 
ag

2. All conclusions follow. 

3.  Only conclusions I and III follow.

4. Only conclusions I and II follow. 


G

Question ID : 26433095996
Status : Answered
Chosen Option : 4

Section : General Awareness


Join @MathsByGaganPratap Telegram Channel for free PDFs
Q.1 What is another name for centre-seeking force?
Ans 1. Van der Waals

2. Centrifugal

3. Gravitational

4. Centripetal

Question ID : 26433064737
Status : Not Answered
Chosen Option : --

Q.2 Which cell organelle containing millions of membrane-bound ribosomes is involved in


the production, folding, quality control and dispatch of certain proteins?

hs
Ans 1. Peroxisome

2. Rough endoplasmic reticulum

3. Mitochondrion

at
4. Golgi apparatus

Question ID : 264330142715
Status : Not Answered

M
Chosen Option : --

Q.3 Which is the oldest oil producing state of India?


Ans 1. Gujarat ap
2. Maharashtra

3. Assam

4. Telangana
at
Question ID : 264330110647
Status : Answered
Chosen Option : 3
Pr

Q.4 Who among the following Gupta king adopted the title of Vikramaditya?
Ans 1. Chandragupta I

2. Chandragupta II

3. Skandagupta
an

4. Samudragupta

Question ID : 264330108994
Status : Answered
Chosen Option : 2
ag

Q.5 Which mobile app was launched to strengthen the pre-litigation mechanism in the
country in November 2021?
Ans 1. Citizens’ Tele-Law Mobile App

2. Citizens’ e-salah Mobile App


G

3. Citizens’ e-Law Mobile App

4. Citizens’ e-legal Mobile App

Question ID : 26433081805
Status : Not Answered
Chosen Option : --
Join @MathsByGaganPratap Telegram Channel for free PDFs
Q.6 What does the Lincoln index measure?
Ans 1. Population Size

2. Population Natality Rate

3. Population Mortality Rate

4. Population Density

Question ID : 26433064444
Status : Answered
Chosen Option : 4

Q.7 Which of the following Articles of the Constitution of India lays down the maximum
strength of the Rajya Sabha as 250?

hs
Ans 1. Article 85

2. Article 80

3. Article 83

at
4. Article 81

Question ID : 264330113432
Status : Answered

M
Chosen Option : 2

Q.8 In January 2022, which Indian badminton player won the Yonex-Sunrise India Open
with a straight-game victory over the reigning world champion Loh Kean Yew of
Singapore in the men's singles final in Delhi?
Ans 1. Srikanth Kidambi 
ap
2. Prannoy Kumar 

3. B Sai Praneeth

4. Lakshya Sen 
at

Question ID : 26433054456
Status : Not Answered
Chosen Option : --
Pr

Q.9 Who among the following was given the Filmfare Award in the Best Choreographer
category in 2008?
Ans 1. Saroj Khan

2. Remo D’Souza
an

3. Geeta Kapoor

4. Farah Khan

Question ID : 26433074133
ag

Status : Not Answered


Chosen Option : --

Q.10 Which of the following states has the highest literacy rate as per the census of India
2011?
G

Ans 1. Kerala

2. Tamil Nadu

3. West Bengal

4. Maharashtra

Question ID : 26433089252
Status : Answered
Chosen Option : 1
Join @MathsByGaganPratap Telegram Channel for free PDFs
Q.11 The second session of Round Table Conference was held in:
Ans 1. 1932

2. 1929

3. 1931

4. 1930

Question ID : 264330106572
Status : Answered
Chosen Option : 3

Q.12 Which clause of Article 51A of the Constitution of India requires to value and preserve
the rich heritage of our composite culture?

hs
Ans 1. Clause d

2. Clause c

3. Clause a

at
4. Clause f

Question ID : 26433086791
Status : Not Answered

M
Chosen Option : --

Q.13 Which sea, located in the north-western part of the Indian Ocean, joins the Gulf of
Oman in the north-west and the Gulf of Aden in the south-west and covers a total area
of 1,491,000 square miles?
Ans 1. Arabian Sea
ap
2. Mediterranean Sea

3. Caribbean Sea

4. Coral Sea
at

Question ID : 264330142709
Status : Not Answered
Chosen Option : --
Pr

Q.14 Who among the following musicians was awarded Grammy in Music Album category
for his collaborative album ‘Global Drum Project’ along with Mickey Hart, Sikiru
Adepoju and Giovanni Hidalgo?
Ans 1. Ricky Kej
an

2. Vikku Vinayakram

3. Zakir Hussain

4. A R Rahman
ag

Question ID : 264330141388
Status : Answered
Chosen Option : 1

Q.15 Who among the following succeeded K Sivan as the ISRO chairman, in January 2022?
G

Ans 1. Shailesh Nayak

2. A S Kiran Kumar

3. VK Saraswat

4. S Somanath

Question ID : 26433082813
Status : Answered
Chosen Option : 4
Join @MathsByGaganPratap Telegram Channel for free PDFs
Q.16 Who among the following sportspersons is credited with the autobiography
‘Undisputed Truth’?
Ans 1. Mike Tyson

2. Matthew Hayden

3. Muhammad Ali

4. Kevin Pietersen

Question ID : 264330141413
Status : Not Answered
Chosen Option : --

Q.17 ‘Amuktamalyada’, a Telugu work, is composed by which of the following rulers?

hs
Ans 1. Sadasiva Raya

2. Krishnadevaraya

3. Vira Narasimha Raya

at
4. Achyuta Deva Raya 

Question ID : 26433055611
Status : Answered

M
Chosen Option : 2

Q.18 What was the minimum consumption expenditure (₹ per capita per month) set as a
benchmark of the poverty line for rural India in 1979?
Ans 1. 49.09

2. 43.5
ap
3. 56.7

4. 62.1
at
Question ID : 26433092888
Status : Not Answered
Chosen Option : --
Pr

Q.19 In which of the following sessions of the Indian National Congress was Dadabhai
Naoroji the President?
Ans 1. Calcutta Session, 1886

2. Madras Session, 1887


an

3. Bombay Session, 1889

4. Allahabad Session, 1892

Question ID : 26433065559
Status : Answered
ag

Chosen Option : 1

Q.20 How many urban post offices are there in India in 2020?
Ans 1. 17503
G

2. 15703

3. 15793

4. 15907

Question ID : 264330142772
Status : Not Answered
Chosen Option : --
Join @MathsByGaganPratap Telegram Channel for free PDFs
Q.21 The first ICC T20 Cricket World Cup was held in the year_______.
Ans 1. 2009

2. 2003

3. 2006

4. 2007

Question ID : 26433054678
Status : Answered
Chosen Option : 1

Q.22 Which of the following terms is NOT related to the structure of a stupa?

hs
Ans 1. Harmika

2. Vedika

3. Chhatra

4. Vimana

at
Question ID : 264330132974
Status : Answered
Chosen Option : 2

M
Q.23 Who among the following has won the ICC Women's Cricketer of the Year 2021?
Ans 1. Lizelle Lee

2. Harmanpreet Kaur

3. Tammy Beaumont
ap
4. Smriti Mandhana

Question ID : 264330142686
at
Status : Answered
Chosen Option : 4

Q.24 Which of the following steps must be performed first in the classical Odissi dance?
Pr

Ans 1. Mangalacharan

2. Pallavi

3. Abhinaya

4. Batu
an

Question ID : 264330141321
Status : Answered
Chosen Option : 1
ag

Q.25 In which year did Rajpath become the permanent venue for the Republic Day parade?
Ans 1. 1956

2. 1957

3. 1954
G

4. 1955

Question ID : 264330141330
Status : Answered
Chosen Option : 1

Section : Quantitative Aptitude


Join @MathsByGaganPratap Telegram Channel for free PDFs
Q.1 S and M can do a piece of work in 9 days, while M alone can finish it in 12 days. In how
many days can S alone finish the work?
Ans 1. 46 days 

2. 35 days 

3. 36 days 

4. 27 days 

Question ID : 26433063495
Status : Answered
Chosen Option : 3

Q.2 What is the average marks of all the students if the average marks of three batches of

hs
55, 60, and 45 students is 50, 55, and 60, respectively?
Ans 1. 51.23

2. 55.98

3. 57.42

at
4. 54.68

Question ID : 264330142581

M
Status : Answered
Chosen Option : 3

Q.3 A starts a business with ₹45,000 and was joined afterwards by B with ₹30,000. When
did B join if the profit at the end of the year was divided in the ratio 2 : 1?
Ans 1. After 6 months
ap
2. After 12 months

3. After 3 months

4. After 9 months
at

Question ID : 264330142670
Status : Answered
Chosen Option : 4
Pr

Q.4

Ans
1.
an

2.
ag

3.

4.
G

Question ID : 264330142257
Status : Answered
Chosen Option : 3
Join @MathsByGaganPratap Telegram Channel for free PDFs
Q.5 Ram reads 200 pages at the rate of 70 pages/hour in the morning. In the evening, when
he was tired, he read 200 pages at the rate of 30 pages/hour. What was his average
rate of reading, in pages per hour?
Ans 1. 42 pages/hour

2. 50 pages/hour

3. 62 pages/hour

4. 30 pages/hour

Question ID : 26433061243
Status : Answered
Chosen Option : 1

hs
Q.6 The six-digit number 7x1yyx is a multiple of 33 for non-zero digits x and y. Which of
the following could be a possible value of (x + y)?
Ans 1. 5

2. 4

at
3. 2

4. 3

M
Question ID : 26433083028
Status : Not Answered
Chosen Option : --

Q.7 A book seller bought a book for Rs.40 and sold it for Rs.45. Find his gain percentage.
Ans 1. 20%
ap
2. 15%

3. 12.5%

4. 17.5%
at

Question ID : 26433063860
Status : Answered
Chosen Option : 3
Pr

Q.8 Despite giving 23% discount on the book, Raj makes 10% profit. If Raj earns Rs.63
profit, then find the marked price of the book.
Ans 1. Rs.900

2. Rs.880
an

3. Rs.988

4. Rs.852

Question ID : 26433065279
ag

Status : Not Answered


Chosen Option : --

Q.9 A family member spends 25% of his monthly income on food, 35% of the remaining on
a home loan and other bank debts, and half of the rest on the education of the
G

children. If the member saves $46,215 every year, then the monthly income of the
member is:
Ans 1. $15,800

2. $190,248

3. $15,520

4. $19,200

Question ID : 26433075435
Status : Not Answered
Chosen Option : --
Join @MathsByGaganPratap Telegram Channel for free PDFs
Q.10 In a grocery shop, Samarth sold 950 g of sugar to a customer in place of 1 kg for ₹54.
The cost price of 1 kg sugar was ₹36. How much profit (in ₹) did he earn on selling the
sugar?
Ans 1. 16.2

2. 18.7

3. 20.2

4. 19.8

Question ID : 26433091620
Status : Answered
Chosen Option : 3

hs
Q.11 Find the simple interest on ₹24,000 at 18% per annum for a period of 7 months.
Ans 1. ₹2,525

2. ₹2,520

3. ₹2,530

at
4. ₹2,535

Question ID : 264330142590

M
Status : Answered
Chosen Option : 2

Q.12 In an 800 m race, the ratio of the speeds of two contestants Alka and Mira is 4 : 7. If
Alka has a start of 520 m, then, Alka will win by:
Ans 1. 490 m 
ap
2. 280 m 

3. 310 m 

4. 210 m 
at

Question ID : 26433081477
Status : Answered
Chosen Option : 2
Pr

Q.13
an

Ans 1. 4 unit2

2. 1 unit2

3. 3 unit2

4. 2 unit2
ag

Question ID : 26433071345
Status : Not Answered
Chosen Option : --
G

Q.14

Ans 1. 12 : 14 : 17

2. 10 : 12 : 15

3. 5 : 9 : 12

4. 3 : 4 : 5

Question ID : 26433063939
Status : Answered
Chosen Option : 2
Join @MathsByGaganPratap Telegram Channel for free PDFs
Q.15 A certain amount of work can be done by a man, a woman and a boy in 20, 30 and 60
days, respectively. How many boys should be there in order to complete the work in 2
days with 2 men and 8 women?
Ans 1. 5 boys

2. 10 boys

3. 12 boys

4. 8 boys

Question ID : 26433073114
Status : Not Answered
Chosen Option : --

hs
Q.16 What annual payment will discharge a debt of ₹1,936 in four annual equal instalments
at the rate of 14% on simple interest?
Ans 1. ₹500

2. ₹475

at
3. ₹425

4. ₹400

M
Question ID : 264330104195
Status : Not Answered
Chosen Option : --

Q.17 Which of the following numbers is divisible by 12?


Ans 1. 5409844
ap
2. 4298123

3. 4512984

4. 3215678
at

Question ID : 264330142547
Status : Answered
Chosen Option : 4
Pr

Q.18 A person covers 35 km distance at the speed of 45 km/h, and after that he covers 95
km distance at the speed of 105 km/h. What is the average speed during the whole
journey?
Ans
1.
an

2.
ag

3.

4.
G

Question ID : 26433082794
Status : Answered
Chosen Option : 4
Join @MathsByGaganPratap Telegram Channel for free PDFs
Q.19 To pack a set of books, Surbhi made a carton that was 48 inches long, 27 inches wide
and 30 inches tall. What will be the total surface area (in square feet) of this carton,
once it closed from all sides? [Use 1 foot = 12 inches]
Ans 1. 49.25

2. 48.25

3. 48.75

4. 49.75

Question ID : 26433075300
Status : Not Answered
Chosen Option : --

hs
Q.20 Simple interest gained on ₹9,000 at the rate of 8% is equal to ₹2,160. Calculate the
time.
Ans 1. 5 years

2. 4 years

at
3. 2 years

4. 3 years

M
Question ID : 264330142534
Status : Answered
Chosen Option : 4

Q.21 The ratio of the number of boys to that of girls in a village is 5 : 3. If 40% of the boys
ap
and 60% of the girls appeared in an examination, then the ratio of the number of boys
and girls who appeared in the examination to the number who did not appear in the
same examination, is:
Ans 1. 17 ∶ 21

2. 19 ∶ 21
at
3. 17 ∶ 27

4. 19 ∶ 23
Pr

Question ID : 26433074984
Status : Answered
Chosen Option : 2

Q.22 After offering two successive discounts of 10% each, the seller earns 10% profit. The
cost price is what per cent (rounded off to 2 decimal places) of the tag price?
an

Ans 1. 70.64%

2. 76.64%

3. 73.64%

4. 79.64%
ag

Question ID : 264330130174
Status : Answered
Chosen Option : 3
G

Q.23 The cost price of 15 apples is the same as the selling price of 12 apples. Find the
percentage profit.
Ans 1. 30

2. 20

3. 15

4. 25

Question ID : 26433081484
Status : Answered
Chosen Option : 3
Join @MathsByGaganPratap Telegram Channel for free PDFs
Q.24 Rekha has two laptops of the same kind. She sold one of them at ₹32,000 and incurred
a loss of 20%. At what price (in ₹) should she sell the second laptop to gain a profit of
20%?
Ans 1. 38,400

2. 41,600

3. 48,000

4. 44,000

Question ID : 264330142671
Status : Answered
Chosen Option : 3

hs
Q.25 A runs 4/3 times as fast as B. In a race, if A gives a lead of 80 m to B, find the distance
from the starting point where they both will meet.
Ans 1. 300 m

2. 360 m

at
3. 320 m

4. 340 m

M
Question ID : 264330142265
Status : Answered
Chosen Option : 3

Section : English Language


ap
Q.1 Select the option that expresses the given sentence in active voice.
The first prize will be won by Williams in the singing competition.
Ans 1. Williams will win the first prize in the singing competition.

2. Williams will be winning the first prize in the singing competition.


at
3. Williams had won the first prize in the singing competition.

4. Williams wins the first prize in the singing competition.


Pr

Question ID : 264330135921
Status : Answered
Chosen Option : 1

Q.2 Select the most appropriate option to fill in the blank.


The Thar desert is _____________ in the state of Rajasthan.
an

Ans 1. bounded

2. seen

3. situated
ag

4. placed  

Question ID : 26433060379
Status : Answered
Chosen Option : 3
G
Join @MathsByGaganPratap Telegram Channel for free PDFs
Q.3 Select the most appropriate meaning of the given idiom.

A piece of information received by word of mouth has no authenticity.


Ans 1. through an official order

2. through rumours

3. in written form

4. in spoken form

Question ID : 26433099088
Status : Answered
Chosen Option : 2

hs
Q.4 Select the most appropriate option to fill in the blank.
I hope you ___________ what I taught you today.
Ans 1. cleared 

2. divided 

at
3. summoned

4. understood 

M
Question ID : 26433059691
Status : Answered
Chosen Option : 4

Q.5 Select the INCORRECTLY spelt word.


Ans 1. Authority
ap
2. Illegitimate

3. Inaffectual

4. Authentic
at

Question ID : 26433087937
Status : Answered
Chosen Option : 2
Pr

Q.6 Select the idiom that gives the most appropriate meaning of the underlined phrase in
the following sentence.
I finally understood how to operate the new smart television.
Ans 1. Cast aside
an

2. Hit the sack

3. Don’t know

4. Figured out
ag

Question ID : 26433086441
Status : Answered
Chosen Option : 4

Q.7 Select the most appropriate option to fill in the blank.


G

The propaganda was made to invade ________ neighbouring countries.


Ans 1. smaller

2. largest

3. stranger

4. strongest

Question ID : 26433060789
Status : Answered
Chosen Option : 2
Join @MathsByGaganPratap Telegram Channel for free PDFs
Q.8 Select the most appropriate synonym of the given word.

Awful
Ans 1. Inoffensive

2. Delightful

3. Acceptable

4. Terrible

Question ID : 26433071874
Status : Answered
Chosen Option : 2

hs
Q.9 Select the most appropriate option that can substitute the underlined segment in the
given sentence.

Athira would like to get him a special birthday present, but nothing springs to mind.
Ans 1. almost always

at
2. forget about something

3. dawns on

4. from the beginning of time

M
Question ID : 264330133177
Status : Not Answered
ap Chosen Option : --

Q.10 Identify the sentence that contains no spelling errors.


Ans 1. Games insteal in people the values of timliness, honesty and consistency in their
routines. 
2. Games instil in people the values of timeliness, honesty and consistensy in their
routeenes.
at
3. Games instil in people the values of timeliness, honesty and consistency in their
routines.
4. Games insteal in people the values of timeliness, honesty and consistensy in their
routines.
Pr

Question ID : 26433088541
Status : Answered
Chosen Option : 4
an

Q.11 Select the option that expresses the given sentence in indirect speech.
Arya said, “I am very busy now.”
Ans 1. Arya said that she is being very busy now.  

2. Arya said that she is be very busy now.  

3. Arya said that she was very busy then. 


ag

4. Arya said that she was being very busy then.  

Question ID : 264330135983
Status : Answered
G

Chosen Option : 3
Join @MathsByGaganPratap Telegram Channel for free PDFs
Q.12 Select the most appropriate option that can substitute the underlined words in the
given sentence.
Students were asked to write the historical events in the order of their occurrence.
Ans 1.  aesthetic order

2.  historic order

3. chronological order

4.  diachronic order

Question ID : 26433059721
Status : Answered
Chosen Option : 3

hs
Q.13 Identify the idiom/phrase that can best substitute the underlined segment.

They discussed everything in the meeting and finalised the next step.
Ans 1. the home stretch 

2. through thick and thin 

at
3. the eleventh hour 

4. the whole nine yards 

M
Question ID : 26433088459
Status : Answered
Chosen Option : 2
ap
Q.14 Select the most appropriate synonym of the underlined word in the following
sentence.

UNESCO is committed to protecting individual privacy and securing the personal


information made available to us when you visit unesco.org, as well as UNESCO
pages on other sites.
at
Ans 1. disloyal 

2.  devoted

3. unfaithful 
Pr

4. inconstant 

Question ID : 26433059799
Status : Answered
Chosen Option : 2
an

Q.15 The following sentence has been split into four segments. Identify the segment that
contains an INCORRECTLY spelt word.
Since the health hazards have reduced, / the ministry has relaxed the earlier
regulations / and has suggested only a short period / of quarrantine for the affected
people.
ag

Ans 1. of quarrantine for the affected people

2. the ministry has relaxed the earlier regulations

3. Since the health hazards have reduced 

4. and has suggested only a short period 


G

Question ID : 264330143249
Status : Answered
Chosen Option : 1
Join @MathsByGaganPratap Telegram Channel for free PDFs
Comprehension:
In the following passage, some words have been deleted. Read the passage carefully and
select the most appropriate option to fill in each blank.
The white tiger is an offspring of Bengal tiger, which has the necessary gene for white
colouring. It has beautiful black stripes on (1) _____ white coat. (2)______ species of animal
has blue eyes and a pink nose.
A pure white tiger is totally white without any stripes, which is due to the presence of double
recessive allele in the genetic code. It happens only in the Bengal tiger subspecies, and only
one in 10,000 births can have it naturally. White tigers are rarely seen in the wild, and only
twelve of them have been spotted in India (3)______ the last one-hundred years. Tigers have
a lifespan of 10-15 years. They are (4) _______ animals, and are fond of hunting at night.
Their diet consists of any animals that they can catch, and can eat up to 40 pounds of meat at
one time. After such a big meal, they usually remain away for several days from food.
They are slow runners, (5) ______ good swimmers. They are poor climbers and cannot climb
a tree easily. They live in an area ranging from 10 to 30 square miles depending upon the
availability of prey, water and shelter. Habitat loss, poaching and expansion of the human

hs
population have together contributed to their becoming endangered. They are widely hunted
for their body-parts, which are used in traditional Chinese medicines.

SubQuestion No : 16
Q.16 Select the most appropriate option to fill in blank number 1.
Ans 1. a

at
2. an

3. the

4. of

M
Question ID : 26433094225
Status : Answered
ap Chosen Option : 1

Comprehension:
In the following passage, some words have been deleted. Read the passage carefully and
select the most appropriate option to fill in each blank.
The white tiger is an offspring of Bengal tiger, which has the necessary gene for white
colouring. It has beautiful black stripes on (1) _____ white coat. (2)______ species of animal
at
has blue eyes and a pink nose.
A pure white tiger is totally white without any stripes, which is due to the presence of double
recessive allele in the genetic code. It happens only in the Bengal tiger subspecies, and only
one in 10,000 births can have it naturally. White tigers are rarely seen in the wild, and only
twelve of them have been spotted in India (3)______ the last one-hundred years. Tigers have
Pr

a lifespan of 10-15 years. They are (4) _______ animals, and are fond of hunting at night.
Their diet consists of any animals that they can catch, and can eat up to 40 pounds of meat at
one time. After such a big meal, they usually remain away for several days from food.
They are slow runners, (5) ______ good swimmers. They are poor climbers and cannot climb
a tree easily. They live in an area ranging from 10 to 30 square miles depending upon the
availability of prey, water and shelter. Habitat loss, poaching and expansion of the human
population have together contributed to their becoming endangered. They are widely hunted
an

for their body-parts, which are used in traditional Chinese medicines.

SubQuestion No : 17
Q.17 Select the most appropriate option to fill in blank number 2.
Ans 1. This

2. These
ag

3. Those

4. Them

Question ID : 26433094226
G

Status : Answered
Chosen Option : 2
Join @MathsByGaganPratap Telegram Channel for free PDFs
Comprehension:
In the following passage, some words have been deleted. Read the passage carefully and
select the most appropriate option to fill in each blank.
The white tiger is an offspring of Bengal tiger, which has the necessary gene for white
colouring. It has beautiful black stripes on (1) _____ white coat. (2)______ species of animal
has blue eyes and a pink nose.
A pure white tiger is totally white without any stripes, which is due to the presence of double
recessive allele in the genetic code. It happens only in the Bengal tiger subspecies, and only
one in 10,000 births can have it naturally. White tigers are rarely seen in the wild, and only
twelve of them have been spotted in India (3)______ the last one-hundred years. Tigers have
a lifespan of 10-15 years. They are (4) _______ animals, and are fond of hunting at night.
Their diet consists of any animals that they can catch, and can eat up to 40 pounds of meat at
one time. After such a big meal, they usually remain away for several days from food.
They are slow runners, (5) ______ good swimmers. They are poor climbers and cannot climb
a tree easily. They live in an area ranging from 10 to 30 square miles depending upon the
availability of prey, water and shelter. Habitat loss, poaching and expansion of the human

hs
population have together contributed to their becoming endangered. They are widely hunted
for their body-parts, which are used in traditional Chinese medicines.

SubQuestion No : 18
Q.18 Select the most appropriate option to fill in blank number 3.
Ans 1. following

at
2. next to

3. behind

4. in

M
Question ID : 26433094227
Status : Answered
ap Chosen Option : 4

Comprehension:
In the following passage, some words have been deleted. Read the passage carefully and
select the most appropriate option to fill in each blank.
The white tiger is an offspring of Bengal tiger, which has the necessary gene for white
colouring. It has beautiful black stripes on (1) _____ white coat. (2)______ species of animal
at
has blue eyes and a pink nose.
A pure white tiger is totally white without any stripes, which is due to the presence of double
recessive allele in the genetic code. It happens only in the Bengal tiger subspecies, and only
one in 10,000 births can have it naturally. White tigers are rarely seen in the wild, and only
twelve of them have been spotted in India (3)______ the last one-hundred years. Tigers have
Pr

a lifespan of 10-15 years. They are (4) _______ animals, and are fond of hunting at night.
Their diet consists of any animals that they can catch, and can eat up to 40 pounds of meat at
one time. After such a big meal, they usually remain away for several days from food.
They are slow runners, (5) ______ good swimmers. They are poor climbers and cannot climb
a tree easily. They live in an area ranging from 10 to 30 square miles depending upon the
availability of prey, water and shelter. Habitat loss, poaching and expansion of the human
population have together contributed to their becoming endangered. They are widely hunted
an

for their body-parts, which are used in traditional Chinese medicines.

SubQuestion No : 19
Q.19 Select the most appropriate option to fill in blank number 4.
Ans 1. solitary

2. separate
ag

3. loner

4. unaccompanied

Question ID : 26433094228
G

Status : Answered
Chosen Option : 1
Join @MathsByGaganPratap Telegram Channel for free PDFs
Comprehension:
In the following passage, some words have been deleted. Read the passage carefully and
select the most appropriate option to fill in each blank.
The white tiger is an offspring of Bengal tiger, which has the necessary gene for white
colouring. It has beautiful black stripes on (1) _____ white coat. (2)______ species of animal
has blue eyes and a pink nose.
A pure white tiger is totally white without any stripes, which is due to the presence of double
recessive allele in the genetic code. It happens only in the Bengal tiger subspecies, and only
one in 10,000 births can have it naturally. White tigers are rarely seen in the wild, and only
twelve of them have been spotted in India (3)______ the last one-hundred years. Tigers have
a lifespan of 10-15 years. They are (4) _______ animals, and are fond of hunting at night.
Their diet consists of any animals that they can catch, and can eat up to 40 pounds of meat at
one time. After such a big meal, they usually remain away for several days from food.
They are slow runners, (5) ______ good swimmers. They are poor climbers and cannot climb
a tree easily. They live in an area ranging from 10 to 30 square miles depending upon the
availability of prey, water and shelter. Habitat loss, poaching and expansion of the human

hs
population have together contributed to their becoming endangered. They are widely hunted
for their body-parts, which are used in traditional Chinese medicines.

SubQuestion No : 20
Q.20 Select the most appropriate option to fill in blank number 5.
Ans 1. but

at
2. either

3. nor

4. or

M
Question ID : 26433094229
Status : Answered
ap Chosen Option : 1

Comprehension:
Read the given passage and answer the questions that follow.
Fine motor skills development allows children to execute vital tasks such as feeding
themselves, writing, zipping their garments, and more. Small muscles that govern the hand,
fingers, and thumbs are used in fine motor activity. Fine motor abilities assist youngsters in
at
doing crucial tasks such as self-feeding, handling objects, and writing. The capacity to do self-
care and everyday chores utilising fine motor skills promotes the development of a child's
self-esteem and confidence.
Fine motor skills develop systematically in children. Most new-borns have a reflexive grip at
birth and begin to reach for items about three months of age. They will also begin to practise
Pr

their voluntary grip and two-handed palmar grasps. At 5 months, they should be able to
demonstrate a one-handed palmar grip, and at 6 months, they should be able to create a
controlled reach. Most new-borns begin to reach and grab for items to put in their mouth
between the ages of 6 and 12 months, and they begin to gain the capacity to regulate the
release of objects they are gripping. They will also develop a pincer grasp and use it to pick
objects up (thumb and one finger).They will also be able to move an object from one hand to
the other and drop and pick up their toys.
an

Most youngsters are interested in stacking building blocks between the ages of 12 and 24
months. They will also practise putting rings on a stick, putting pegs on a pegboard, and
turning pages a few at a time. They can also begin to draw, paint with their entire arm
movement, moving their hands and producing strokes, and feed themselves with little to no
help.
Toddlers at the age of two can begin to string big beads, turn single pages, cut with scissors,
and handle crayons with their thumb and fingers (instead of their fist). They will also be able
ag

to use only one hand for most tasks and begin to paint with their wrists, forming dots, lines,
and circular strokes. Your youngster will also be able to eat consistently and without help.
Most youngsters will be able to cut along the dotted lines on paper by the age of four. They
will be able to draw a cross or square shape and write their name and the numbers 1 to 5.
They will be able to copy letters and will have mastered their handedness. They will also be
able to dress themselves for the first time.
G

SubQuestion No : 21
Q.21 Select the most appropriate title for the passage.
Ans 1. Development of Fine Motor Skills in Children

2. Importance of Fine Motor Skills

3. Systematic Development of Children

4. Development of New-borns

Question ID : 26433099661
Status : Not Answered
Chosen Option : --
Join @MathsByGaganPratap Telegram Channel for free PDFs
Comprehension:
Read the given passage and answer the questions that follow.
Fine motor skills development allows children to execute vital tasks such as feeding
themselves, writing, zipping their garments, and more. Small muscles that govern the hand,
fingers, and thumbs are used in fine motor activity. Fine motor abilities assist youngsters in
doing crucial tasks such as self-feeding, handling objects, and writing. The capacity to do self-
care and everyday chores utilising fine motor skills promotes the development of a child's
self-esteem and confidence.
Fine motor skills develop systematically in children. Most new-borns have a reflexive grip at
birth and begin to reach for items about three months of age. They will also begin to practise
their voluntary grip and two-handed palmar grasps. At 5 months, they should be able to
demonstrate a one-handed palmar grip, and at 6 months, they should be able to create a
controlled reach. Most new-borns begin to reach and grab for items to put in their mouth
between the ages of 6 and 12 months, and they begin to gain the capacity to regulate the
release of objects they are gripping. They will also develop a pincer grasp and use it to pick
objects up (thumb and one finger).They will also be able to move an object from one hand to

hs
the other and drop and pick up their toys.
Most youngsters are interested in stacking building blocks between the ages of 12 and 24
months. They will also practise putting rings on a stick, putting pegs on a pegboard, and
turning pages a few at a time. They can also begin to draw, paint with their entire arm
movement, moving their hands and producing strokes, and feed themselves with little to no
help.
Toddlers at the age of two can begin to string big beads, turn single pages, cut with scissors,

at
and handle crayons with their thumb and fingers (instead of their fist). They will also be able
to use only one hand for most tasks and begin to paint with their wrists, forming dots, lines,
and circular strokes. Your youngster will also be able to eat consistently and without help.
Most youngsters will be able to cut along the dotted lines on paper by the age of four. They
will be able to draw a cross or square shape and write their name and the numbers 1 to 5.

M
They will be able to copy letters and will have mastered their handedness. They will also be
able to dress themselves for the first time.

SubQuestion No : 22
Q.22 In light of the above passage, which of the following toys may help in the development
of fine motor skills?
Ans 1. Crayons
ap
2. All  of the given options

3. String beads

4. Stacks and building blocks


at

Question ID : 26433099664
Status : Not Answered
Pr

Chosen Option : --
an
ag
G
Join @MathsByGaganPratap Telegram Channel for free PDFs
Comprehension:
Read the given passage and answer the questions that follow.
Fine motor skills development allows children to execute vital tasks such as feeding
themselves, writing, zipping their garments, and more. Small muscles that govern the hand,
fingers, and thumbs are used in fine motor activity. Fine motor abilities assist youngsters in
doing crucial tasks such as self-feeding, handling objects, and writing. The capacity to do self-
care and everyday chores utilising fine motor skills promotes the development of a child's
self-esteem and confidence.
Fine motor skills develop systematically in children. Most new-borns have a reflexive grip at
birth and begin to reach for items about three months of age. They will also begin to practise
their voluntary grip and two-handed palmar grasps. At 5 months, they should be able to
demonstrate a one-handed palmar grip, and at 6 months, they should be able to create a
controlled reach. Most new-borns begin to reach and grab for items to put in their mouth
between the ages of 6 and 12 months, and they begin to gain the capacity to regulate the
release of objects they are gripping. They will also develop a pincer grasp and use it to pick
objects up (thumb and one finger).They will also be able to move an object from one hand to

hs
the other and drop and pick up their toys.
Most youngsters are interested in stacking building blocks between the ages of 12 and 24
months. They will also practise putting rings on a stick, putting pegs on a pegboard, and
turning pages a few at a time. They can also begin to draw, paint with their entire arm
movement, moving their hands and producing strokes, and feed themselves with little to no
help.
Toddlers at the age of two can begin to string big beads, turn single pages, cut with scissors,

at
and handle crayons with their thumb and fingers (instead of their fist). They will also be able
to use only one hand for most tasks and begin to paint with their wrists, forming dots, lines,
and circular strokes. Your youngster will also be able to eat consistently and without help.
Most youngsters will be able to cut along the dotted lines on paper by the age of four. They
will be able to draw a cross or square shape and write their name and the numbers 1 to 5.

M
They will be able to copy letters and will have mastered their handedness. They will also be
able to dress themselves for the first time.

SubQuestion No : 23
Q.23 When do children learn to regulate the release of the objects that they are gripping?
ap
Ans 1. 3-6 months

2. 2-4 years

3. 12-24 months

4. 6-12 months
at
Question ID : 26433099663
Status : Answered
Chosen Option : 4
Pr
an
ag
G
Join @MathsByGaganPratap Telegram Channel for free PDFs
Comprehension:
Read the given passage and answer the questions that follow.
Fine motor skills development allows children to execute vital tasks such as feeding
themselves, writing, zipping their garments, and more. Small muscles that govern the hand,
fingers, and thumbs are used in fine motor activity. Fine motor abilities assist youngsters in
doing crucial tasks such as self-feeding, handling objects, and writing. The capacity to do self-
care and everyday chores utilising fine motor skills promotes the development of a child's
self-esteem and confidence.
Fine motor skills develop systematically in children. Most new-borns have a reflexive grip at
birth and begin to reach for items about three months of age. They will also begin to practise
their voluntary grip and two-handed palmar grasps. At 5 months, they should be able to
demonstrate a one-handed palmar grip, and at 6 months, they should be able to create a
controlled reach. Most new-borns begin to reach and grab for items to put in their mouth
between the ages of 6 and 12 months, and they begin to gain the capacity to regulate the
release of objects they are gripping. They will also develop a pincer grasp and use it to pick
objects up (thumb and one finger).They will also be able to move an object from one hand to

hs
the other and drop and pick up their toys.
Most youngsters are interested in stacking building blocks between the ages of 12 and 24
months. They will also practise putting rings on a stick, putting pegs on a pegboard, and
turning pages a few at a time. They can also begin to draw, paint with their entire arm
movement, moving their hands and producing strokes, and feed themselves with little to no
help.
Toddlers at the age of two can begin to string big beads, turn single pages, cut with scissors,

at
and handle crayons with their thumb and fingers (instead of their fist). They will also be able
to use only one hand for most tasks and begin to paint with their wrists, forming dots, lines,
and circular strokes. Your youngster will also be able to eat consistently and without help.
Most youngsters will be able to cut along the dotted lines on paper by the age of four. They
will be able to draw a cross or square shape and write their name and the numbers 1 to 5.

M
They will be able to copy letters and will have mastered their handedness. They will also be
able to dress themselves for the first time.

SubQuestion No : 24
Q.24 Select the most appropriate ANTONYM of the given word.
Systematically
Ans 1. Evenly
ap
2. Selectively

3. Oddly

4. Haphazardly
at

Question ID : 26433099665
Status : Answered
Pr

Chosen Option : 4
an
ag
G
Join @MathsByGaganPratap Telegram Channel for free PDFs
Comprehension:
Read the given passage and answer the questions that follow.
Fine motor skills development allows children to execute vital tasks such as feeding
themselves, writing, zipping their garments, and more. Small muscles that govern the hand,
fingers, and thumbs are used in fine motor activity. Fine motor abilities assist youngsters in
doing crucial tasks such as self-feeding, handling objects, and writing. The capacity to do self-
care and everyday chores utilising fine motor skills promotes the development of a child's
self-esteem and confidence.
Fine motor skills develop systematically in children. Most new-borns have a reflexive grip at
birth and begin to reach for items about three months of age. They will also begin to practise
their voluntary grip and two-handed palmar grasps. At 5 months, they should be able to
demonstrate a one-handed palmar grip, and at 6 months, they should be able to create a
controlled reach. Most new-borns begin to reach and grab for items to put in their mouth
between the ages of 6 and 12 months, and they begin to gain the capacity to regulate the
release of objects they are gripping. They will also develop a pincer grasp and use it to pick
objects up (thumb and one finger).They will also be able to move an object from one hand to

hs
the other and drop and pick up their toys.
Most youngsters are interested in stacking building blocks between the ages of 12 and 24
months. They will also practise putting rings on a stick, putting pegs on a pegboard, and
turning pages a few at a time. They can also begin to draw, paint with their entire arm
movement, moving their hands and producing strokes, and feed themselves with little to no
help.
Toddlers at the age of two can begin to string big beads, turn single pages, cut with scissors,

at
and handle crayons with their thumb and fingers (instead of their fist). They will also be able
to use only one hand for most tasks and begin to paint with their wrists, forming dots, lines,
and circular strokes. Your youngster will also be able to eat consistently and without help.
Most youngsters will be able to cut along the dotted lines on paper by the age of four. They
will be able to draw a cross or square shape and write their name and the numbers 1 to 5.

M
They will be able to copy letters and will have mastered their handedness. They will also be
able to dress themselves for the first time.

SubQuestion No : 25
Q.25 When do children get interested in playing with building blocks?
ap
Ans 1. 12-24 months

2. 2-4 years

3. 6-12 months

4. 0-6 months
at
Question ID : 26433099662
Status : Answered
Chosen Option : 1
Pr
an
ag
G

You might also like